Sei sulla pagina 1di 221

 

1. ¿QUÉ ESTRUCTURA SE LESIONA MÁS FRECUENTEMENTE EN UN ESGUINCE DE


TOBILLO?:

1. 1. LIGAMENTO DELTOIDEO
2. 2. LIGAMENTO PERONEO-CALCÁNEO
3. 3. LIGAMENTO PERONEO-ASTRAGALINO ANTERIOR
4. 4. LIGAMENTO PERONEO-ASTRAGALINO POSTERIOR
Gráfico de respuestas
Comentario
La mayoría de los esguinces de tobillo siguen un patrón lesional en supinación y rotación externa.
Con este gesto, el ligamento que va a soportar la tensión es el lateral externo. Por la posición de
sus tres fascículos, el primero en tensarse, y por tanto en lesionarse (estiramiento, rotura parcial o
completa) es el fascículo peroneo- astragalino anterior. Si la fuerza es suficientemente grande para
sobrepasar la resistencia tensil de este fascículo, se lesionarán consecutivamente el peroneo-
calcáneo y el peroneo- astragalino posterior. El ligamento deltoideo (lateral interno) suele
lesionarse con menor frecuencia (inversión y rotación interna).(R3)

2. En el síndrome de apnea del sueño hay hipoxia, hipercapnia y acidosis y puede


asociarse con las siguientes alteraciones EXCEPTO:

1. 1. Hipertensión arterial.
2. 2. Hipertensión pulmonar.
3. 3. Hipoventilación alveolar.
4. 4. Aumento de las fases de ondas lentas del sueño profundo.
Gráfico de respuestas
Comentario

Pregunta fácil acerca de un tema de moda: el SAOS.

El síndrome de apnea obstructiva del sueño (SAOS) apareció en el MIR por primera vez hace
pocas convocatorias, así pues se trata de un tema que merece cierta atención. En este síndrome,
existe hipertensión arterial con mayor frecuencia que en la población general, entre otras razones
porque suele tratarse de pacientes obesos en los que además coexisten otros factores de riesgo
cardiovascular.

La hipertensión pulmonar también es más frecuente que en el paciente sano, debido a la hipoxia
episódica a la que están sometidos durante sus apneas, lo que condiciona una vasoconstricción
pulmonar. La hipoventilación alveolar (opción 3) es consecuencia directa de la apnea, y condiciona
un aumento de la PaCO2, que es la traducción gasométrica de la hipoventilación. En cuanto a la
bradicardia, también puede registrarse durante la noche en estos enfermos, así como otras
alteraciones cardiológicas, como mayor incidencia de arritmias, extrasístoles, etc. Esto se debe al
efecto arritmogénico que tiene la hipoxia sobre el tejido de conducción cardíaco.

Sin embargo, las fases de ondas lentas (sueño profundo)no están aumentadas, sino todo lo
contrario, puesto que la calidad del sueño de estos pacientes está claramente disminuida respecto
a la población general (recuerda que, en ocasiones, incluso se despiertan varias veces por noche,
como consecuencia de las apneas si son muy prolongadas, o por el propio ronquido), de modo que
la respuesta correcta es la 5.

A continuación un esquema que resume lo más destacado de este tema.(R4)

 
 
 
 
Tratamiento del síndrome de apnea del sueño

3. Mujer de 24 años, primigesta, que en una revisión en la 24ª semana de gestación


presenta cifras tensionales de 140/85 (cifras en la primera consulta de 120/65) que se
repiten en la siguiente consulta. En las últimas dos semanas ha aumentado 2 kg de peso.
En la examen de orina hay proteinuria de +++. No presenta ninguna otra sintomatología.
¿Cuál de las siguientes actuaciones NO sería correcta en esta situación?

1. 1. Tratamiento hipotensor con metildopa.


2. 2. Reposo en decúbito lateral izquierdo.
3. 3. Control de la tensión arterial 4 veces al día.
4. 4. Control diario de peso.
Gráfico de respuestas
Comentario

Lo primero es hacer un diagnóstico de la patología de la paciente: hipertensión presente a partir de


la 20º semana de gestación y proteinuria es compatible con preeclampsia. De las 4 opciones

 
 
 
 
terapéuticas no optaríamos por administrar un hipotensor (metildopa) dado que la paciente
mantiene una TA menor a 150/100 (indicado si la TA es superior a este límite de forma
persistente).(R1)

4. A 60-year-old female is tested for the BRCA1 gene after her sister is diagnosed with
BRCA1-positive breast cancer. The test is positive for BRCA1 in our patient too. Which of
the following antigens should we consider testing her for?

1. 1. CEA
2. 2. CA125
3. 3. CA19-9
4. 4. CA15.3

(R2)

5. ¿Cuál de los siguientes NO es criterio de Stegen y Toledo para el diagnostico de


tuberculosis infantil?

1. 1. Cuadro clínico.
2. 2. Test de ADA positivo.
3. 3. Radiografía sugestiva.
4. 4. Antecedente epidemiológico de contacto.

(R2)

 
 
 
 
6. Acerca de las siguientes afirmaciones sobre la disección enzimática de las
inmunoglobulinas, una de ellas es FALSA:

1. 1. Con papaína se obtienen tres fragmentos: 2 Fab idénticos y 1 Fc (cristalizable).


2. 2. En la fracción Fc se encuentran los extremos aminoterminales de las cadenas pesadas.
En la digestión por pepsina se consigue un fragmento bivalente (F(ab)2) y 2 péptidos Fc
3. 3.
grandes, así como otros pequeños.
La fracción Fab está constituida por las cadenas ligeras y parte de las cadenas pesadas; la
4. 4.
Fc sólo por una porción de las cadenas pesadas.
Gráfico de respuestas
Comentario
Pregunta difícil ya que la digestión enzimática de las Igs no se ha preguntado nunca en el MIR, por
lo que no te debe importar mucho haberla fallado. Estudia bien el tema de las Igs porque es clave
en el MIR. Las inmunoglobulinas son glucoproteínas sintetizadas por los linfocitos B y células
plasmáticas en respuesta al estímulo antigénico. La estructura de una Ig es un tetrámero formado
por dos cadenas pesadas y dos ligeras, que se ensamblan adoptando la forma de Y. Las cadenas
de Ig, tanto pesadas como ligeras, presentan una parte variable (V) que difiere de un clon a otro y
es en la que reside la especificidad para el antígeno y otra región constante (C) común a los
diversos clones. Cuando sometemos a disección enzimática con papaína las Igs obtenemos dos
fragmentos idénticos Fab (formados por parte de las cadenas pesada y ligera) donde reside la
función de unión al antígeno y una fracción Fc (formado por cadena pesada sólo) que se encuentra
en la zona carboxiterminal y que es donde reside las distintas funciones de la misma como la de
atravesar la placenta (Papa enseña el ABC => papaína da 2 fragmentos Fab y uno Fc). Hay otra
digestión enzimática importante que es la pepsina en la que se forman un fragmento F(ab)2 y un
2Fc?.(R2)

7. Señale la opción FALSA:

El primer signo de aparición de pubertad en la mujer es la aparición de botón mamario o


1. 1.
telarquia.
El primer signo de aparición de pubertad en el varón es la aparición de vello en la zona
2. 2.
púbica o pubarquia.
El estirón puberal es algo más precoz en las niñas frente al varón, coincidiendo con el
3. 3.
estadio de Tanner II.
4. 4. Un volumen testicular inferior a 3 cc indica estadio Tanner I.
Gráfico de respuestas
Comentario

En el ENARM, alguna vez se ha hecho alusión al del desarrollo puberal, definido por los estadios
de Tanner (I- V), siendo el Tanner I el estadio prepuberal o infantil. Recuerde que el signo físico
inicial que marca el comienzo de la pubertad en el varón es el incremento del tamaño testicular por
encima de 4 cc, y en la mujer la aparición de telarquia o botón mamario (opción 1 verdadera). La
aparición de vello axilar y pubiano es fruto de un fenómeno independiente denominado
adrenarquia. Durante la pubertad acaece el estirón puberal, siendo más precoz en la mujer (Tanner
II) frente el varón (estadios III-IV), generalmente siendo máximo previo a la aparición de la
menarquia (edad aproximada 12 años). Por último un dato que muestra la impregnación
estrogénica de la pubertad es el incremento de los ovarios y cuerpo uterino en la mujer,
invirtiéndose la relación cuerpo-cuello existente en estadios infantiles.(R2)

8. Lactante de tres meses de vida que desde hace un mes presenta episodios
intermitentes de distensión abdominal, dolores de tipo cólico y algunos vómitos.
Tendencia al estreñimiento. Entre sus antecedentes personales hay que destacar que fue

 
 
 
 
prematuro, pesó 900 g al nacimiento y tuvo dificultad respiratoria importante que precisó
ventilación asistida durante 15 días. ¿Cuál es el diagnóstico más probable de su cuadro
clínico?

1. 1. Estenosis cólica secundaria a enterocolitis necrotizante.


2. 2. Megacolon congénito.
3. 3. Vólvulo intestinal intermitente.
4. 4. Adenitis mesentérica secundaria a neumopatía crónica.
Gráfico de respuestas
Comentario

La enterocolitis necrotizante suelen presentarla en el ENARM como un cuadro de distensión


abdominal y evacuaciones sanguinolentas. Sin embargo, esta pregunta va un poco más allá.
Observa que nos dan tres antecedentes que implicarían un mayor riesgo para esta enfermedad:
prematuridad, bajo peso, necesidad de ventilación asistida... Si a esto añadimos un cuadro clínico
compatible con obstrucción intestinal (estreñimiento, vómitos, distensión y dolor cólico), la opción
que mejor encaja es la 1.

De hecho, una de las complicaciones más habituales de esta entidad son las estenosis
intestinales, que generan cuadros suboclusivos, como en este lactante que nos comentan.(R1)

Enterocolitis necrotizante. Radiología

9. En relación con el crecimiento y desarrollo del niño en sus primeros años de vida,
todas las afirmaciones siguientes son ciertas, EXCEPTO:

 
 
 
 
1. 1. A los 12 meses de vida, el niño aproximadamente duplica su peso al nacimiento.
2. 2. La talla al nacer se duplica, aproximadamente, a los cuatro años de vida.
La velocidad de crecimiento en estatura es máxima durante los cuatro primeros años de
3. 3.
vida.
4. 4. Hacia los 2 meses suele haber hecho su aparición la sonrisa voluntaria o social.
Gráfico de respuestas
Comentario

El peso al nacimiento se duplica al 5º mes. Para el 12º mes el peso al nacimiento se ha triplicado,
por lo que la respuesta 1 es incorrecta.

Este tipo de preguntas no se pueden allar en el ENARM!(R1)

10. Si al hacer una ecografía cerebral a un prematuro nos encontramos con imágenes
anómalas periventriculares e hiperecogenicidad en más del 50% del espacio ventricular
con dilatación ventricular, el diagnóstico es:

1. 1. Hemorragia intraventricular tipo II y áreas de leucomalacia.


2. 2. Hemorragia intraventricular tipo III y áreas de leucomalacia.
3. 3. Hemorragia intraventricular tipo IV y áreas de leucomalacia.
4. 4. Hemorragia intraventricular tipo V y áreas de leucomalacia.
Gráfico de respuestas
Comentario

Los estudios Ultrasonográficos permiten clasificar las HIV en cuatro grupos fundamentales:

Grado I: Cuandola hemorragia está localizada en la matriz germinal, sin o mínima hemorragia
intraventricular.

Grado II: El sangrado es intraventricular y ocupa entre el 10 y el 50 % del ventrículo.

Grado III: El sangrado intraventricular es mayor del 50 % y el ventrículo lateral está distendido.

Grado IV: a lo anterior se asocia sangramiento intraparenquimatoso.(R2)

11. Uno de los siguientes criterios no es sugestivo de fibrosis pulmonar idiopática:

1. 1. Alteración ventilatoria restrictiva.


Ausencia de hallazgos histopatológicos en biopsia transbronquial o lavado broncoalveolar
2. 2.
sugestivos de un diagnóstico alternativo.
3. 3. Exclusión de cualquier causa de enfermedad pulmonar intersticial difusa.
TC torácico de alta resolución con lesión del tipo “panal de miel”, asociada de forma
4. 4.
constante a imágenes en “vidrio esmerilado” extensos.
Gráfico de respuestas
Comentario

La lesión en panal no se asocia de forma constante al vidrio esmerilado… Sólo esta forma de
expresarlo, tan restrictiva, debería hacerte sospechar lo peor. La imagen en vidrio esmerilado
aparece en fases más incipientes, cuando predomina la inflamación.

 
 
 
 
En cambio, la imagen en panal se ve en fases muy tardías, con predominio fibrótico y de
destrucción parenquimatosa, de modo que no siempre coexisten (de hecho, no suelen hacerlo).
Repasa los patrones radiológicos de las enfermedades intersticiales en el comentario previo a
éste.(R4)

12. Un hombre 56 años, diabético y con sobrepeso moderado, presenta una discreta
hepatomegalia. El enfermo indica que no es bebedor habitual, sólo excepcionalmente
toma algo de vino. Una biopsia de hígado muestra acúmulos de vacuolas claras
citoplásmicas en un 50% de los hepatocitos. Focos inflamatorios múltiples con
neutrófilos, degeneración hialina de Mallory y fibrosis alrededor de las vénulas hepáticas
terminales. Señale la respuesta correcta en este caso:

1. 1. Debe repetirse la biopsia para saber si los cambios son difusos.


2. 2. Los cambios histológicos son irreversibles y acabarán produciendo una cirrosis.
3. 3. La lesión está causada claramente por el alcohol a pesar de la negativa del enfermo.
Es importante disminuir peso y controlar adecuadamente la diabetes para evitar el posible
4. 4.
desarrollo de una cirrosis.
Gráfico de respuestas
Comentario

Pregunta importante en forma de caso clínico sobre el diagnóstico diferencial de las hepatopatías.

En este caso, debemos centrar el diagnóstico diferencial de la lesión del paciente en base a los
hallazgos histológicos encontrados. Los acúmulos de vacuolas claras denotan el componente
graso o esteatósico implicado, sobre el que se añade el infiltrado por células inflamatorias
compatible con el diagnóstico de hepatitis (respuesta 2 falsa). La causa más frecuente de daño
hepático y de evolución a cirrosis en nuestro medio es, actualmente, el consumo de alcohol. La
presencia de degeneración hialina de Mallory en la biopsia es característica, pero no específica, de
la hepatitis alcohólica. Puesto que el paciente ha negado el consumo de alcohol y se destaca al
principio que es diabético y con sobrepeso, se deben considerar otros diagnósticos (respuesta 4
falsa) sin necesidad de repetir la biopsia (respuesta 1 falsa).

Las respuestas 3 y 5 se contraponen, por lo que la respuesta correcta al caso clínico debe estar
entre estas opciones. La esteatosis y la hepatitis se consideran lesiones precursoras del daño
hepático irreversible, sin embargo, no todas acaban alcanzando la cirrosis, por su potencial
carácter reversible (respuesta 3 falsa). El diagnóstico del paciente probablemente se trate de una
esteatohepatitis no alcohólica o NASH, diagnóstico de exclusión de causa alcohólica que aparece
generalmente en pacientes diabéticos y obesos. Es por ello que en estos pacientes es importante
disminuir el peso y controlar la diabetes para evitar la evolución a cirrosis por lo que la respuesta 5
es la correcta.(R4)

13. Entre los efectos de la adicción a la cocaína en una mujer gestante, es FALSO que
exista:

1. 1. Mayor riesgo de desprendimiento de placenta.


2. 2. Mayor incidencia de parto post-término.
3. 3. Mayor incidencia de hipertensión materna.
4. 4. Mayor incidencia de retraso del crecimiento intrauterino y bajo peso al nacer.
Gráfico de respuestas
Comentario

 
 
 
 
La cocaína no suele producir síndrome de abstinencia en el recién nacido, a diferencia de los
opiáceos. Sin embargo, pueden aparecer complicaciones obstétricas, como el parto pretérmino, el
desprendimiento prematuro de placenta, etc.

Por otra parte, puede afectar directamente al feto, manifestándose como CIR, microcefalia,
hemorragias intracraneales, anomalías digestivas o renales, muerte súbita, alteraciones
conductuales o neurológicas.

La respuesta correcta, por tanto, es la 2. Lo que produce la cocaína no es parto postérmino, sino
todo lo contrario: pretérmino.(R2)

14. Una mujer de 67 años, con el diagnóstico previo de cirrosis hepática por infección por
el virus de la hepatitis C, acude al Hospital por descompensación hidrópica en forma de
ascitis, como había tenido en otras ocasiones, pero con dolor abdominal difuso, que se
confirma a la palpación, y fiebre. Se decide realizar paracentesis diagnóstica, cuyo
resultado es el siguiente: LDH 875, glucosa 25, albúmina 0,26, células 15.700 con 90% de
PMN. Ante estos resultados, ¿qué respuesta considera correcta?

Se trata de una peritonitis bacteriana espontánea, causa frecuente de descompensación


1. 1. hidrópica y se debe iniciar tratamiento con una cefalosporina de segunda generación
(cefoxitina o cefuroxina) por vía intravenosa.
Se trata de una peritonitis bacteriana espontánea, causa frecuente de descompensación
2. 2.
hidrópica, siendo la causa más probable el enterococo.
Se trata probablemente de una peritonitis bacteriana espontánea, del paciente cirrótico,
3. 3. probablemente producida por E.coli, y que debe tratarse con cefalosporinas de tercera
generación.
Es poco probable que se trate simplemente de una peritonitis bacteriana espontánea,
4. 4.
siendo obligatorio que se descarten causas de peritonitis secundaria.
Gráfico de respuestas
Comentario

En un paciente con ascitis y hepatopatía crónica, se debe sospechar la presencia de peritonitis


bacteriana secundaria (es decir, debida a un foco infeccioso abdominal) si existe cualquiera de los
siguientes datos en el líquido ascítico:

Leucocitos mayores de 10.000 por mm3.

Glucosa menor de 50 mg/dl.

Proteínas mayores de 2,5 g/dl.

LDH mayor de 225.

Esta peritonitis requiere habitualmente tratamiento quirúrgico y se debe valorar la causa con
pruebas de imagen, a diferencia de la bacteriana espontánea, que se trataría con antibióticos.(R4)

15. ¿Cuál es el tipo de vaso sanguíneo que se afecta sobre todo en las vasculitis por
hipersensibilidad?:

1. 1. Arterias de pequeño tamaño.

 
 
 
 
2. 2. Arterias de mediano tamaño.
3. 3. Venas de mediano tamaño.
4. 4. Vénulas postcapilares.
Gráfico de respuestas
Comentario
Las vasculitis por hipersensibilidad, también llamada vasculitis con afectación predominantemente
cutánea o vasculitis cutánea leucocitoclástica se caracterizan por la afectación de pequeños vasos
de la piel, siendo las vénulas poscapilares los vasos más frecuentemente afectados, aunque las
arteriolas y capilares también pueden afectarse con menor frecuencia. Los hallazgos
anatomopatológicos consisten generalemente en una vasculitis con leucocitoclastia (restos de los
núcleos de los polmorfonucleares fragmentados en la pared de los vasos) y necrosis fibrinoide.(R4)

16. Respecto a las benzodiacepinas, señale la opción FALSA:

1. 1. Pueden producir dependencia, problemas psicomotores y amnesia anterógrada.


Son usados para subyugar las crisis de pánico, y alprazolam y clonacepam incluso para
2. 2.
prevenirlas.
3. 3. Son útiles en la intoxicación alcohólica, evitando las convulsiones o "crisis de ron".
Pueden usarse en el insomnio, especialmente triazolam y flunacepam, y en el síndrome de
4. 4.
piernas inquietas.
Gráfico de respuestas
Comentario

El tema de psicofármacos es el más preguntado en Psiquiatría. No constituye una unidad temática


en nuestro manual, sino que aparece separadamente, en la sección del tratamiento de cada
enfermedad.

Los efectos principales de las benzodiacepinas son los hipnóticos (opción 5 correcta),
miorrelajantes y anticonvulsivantes (respuesta 1 cierta). Sin embargo, esto no entra en conflicto
con la respuesta 4. Aunque las benzodiacepinas tienen carácter anticonvulsivante, el tratamiento
preventivo de las “crisis del ron” es el sulfato de magnesio (las benzodiacepinas serían el
tratamiento, no la profilaxis). Recuerda que el sulfato de magnesio también se emplea como
anticonvulsivante en el tratamiento de las convulsiones de la eclampsia.

Respecto a la respuesta 3, es cierto que se emplean como tratamiento de las crisis de pánico. Para
la prevención de estas crisis, es más frecuente emplear fármacos antidepresivos, pero el
clonacepam y el alprazolam también se han empleado con este fin, por lo que no es
incorrecta.(R3)

17. A 5-year-old child with a medical history of gingivitis and periodontal abscesses
comes to the emergency department presenting with cephalea and fever. He complains
of double vision and intense pain of left eye. Physical examination shows intense swelling
and erythema of the left eyelid. Temperature is 39ºC. Which of the following is the most
likely cause?

1. 1. Bacterial sinusitis
2. 2. Temporal abscess
3. 3. Periodontal abscess
4. 4. Anterior uveitis
Gráfico de respuestas
Comentario

 
 
 
 
Bacterial sinusitis. This patient is suffering from orbital cellulitis, which is suggested by the intense
inflammatory signs and symptoms. This is a severe infection with potential complications that
include hearing loss, meningitis or a brain abscess.(R1)

18. Hombre de 65 años con disnea progresiva, y cansancio que acude a la consulta
porque desde hace 3 meses presenta disnea de pequeños esfuerzos y ortopnea. A la
exploración se detecta un soplo pansistólico en foco mitral y por ecocardiografía se
comprueba la existencia de una insuficiencia mitral degenerativa con prolapso del velo
posterior por rotura de cuerdas tendinosas. La fracción de eyección ventricular izquierda
era 40% y el estudio hemodinámico demostró que las arterias coronarias no presentaban
lesiones significativas. Indique el tratamiento electivo en este caso clínico:

Tratamiento médico hasta que se detecte que la fracción de eyección ventricular izquierda
1. 1.
sea menos de 30%.
Reparación de la válvula mitral mediante resección del segmento del velo posterior
2. 2.
afectado por la rotura de las cuerdas y anuloplastia mitral.
3. 3. Reparación de las cuerdas rotas.
4. 4. Sustitución de la válvula mitral por bioprótesis.
Gráfico de respuestas
Comentario

Pregunta de dificultad media del tema de valvulopatías.

La clínica del paciente orienta a una ICC en mala clase funcional, y la exploración con soplo
pansistólico en foco mitral sugiere IM, como se comprueba con el ECO. En atención a ello, nos
encontramos ante un paciente con IM severa sintomática por ICC y que, igualmente, manifiesta
deterioro de la fracción de eyección del VI. Así pues, tiene indicación de cirugía valvular, y siempre
que sea posible, el tratamiento de elección es la reparación valvular mitral (enunciado 2), y sólo si
esto no fuera posible, se realizará sustitución valvular por válvula protésica. En el esquema de la
página anterior, te presentamos el algoritmo terapéutico de la insuficiencia mitral.(R2)

Algoritmo de manejo de la insuficiencia mitral

 
 
 
 
19. ¿Cuál es el tipo de glomerulonefritis que evoluciona MENOS frecuentemente hacia
glomerulonefritis crónica?:

1. 1. GN IgA.
2. 2. GN membranosa.
3. 3. GN por cambios mínimos.
4. 4. GN segmentaria y focal.
Gráfico de respuestas
Comentario

Sencilla pregunta acerca de la evolución de las GN que se puede contestar teniendo simplemente
una fotografía de cada GN. En este caso, la GN que menos frecuentemente evoluciona a una
forma crónica es la opción 3, la GN por cambios mínimos. De ella debes saber que es una GN de
buen pronóstico, y que aparece típicamente en niños de 2 a 6 años (siendo la causa más frecuente
de síndrome nefrótico en la infancia). No está indicada la biopsia en ausencia de complicaciones;
su manejo es iniciar tratamiento con esteroides, gracias a los cuales la mayoría de los niños se
recuperan completamente, incluso existen un 30- 40% de remisiones espontáneas.(R3)

20. El Streptococcus viridans es el agente productor más frecuente de:

1. 1. Reumatismo poliarticular agudo.


2. 2. Glomerulonefritis difusa.
3. 3. Cuadros erisipelosos.
4. 4. Endocarditis bacteriana subaguda.
Gráfico de respuestas
Comentario

El agente productor más frecuente de las endocarditis subagudas -opción 4- es el estreptococo


(que ocasiona el 55% de todos los casos en válvulas naturales), y de ellas, el estreptococo viridans
produce el 75% - opción 4- . Debe recordar que la erisipela - opción 3- está producida por el
estreptococo pyogenes, que también puede causar, de forma reactiva, cuadros de glomerulonefritis
posestreptocócica - opción 2- . Un estreptococo causante frecuente de meningitis bacteriana es el
Strep. Pneumoniae (causa más frecuente de meningitis en adultos, de meningitis con fístula de
LCR y de meningitis recurrente). Aprovecho para comentarte otro matiz importante de la respuesta
4: se refiere a cuadro subagudo; si le dijera cuadro fulminante o hiperagudo sería falsa porque la
causa más frecuente sería S. aureus.(R4)

21. ¿Cómo se denomina el tumor androgénico que deriva de la médula ovárica?:

1. 1. Androblastoma.
2. 2. Gonadoblastoma.
3. 3. Meduloblastoma.
4. 4. Tumor de la teca-fibroma.
Gráfico de respuestas
Comentario

Esta pregunta sobre tumores ováricos no tiene mucha relevancia.

Los tumores de cordones - estroma normalmente producen hormonas que ayudan a


diagnosticarlos. El 50% son androblastomas o tumores de celulas de Sertoly-Leydig. Recuerde que
es la primera causa de virilización de origen ovárico. Los tumores de la teca-fibroma producen

 
 
 
 
estrógenos y andrógenos, los de la granulosa estrógenos. Los gonadoblastomas son muy poco
frecuentes y aparecen casi exclusivamente en individuos con disgenesia gonadal. Contienen zonas
semejantes al disgerminoma y elementos del estroma gonadal y derivan de la célula germinal. El
meduloblastoma es tumor cerebral maligno más frecuente en la infancia.(R1)

22. ¿Cuál es la cardiopatía congénita más frecuente en la infancia?

1. 1. Tetralogía de Fallot.
2. 2. Persistencia del conducto arterioso.
3. 3. Coartación de aorta.
4. 4. Comunicación interventricular.
Gráfico de respuestas
Comentario

Pregunta muy importante en el ENARM. La cardiopatía congenita más frecuente es la CIV siempre
que se excluya la válvula bicúspide.

La más frecuente de las cianóticas es la D-transposición de los grandes vasos, pero a partir del
año de vida, la cianótica más comun es la tetralogia de Fallot.(R4)

23. En la utilización de los corticoides en la ruptura prematura de membranas antes de


las 34 semanas de gestación, las siguientes afirmaciones son ciertas menos una. Indique
cuál:

1. 1. Aumenta la mortalidad perinatal por infección.


2. 2. Disminuye la aparición de distrés respiratorio del RN.
3. 3. Disminuye el riesgo de enterocolitis necrotizante.
4. 4. Acelera la maduración pulmonar.
Gráfico de respuestas
Comentario

Respuesta incorrecta la número 1, ya que el uso de corticoides no incrementa la mortalidad


perinatal secundaria a infección. Entre los beneficios del uso de corticoides: acelera la maduración
pulmonar, disminuye el riesgo de enterocolitis necrotizante y el riesgo de hemorragia de la matriz
germinal.(R1)

24. A male patient with a history of long-standing arterial hypertension for the past 20
years presents with symptoms of heart failure. Echocardiogram shows concentric left
ventricular hypertrophy. Which of the following would be the least appropriate treatment
for this patient?

1. 1. Verapamil.
2. 2. Atenolol.
3. 3. Enalapril.
4. 4. Digoxin.
Gráfico de respuestas
Comentario

En este paciente, debemos asumir una insuficiencia cardíaca diastólica (deterioro de la relajación
ventricular), como consecuencia de la hipertrofia debida a su cardiopatía hipertensiva. En otras
palabras, la rigidez del

 
 
 
 
miocardio dificulta su llenado. Por este motivo, serán beneficiosos los fármacos que disminuyan la
frecuencia cardíaca (verapamil, diltiazem, atenolol), que además son inotropos negativos, con lo
que favorecen esta relajación. Por otra parte, merece la pena utilizar un IECA, que ha demostrado
prolongar la supervivencia en la insuficiencia cardíaca.

En este caso, la digoxina no sería útil. No ha demostrado prolongar la supervivencia en la


insuficiencia cardíaca y, para este paciente en concreto, su carácter inotrópico positivo no nos
aporta nada. Recuerde que está más indicada en la insuficiencia cardíaca sistólica que en la
diastólica.(R4)

25. Lactante de un mes que ha sufrido de conjuntivitis permanente desde el nacimiento.


Hace 10 días presenta tos con taquipnea progresiva. No se encuentra en estado crítico y
se ausculta sibilantes en ambos campos pulmonares. Radiografía de tórax: infiltrado
bilateral. ¿Cuàl es el agente causal más probable?

1. 1. Hemophilus influenzae.
2. 2. Streptococo pneumoniae.
3. 3. Chlamydia trachomatis.
4. 4. Mycoplasma pneumoniae.
Gráfico de respuestas
Comentario

Es una lactante de un mes de edad que presenta conjuntivitis de larga evolución, asociado a
neumonía bilateral, por lo que debe pensar inmediatamente en infección por Chlamydia
trachomatis.

La infección por H. influenza y S. pneumoniae no sería tan larvada con lo que podría descartarlo.

La infección por M pneumoniae es típico de jóvenes, con lo que podrías descartar esta opción con
cierta seguridad.(R3)

26. Paciente primigesta Rh negativa no sensibilizada con esposo Rh positivo, recién


nacido Rh positivo. La conducta a seguir es:

1. 1. Vacunarla a la semana del parto.


2. 2. No requiere vacunación hasta el próximo parto.
3. 3. Vacunarla dentro de las 72 horas posparto.
4. 4. Vacunarlas en cualquier momento.
Gráfico de respuestas
Comentario

La respuesta correcta es la 3. Revise el siguiente cuadro.(R3)

 
 
 
 

27. Agente etiológico más frecuentemente causante de bronquiolitis y neumonía en niños


menores de 1 año.

1. 1. Virus parainfluenzae.
2. 2. Haemophilus influenzae.
3. 3. Virus Sincitial Respiratorio.
4. 4. Virus influenzae.
Gráfico de respuestas
Comentario

El agente más frecuentemente implicado en la bronquiolitis aguda es el virus respiratorio sincitial.


Respuesta 3 correcta.(R3)

28. La causa más frecuente de esterilidad femenina es:

1. 1. Ovárica.
2. 2. Tubárica.
3. 3. Uterina.
4. 4. Cervical.
Gráfico de respuestas
Comentario

En un 40% de los casos, el factor implicado en la esterilidad es el masculino; en otro 40%, es el


factor femenino (la mayoría de los casos por alteración tubárica) y en un 20%, es un factor mixto o
de origen desconocido.(R2)

29. Con relación al examen físico marque verdadero o falso en los siguientes enunciados
y señale la respuesta correcta ( ) Reconocer patologías que pueden requerir
intervenciones inmediatas ante la posibilidad de lesión. ( ) Reconocer posibles factores
que pueden explicar condiciones patológicas en el recién nacido. ( ) Nos permite entender
el estado en que se encuentra el recién nacido y realizar un pronostico de cómo se
encontrara a corto plazo. ( ) No nos ayuda a tomar decisiones terapéuticas urgentes
adecuadas. ( ) El primer examen físico se debe realizar a las 8 horas de vida extrauterina.

 
 
 
 
1. 1. VVVFF.
2. 2. VFVFV.
3. 3. VVFFV.
4. 4. FVFVF.
Gráfico de respuestas
Comentario

Es una pregunta que usando la lógica se puede responder. El primer examen físico se debe
realizar a las 3 horas desde el nacimiento.(R1)

30. Niño de 10 años con episodios breves de distracciones (< 1 minuto) en los que no
responde a llamadas y parpadea. Un EEG muestra descargas de punta-onda a 3 ciclos
por segundo. El tratamiento electivo de primera línea lo haría con:

1. 1. Valproato.
2. 2. Carbamacepina.
3. 3. Gabapentina.
4. 4. Clonazepam.
Gráfico de respuestas
Comentario
Nos describen a un niño de 10 años con cuadros reiterados de desconexión del medio de poco
tiempo de duración (menor a un minuto), durante los cuales presenta automatismos sin responder
a estímulos. En el EEG un patrón caracteristico generalizado de Punta-Onda a 3 Hz. Con los datos
descritos, tanto edad, como patrón EEG debermeos pensar en un tipo de epilepsia generalizada
característico de niños, la epilepsia de ausencias. Clásicamente se trataba con Etoxusimida, pero
actualmente se prefiere Ácido Valproico por presentar el mismo efecto entiepiléptico y menos
efectos secundarios.(R1)

31. En relación con las esofagitis víricas, indique la respuesta FALSA:

Las inclusiones A de Cowdry (intranucleares y eosinófilas) en la citología indica


1. 1.
infección por virus herpes.
El VIH puede provocar una esofagitis en la seroconversión asociada a úlceras bucales y
2. 2.
eritema máculopapuloso cutáneo.
3. 3. El virus herpes simple tipo II no produce esofagitis.
El CMV produce úlceras serpiginosas que luego confluyen formando úlceras gigantes en
4. 4.
tercio inferior del esófago sólo en enfermos inmunodeprimidos.
Gráfico de respuestas
Comentario

Las esofagitis por virus del grupo herpes tienen tendencia a provocar ulceraciones.
Histológicamente, suele demostrarse inclusiones intranucleares y eosinófilas, conocidas como
inclusiones o cuerpos A de Cowdry. El CMV produce en pacientes VIH y, en general, en
inmunodeprimidos, grandes úlceras en el tercio inferior del esófago. El aciclovir es el tratamiento de
elección para las del grupo herpético, salvo para el CMV, que se trata con ganciclovir. El virus
herpes simple tipo II es causa de uretritis, vaginitis y balanitis. Se adquiere por transmisión sexual y
se han identificado lesiones en mucosa oral, e incluso esofagitis, si bien es un hallazgo muy
infrecuente.(R3)

32. Embarazada en el primer trimestre de gestación, con antecedentes de hijos fallecidos


por una inmunodeficiencia y cuyo defecto molecular en la familia ya es conocido. ¿Qué

 
 
 
 
prueba diagnóstica es la más idónea para el diagnóstico prenatal de la
inmunodeficiencia?

1. 1. Estudio molecular en los padres.


2. 2. Biopsia de corion en la 9.ª semana.
3. 3. Determinación de alfa-fetoproteína en la madre.
4. 4. Estudio de subpoblaciones de linfocitos en la madre.
Gráfico de respuestas
Comentario

Concepto poco preguntado en el ENARM. La biopsia corial se basa en la obtención de


vellosidades coriales por el cuello uterino o vía transabdominal. Obtiene directamente tejidos
fetales placentarios y los resultados genéticos pueden obtenerse en 48-72 horas. Presenta mayor
número de pérdidas fetales. Es el método que permite el diagnóstico más precoz de
cromosomopatías como la trisomía del 21 y, consecuentemente, de elección si queremos un
diagnóstico citogenético prenatal antes de las 12 semanas de gestación. Se realiza a partir de las
ocho semanas de gestación.(R2)

33. En una enferma de 40 años, con insuficiencia renal crónica secundaria a nefrectomía
bilateral y antecedentes personales de gastrectomía por úlcera péptica, apendicectomía
e histerectomía, ¿cuál sería el tratamiento sustitutivo más adecuado?

No es candidata a tratamiento sustitutivo por tener un pronóstico vital ominoso a corto


1. 1.
plazo.
2. 2. Hemodiálisis periódica.
3. 3. Diálisis peritoneal continua ambulatoria.
4. 4. Trasplante renal urgente.
Gráfico de respuestas
Comentario

Una pregunta de dificultad media sobre el tratamiento de la insuficiencia renal crónica.

Dado que se trata de una paciente joven, habría que ofrecerle un tratamiento sustitutivo para su
función renal, que será la hemodiálisis periódica, el trasplante renal o la diálisis peritoneal. En este
caso, por supuesto podría realizarse un trasplante renal, pero no es necesario realizarlo de forma
urgente (respuesta 4 falsa). Por otra parte, en esta paciente no podemos plantearnos la diálisis
peritoneal, puesto que tiene antecedentes de cuatro intervenciones quirúrgicas abdominales, por lo
que es de suponer que la capacidad de intercambio de su peritoneo no será óptima. Teniendo en
cuenta todo esto, la única opción posible es la respuesta 2.(R2)

34. Neonato pretérmino que presenta al nacer un episodio de intensa dificultad


respiratoria. Tiene tiraje subcostal e intercostal, disociación toraco-abdominal y cianosis
generalizada que mejora cuando el niño llora. Además tiene rinorrea mucosa bilateral,
que se aspira, pero la sonda no pasa más allá de la cavidad nasal. ¿Qué tratamiento sería
el más adecuado para este paciente?

1. 1. Quirúrgico, diferido a que el niño pese más de 5 kg.


2. 2. Surfactante pulmonar.
3. 3. Tetina de McGovern.
4. 4. Cirugía urgente.
Gráfico de respuestas

 
 
 
 
Comentario

Esperamos que, el día del ENARM, no tenga ningún problema si aparece una atresia de coanas.
Lógicamente, al ser la atresia de coanas un problema estructural, el tratamiento de la misma es
quirúrgico. Respuesta correcta 4.(R4)

35. A 3 months old female infant is brought to the clinic for a routine visit. The girl shows
a marked growth retardation and a bluish colour of skin and mucosas. The mother says
that she often becomes very fussy and do not respond to stimulation, and seems hardly
breathing. She gets easily tired. Cardiac auscultation reveals a harsh pansystolic murmur
at the left sternal border with a single S2. What is the most likely diagnosis?

1. 1. Isolated ventricular septal defect


2. 2. Ostium primun
3. 3. Tricuspid atresia
4. 4. Tetralogy of Fallot
Gráfico de respuestas
Comentario
Tetralogy of Fallot. Fallot disease is a cyanotic congenital heart disease and accounts for 7-10% of
congenital heart defects. The classic tetralogy includes the pulmonary stenosis, ventricular septal
defect, right ventricular hypertrophy and an overriding aorta. Physical examination shows a high
intensity pulmonic murmur, a bluish skin color and digital clubing. Failure to thrive is common.
Children with tetralogy of Fallot may develop "tet spells”, as in this clinical case, which are
characterized by sudden, marked increase in cyanosis followed by syncope, and may result in
hypoxic brain injury and death.(R4)

36. Una de las siguientes asociaciones referentes al pulso arterial NO es cierta:

1. 1. Pulso pequeño o parvus: shock.


2. 2. Pulso dícroto: estenosis aórtica.
3. 3. Pulso grande o magnus: insuficiencia aórtica.
4. 4. Pulso grande o magnus: tetralogía de Fallot.
Gráfico de respuestas
Comentario
Pregunta de moderada dificultad de un tema muy preguntado en el MIR como es la semiología
cardiaca, en este caso el pulso arterial. El pulso hipercinético se asocia a un aumento del volumen
de eyección del VI o a la disminución de las resistencias periféricas, como por ejemplo en la
insuficiencia aórtica, en la comunicación interventricular y en la circulación hiperdinámica (anemia,
fiebre, ansiedad, fístulas arteriovenosas, etc...). En el pulso dícroto se palpan dos picos en el pulso,
uno sistólico y otro diastólico. Es característico de situaciones en las que hay una disminución del
gasto cardíaco como en la insuficiencia cardíaca o en el taponamiento cardíaco. El pulso tardus o
parvus se produce en situaciones de disminución del volumen/latido, como es el shock y en
situaciones de obstrucción a la eyección ventricular, como es la estenosis aórtica o la HTA.(R2)

37. Recién nacida de 36 horas de vida trasladada a la unidad de neonatología de su


hospital para estudio y tratamiento de ictericia progresiva y masa abdominal izquierda.
El embarazo ha sido controlado y normal. El parto ha tenido lugar a las 42 semanas de
gestación, tras un tiempo de rotura de membranas de 9 horas y sin fiebre materna
intraparto. Fue eutócico, en presentación cefálica, prolongado y traumático, precisando

 
 
 
 
instrumentación. Apgar 9/10. PRN: 4,180 g. Talla: 51 cm. PC: 37 cm. A su llegada se realiza
una exploración neonatal completa en la que se detecta coloración ictérica de piel y
mucosas, masa abdominal a nivel de hipocondrio izquierdo y crepitación de clavícula
izquierda. Se realizan exámenes de laboratorio donde se observa una hemoglobina de 12
g/dl, bilirrubina total 14.3 mg/dl, bilirrubina indirecta 13.9 mg/dl. En la bioquímica
destacan un sodio de 131 mEq/l, un potasio de 5.9 mEq/l y una glucemia 55 mg/dl.
Respecto a la patología que sospecha en este neonato, señale la opción INCORRECTA

1. 1. El diagnóstico se confirma mediante un ultrasonido abdominal.


2. 2. Es más frecuente en fetos macrosómicos nacidos de nalgas.
3. 3. El tratamiento es sintomático y de sostén.
4. 4. Se debe a una trombosis de la vena renal.
Gráfico de respuestas
Comentario

Tema muy rentable. El caso que se describe es el típico de una hemorragia suprarrenal. El
sangrado de esta víscera ocurre en fetos macrosómicos que nacen de nalgas (aunque también
puede ocurrir en partos en cefálica como es el caso) y han sufrido un expulsivo traumático (nuestro
paciente requirió instrumentación y tiene una fractura de clavícula). La clínica se debe por un lado
al sangrado, que da lugar a la anemización y la ictericia (el neonato no logra aclarar toda la
hemoglobina y por ello se elevan la bilirrubina total y la indirecta); y por otro lado a la lesión de la
glándula que da lugar a un déficit de cortisol (hormona contrarreguladora que eleva la glucemia,
por lo que el paciente se encuentra hipoglucémico) y aldosterona (que en condiciones normales
retiene sodio en el túbulo renal a expensas de perder potasio y protones, por lo que si esta
hormona es deficitaria se producirá lo contrario: aumentarán potasio y protones (hiperpotasemia y
acidosis) y disminuirá el sodio (hiponatremia)). El diagnóstico se confirma con ultrasonido y no
precisa cirugía de entrada, siendo suficiente un tratamiento de sostén. La afirmación claramente
falsa es la 4, ya que el cuadro se debe a la lesión directa y sangrado posterior de la suprarrenal, no
a la trombosis de la vena renal.(R4)

38. Una mujer de 54 años acude a la consulta de urología para valoración de


incontinencia. Refiere haber sido derivada por su neurólogo, que la sigue por esclerosis
múltiple desde hace 10 años. Al interrogatorio refiere incontinencia leve, de varias gotas,
que sucede tras la aparición de necesidad imperiosa de orinar. En esta paciente no es
cierto que:

La disfunción urinaria es común en la EM, aunque no todos los pacientes con EM la


1. 1.
padecen.
2. 2. Suele estabilizarse en periodos estables de la enfermedad.
Dado que tras su aparición no varía en el tiempo, será suficiente con una batería de
3. 3.
pruebas inicial que nunca habrá que repetir.
Las complicaciones graves suelen en general poderse prevenir con una buena atención y
4. 4.
exploraciones en el seguimiento.
Gráfico de respuestas
Comentario
En los pacientes con esclerosis múltiple el funcionamiento de la vejiga puede variar con el tiempo,
por lo que puede ser necesario repetir exploraciones, diagnósticos y variar estrategias.(R3)

39. En un paciente con traumatismo abdominal cerrado, sin pérdida de conciencia, lo más
indicado para determinar la ruptura de una víscera solida es:

 
 
 
 
1. 1. Lavado peritoneal diagnóstico.
2. 2. Control de hemoglobina.
3. 3. Radiografía simple de abdomen.
4. 4. Laparotomía exploradora.
Gráfico de respuestas
Comentario

Dentro del manejo del paciente con traumatismo abdominal cerrado las pruebas diagnósticas de
elección son: TAC abdominal si el paciente se encuentra estable; ECO-FAST o LPD (punción
lavado peritoneal) si el paciente está inestable.

Entre las opciones no se encuentran ni la ECO ni el TAC pero sí el lavado peritoneal diagnóstico
(respuesta 1) que será por tanto la opción correcta. Además mediante esta técnica podría
determinar la presencia o no de líquido libre en fosa ilíaca que indica la posible ruptura de una
víscera con el consiguiente sangrado.(R1)

40. A 47-year-old female, comes to your office complaining of pain on her left ear for the
past seven days. She refers hearing loss and tinnitus in the same ear for a few months.
She denies otorrhea. There are no other findings on examination except for the presence
of a pulsatile mass in her left neck. Otoscopy is shown in the image. What would be the
most appropiate description of the diagnosis?

1. 1. Erytematous mass in the hipotimpanic area. Otosclerosis.


2. 2. Erythematous and bulging tympanic membrane. Acute Otitis Media.
Retrotympanic occupation with seromucous material and myringosclerosis. Chronic
3. 3.
media otitis
4. 4. Erythematous mass occupying anterior portion of tympanic box. Chemodectoma.
Gráfico de respuestas
Comentario

El paraganglioma o quemodectoma es un concepto poco preguntado en el nacional, pero debe


saber reconocer el caso clínico, como lo describen es más frecuentes en mujeres, son benignos,
estan muy vascularizados por ramas de la carótida externa y los síntomas típicos serán acufenos
pulsátiles, sincrónicos con el pulso.(R4)

41. La trombosis venosa del seno lateral cerebral suele ser una complicación de:

1. 1. Otitis media o mastoiditis.


2. 2. Meningitis.
3. 3. Hemorragia subaracnoidea.
4. 4. Infarto isquémico.
Gráfico de respuestas
Comentario

El seno lateral está en contacto con la pared posterior de la mastoides. Aunque hoy en día es
excepcional, puede originarse desde una infección del oído medio, sobre todo de la mastoides.
Inicialmente, se produce una flebitis por la infección de su pared y posteriormente el seno se
trombosa. Este trombo puede crecer hacia el endocráneo y el cuello a través de la vena yugular
interna. El tratamiento es médico y en algún caso quirúrgico.(R1)

 
 
 
 
Un niño de 12 años acude por presentar estornudos, rinorrea y prurito nasal. La
exploración física revela intenso edema nasal, mucosa pálida y con secreción clara. El
diagnóstico más probable es:

1. 1. Rinitis vasomotora.
2. 2. Rinitis alérgica.
3. 3. Mastocitosis nasal.
4. 4. Cuerpo extraño.
Gráfico de respuestas
Comentario

Rinitis alérgica: muy predominante (25% población). Dependiendo del tipo de neumoalergeno, la
rinitis podrá ser estacional o perenne. La clínica típica consiste en estornudos en salvas,
obstrucción nasal, rinorrea acuosa y prurito nasal y/o ocular.

La mucosa que recubre los cornetes muestra una palidez característica. Dado que la etiología es
alérgica, habrá un incremento de eosinófilos en sangre y exudado nasal, así como de IgE en
sangre tanto total como específica (RAST). Las pruebas cutáneas alérgicas (Prick test) confirman
el diagnostico.

Su tratamiento es el de cualquier proceso alérgico: evitar la exposición a alérgenos, usar


descongestionantes, antihistamínicos, corticoides intranasales.(R2)

42. ¿Cuál de los siguientes tratamientos ha demostrado ser de mayor utilidad en la


terapéutica de la esclerosis múltiple?

1. 1. Prednisona.
2. 2. Piridostigmina.
3. 3. Interferon alfa.
4. 4. Cirugía estereotáxica.
Gráfico de respuestas
Comentario

La esclerosis múltiple es un tema que debes conocer bien. Es importante que tengas en cuenta las
siguientes ideas:

Las formas recurrente- remitentes son las que se benefician del tratamiento de fondo con
interferón, no las crónicas progresivas. No confundas el interferón beta, que es el que se emplea
en esta enfermedad, con el interferón alfa (usado frente a ciertos tumores sólidos, como el
melanoma o el hipernefroma, y también empleado hasta hace poco para la leucemia mieloide
crónica).

El tratamiento de los brotes se realiza con corticoides a altas dosis, casi siempre por vía sistémica,
con la excepción de las formas leves (síntomas únicamente sensitivos, aquí sería posible por vía
oral). De las opciones que nos mencionan, debemos elegir la prednisona, por su utilidad para los
brotes agudos.(R1)

43. A 30-year-old patient with a history of gastric hypersecretion comes to the emergency
department with perforated jejunal ulcers. The physician obtains an ultrasound that
shows a mass measuring 4 cm in diameter located in the pancreas. Which of the following

 
 
 
 
is the most likely diagnosis?

1. 1. Insulinoma.
2. 2. Vipoma.
3. 3. Somatostatinoma.
4. 4. Gastrinoma.
Gráfico de respuestas
Comentario

Ante un paciente con úlcera en una localización poco habitual (en este caso, yeyuno), junto con
hipersecreción ácida, la primera sospecha diagnóstica es el Gastrinoma (síndrome de
Zollinger- Ellison). La localización más frecuente de este tumor es la pared duodenal, seguido del
páncreas. Además, es frecuente la extensión a distancia del tumor, ya que tiene un alto índice de
diseminación, lo que implica mal pronóstico.(R4)

44. El pulso arterial paradójico es típico, aunque no patognomónico, de la siguiente


entidad:

1. 1. Insuficiencia cardiaca congestiva.


2. 2. Taponamiento pericárdico.
3. 3. Insuficiencia aórtica.
4. 4. Insuficiencia tricúspide.
Gráfico de respuestas
Comentario

El pulso paradójico consiste en la disminución en más de 10 mmHg de la presión sistólica con la


inspiración profunda. Es la exageración de un fenómeno fisiológico por el abombamiento del septo
interventricular hacia el VI al aumentar el llenado del VD, cuya pared lateral no puede distenderse.
El ejemplo más típico es el taponamiento cardíaco, pero también puede aparecer en otras
patologías como obstrucciones bronquiales graves, tromboembolismo pulmonar, obstrucción de la
cava superior, pericarditis constrictiva, etc...(R2)

45. Hombre de 62 años de edad, diagnosticado desde hace 2 años de hemorroides,


tratado con pomadas y modificaciones alimentarias. Desde hace 2 meses se le prolapsan
y tiene que reducirlas manualmente. ¿Qué grado de hemorroides presenta ahora?

1. 1. Primer grado.
2. 2. Segundo grado.
3. 3. Tercer grado.
4. 4. Quinto grado.
Gráfico de respuestas
Comentario

Pregunta poco significativa y bastante difícil, puesto que los grados de las hemorroides nunca
habían sido preguntados en el ENARM.Éstos son:

•   Grado I: permanecen en recto, sin sobrepasar el ano.


•   Grado II: prolapso a través del ano cuando el paciente puja, pero se reducen
espontáneamente.
•   Grado III: prolapso por el ano cuando el paciente puja. La reducción requiere ayuda
manual.

 
 
 
 
•   Grado IV: prolapso persistente.

(R3)

46. A 9-year-old boy visits pediatrician’s consultation for a routine health check-up. His
mother is worried about sunlight damage of his skin and eyes. The boy has a low
phototype (2), blonde hair, blue eyes and some junctional acquired nevi. Which of the
following is the best advice in this case?

1. 1. To avoid sun exposure between 12 am and 5 pm.


2. 2. Employment of total sunscreen protection.
3. 3. Vit A supplements
4. 4. Dermatoscopic examination at least once a year.
Gráfico de respuestas
(R1)

47. Síntomas o signo más frecuente en el adenocarcinoma de endometrio.

1. 1. Aumento de tamaño del útero.


2. 2. Sangrado post menopáusico.
3. 3. Flujo vaginal.
4. 4. Ninguna de las anteriores.
Gráfico de respuestas
Comentario

En mujeres postmenopáusicas en las que se sospeche de adenocarcinoma de endometrio el signo


clínico más frecuente es el sangrado transvaginal post-menopáusico, respuesta 2 correcta.(R2)

48. Un hombre de 70 años es diagnosticado de arteritis temporal de células gigantes. Se


debería iniciar el tratamiento con:

1. 1. Prednisona 20 mg cada día.


2. 2. Prednisona 40-60 mg cada día.
3. 3. Prednisona 40-60 mg cada día, calcio y vitamina D y bisfosfonatos.
4. 4. Ciclofosfamida oral.
Gráfico de respuestas
Comentario

El tratamiento de la arteritis de la temporal es, en realidad, la prednisona, normalmente en dosis de


0,5 a 1 mg/kg/día. Sin embargo, es fundamental hacer una valoración completa de todas las
opciones que nos presentan.

No siempre nos encontraremos una respuesta correcta y las otras completamente falsas. En
muchas ocasiones debemos elegir la más conveniente de las respuestas, aunque haya algunas
que no sean estrictamente falsas.

En esta pregunta, la mejor respuesta sería la 3. Por supuesto, la 2 es también correcta… Pero no
hace nada por evitar las complicaciones a largo plazo de los corticoides sistémicos en altas dosis.
Teniendo en cuenta que se trata de un paciente anciano, encontramos una razón añadida para
pensar en la prevención de una posible osteoporosis. La respuesta correcta (o mejor dicho, más
correcta), es la 3.(R3)

 
 
 
 
49. De los siguientes parámetros funcionales respiratorios, ¿cuál estará más disminuido
en la enfermedad pulmonar obstructiva crónica tipo enfisema que en la tipo bronquitis?:

1. 1. VEMS.
2. 2. Capacidad de difusión de monóxido de carbono.
3. 3. Capacidad pulmonar total.
4. 4. Flujo espiratorio máximo.
Gráfico de respuestas
Comentario
Pregunta típica del MIR y que no debes dudar, pues ya ha sido muchas veces motivo directo de
cuestión MIR y otras ha sido un dato clave para diferenciar entre los dos tipos de EPOC. Ambos
tipos comparten la característica fundamental de ser enfermedades obstructivas por lo que habrá
cambios en las pruebas funcionales que compartirán: Tiffeaneau bajo, aumento del VR,
disminución del VEMS, leve aumento de la CPT, disminución del flujo espiratorio máximo,... . Sin
embargo debes tener claro, que en general tanto las alteraciones funcionales como las
gasométricas son mucho más acusadas en el paciente enfisematoso que en la bronquitis crónica.
Como excepción (recuerda que es clave para el MIR) está la DLCO que debido a la destrucción del
parénquima pulmonar que tiene lugar en el enfisema se encuentra muy disminuido aquí, mientras
que en la bronquitis crónica se altera muy poco(R2)

50. Teniendo en cuenta que el crecimiento intrauterino retardado de tipo II (CIR tipo II) es
el más frecuente de los retardos del crecimiento (aproximadamente 70% del total) y tiene
como principal responsable del cuadro la insuficiencia útero-placentaria, que no suele
afectar al crecimiento fetal hasta el tercer trimestre de la gestación, ocasionando una
redistribución del gasto cardíaco fetal hacia los órganos vitales ¿cuál es el principal
parámetro en el estudio del crecimiento intrauterino retardado (CIR) de tipo II?:

1. 1. Altura de fondo uterino.


2. 2. Diámetro biparietal.
3. 3. Diámetro abdominal.
4. 4. Relación perímetro cefálico/perímetro abdominal.
Gráfico de respuestas
Comentario
El retraso de crecimiento que aparece en las últimas semanas del embarazo se denomina CIR
asimétrico o tipo II. Este se caracteriza por ADELGAZAMIENTO o estacanmiento de los diámetros
abdominales. Por tanto, en el caso del CIR II el parámetro ecográfico más fiable es la relación entre
el perímetro cefálico y el perímetro abdominal, dado que puedes valorar la desproporción que sufre
el feto (que no valoras si solo usas el diámetro abdominal).(R4)

 
 
 
 

51. Lactante de 1 mes de vida que


es llevada a consulta por presentar erupción cutánea localizada en el área del pañal
(imagen nº ##). Nacida a término tras embarazo normal y parto eutócico sin otros
antecedentes de interés. Ante el cuadro que sospecha, indique la respuesta FALSA:

La lesión típica observada se presenta como un eritema papular y confluente con pápulas
1. 1.
rojas satélites en el área del pañal.
2. 2. Se observa a menudo como complicación del tratamiento antibiótico oral.
3. 3. Puede infectar de forma secundaria cualquier otra dermatitis del pañal no infecciosa.
4. 4. El tratamiento de primera elección consiste en glucocorticoides orales.
Gráfico de respuestas
Comentario

Se trata de una sobreinfección por Cándida albicans que se caracteriza clínicamente como un
eritema intenso de color rojo violáceo con formación de pápulas y pápuló-pústulas de extensión
periférica (cuadro clínico encuadrado dentro del espectro clínico de la dermatitis del panal).

En muchas ocasiones se benefician de la aplicación de un antifúngico tópico y, ocasionalmente, de


un corticoide tópico.

Los factores de riesgo generales para la aparición de candidiasis mucocutánea son:


antibioticoterapia, inmunosupresión, diabetes, corticoterapia...(R4)

52. En el paciente de la pregunta anterior, ¿cuál sería la opción terapéutica más


recomendable?:

1. 1. Nistatina tópica 10 días y cuidados básicos del área perianal.


Dado que no se observan complicaciones, esperaría la resolución espontánea de las
2. 2.
lesiones.
3. 3. Indicaría antifúngicos por vía sistémica.
4. 4. Ninguna medicación es útil en la resolución completa.
Gráfico de respuestas
Comentario

El abordaje de la dermatitis irritativa del pañal debe orientarse tanto a la prevención como al
tratamiento sintomático de las lesiones establecidas.

Se aconseja realizar una buena higiene de la zona, que debe permanecer siempre seca y limpia.
La limpieza se realizará con agua tibia o con un jabón ácido o neutro.

 
 
 
 
Asimismo se deben efectuar cambios frecuentes de los pañales, e incluso en niños con episodios
repetidos, puede ser útil mantenerlos unas horas al día sin pañales.

Parece claro que el factor crucial en la prevención de la dermatitis irritativa del pañal es el número
de cambios de pañal al día.

La maceración puede evitarse con polvos de talco u óxido de zinc, aunque estos se encuentran
desaconsejados en erosiones de la piel, ya que podrían generar granulomas por cuerpo extraño.

No se utilizarán de modo profiláctico corticoides, antifúngicos ni antibióticos tópicos, por el riesgo


de sensibilización.(R1)

53. Señale la opción FALSA en relación al tratamiento de la malnutrición energético-


proteica:

Hay que tener cuidado con el síndrome de realimentación, en el que se puede producir
1. 1. hipofosfatemia severa, junto con otras alteraciones electrolíticas, cuando se reintroduce la
alimentación, tanto oral como parenteral, en niños gravemente afectados.
En las malnutriciones secundarias se aconseja dar suplementos de zinc ya que contribuye
2. 2.
al aumento de peso y a la mejoría de la inmunidad.
En las formas agudas de malnutrición los objetivos primeros del tratamiento son vencer la
3. 3.
hipotermia, el colapso y las hipoglucemias, que pueden ser muy severas.
Es fundamental evitar la leche en las formas graves de malnutrición calórico-proteica
4. 4.
porque puede perpetuar la diarrea por malabsorción.
Gráfico de respuestas
Comentario

Esta pregunta no tiene especial importancia. Cuando se reintroduce la alimentación debe iniciarse
despacio aportando el 50-70% de las calorías que necesitaría e ir aumentando progresivamente.

Se debe suplementar cualquier déficit de oligoelementos y vitaminas antes de iniciarla ya que el


síndrome de realimentación es una complicación muy grave y de mal pronostico en el que, entre
otras alteraciones hidroelectroliticas hay hiperglucemia, hiponatremia, hipokalemia, hipofosfatemia
y acidosis metabólica.

Puede ser útil recordar que, al igual que en gastroenteritis aguda no se suprime la lactancia por su
efecto trófico sobre la vellosidades intestinales, en las formas graves de malnutrición tampoco se
retira.(R4)

54. Una paciente de 13 años acude a Urgencias de Pediatría por dolor hipogástrico
severo, que ya le había sucedido otras veces en el último año. Se detecta una masa de
15 cm en la pelvis menor. Se realiza un ultrasonido que demuestra contenido de
ecogenidad intermedia, y no se observa útero ni ovarios. Los genitales externos son
normales, así como el vello genital. Entre los labios menores se ve una zona oscurecida
a tensión. ¿Cuál es el diagnóstico probable?:

1. 1. Tumor de estirpe ectodérmica.


2. 2. Tabique vaginal transverso.
3. 3. Quiste de Gardner.
4. 4. Himen imperforado.
Gráfico de respuestas

 
 
 
 
Comentario

Pregunta sencilla, el cual corresponde a un himen imperforado respuesta 4.

Le mencionan los datos clave paciente en edad de menarquia, con dolor hipogastrico, que se ha
repetido en otras ocasiones, con masa en pelvis.(R4)

55. Paciente de 48 años diagnosticado hace 10 de esclerosis múltiple. Ha tenido un brote


hace dos meses del que se ha recuperado bastante bien. Acude a nuestra consulta
porque, coincidiendo con una infección urinaria, presenta una clínica muy similar a la del
último brote. ¿Cuál es el tratamiento más apropiado para este paciente?:

1. 1. Metilprednisolona intravenosa a altas dosis.


2. 2. Prednisona oral.
3. 3. Tratamiento de la infección exclusivamente.
4. 4. Copolímero 1.
Gráfico de respuestas
Comentario

Pregunta de dificultad moderada de un tema poco importante en el ENARM. Nos presentan un


paciente diagnosticado de esclerosis múltiple, pero en este caso no es un nuevo brote de la
enfermedad sino un empeoramiento transitorio (pseudobrote) debido a la infección concomitante.
También puede aparecer asociado a fiebre, elevación de la temperatura ambiental o efecto
adverso al tratamiento. En este caso está contraindicada la administración de corticoides, es
necesario identificar la causa y realizar un tratamiento etiológico. Por tanto, la respuesta correcta
es la 3 (tratamiento de la infección exclusivamente).(R3)

56. La primera manifestación del climaterio es:

1. 1. Sensación de bochorno.
2. 2. Trastornos en el ciclo menstrual.
3. 3. Cambios atróficos en los genitales externos.
4. 4. Cambios atróficos en las mamas.
Gráfico de respuestas
Comentario

El climaterio es una etapa de la vida de la mujer que dura años, durante la cual ocurre el paso de la
edad fértil a la no fértil. Se produce un agotamiento de los folículos primoridales y, como
consecuencia, el suceso clave de la menopasua, la caída de estrógenos.

En la perimenopausia que comprende el periodo de meses o de años que precede a la


menopausia y que suele ir acompañado de alteraciones del ciclo tanto en cantidad como en
frecuencia. Respuesta 2 correcta.(R2)

57. Ante una paciente amenorreica con test de gestágenos positivo, ¿cuál de las
siguientes etiologías de la amenorrea es la más factible?:

1. 1. Anovulación crónica.
2. 2. Síndrome de Asherman.
3. 3. Disgenesia gonadal.

 
 
 
 
4. 4. Fallo ovárico precoz.
Gráfico de respuestas
Comentario

Ante una amenorrea secundaria, lo primero que debe solicitarse es un test de embarazo. En caso
de ser positivo, ya tendríamos cuál es la causa de su amenorrea. En caso de ser negativo, se debe
practicar una determinación de TSH y PRL. Si son normales, el siguiente paso consiste en
administrar progesterona.

El test de la progesterona consiste en administrar una pequeña cantidad esta hormona, durante 5
días. Si la paciente ha producido con normalidad la primera fase del ciclo (proliferativa), pero no
llega a ovular y por tanto no llega a producir progesterona, al administrarle esta progesterona que
le faltaba, tendrá la regla y concluimos que la causa era la anovulación (respuesta 1 correcta)

En caso de no conseguirse la regla con la administración de progesterona, el paso siguiente es


combinar estrógenos y progestágenos para continuar el estudio.(R1)

58. Mujer de 19 años que en los últimos 15 días presenta febrícula, sudoración nocturna,
astenia y dolor en costado derecho. En la Rx de tórax se aprecia derrame pleural derecho
importante, sin otras alteraciones. Se realiza toracocentesis diagnóstica, obteniéndose
un líquido serohemático con: pH 7,36, proteínas 4,5 g/dl (sangre 7,5), LDH 850 U/l(sangre
150), glucosa 40 mg/dl y amilasa en rango de normalidad. El Gram es negativo. La
citología muestra predominio de linfocitos. ¿Cuál sería el siguiente paso a dar?

1. 1. Realizar biopsia pleural ciega.


2. 2. Realizar toracoscopia.
3. 3. Iniciar tratamiento antibiótico y esperar respuesta.
4. 4. Esperar el resultado del cultivo de Lowenstein.
Gráfico de respuestas
Comentario

Los derrames pleurales se clasifican en exudados o trasudados en atención a su contenido en


proteínas y LDH (criterios de Light). En este caso, se trata claramente de un exudado, dado su alto
contenido en proteínas y el aumento del LDH. Con los datos que nos aportan, el diagnóstico más
probable es el derrame pleural tuberculoso. Vamos a enumerar algunas características típicas del
mismo:

- Predominio de linfocitos entre las células.

- Ausencia o pobreza en células mesoteliales.

- Glucosa disminuida.

- Adenosindeaminasa aumentada (ADA +).

- Características de exudado.

Recuerda que la negatividad de la baciloscopia y del cultivo no excluye este diagnóstico. Recuerda
que, en muchas ocasiones, estas pruebas son negativas aunque se trate de un verdadero derrame
tuberculoso, lo que nos obliga a realizar una biopsia pleural con aguja. Lógicamente, no puedes

 
 
 
 
optar por la respuesta 5 (esperar al cultivo), porque M. tuberculosis tarda varias semanas en
crecer.(R1)

59. Paciente de 20 años que acude a su consulta por amenorrea primaria. Usted le realiza
un USG ginecológico en la primera visita, en la que no visualiza útero ni ovarios. El
fenotipo es femenino normal. Le solicita un cariotipo, siendo 46XY normal. Respecto a la
patología que sospecha, es CIERTO que:

1. 1. Se debe realizar extirpación de las gónadas por riesgo de gonadoblastoma.


2. 2. Las anomalías del sistema renoureteral son frecuentes.
3. 3. Los niveles de testosterona son acordes a los del varón.
4. 4. Cursa con cuadros de dolor cólico mensualmente.
Gráfico de respuestas
Comentario

En este caso clínico estamos ante un síndrome de Morris o de feminización testicular, el cual se
caracteriza por genitales externos normales, ausencia de vello axilar y pubiano, cariotipo 46XY y
niveles de testosterona normales para un varón, los cuales no hacen su efecto al carecer las
células del receptor nuclear de la testosterona. Los testículos intraabdominales pueden degenerar
en una tumoración de extirpe germinal, como el disgerminoma (el gonadoblastoma es otra
tumoración germinal, característica del síndrome de Swyer). En el síndrome de Swyer sí existe
útero, dado que se comporta como un 45 X0, dado que aunque se trata de un 46XY también, el
cromosoma Y no es funcionante. El síndrome de Morris no cursa con dolor, lo contrario que en el
himen imperforado. La etiología enzimática de la 21 hidroxilasa se da en la hiperplasia suprarrenal
congénita, que cursa con virilización de los genitales externos.(R3)

60. Señale cuál de las siguientes afirmaciones NO es correcta en relación con la aplasia
medular:

Un paciente con pancitopenia y reticulocitos descendidos podría corresponder a este


1. 1.
diagnóstico.
El parvovirus B19 se ha relacionado con la aparición de crisis aplásicas en pacientes con
2. 2.
anemias hemolíticas crónicas.
El tratamiento de elección es el trasplante de médula ósea, especialmente en mayores de
3. 3.
50 años.
Se ha relacionado con ciertos fármacos, como el cloranfenicol, las pirazolonas y los
4. 4.
antitiroideos.
Gráfico de respuestas
Comentario

En los pacientes de edades avanzadas, por lo general por encima de 50 años, no se realiza
transplante de médula ósea. En caso de presentar una aplasia medular se opta por otros
tratamientos como la globulina antitimocítica con ciclosporina.(R3)

61. ¿Cuál de las siguientes patogenias es FALSA en las alopecias no cicatriciales?

1. 1. Alopecia areata: inducción telogénica y paro en anagen precoz.


2. 2. Atriquia: aplasia folicular.
3. 3. Efluvio telogénico: inducción telogénica.
4. 4. Alopecia por quimioterapia: desprendimiento en catagen.
Gráfico de respuestas

 
 
 
 
Comentario

Una de las reacciones adversas más frecuentes de la quimioterapia es la alopecia. Se debe a


un efluvio anagénico secundario a la detención brusca de la actividad mitótica de las células de la
matriz del pelo. Aunque suele ocurrir entre 7- 10 días tras el comienzo de la quimioterapia, se
manifiesta clínicamente 1 ó 2 meses después. Fundamentalmente, se observa en el cuero
cabelludo; pero también es posible la afectación de otras zonas pilosas como las cejas, axilas y
pubis. La pérdida del pelo suele ser reversible y la severidad depende sobre todo del fármaco
administrado. La atriquia congénita es una enfermedad que se suele heredar de forma autosómica
recesiva. El pelo del cuero cabelludo con frecuencia es normal al nacimiento. Se desprende entre
el primer y sexto mes de vida, no apareciendo posteriormente. En la vida adulta no hay folículos
pilosos. También es posible que haya ausencia de pelo en otras localizaciones.(R4)

62. Señalar cuál de los siguientes NO es un criterio diagnóstico de la neurofibromatosis:

1. 1. Presencia de 2 o más máculas café con leche, de más de 1.5 cms.


2. 2. Glioma óptico.
3. 3. Presencia de un neuroma plexiforme.
4. 4. Dos o más hamartomas del iris (nódulos de Lisch).
Gráfico de respuestas
Comentario

El tema de las facomatosis tiene una importancia relativamente baja de cara al ENARM por no ser
objeto de muchas preguntas.

En esta pregunta se resumen las principales lesiones de la neurofibromatosis, no está de más que
las recuerde.

La neurofibromatosis es la facomatosis más frecuente y se hereda de forma autosómica


dominante. Se expresa clínicamente en forma de la triada de manchas café con leche,
neurofibromas cutáneos múltiples y nódulos de Lisch en el iris. Para el diagnóstico los pacientes
deben cumplir al menos dos de los siguientes criterios: seis o más manchas café con leche
mayores de 5 mm en prepúberes y de 15 mm en postpúberes, dos o más neurofibromas o un
neurofibroma plexiforme, efélides en pliegues axilares o inguinales (signo de Crowe), dos o más
hamartomas del iris (nódulos de Lisch), glioma óptico, alteraciones óseas distintivas (displasia
esfenoidal, pseudoartrosis, adelgazamiento de la cortical de huesos largos), un familiar afecto de
primer grado.(R1)

63. Varón de 60 años de edad que es derivado a su consulta por hallazgo de masa
pulmonar en Rx tórax. Se trata de un paciente fumador, que desde hace 3 meses refiere
cuadro constitucional y algún episodio aislado de hemoptisis. En la TAC torácica se
observa una masa pulmonar de 5 cm de diámetro en el LID, con adenopatías patológicas
en hilio derecho y en mediastino homolateral y contralateral. Tiene además una
adenopatía supraclavicular derecha palpable. Se le realiza una broncoscopia donde se
observa una masa endobronquial en el bronquio intermediario, que se biopsia, con
resultado de carcinoma microcítico. En la PET se objetiva captación patológica a nivel de
la masa pulmonar y de las adenopatías mediastínicas y supraclavicular, y a ningún otro
nivel. Una RNM cerebral descarta la existencia de metástasis cerebrales. Señale la
respuesta correcta:

1. 1. El tratamiento correcto es la quimiorradioterapia.

 
 
 
 
2. 2. El tratamiento correcto es la quimioterapia.
3. 3. El tratamiento correcto es la lobectomía inferior derecha
4. 4. El tratamiento correcto es la neumonectomía derecha.
Gráfico de respuestas
Comentario
Se trata de un paciente con un carcinoma microcítico en el LID. Existen adenopatías patológicas
en mediastino, pero no es necesario realizar mediastinoscopia porque en el microcítico el valor
predictivo positivo de la TAC y de la PET es muy alto. Las adenopatías se localizan en hilio
homolateral, mediastino homolateral, mediastino contralateral y supraclavicular homolateral. Todo
ello define la existencia de una enfermedad limitada, por lo que el tratamiento correcto es
quimiorradioterapia. La existencia de adenopatías hiliares contralaterales, supraclaviculares
contralaterales, derrame pleural tumoral (homo o contralateral) o metástasis a distancia definirían la
existencia de enfermedad extendida, en cuyo caso el tratamiento correcto sería quimioterapia.(R1)

64. A 65-year-old male comes to the ER complaining of extertional shortness of breath


and ankle edema for the last two months. His previous medical history is significant for
hypertension, dyslipidemia and obesity. He smokes a pack of cigarettes a day and drinks
two or three cans of beer a day. He also reports oppressive epigastric pain radiated to the
left shoulder and jaw that appears when he walks 100 meters and disappears after several
minutes of rest. Blood pressure is 140/85 mmHg, pulse is 80/min and temperature is
36.8ºC. Physical examination reveals obesity, mild bibasilar crackles and pitting edema
in both ankles. EKG shows no abnormalities and lab results are within normal limits.
Stress test is positive and the patient is admitted to perform a coronariography. Further
inquiries reveal excessive daytime somnolence (episodes of falling asleep at work or
while he stops his car at a red light). His wife says that he has always been a snorer and
that he occasionally stops breathing while sleeping. Which of the following is the best
next step in management?

1. 1. Add caffeine to his treatment.


2. 2. CT of the chest.
3. 3. Polysomnography.
4. 4. MRI of the upper airways.
Gráfico de respuestas
Comentario

Pregunta sencilla. La respuesta correcta es la 3, realizar una polisomnografía, ya que se sospecha


de un SAHOS. Esta enfermedad se ha asociado a múltiples complicaciones como hipertensión
arterial, e infartos.(R3)

65. RN de 35 semanas de edad gestacional que, en las primeras 12 horas de vida, presenta
taquipnea, quejido audible y cianosis progresiva, refractaria a la administración de
oxígeno. A la auscultación encontramos crepitantes, sobre todo en bases pulmonares.
En la gasometría aparece hipoxemia progresiva, aumento de la PCO2 y acidosis
metabólica. ¿Qué radiografía de tórax esperaría encontrar en este momento?:

1. 1. Infiltrado de patrón reticulogranular con broncograma aéreo.


2. 2. Hiperinsuflación y aumento de las marcas vasculares, sin broncograma.
3. 3. Patrón de esponja.
4. 4. Borde de ambos pulmones colapsados.
Gráfico de respuestas
Comentario(R1)

 
 
 
 

66. Un niño de 2 años, durante una gastroenteritis aguda presenta diarrea líquida intensa, vómitos
continuos, sed intensa, diuresis escasa y somnolencia. Pesaba 17.5 kg antes del episodio y ahora pesa
15 kg. ¿Qué grado de deshidratación tiene y cuál es la vía de elección para rehidratarle?:

1. 1. Deshidratación leve - rehidratación oral.


2. 2. Deshidratación moderada - rehidratación oral.
3. 3. Deshidratación severa - rehidratación oral.
4. 4. Deshidratación severa - rehidratación intravenosa.
Gráfico de respuestas
Comentario

Tema muy importante para el ENARM. En un niño de esta edad, una pérdida superior al 10% de su
peso se considera una deshidratación severa. En este caso, hemos sobrepasado el límite
ampliamente, llegando casi al 15%

Observe que, por otra parte, está teniendo una importante repercusión clínica (escasa diuresis y
somnolencia). La respuesta correcta sería, por tanto, la 4, ya que en este caso se trata de un caso
lo suficientemente grave como para plantearse la rehidratación intravenosa.(R4)

 
 
 
 

67. Mujer de 28 años que acude a consulta ginecológica para revisión rutinaria. Durante
la anamnesis, la paciente refiere eritema vulvovaginal, secundario a lo que cree que son
pérdidas de orina. La paciente refiere una necesidad imperiosa de orinar en situaciones
tales como estar en la alberca, entrar en casa o frío exterior. Al preguntarla sobre si esos
episodios van acompañados de incontinencia, responde que piensa que si. Refiere
necesitar ir al baño cada hora y media y nicturia de 2 veces, que no le ha llamado la
atención porque siempre ha sido así. Ante su sospecha diagnóstica, decide solicitar una
urodinamia que es informada como presencia de contracciones no inhibidas del detrusor,
sin episodios de incontinencia. ¿Qué patología presenta la paciente y que tratamiento
propondría?:

1. 1. Vejiga hiperactiva. Tratamiento quirúrgico mediante colocación banda suburetral.


2. 2. Incontinencia de esfuerzo. Tratamiento quirúrgico mediante colocación banda suburetral.
3. 3. Incontinencia de esfuerzo. Tratamiento con anticolinérgicos.
4. 4. Vejiga hiperactiva. Tratamiento con anticolinérgicos.
Gráfico de respuestas
Comentario
La vejiga hiperactiva se caracteriza por el síntoma principal de urgencia miccional, que puede ir
asociado o no de incontinencia. Además se puede acompañar de frecuencia y nicturia. En el
estudio urodinámico, que junto con el cultivo de orina, son las dos pruebas de elección para el
correcto diagnóstico, podremos ver las contracciones no inhibidas del músculo detrusor,
característico de este tipo de patología. El tratamiento de elección de la incontinencia de urgencia
es médico, siendo los anticolinérgicos los que mejores resultados presentan en la vejiga
hiperactiva.(R4)

68. ¿Cuál de los siguientes supuestos NO es cierto?:

1. 1. La glucosa pasa de la madre al feto a través de la placenta.


2. 2. La hiperglucemia fetal estimula la secreción de insulina.
3. 3. Los cuerpos cetónicos no atraviesan la placenta.
4. 4. Los aminoácidos atraviesan la placenta.
Gráfico de respuestas

 
 
 
 
Comentario

Pregunta complicada sobre la transferencia placentaria. La glucosa pasa de la madre al feto por
difusión facilitada, habiendo mayor concentración en la madre. La hiperglucemia fetal estimula la
producción de insulina fetal (evento que se ve en los fetos de gestantes diabéticas, de ahí que
exista mayor macrosomía fetal). Los aminoácidos atraviesan la barrera placentaria por transporte
activo. El glucagón no atraviesa la placenta, al igual que la insulina. Por lo tanto, la opción 3 es la
incorrecta, ya que los cuerpos cetónicos sí atraviesan la placenta.(R3)

69. Paciente varón de 69 años con cifras de hemoglobina de 8,2 g/dl y VCM de 126 fl.
¿Cuál de las siguientes causas de macrocitosis es la más probable que presente este
paciente?:

1. 1. Alcoholismo.
2. 2. Hepatopatía.
3. 3. Hipotiroidismo.
4. 4. Anemia megaloblástica.
Gráfico de respuestas
Comentario
Esta pregunta es de nivel de dificultad intermedio. Recordad que con el VCM se clasifican a las
anemias en microciticas, normociticas o macrocíticas. Todas las opciones citadas tienen un VCM
>100, por esto nos preguntan por el diagnostico más probable. Las anemias megaloblasticas
(opción 4 correcta) son las que cursan con mayor frecuencia con VCM superior a 120 fL.(R4)

70. A 54-year-old woman comes to your office concerned about her risk of developing
pancreatic cancer after her older brother died from it two months ago. Her previous
medical history is significant for hypertension, type 2 diabetes mellitus, obesity, and a
miocardial infarction two years ago. Her current medication includes, metformin and
glargine insulin, enalapril, aspirin, atorvastatin and atenolol. She smokes half a pack of
cigarrettes everyday and drinks 5 or 6 cans of beers on weekends. Which of the following
interventions would decrease his risk of pancreatic cancer the most?

1. 1. Alcohol cessation
2. 2. Quitting smoking
3. 3. Low fat diet
4. 4. Stop glargine insuline and start treatment with fast insuline
Gráfico de respuestas
Comentario
Quitting smoking. Remember that smoking is the most important risk factor for pancreatic
carcinoma.(R2)

71. A 48-year-old woman was diagnosed with breast cancer 5 years ago. She underwent
surgery, and she has just recently finished her hormone therapy. Her oncologist has told
her that she has a very good prognosis. However, ever since she was diagnosed with
cancer, she can't stop thinking about her imminent death, and because of this, she won't
go out any more with her friends or family. She complains of insomnia and anorexia. Her
husband is very worried, because her wife "is not who she used to be", and demands that
you prescribe an antidepressant. Which of the following is an absolute contraindication
to the use of bupropion?

1. 1. Epilepsy

 
 
 
 
2. 2. Hypertension
3. 3. Coronary artery disease
4. 4. Cancer
Gráfico de respuestas
Comentario
Epilepsy. Bupropion serves as an atypical antidepressant different from most commonly prescribed
antidepressants such as SSRIs. It is an effective antidepressant on its own, but it is also popular as
an add-on medication in cases of incomplete response to first-line SSRI antidepressants. In
contrast to many other antidepressants, it does not cause weight gain or sexual dysfunction. The
most important side effect is an increase in risk for epileptic seizures, which caused the drug to be
withdrawn from the market for some time and then caused the recommended dosage to be
reduced.(R1)

72. El padecimiento de parotiditis en el primer trimestre de la gestación se asocia


típicamente con una alteración cardíaca, ¿cuál?:

1. 1. Prolapso de la válvula mitral.


2. 2. Fibroelastosis endocárdica.
3. 3. Aneurismas de las arterias coronarias.
4. 4. Comunicación interventricular múltiple.
Gráfico de respuestas
Comentario

Las asociaciones que debe recordar de la parotiditis son: el aumento de la amilasa (sin correlato
clínico con pancreatitis) y la orquitis/ooforitis. Recuerde, además, que la parotiditis es la causa más
frecuente de hipoacusia neurosensorial unilateral adquirida. A veces se asocia con fibroelastosis
endocárdica pero es casi una anécdota.(R2)

73. Un paciente varón de 46 años va a ser intervenido quirúrgicamente de estenosis


aórtica severa y sintomática. Señale lo cierto con respecto al tratamiento de este
paciente:

1. 1. Si tiene disfunción ventricular, está contraindicada la cirugía.


2. 2. La prótesis aórtica será preferiblemente biológica.
3. 3. El tratamiento de elección puede ser una comisurotomía.
4. 4. Si tiene disfunción ventricular izquierda, la mortalidad quirúrgica será mayor.
Gráfico de respuestas
Comentario
Pregunta útil para repasar el tratamiento quirúrgico en la Estenosis Aórtica, un tema bastante
preguntado. Debes recordar que la cirugía se indica en la EAo en cuanto aparecen los primeros
síntomas, ya que este hecho empeora mucho el pronóstico. Si ya existe insuficiencia ventricular
izquierda, el riesgo operatorio es alto, pero está justificado, ya que la esperanza de vida en los
sujetos que sobreviven a la intervención es, con mucho, superior a la que existiría con tratamiento
médico. Por tanto, la disfunción ventricular no contraindica la cirugía, aunque sí aumenta la
mortalidad quirúrgica. En este paciente, de 46 años, la prótesis será preferiblemente mecánica, ya
que la biológica tiene una duración limitada. La prótesis biológica se indica en casos de pacientes
ancianos, o con contraindicación para la anticoagulación (la mecánica sí requiere anticoagulación,
mientras que la biológica no). Después de la intervención sigue siendo necesaria la profilaxis
contra la endocarditis. La comisurotomía se emplearía sólo en caso de EAo no calcificada, por lo
que se emplea casi exclusivamente en las de etiología congénita.(R4)

 
 
 
 
74. Un paciente de 45 años consulta por molestias oculares de varios meses de
evolución. Refiere picor y pinchazos en ambos ojos, así como sensación de quemazón
en los párpados. En la exploración se observan escamas blanquecinas en la base de las
pestañas. ¿Cuál es el diagnóstico más probable?:

1. 1. Ectropion senil.
2. 2. Blefaritis escamosa.
3. 3. Blefaritis estafilocócica.
4. 4. Meibomitis crónica.
Gráfico de respuestas
Comentario
Ante un cuadro de blefaritis anterior con escamas blanquecinas adheridas a las pestañas y signos
de inflamación debes pensar en una blefaritis seborreica (eccematosa o escamosa) y el
diagnóstico diferencial debes hacerlo principalmente con la blefaritis ulcerosa (estafilocócica).(R2)

75. El cáncer de cérvix se origina preferentemente en:

1. 1. Ectocérvix.
2. 2. Unión escamo-columnar.
3. 3. Glándulas cervicales.
4. 4. Quistes de Naboth.
Gráfico de respuestas
Comentario

La unión escamo-columnar es el sitio en donde se encuentra el mayor número de cáncer de cérvix.


Respuesta 2 correcta.(R2)

76. ¿Cuál de los siguientes antibióticos NO utilizaría en un paciente con una neumonía
nosocomial en un paciente EPOC con varios ingresos?:

1. 1. Meropenem.
2. 2. Cefepima.
3. 3. Tigeciclina.
4. 4. Piperacilina-tazobactam.
Gráfico de respuestas
Comentario
La neumonía nosocomial presenta un riesgo aumentado de infección por microorganismos
gramnegativos. Además cuando existen otros factores de riesgo como en este caso los ingresos y
tratamientos previos el riesgo de infección por Pseudomonas es importante. La tigeciclina es una
tetraciclina que cubre gramnegativos y grampositivos con amplio espectro pero no Pseudomonas
por lo que no puede utilizarse en este caso.(R3)

77. A 7-year-old girl is referred to your office for vaginal bleeding for the last two months.
The mother denies previous exposure to exogenous estrogens. Physical examination
reveals a body height that is in the 98th percentile, Tanner III breast development and no
axillary or pubic hair. No abdominal or pelvic masses are noted upon palpation.
Neurological examination is normal. Skull X-ray film shows no enlargement or distortion
of the sella turcica and absence of intracerebral calcifications; Bone age; 10
years.Laboratory studies reveal: 17-ketosteroids: 17 umol. Gonadotropins in urine
undetectable. Which of the following is the most appropriate next step in management?

 
 
 
 
1. 1. Exploratory laparotomy.
2. 2. Treatment with medroxyprogesterone.
3. 3. Measurement of plasma androstenedione.
4. 4. Abdominal CT and pelvic ultrasound.
Gráfico de respuestas

(R4)

78. Una mujer de 70 años consulta por astenia y anemia ferropénica que no presentaba
en los laboratorios del año anterior. En el estudio se demuestra un cáncer intestinal. ¿En
qué lugar estará más frecuentemente localizado?

1. 1. Duodeno.
2. 2. Yeyuno.
3. 3. Ciego.
4. 4. Sigma.
Gráfico de respuestas
Comentario

El 70-80% de los cánceres colorrectales aparecen en colon descendente, sigma y recto. No


obstante, los tumores que aparecen en estas localizaciones se manifiestan con síntomas
obstructivos, lo que no encaja con la clínica que nos describen en esta pregunta. Sin embargo, los
cánceres de localización más proximal (ciego, colon ascendente) se manifiestan con signos y
síntomas derivados del sangrado (anemia ferropénica, en este caso), así que la respuesta correcta
es la 3. Importante: el colon derecho sangra; el izquierdo, se obstruye.

Las opciones 1 y 2 son muy improbables, dado que los tumores primarios de esa localización son
mucho menos frecuentes que el cáncer de colon.(R3)

79. Respecto a la menopausia, la afirmación CORRECTA es:

1. 1. Lo más eficaz es el estradiol oral.


Los suplementos de calcio son fundamentales, ya que se ha demostrado que reducen
2. 2.
considerablemente el índice de fracturas de cadera.
Si hay contraindicación para los estrógenos, debemos al menos administrar
3. 3.
progestágenos.
4. 4. Si aparece tensión mamaria, debemos bajar la dosis de estrógenos.
Gráfico de respuestas
Comentario

La tensión y el dolor mamario son síntomas derivados de un hiperestrogenismo, como también


puede ocurrir en la mastopatía fibroquística. Por tanto, en las pacientes con terapia hormonal
sustitutiva (THS), habría que disminuir la dosis de estas hormonas.

Respecto a la opción 1, conviene decir que el estradiol es igualmente eficaz independientemente


de la vía. Lo que puede variar en función de ésta son los efectos secundarios.

En pacientes histerectomizadas, no es necesario añadir progestágenos, ya que no existiría riesgo


de cáncer de endometrio, como sucede en otros casos. Por otra parte, carece de sentido
administrar progestágenos solos. Los síntomas climatéricos dependen del déficit estrogénico. La

 
 
 
 
razón para administrar progestágenos es para evitar el aumento de riesgo de cáncer de
endometrio.

En cuanto al calcio, los suplementos no son necesarios si en la dieta hay cantidad suficiente. En
caso de administrarse calcio, se asocia a vitamina D. Los suplementos de calcio sí que han
demostrado una menor pérdida en la densidad mineral ósea… Pero no alteran la incidencia de la
fractura de cadera.(R4)

80. La existencia en una mujer de 40 a. de nefrolitiasis y nefrocalcinosis, osteitis fibrosa


quística, así como resorción subperiostica de falanges y pequeñas lesiones en
sacabocado en cráneo, evidenciables con Rx simple; cloro 110 y calcio 11,5 mgr/dl
potasio 3,2, harían sospechar que padece:

1. 1. Sarcoidosis.
2. 2. Hipercalcemia secundaria a ca. mama.
3. 3. Hiperparatiroidismo primario.
4. 4. Pseudohiperparatiroidismo.
Gráfico de respuestas
Comentario
Los trastornos del metabolismo del calcio constituyen un tema bastante preguntado en el MIR. Esta
pregunta es sencilla. Las cifras normales de Ca son 9 a 10.5 mg/dl. Esta pacte tiene por lo tanto
hipercalcemia. Por probabilidad acertaríamos la pregunta ya que la causa más frecuente de
hipercalcemia es el hiperparatiroidismo primario (por un adenoma paratiroideo), pero es que
además la pacte presenta la clínica característica de esta entidad: afectación ósea puesto que la
PTH estimula la resorción, con osteítis fibrosa quística (quistes óseos en los huesos largos y
tumores pardos, formados por acúmulos de osteoclastos gigantes multinucleares), resorción
subperióstica de las falanges y osteopenia localizada (cráneo en sal y pimienta). También presenta
afectación renal con litiasis recidivante y nefrocalcinosis secundarias a la hipercalciuria
persistente.(R3)

81. ¿Cuál de las siguientes afirmaciones acerca de la infección nosocomial considera


FALSA?:

La infección del tracto urinario constituye la forma más prevalente de infección


1. 1.
nosocomial.
La duración de la intervención es un factor de riesgo para el desarrollo de infección de la
2. 2.
herida quirúrgica.
Los microorganismos más frecuentemente implicados en la infección asociada a catéter
3. 3. intravascular son los estafilococos coagulasa negativos, Staphylococcus aureus, Candida
y algunos gramnegativos.
4. 4. La bacteriemia por catéter supone la infección nosocomial de peor pronóstico.
Gráfico de respuestas
Comentario

La neumonía justifica hasta el 20% de todas las infecciones nosocomiales y constituye la forma
asociada a una mayor mortalidad, que puede superar el 50% entre los pacientes sometidos a
ventilación mecánica. Entre sus grupos específicos de riesgo figuran los pacientes ingresados en
UCI (sobre todo los intubados), de edad avanzada, con alteración del nivel de conciencia,
portadores de sonda nasogástrica y sometidos a tratamiento con inhibidores de la bomba de
protones. Los agentes más frecuentemente identificados son Pseudomonas aeruginosa,
Staphylococcus aureus, S. pneumoniae y algunos bacilos gramnegativos.(R4)

 
 
 
 
82. Paciente que acude por dolor epigástrico que cede con la ingesta de un mes de
evolución. Se realiza tránsito intestinal descubriéndose pequeño lecho ulceroso sin
signos de malignidad en la curvadura mayor de cuerpo gástrico. En el test del aliento el
Helicobacter pylori fue negativo. ¿Qué debería hacer a continuación?:

1. 1. Tratamiento con omeprazol durante un mes.


2. 2. Gastroscopia antes de iniciar tratamiento.
3. 3. Tratamiento quirúrgico.
4. 4. Tratamiento con antisecretores, antiácidos y antibióticos durante 1 mes.
Gráfico de respuestas
Comentario
En las úlceras gástricas el estudio endoscópico es obligatorio, ya que casi un 5 % de ellas
"esconden" un carcinoma. Éste es el motivo por el que en el diagnóstico de cualquier úlcera
gástrica, antes de iniciar un tratamiento, es obligado el estudio endoscópico con toma de biopsias
de los bordes de la úlcera. Y en este caso, al no tener infección por Helicobacter pylori, el
tratamiento sería únicamente con antisecretores.(R2)

83. ¿Cuál de las siguientes afirmaciones con respecto al síndrome de Barter es falsa?

1. 1. Asocia aumento de PGE2.


2. 2. El tipo IV y V asocian sordera.
3. 3. El tipo III se produce por mutaciones en el canal apical de K.
4. 4. El tipo I se produce generalmente por mutaciones en el cotransportador Na-K-2Cl.
Gráfico de respuestas
Comentario

Pregunta muy difícil sobre mutaciones genéticas del síndrome de Barter. La respuesta falsa es la
3, el tipo II es el que tiene mutación en el canal apical de K.(R3)

84. La causa más frecuente de isquemia mesentérica aguda es:

1. 1. Trombosis arterial mesentérica.


2. 2. Trombosis venosa mesentérica.
3. 3. Embolia arterial mesentérica.
4. 4. Choque hipovolémico.
Gráfico de respuestas
Comentario

Pregunta muy importante para fines del ENARM, es frecuente que lo pregunten en forma de caso
clínico, como el típico anciano con algún tipo de arritmia que inicia de manera súbita con dolor
abdominal. La isquemia mesentérica aguda es un proceso obstructivo brusco de la arteria
mesentérica debido con más frecuencia por una embolia en pacientes con fibrilación auricular u
otro proceso embólico. Con menos frecuencia se debe a una trombosis in situ de una placa de
arteriosclerosis o a bajo gasto.(R3)

85. No sin mucha reticencia, consigue que su madre acuda al psiquiatra. El motivo de
consulta son las interminables quejas somáticas que viene padeciendo desde su
adolescencia, quejas que han desembocado en numerosas exploraciones
complementarias (endoscopias, ecografías) y en dos intervenciones quirúrgicas por
“piedras en la vesícula” y “gastritis”; está diagnosticada de “colon irritable” y dice tener
continuos mareos que atribuye a las “cervicales”; abusa de los analgésicos menores

 
 
 
 
para controlar una “cefalea tensional” crónica. Señale cuál de los siguientes datos NO se
suele encontrar en estos pacientes:

1. 1. Este trastorno suele iniciarse antes de los 30 años y tener un curso crónico.
2. 2. Hay un claro predominio de las mujeres.
3. 3. Existe una buena respuesta a antidepresivos en cerca del 60% de los casos.
4. 4. Existe un riesgo de suicidio aumentado.
Gráfico de respuestas
Comentario

Es un cuadro muy típico, mujer multisintomática, habiéndose descartado patología orgánica:


trastorno por somatización o Síndrome de Briquet.

Suele predominar en mujeres (5:1, R2), de inicio en jóvenes (<30años generalmente, R1), con
ansiedad y depresión asociados, lo que incrementa el riesgo suicida (R4).

La respuesta al tratamiento farmacológico es mala, aunque hay autores que abogan por realizar
una prueba con antidepresivos por la posibilidad e una depresión enmascarada, aunque el objetivo
más realista es el de evitar la yatrogenia.(R3)

86. Todo lo siguiente es apropiado para el tratamiento de la laringotraqueobronquitis viral


aguda, EXCEPTO:

1. 1. Antibióticos.
2. 2. Antipiréticos.
3. 3. Adrenalina racémica.
4. 4. Corticosteroides.
Gráfico de respuestas
Comentario

Esta pregunta acerca del tratamiento de la LAV es interesante.

La humidificación del ambiente tiene como objetivo fluidificar las secreciones y evitar la desecación
de la mucosa pero no hay estudios que avalen su eficacia. Debe ser evitada cuando se demuestre
estresante para el niño.

Los corticoides suelen formar parte del tratamiento habitual. Si el crup es leve: dexametasona vo
en dosis única, si es moderado se añade budesonida inhalada.

La adrenalina se considera efectiva por su efecto vasoconstrictor además de relajar la musculatura


bronquial y se reserva para casos graves.

Los antibióticos solo están indicados en caso de sobreinfección bacteriana documentada.(R1)

87. ¿Cuál es el tratamiento de elección en un paciente al que se le sometió a un by-pass


de mamaria interna que padece angina recurrente tras la derivación quirúrgica por
estenosis del injerto?:

1. 1. Sustituir el injerto de mamaria por uno de safena interna


2. 2. Practicar una ACTP del injerto estenosado.

 
 
 
 
3. 3. Trasplante cardíaco.
4. 4. Papaverina intracoronaria.
Gráfico de respuestas
Comentario
Las indicaciones de ACTP que vienen en el Manual son las siguientes: - Enfermedad coronaria de
2 vasos que no afecte a la DA proximal. - Enfermedad de un vaso. - Enfermedad de injertos de
derivación quirúrgica. Por tanto, en este paciente, habría que hacer una coronariografía, localizar la
lesión y realizar la ACTP. Actualmente, para este tipo de lesiones, se están colocando stents
recubiertos por fármacos, lo que disminuye de forma muy importante la tasa de reestenosis.(R2)

88. Un paciente de 80 años acude al médico tras haber presentado un síncope brusco
mientras subía un tramo de escaleras. La exploración física muestra un soplo sistólico
eyectivo 3 sobre 6, y en el electrocardiograma se observa un ritmo sinusal normal y
signos de hipertrofia del ventrículo izquierdo. ¿Qué exploración diagnóstica solicitaría
en primer lugar?

1. 1. Un Holter de 24 horas.
2. 2. Un ecocardiograma-Doppler.
3. 3. Un estudio electrofisiológico.
4. 4. Una prueba de esfuerzo.
Gráfico de respuestas
Comentario

Pregunta fácil. Tema valvulopatías.

Nos presentan un caso clínico de un paciente mayor que tiene un síncope de esfuerzo, destacando
a la exploración un soplo sistólico significativo y signos de hipertrofia ventricular izquierda (HVI).
Todos estos datos nos sugieren la sospecha diagnóstica de estenosis aórtica. La estenosis aórtica
degenerativa es frecuente en personas mayores. La tríada clínica de esta entidad está compuesta
por angina, síncope y disnea, confiriendo el síncope un pronóstico de supervivencia de
aproximadamente tres años. Es normal la aparición de signos de HVI en el ECG por la sobrecarga
de presión que acarrea la obstrucción crónica a la eyección del ventrículo izquierdo. El soplo es
sistólico, rudo, más intenso en foco aórtico e irradiado a carótidas. La prueba diagnóstica, que es lo
que nos piden para confirmar la sospecha clínica, aquí como en todas las valvulopatías es el
ecocardiograma-doppler.(R2)

89. La causa más frecuente de amenorrea secundaria es:

1. 1. Anovulación.
2. 2. Anorexia nerviosa.
3. 3. Realización de deporte continuado.
4. 4. Gestación.
Gráfico de respuestas
Comentario

Pregunta básica que no puede fallar. La causa más frecuente de amenorrea secundaria es el
embarazo.!!!!(R4)

90. Paciente de 42 años con carcinoma de cérvix, diagnosticado por biopsia, que invade
el labio anterior y 1 cm de la cúpula vaginal; los parametrios no están invadidos; la

 
 
 
 
rectoscopía, la urografía descendente y la cistoscopía son normales. En este caso, el
tratamiento más CORRECTO sería:

1. 1. Radioterapia externa más quimioterapia.


2. 2. Histerectomía total ampliada tipo Wertheim-Meigs más radioterapia externa.
3. 3. Radioterapia de contacto, seguida de histerectomía total simple.
4. 4. Ninguna de las anteriores.
Gráfico de respuestas
Comentario

Pregunta teórica acerca del tratamiento del carcinoma de cérvix, debiendo conocer datos básicos
del TNM. Recuerde que el estadio IIB implica afectación de parametrios. En este caso nos
presentan un carcinoma cervical estadio IIA por lo que el tratamiento de elección es la
histerectomía radical con linfadenectomía pélvica (cirugía de Wertheim-Meigs). La única opción
que te ofrece esta posibilidad es la respuesta número 2 (con RT externa adyuvante).(R2)

91. Aunque no todos los autores han llegado a las mismas conclusiones, determinados
trabajos han encontrado que algunas obras de Mozart pueden producir un efecto
protector frente a la aparición de crisis epilépticas. Este efecto se ha descrito
especialmente en relación con una de ellas:

1. 1. Sonata para dos pianos en Re mayor K448.


2. 2. Cuarteto en La mayor K464.
3. 3. Leck mir den Arsch K233.
4. 4. Serenata nº 13 en Sol mayor K525.
Gráfico de respuestas
Comentario

Una pregunta muy complicada, que muy pocas personas podrán contestar, sin embargo, por
técnica de examen, podrías elegir la respuesta más extensa, y sería un acierto. El llamado “efecto
Mozart” (que hoy día está puesto en duda) se ha descrito en muchas de sus obras, entre ellas
varias de las que aquí figuran, pero la que más se ha estudiado en relación con esto es la opción
1.(R1)

92. Las pruebas básicas empleadas habitualmente en la práctica clínica para el estudio
de la función pulmonar son:

Espirometría forzada con prueba broncodilatadora, medición de volúmenes pulmonares


1. 1.
estáticos y transferencia de monóxido de carbono.
Espirometría forzada, medición de volúmenes pulmonares estáticos y determinación de
2. 2.
las relaciones ventilación/perfusión.
3. 3. Espirometría forzada, pruebas de broncoprovocación y prueba de esfuerzo.
Transferencia de monóxido de carbono, gasometría arterial y determinación de las
4. 4.
relaciones ventilación/perfusión.
Gráfico de respuestas
Comentario

Para el estudio de función pulmonar son utiles la espirometria forzada con prueba broncodilatadora
(patología obstructiva), la medicion de volúmenes pulmonares estaticos (sospecha de restricción),
y la DLCO (capacidad de difusión pulmonar).(R1)

 
 
 
 
93. Un paciente sufrió una herida por proyectil de arma de fuego, que penetró en el sexto
espacio intercostal izquierdo por delante y salió en el mismo espacio por detrás. No hay
hipotensión ni signos de irritación peritoneal. El tratamiento de elección es:

1. 1. Ingreso en el hospital para vigilancia estrecha.


Exploración local de ambas heridas, e ingreso en el hospital para observación si no se
2. 2.
puede comprobar la penetración de la cavidad peritoneal.
3. 3. Lavado peritoneal, y si resulta positivo, laparotomía exploratoria.
4. 4. Laparotomía exploratoria.
Gráfico de respuestas
Comentario

Es una pregunta que no se volverá a fallar en cuanto estudiemos el manual o se te repita. Todas
las heridas por asta de toro o por arma de fuego necesitan laparotomía exploradora (opción 4), sea
evidente o no la perforación e independientemente de los síntomas. Se deben incluir aquellas de
tórax bajo, espalda y flancos. Esto es debido a que hay lesión en el 90% de los casos. Además,
trayectos extraperitoneales en una herida por arma de fuego pueden originar lesiones
intraperitoneales por el efecto de onda expansiva, de ahí la necesidad de la laparotomía
exploradora.(R4)

94. Es característica de malignidad en tumores de ovario EXCEPTO:

1. 1. Papilas intraquísticas.
2. 2. Ascitis.
3. 3. Multilocular.
4. 4. Contenido homogéneo refringente.
Gráfico de respuestas
Comentario

Pregunta bastante específica, pero que puede contestar por descarte sindo lógica la respuesta.
Respuesta 4 incorrecta.

Los siguientes datos son sospechosos de malignidad:

Ascitis

Palpación de tumoración pélvica

Poca movilidad por adherencias

Índice de crecimiento rápido

Edad no reproductiva

Nódulos en fondo de saco de Douglas

Bilateralidad

Hallazgos ultrasonográficos que harán sospechar malignidad:

 
 
 
 
Tamaño tumoral superior a 5-10 cm

Contenido heterogéneo refringente

Presencia de tabiques o septos gruesos mayores de 3 mm, papilas, o partes sólidas en su interior.

Presencia de líquidos en cantidad moderada en Douglas o ascitis.

Multilocular(R4)

95. Paciente de 47 años que acude a urgencias porque, desde hace 15 días, presenta un
sangrado que comenzó como una menstruación normal, pero que no ha cedido, incluso
llegando a sangrar más que una menstruación. Como antecedentes ginecológicos
destacan 2 partos eutócicos y un legrado por aborto diferido; menstruaciones cada 26
días, de 5 días de duración, no dolorosas. Le solicita una prueba de embarazo en orina,
siendo negativo su resultado. En la exploración se detecta un cérvix de multípara, con
restos hemáticos en vagina en cantidad similar a una menstruación. Le realiza un USG
transvaginal en la que se observa un útero regular en anteversión de 8 cm, con un
endometrio de aspecto secretor de 13 mm sin que parezca observarse patología. Anexo
derecho normal, anejo izquierdo con formación econegativa de 38 mm en ovario
izquierdo de aspecto benigno. Respecto al cuadro clínico que presenta la paciente, es
FALSO que:

1. 1. Para cohibir la hemorragia, se debe pautar tratamiento estrogénico.


2. 2. Es recomendable el uso de AINEs y preparados con hierro oral.
3. 3. El cuadro clínico puede explicarse por ciclos disovulatorios.
Si persisten los cuadros de sangrado, se puede plantear el uso de anticonceptivos
4. 4.
hormonales, DIU de levonorgestrel o cirugía.
Gráfico de respuestas
Comentario

Este es un caso clínico muy frecuente en la práctica diaria. Tenemos una paciente en edad
premenopáusica, las cuales son más candidatas a metrorragias disfuncionales, debidas muchas
veces a ciclos anovulatorios. Nuestra paciente presenta una metrorragia que en ecografía se
observa una formación funcional ovárica que puede ser un folículo luteinizado no roto, folículo
persistente, … en tal caso, formación funcional de aspecto benigno, la cual debe vigilarse en unos
tres meses para demostrar su desaparición; esta tumoración ovárica secretora de hormonas hace
que un endometrio secretor pueda sangrar, por ello debemos pautar gestágenos para cohibir la
hemorragia. Si fuera atrófico el endometrio, deberíamos dar estrógenos. También podemos pautar
hierro para prevenir anemia y los AINEs al disminuir las prostaglandinas disminuirán el sangrado.
Para regular el ciclo se pueden usar anticonceptivos hormonales, o se puede usar DIU de LNG
para hacer que el endometrio prolifere menos y tener menos regla.(R1)

96. ¿Cuál de las siguientes opciones acerca de la bronquiolitis es CORRECTA?:

El principal agente causal es el virus respiratorio sincitial, del que hay seis tipos, que van
1. 1.
del A al F.
El único tratamiento farmacológico que se ha demostrado eficaz es la adrenalina en
2. 2.
aerosol.
3. 3. La epidemia de bronquiolitis ocupa, típicamente, los meses de septiembre a enero.
4. 4. El diagnóstico se suele realizar por la detección mediante cultivo del VRS en moco nasal.

 
 
 
 
Gráfico de respuestas
Comentario

Esta pregunta de bronquiolitis es bastante importante ya que resume varios aspectos de interés.

El principal agente causal es el virus respiratorio sincitial, del que hay sólo dos grupos antigénicos,
el A y el B. La mayoría de los casos se producen en forma de epidemias anuales entre los meses
de octubre a abril (invierno y principios de primavera).

El diagnóstico se suele realizar por la detección de antígeno de VRS en moco nasal mediante
técnicas de diagnostico rápido (inmunofluorescencia). El dato auscultatorio más típico es la
presencia de sibilancias en el lactante.

A pesar de que el salbutamol es el fármaco más utilizado en la bronquiolitis, numerosos estudios


confirman que los beta- agonistas no son útiles en su tratamiento pero, dado que pueden mejorar a
algunos pacientes se utiliza inicialmente. La mejoría puede deberse a que sea el primer episodio
de broncoespasmo de un niño asmático o que el episodio actual sea una crisis de sibilancias
inducido por otro virus diferente al VRS. El único tratamiento farmacológico que parece eficaz es la
adrenalina en aerosol, que disminuye el edema de la pared.(R2)

97. Paciente adolescente con NIC I más HPV cervicovaginal. ¿Cuál de las siguientes sería
la conducta adecuada?

1. 1. Cono leep o cono frio.


2. 2. Vacunación inmediata contra cepas 16, 18, 6 y 11.
3. 3. Histerectomía ampliada a vagina.
4. 4. Observación y control periódico.
Gráfico de respuestas
Comentario

La actitud diagóstica que tomaremos frente a una neoplasia intraepitelial cervical va a depender de
las características de la paciente:

- Chica joven, sin NIC previa, de bajo grado, inmunocompetente: observación

- Si no cumple los anteriores requisitos: crioterapia, láser o termocoagulación.

Esto es debido a que en las NIC I no existe acuerdo sobre cual debe ser la mejor conducata, si es
tratamiento o es seguimiento. Dado el bajo riesgo de progresión a un cáncer invasor, <1%, y el alto
porcentaje de regresión espontánea, especialmente en pacientes jóvenes, el tratamiento
sistemático representa un sobretratamiento, no extento de complicaciones. Por ello la conducta se
individualiza en cada caso.(R4)

98. La laparoscopia se considera la técnica de elección como abordaje en una de las


siguientes patologías:

1. 1. Como técnica diagnóstica en la herida por arma blanca.


2. 2. Hernioplastia inguinal.
3. 3. Exploración de la vía biliar.
4. 4. Eventración.

 
 
 
 
Gráfico de respuestas
Comentario

Pese a que la laparoscopia se está convirtiendo en un abordaje aceptado para múltiples patologías
(colectomías, sutura úlcera perforada, exploraciones de la vía biliar y drenaje de quistes hepáticos,
pancreatectomías distales…), su uso como tratamiento de elección sólo ha demostrado ventajas
sobre el abordaje abierto en: colecistectomía, laparoscopia diagnóstica (dolor crónico abdominal,
estadiaje preoperatorio, evaluación de la integridad peritoneal en la herida penetrante por arma
blanca), apendicitis en mujeres jóvenes y obesos, esplenectomía, adrenalectomía por patología
benigna y nefrectomía.(R1)

99. ¿Cuál de estos órganos se afecta clínicamente con MENOR frecuencia en pacientes
con PAN clásica?

1. 1. Riñón.
2. 2. Nervios periféricos.
3. 3. Piel.
4. 4. Sistema nervioso central.
Gráfico de respuestas
Comentario

Recuerda que la PAN clásica es una vasculitis sistémica necrotizante que puede afectar a casi
cualquier órgano, con excepción del pulmón. En orden de frecuencia, los órganos más afectados
son el riñón (70%), con desarrollo de insuficiencia renal e HTA; el musculoesquético ( 64%), con
artralgias, mialgias o artritis; el SN periférico (51%) con neuropatía periférica o mononeuritis
múltiple; el tubo digestivo (44%) con dolor abdominal o hemorragias, infartos, perforaciones, etc.
En la piel (43%), es típica la aparición de púrpura, nódulos, livedo, Raynuad… A nivel cardíaco
(36%), con ICC, infarto o pericarditis. Respecto al genitourinario (25%), dolor testicular u ovárico. El
SNC se afecta con poca frecuencia (23%) con ACVs, alteraciones del estado mental o
convulsiones.(R4)

100. Le avisan de de un registro cardiotocográfico patológico, en una gestante de 42


semanas que está de parto. Al romper la bolsa, detecta un líquido amniótico intensamente
teñido de meconio. El RN nace con un Apgar al minuto y a los cinco minutos de 4 y 7,
precisando aspiración de secreciones y ventilación con mascarilla con presión positiva
intermitente. A las dos horas de vida el niño presenta dificultad respiratoria con quejido
espiratorio audible sin estetoscopio, disociación tóraco-abdominal ligera, retracción
xifoidea y tiraje intercostal marcados, sin aleteo nasal. En la gasometría muestra
hipoxemia con hipercapnia. ¿Cuál de las siguientes es CORRECTA?

1. 1. El paciente tiene un score de Silverman de 5.


2. 2. En la radiografía de tórax es frecuente hallar derrame en las cisuras y broncograma aéreo.
Otro hallazgo radiológico típico es la hipoventilación, con menos de 7 espacios
3. 3.
intercostales en la radiografía AP de tórax.
4. 4. Algunos pacientes se benefician del tratamiento con surfactante intratraqueal.
Gráfico de respuestas
Comentario
El cuadro descrito corresponde a un síndrome de aspiración meconial. Es característico de RN a
término o postérmino, con datos de sufrimiento fetal agudo, Apgar bajo al nacer y líquido amniótico
teñido de meconio (el RN hijo de madre diabética es propenso a padecer enfermedad de
membrana hialina). La clínica consiste en dificultad respiratoria severa (el Silverman del

 
 
 
 
paciente es de 7), con abombamiento torácico por atrapamiento aéreo (y, por tanto, con signos
radiológicos de hiperinsuflación), hipoxemia e hipercapnia. En la radiografía es frecuente
encontrar hiperinsuflación, infiltrados algodonosos, atelectasias y aire extraalveolar (el líquido en
cisuras es un dato sugestivo de taquipnea transitoria del RN y el broncograma aéreo de
enfermedad de membrana hialina). El tratamiento consiste en asistencia respiratoria,
antibioterapia y, en ocasiones, surfactante para mejorar la mecánica pulmonar.(R4)

101.
Respecto al ECG (IMAGEN), señale la opción correcta.

1. 1. Muestra un patrón de preexcitación ventricular.


2. 2. Se trata de un infarto antiguo de miocardio inferior.
3. 3. Existe bloqueo completo de la rama izquierda.
4. 4. Existe un flutter auricular.
Gráfico de respuestas
Comentario

En este ECG, se muestra claramente un bloqueo completo de rama izquierda. Si se tratase de un


bloqueo completo de rama derecha, como dice la respuesta 1, la imagen estaría en derivaciones
como aVR, V1 o V2.(R3)

102. El epitelio que recubre las paredes del cistoadenoma mucinoso recuerda el epitelio:

1. 1. Tubárico.
2. 2. Endocervical.
3. 3. Endometrial.
4. 4. Exocervical.
Gráfico de respuestas
Comentario

Esta pregunta sobre la histologia de los tumores de ovario no tiene mucha importancia.

El epitelio que recubre las paredes del cistoadenoma mucinos es semejante al epitelio
endocervical, el de los cistoadenomas serosos al tubarico, el de endometrioma al endometrio.

No olvidemos que el epitelio del endocérvix es cilindrico y el del exocérvix escamoso.(R2)

 
 
 
 

103. Respecto al síndrome que


presumiblemente tiene un paciente con la radiografía de tórax que se muestra en la
imagen nº 10, ¿cuál de los siguientes NO constituye un criterio diagnóstico?

1. 1. PaO2/FiO2 inferior o igual a 200 mmHg.


2. 2. Presión capilar pulmonar aumentada.
3. 3. Presencia de infiltrados alveolares bilaterales.
4. 4. Signos de insuficiencia respiratoria.
Gráfico de respuestas
Comentario

El síndrome de insuficiencia respiratorio del adulto se debe sospechar ante una insuficiencia
respiratoria aguda grave de curso evolutivo rápido con un patrón radiológico de infiltrado alveolar
bilateral sin cardiomegalia.

Gasométricamente se usa el cociente PaO2/FiO2 que, por alteración de la difusión se ve reducido,


siendo inferior a 200 mmHg. Es importante diferenciarlo del edema pulmonar cardiogénico, por lo
que en el distrés respiratorio del adulto la presión capilar pulmonar será normal (opción 2
incorrecta).(R2)

104. A 32-year-old woman presents with fever, malaise and altered mental status the day
she returns from a 3-week trip to Chiapas. One week after, she is brought to the
emergency department with a generalized clonic seizure. Laboratory studies show Hb 11
g/dl. Physical examination reveals splenomegaly. Diagnostic suspicion is confirmed with
thin and thick blood smear tests. Which of the following microorganisms is most likely
responsible for her illness?

1. 1. Babesia microti.
2. 2. Plasmodium ovale.
3. 3. Plasmodium malariae.
4. 4. Plasmodium falciparum.
Gráfico de respuestas
Comentario

 
 
 
 
El paludismo cerebral está producido por el P. falciparum. Es una complicación grave de la
infección por este parásito que tiene una mortalidad del 20% aún con tratamiento. El tratamiento es
con quinidina intravenosa.(R4)

105. En relación a la vulvovaginitis, indique cuál de las siguientes alternativas no guarda


correspondencia:

1. 1. Infección por monilia - gestación.


2. 2. Infección por gonococo - flujo vaginal sanguinolento.
3. 3. Infección por Tricomonas - flujo vaginal espumoso.
4. 4. Infección por Herpes - ulceración.
Gráfico de respuestas
Comentario

Pregunta de un tema muy importante. La que no guardia correspondencia es la 2: la gonorrea en la


mujer suele ser asintomática, o como mucho dar cierto flujo purulento.

Recuerden que moniliasis (respuesta 1) es otra forma de llamar a la candidiasis.(R2)

106. Una reducción importante de la capacidad de difusión del monóxido de carbono en


un paciente con una obstrucción ventilatoria marcada significa:

1. 1. Que está afectado de cor pulmonale.


2. 2. Que probablemente está afecto de un enfisema pulmonar.
3. 3. Que tiene bronquiectasias.
4. 4. Que está afecto de una bronquitis crónica moderada.
Gráfico de respuestas
Comentario

Ante un paciente con obstrucción ventilatoria, deberíamos sospechar un EPOC, ya sea en forma
de bronquitis crónica o de enfisema. Si además nos dicen que la difusión de monóxido de carbono
está reducida de forma importante, debemos decantarnos por el enfisema pulmonar (respuesta 2
correcta). Recuerda que, en la bronquitis crónica, el dato clínico típico es la tos con expectoración
abundante.(R2)

107. Durante los últimos años, se ha observado un descenso de la incidencia de


meningitis e infecciones invasivas por neumococo. ¿Qué medida, de las enunciadas a
continuación, ha sido más determinante?

1. 1. Vacunación a los niños con vacuna conjugada antineumocócica (PCV7).


2. 2. Vacunación a los adultos con vacuna conjugada antineumocócica (PCV7).
3. 3. Vacunación a los niños con vacuna polisacárida polivalente antineumocócica.
4. 4. La mejora en las condiciones de vida y un mejor uso de antibióticos.
Gráfico de respuestas
Comentario

La respuesta 1 es correcta, ya que en estudios recientes se ha concluido que la vacunación


antineumocócica masiva de la población infantil menor de 2 años ha reducido significativamente la
tasa de meningitis y enfermedad neumocócica invasiva (por los serotipos incluidos en la vacuna)
en el conjunto de la población.

 
 
 
 
Existen resultados convincentes de que la introducción de la PCV7 en la población infantil en los
EE.UU. ha reducido la incidencia de enfermedad neumocócica invasiva en los niños y en los
adultos, por lo que sería esperable que esto ocurriera en otros países.

La respuesta 2 es falsa, ya que la PCV7 tiene su indicación en niños menores de 2 años, no en


adultos. Las respuesta 3 es falsa porque hacen referencia a la vacuna polisacárida polivalente
(PPV23) que se administra en niños mayores de 2 años y adultos con enfermedades crónicas,
inmunodepresión, mayores de 65 años y personas institucionalizadas. Tiene una inmunidad poco
duradera, por lo que se recomienda en ciertos casos la revacunación cada 3-5 años, y no ha
demostrado reducir la incidencia de meningitis y enfermedad neumocócica invasiva.

La mejora en las condiciones de vida y el mejor uso de los antibióticos (respuesta 4 incorrecta) han
influido en la disminución de la incidencia de meningitis e infecciones invasivas por neumococo
pero sin duda, lo más determinante ha sido la vacunación con PCV7 en niños menores de 2
años.(R1)

108. La adrenalina es uno de los agentes más importantes que se utiliza en la reanimación
neonatal, con respecto a su uso NO es cierto lo siguiente:

1. 1. Se usa en asistolia.
Se usa si la frecuencia cardiaca se encuentra menor de 120 latidos por minuto pero con
2. 2.
una ventilación eficaz con oxigeno al 100%.
3. 3. Es útil por vía endotraqueal.
4. 4. Frecuencia cardiaca menor a 60 latidos por minuto.
Gráfico de respuestas
Comentario

En determinadas situaciones, en la reanimación del RN se recurre a la ventilación a presión


positiva intermitente y a la administración de adrenalina. El parámetro que determina la
administración de estas medidas es que la frecuencia cardíaca sea inferior a un límite definido
(respuesta 2). El uso de drogas como la adrenalina son útiles en situación de asistolia (respuesta
1) y pueden ser administradas por vía endotraqueal (respuesta 3).(R2)

109. Las alteraciones de la variabilidad del registro cardiotocográfico se caracterizan por


todo lo siguiente, EXCEPTO:

1. 1. El aumento de variabilidad constituye el signo más precoz de hipoxia fetal leve.


La hipoxia leve produce aumento de la variabilidad que a corto plazo se debe a un
2. 2.
aumento del tono simpático fetal.
3. 3. La variabilidad baja se debe frecuentemente a sueño fetal.
4. 4. El ritmo sinusoidal suele deberse a isoinmunización Rh grave.
Gráfico de respuestas
Comentario

Una pregunta bastante difícil sobre la interpretación del registro cardiotocográfico. No se preocupe
si la ha fallado.

La hipoxia leve produce un aumento de la variabilidad, pero no es debida (a corto plazo) a un


aumento del tono simpático, sino del parasimpático, lo que permite economizar glucógeno a nivel
miocárdico. Si la hipoxia persiste, entonces aparecería el aumento del tono simpático, pero siempre

 
 
 
 
en segundo lugar, contribuyendo a redistribuir el flujo sanguíneo hacia cerebro, corazón y
suprarrenales.(R2)

110. Una niña recién nacida a término, de adecuado peso, con un Apgar de 9/10, a las 4
horas de vida no ha realizado diuresis, y su creatinina sanguínea es de 1 mg/dl. Señale la
actitud correcta de manejo:

1. 1. Administrar furosemida.
2. 2. Realizar ecografía renal y de vías urinarias.
3. 3. Sondaje vesical y medición horaria de diuresis.
4. 4. Esperar, ya que no existen datos patológicos en ese momento.
Gráfico de respuestas
Comentario

La primera micción ocurre en el primer día, por lo que pasados 4 horas uno no tiene que
alarmarse. Recordad que en el recién nacido a término están reducidas la filtración glomerular y la
capacidad para concentrar la orina.(R4)

111. Un hombre de 78 años de edad, con


buen estado funcional físico y cognitivo, con antecedentes de diabetes mellitus tipo 2 y
cardiopatía en fibrilación auricular acude al servicio de urgencias de un hospital de tercer
nivel por dolor abdominal intenso, continuo, difuso, de unas 4 horas de evolución. Los
signos vitales son normales. El abdomen es blando y levemente doloroso a la palpación
sin defensa ni rebote. El ECG muestra fibrilación auricular sin otros hallazgos y la
glucemia capilar es 140 mg/dl. La imagen 1 muestra la radiografía simple del abdomen a
su llegada a urgencias. En relación con la lectura de la radiografía, señale, entre las
siguientes, la respuesta CORRECTA:

Asas de intestino delgado ligeramente dilatadas, con aire y presencia de heces y gas en
1. 1.
marco cólico.
Asas de intestino delgado dilatadas e imagen de colon cortado asociado a pancreatitis
2. 2.
aguda.
3. 3. Dilatación de asas de intestino delgado y grueso, por fecaloma en ampolla rectal.
4. 4. Asas de intestino delgado dilatadas con aire en la pared (“neumatosis intestinal”).
Gráfico de respuestas

 
 
 
 
Comentario

Una pregunta altamente intimidatoria, pero sencilla en realidad. El hecho de tener asociada una
imagen, añadido a la extensión del enunciado, hace que parezca más difícil de lo que es.

Un anciano con dolor abdominal súbito y una cardiopatía embolígena (fibrilación auricular) es uno
de los casos clínicos más típicos. Con imagen o sin él, es fácil reconocer el diagnóstico más
probable: una embolia mesentérica.

A continuación, debemos señalar la imagen radiológica más compatible. En realidad, ni siquiera


hace falta verla. Las descripciones que nos muestran son suficientemente sugestivas. La
neumatosis intestinal es un hallazgo fuertemente asociado a la embolia mesentérica y, por tanto,
sería la respuesta correcta.(R1)

Imagen: asas de intestino delgado ligeramente dilatadas, con aire y presencia de heces y gasen el marco cólico

112. ¿Cuál de las siguientes es la prueba diagnóstica más adecuada en el estudio del
dolor abdominal de este paciente?

1. 1. TC abdominal sin contraste para disminuir el riesgo de daño renal.


2. 2. Repetir radiografía simple de abdomen para descartar aire libre intraperitoneal.
3. 3. Hacer un ultrasonido abdominal para descartar patología biliar y/o renoureteral.
4. 4. Angiografía por medio de tomografía axial computarizada (angio-TC).
Gráfico de respuestas
Comentario

Una pregunta asociada a la pregunta anterior. El único problema es que precisa que tengamos una
sospecha diagnóstica correcta: la embolia mesentérica.

El diagnóstico de esta enfermedad se realiza mediante angioTC. En ella veríamos una obstrucción
brusca en la mesentérica superior. El tratamiento es quirúrgico: la embolectomía.(R4)

113. Un paciente de 61 años, antiguo bebedor importante, padece cirrosis e hipertensión


portal desde hace varios años. Refiere un aumento progresivo del perímetro abdominal
durante las últimas dos semanas, aunque está en tratamiento con espironolactona. No
dice haber aumentado el consumo de sal. No tiene fiebre ni dolor abdominal. La
exploración revela ascitis franca, aunque no a tensión. Señale la respuesta FALSA en
relación con este caso clínico:

1. 1. Está indicada la realización de una paracentesis diagnóstica.


Un gradiente seroascítico de la albúmina menor de 1,1 g/dL orientaría a hipertensión
2. 2.
portal.
Probablemente habrá que incrementar la dosis de espironolactona, e incluso añadir
3. 3.
furosemida, aunque ésta implica cierto riesgo de encefalopatía hepática.
En pacientes con sospecha de ascitis, el signo de la matidez cambiante apoya el
4. 4.
diagnóstico.
Gráfico de respuestas
Comentario

 
 
 
 
Un gradiente sero- ascítico de albúmina MAYOR de 1,1 g/dL orientaría hacia hipertensión portal.
Recuerda que este es la determinación que más nos ayuda a saber la causa de la ascitis, con una
seguridad diagnóstica del 97%.(R2)

114. Un paciente de 38 años acude a urgencias por un episodio de hematuria


macroscópica tras jugar un partido de fútbol en el que tuvo un choque frontal bastante
brusco con un jugador del equipo contrario. En la exploración destacan dos masas
abdominales palpables. En la analítica básica, el único hallazgo anormal es un
hematocrito de 54%, y en el resto de la exploración, una tensión arterial de 145/98 mm de
Hg. ¿Cuál de las siguientes respuestas le parece correcta?

La cavografía dará el diagnóstico de certeza, y el tratamiento de elección es el tratamiento


1. 1.
es la anticoagulación.
Debería solicitar una determinación de Epo. y la saturación de O2 para descartar una
2. 2.
policitemia vera.
3. 3. Es infrecuente la sobrecarga de líquidos, por la tendencia a perder sal por el riñón.
Sobre todo si el paciente es fumador, lo primero que hay que sospechar es un carcinoma
4. 4.
renal, y descartarlo con una pielografía intravenosa.
Gráfico de respuestas
Comentario

Una pregunta interesante y a la vez difícil. Se trata de una enfermedad renal poliquística del adulto,
que debemos sospechar ante las dos masas palpables, la hipertensión arterial y el aumento del
hematocrito.

La respuesta correcta es la 3, aunque pueda resultar aparentemente contradictoria con la HTA que
suele afectar a estos pacientes. Ten en cuenta que la poliquistosis es una enfermedad túbulo-
intersticial, por lo que efectivamente puede producir un síndrome pierde- sal. Lo que ocurre es que,
con el paso del tiempo, se distorsiona la morfología renal como consecuencia del crecimiento de
estos quistes, lo que acaba produciendo una insuficiencia renal por la desestructuración del
parénquima.(R3)

115. Niño de 7 años de edad que ha participado en una competencia atlética durante la
mañana, se acuesta y se levanta a la una de la madrugada en forma brusca con dolor de
garganta, dificultad para respirar, tos perruna por lo que es llevado a urgencias. ¿Cuál es
su diagnóstico?

1. 1. Absceso amigdalino.
2. 2. Laringitis.
3. 3. Epiglotitis.
4. 4. Crisis asmática.
Gráfico de respuestas
Comentario

La clínica clásica de la laringitis es: dificultad respiratoria, estridor y tos perruna. Existen dos tipos:
la viral y la espasmódica. La epiglotits es siempre infecciosa y cursa con dolor, fiebre alta y mucha
dificultad para respirar.(R2)

116. Varón de 64 años remitido a la consulta por anemia de 10,7 g/dl de hemoglobina,
detectada en un chequeo anual de su empresa. Como único antecedente destaca una
hepatopatía crónica de origen alcohólico. El hemograma confirma la anemia y la

 
 
 
 
bioquímica sólo aporta una velocidad de sedimentación de 104 mm en la primera hora
(normal <10); en el proteinograma destaca la presencia de un pico monoclonal. De los
siguientes diagnósticos, uno parece MENOS probable:

1. 1. Mieloma múltiple.
2. 2. Hipergammaglobulinemia secundaria a su hepatopatía.
3. 3. Macroglobulinemia de Waldeström.
4. 4. Linfoma no Hodgkin de bajo grado.
Gráfico de respuestas
Comentario

Una pregunta totalmente conceptual. La presencia de un pico monoclonal implica que estos
anticuerpos han sido producidos por el mismo clon celular. Por lo tanto, debemos sospechar la
existencia de un tumor o, en su defecto, una gammapatía monoclonal de significado incierto. La
hepatopatía puede justificar una hipergammaglobulinemia, pero sería policlonal.(R2)

117. En cuál de los siguientes casos, no solicitaría usted unos anticuerpos para descartar
enfermedad celíaca:

Niño de 2 años con diarrea de más de 3 semanas de evolución, fallo de medro y anemia
1. 1.
ferropénica.
2. 2. Niño de 8 años con diabetes mellitus tipo 1.
3. 3. Niño de 6 años primo hermano de un celíaco.
4. 4. Mujer de 45 años madre de un celíaco.
Gráfico de respuestas
Comentario

Los anticuerpos en la enfermedad celíaca sirven como método de cribado, de manera que deben
ser estudiados cuando existe sospecha clínica, a los familiares de primer grado de un enfermo
conocido y a todos aquéllos que sufren enfermedades asociadas (hipotiroidismo autoinmune, DM
tipo 1, déficit de IgA, dermatitis herpetiforme…), también en niños con diarrea, falla de medro y
anemia ferroénica. Respuesta 3 correcta.(R3)

118. Acude a su consulta una paciente de 22 años consultando por amenorrea desde
hace un año. Menarquia a los 14 años con reglas cada 29 días de 5 días de duración hasta
hace un año. Las determinaciones hormonales son las siguientes: prolactina 18 ng/ml,
FSH 3 mUI/ml, LH 1 mUI/ml, estradiol 13 pg/ml, progesterona 0.3 ng/ml. Tras la
administración de acetato de noretisterona 5 mg durante 10 días no consigue
menstruación. Con un preparado anticonceptivo con 30 µg de etinilestradiol y 3 mg de
drospirenona se consigue menstruación al suspenderlo. Respecto a la amenorrea
secundaria que tiene la paciente, es CIERTO que:

1. 1. La causa de la amenorrea se debe a etiología ovárica.


2. 2. En la ecografía, el endometrio está atrófico.
3. 3. Se debe pedir RMN de silla turca para descartar la presencia de un prolactinoma.
4. 4. El IMC de la paciente suele estar por encima de 18.
Gráfico de respuestas
Comentario

Caso clínico en que se nos cuenta una amenorrea secundaria en la que sólo con los datos de
laboratorio podemos establecer el diagnóstico. Tenemos una paciente con un hipogonadismo

 
 
 
 
(estradiol <20 pg/ml) hipogonadotropo (FSH y LH <5 mUI/ml), por lo que la causa es central y no
ovárica (hipergonadotropo), por tanto 1 es falsa. Al no tener estradiol, el endometrio no se estimula
y ecográficamente se ve atrófico. En las amenorreas centrales se puede realizar una RM de silla
turca para descartar adenomas de hipófisis, pero en este caso no buscamos un prolactinoma (>100
ng/ml) ya que el valor de la prolactina es normal (<20 ng/ml). Estas amenorreas centrales suelen
deberse a bajo peso de la paciente, entrando en amenorrea secundaria cuando su IMC es <18 o la
masa grasa <23%.(R2)

120. Un paciente varón de 45 años que presenta asma, astenia, anorexia, malestar
general, así como lesiones cutáneas nodulares, con presencia de infiltrados pulmonares
cambiantes en la radiografía de tórax y eosinofilia periférica en la analítica, nos debe
hacer sospechar el diagnóstico de:

1. 1. Panarteritis nodosa.
2. 2. Enfermedad de Churg-Strauss.
3. 3. Granulomatosis de Wegener.
4. 4. Vasculitis leucocitoclástica.
Gráfico de respuestas
Comentario

La enfermedad de Churg- Strauss es una vasculitis sistémica necrotizante granulomatosa que


afecta a numerosos órganos, fundamentalmente al pulmón. Afecta a vasos de pequeño y mediano
calibre, incluidas vénulas. El paciente inicialmente, antes de la aparición del resto de
manifestaciones clínicas suele presentar un asma grave, con crisis severas que requieren
tratamiento corticoideo, así como rinitis. Más adelante, aparece el resto de sus menifestaciones,
con clínica general de fiebre, pérdida de peso, afectación renal, cardíaca, neuropatía, lesiones
cutáneas, etc. La afectación pulmonar domina el cuadro, con aparición de infiltrados migratorios y
las crisis asmáticas mencionadas. No olvides que, en el hemograma, es muy característica la
presencia de eosinofilia.(R2)

121. L.B. es una mujer de 35 años con historia de 3 meses de evolución de dolores
articulares que le afectan manos, hombros y, en menor grado, rodillas y pies. Refiere
rigidez matutina de 3 horas de evolución, astenia intensa hasta el encamamiento,
sudoración nocturna y pérdida de 3 kg de peso. En la exploración presenta tumefacción
articular dolorosa en las articulaciones interfalángicas proximales (IFP), metacarpo-
falángicas (MCF), muñecas, hombros, rodillas y articulaciones metatarsianas (MTF).
Tiene adenopatías periféricas cervicales y axilares indoloras no adheridas, así como un
nódulo en la superficie de extensión del antebrazo izquierdo. Los análisis de laboratorio
muestran VSG 55 mm/hora, FR 1:1280, Hb 10,5 g/dl y aumento de proteínas totales a
expensas de hipergammaglobulinemia policlonal. Las radiografías de manos y muñecas
mostraron osteopenia yuxtaarticular, así como erosiones marginales precoces,
afectando la 4ª IFP de la mano derecha. Fue iniciado tratamiento con AINE, prednisona
en dosis bajas (5 mg/día) y metotrexato (7,5 mg/semana). En esta paciente, la forma de
presentación de la enfermedad induce a tratar precozmente y de forma agresiva la
enfermedad. Por favor, señale cuál de los siguientes enunciados NO es correcto:

Los pacientes femeninos de raza blanca cursan con sinovitis de carácter más persistente y
1. 1.
enfermedad erosiva más rápidamente progresiva que los varones.
La presencia de nódulos subcutáneos, como en el caso descrito, así como la de erosiones
2. 2.
radiológicas, es índice de severidad pronóstica.

 
 
 
 
El patrón de presentación de la enfermedad no parece predecir el desarrollo de
3. 3.
deformidades.
Aproximadamente el 40% de los pacientes con artritis reumatoide cursan con un proceso
4. 4.
inflamatorio agudo que remite sin deformidades mayores.
Gráfico de respuestas
Comentario

La descripción de la paciente deja claro que se trata de un AR, al cumplir todos los criterios
propuestos para el diagnóstico de la enfermedad. La paciente muestra varias características
sugestivas de mala evolución, como el sexo femenino y la raza blanca. Efectivamente, tiene
tendencia a producir sinovitis más persistente y mayor frecuencia de erosiones, la elevación de la
VSG, la presencia de erosiones, nódulos reumatoides y título elevados de FR son también datos
de agresividad y mal pronóstico. No obstante, la evolución de la AR es muy variable y difícil de
predecir en cada paciente, de forma que el patrón de inicio del proceso no parece tener valor
predictivo respecto a la aparición de incapacidad. Las formas autolimitadas no suponen,
lamentablemente, más que el 15% de los casos (respuesta 5 falsa).(R4)

122. Señale cuál de las siguientes infecciones congénitas puede producir síndrome
nefrótico congénito:

1. 1. Citomegalovirus.
2. 2. Rubéola.
3. 3. Sífilis.
4. 4. Hepatitis B.
Gráfico de respuestas
Comentario

La sífilis congéntita tardía puede dar lugar a una tríada (la de Hutchinson, caracterizada por
queratitis, hipoacusia neurosensorial y alteraciones dentarias), a alteraciones neurológicas (paresia
juvenil, tabes juvenil), a lesiones óseas deformantes (molares en mora, dientes de Hutchinson, tibia
en sable, frontal prominente) y también a un síndrome nefrótico.(R2)

123. En relación con el tratamiento de la hipertensión portal, indique cuál de las


siguientes afirmaciones es INCORRECTA:

La profilaxis primaria de la hemorragia digestiva alta por varices esofágicas se realiza con
1. 1.
técnicas de esclerosis.
El propranolol puede asociarse a vasodilatadores venosos, y estos últimos constituyen una
2. 2.
alternativa cuando está contraindicado el propanolol.
Dentro de las técnicas quirúrgicas, la anastomosis esplenorrenal distal o técnica de
3. 3. Warren es una técnica selectiva, conserva el flujo hepático y no conduce a la
encefalopatía hepática como las técnicas no selectivas.
En general, la cirugía de la hipertensión portal disminuye el riesgo de sangrado, pero la
4. 4.
mortalidad de los pacientes no varía.
Gráfico de respuestas
Comentario

La profilaxis primaria de la hemorragia digestiva alta por varices se realiza con revisiones
periódicas únicamente (varices pequeñas) o con betabloqueantes (varices grandes), que son el
método de elección. Sólo se utilizan técnicas endoscópicas si los betabloqueantes están
contraindicados, y en este caso se utiliza la ligadura en bandas, y no la esclerosis (respuesta 1
falsa).(R1)

 
 
 
 
124. Varón de 50 años consulta por un bulto en el cuello. En la exploración aparece un
nódulo firme, duro, en lóbulo derecho de unos 2 cm de diámetro. Las hormonas tiroideas
son normales. En la gammagrafía existe un área que no capta y que coincide con el
nódulo. La PAAF del nódulo muestra abundante celularidad, así como folículos y escaso
coloide. ¿Cuál sería la actitud con respecto al paciente?

1. 1. Seguimiento periódico para descartar crecimiento.


2. 2. No hacer nada, puesto que se trata de un adenoma folicular y es benigno.
Cirugía del nódulo para descartar carcinoma folicular mediante la demostración de
3. 3.
invasión vascular.
4. 4. Ecografía tiroidea para ver si es sólido o quístico.
Gráfico de respuestas
Comentario

El carcinoma folicular representa el 15- 20% de los tumores tiroideos (el más frecuente es el
papilar, no olvides este detalle). Tiene tendencia a presentarse en sujetos de edad avanzada.
Anatomopatológicamente, se asemeja al epitelio tiroideo normal, es encapsulado y sólo se
diferencia del adenoma folicular benigno si hay invasión de la cápsula o invasión vascular.
Por lo tanto, ante una PAAF con celularidad folicular, debe realizarse una resección del nódulo
para descartar este tipo de cáncer, ya que no existe otra forma de distinguirlo del adenoma.

El carcinoma folicular se propaga rápidamente por vía hemática y el paciente puede presentar
metástasis en pulmón, hueso (osteolíticas) o sistema nervioso central. Las metástasis pueden
llegar a producir hiperfunción tiroidea por el exceso de producción de T4 y T3 (aunque esto es
raro). Un subtipo de carcinoma folicular, el carcinoma de células de Hürthle, tiende a ser más
invasor y menos radiosensible.(R3)

125. Con respecto a la neoadyuvancia en el cáncer de recto, indique la respuesta


INCORRECTA:

1. 1. Está indicada cuando el tumor afecta a las vesículas seminales.


2. 2. Está indicada cuando existen adenopatías positivas.
3. 3. Si se da quimioterapia preoperatoriamente, ya no hay que darla en el postoperatorio.
En algunos casos permite hacer resecciones anteriores en pacientes en los que, de entrada,
4. 4.
estaba indicada una amputación abdominoperineal.
Gráfico de respuestas
Comentario
Parece existir evidencia suficiente para recomendar la neoadyuvancia (es decir el tratamiento
médico con radio o quimioradioterapia) en el cáncer de recto en estadio II (T3: afectación fascia
propia del recto y T4: afectación de estructuras vecinas) y III (adenopatías positivas con cualquier
T). El efecto fundamental de la neoadyuvancia lo realiza la radioterapia, disminuyendo las recidivas
locales. En este tumor la quimioterapia preoperatoria, que no siempre se da, tiene un efecto
“sensibilizador” para la radioterapia, es decir la hace más efectiva. Por eso, el haber recibido
quimioterapia en el preoperatorio, en el recto, a diferencia de otros cánceres como el de mama, no
evita su administración de forma adyuvante si está indicada (B2 con factores de riesgo o C y D de
la clasificación de Dukes). La neoadyuvancia permite, en caso de respuestas, realizar Cirugías
radicales (R0: sin restos de enfermedad) y aumenta la tasa de conservación de esfínteres, siendo
en la actualidad poco frecuente tener que realizar una amputación abdomino perineal (sólo se hace
si no hay respuesta, si hay afectación esfinteriana franca, incontinencia previa del paciente o
tumores muy bajos que no permiten margen libre distal).(R3)

126. Señale la afirmación FALSA respecto a la angioplastia coronaria:

 
 
 
 
1. 1. El índice de éxitos es del 90%.
2. 2. La mortalidad es menor del 1%.
Si se implanta un stent tras el procedimiento se ha demostrado disminución en la
3. 3.
incidencia de reestenosis hasta sólo el 20%.
Tras sufrir una reestenosis, si se vuelve a realizar ACTP, se mantienen los índices de
4. 4. riesgo, pero el éxito de la intervención se ve mermado por al alteración anatómica que
origina el procedimiento previo.
Gráfico de respuestas
Comentario

Existe un espectacular avance en las técnicas de revascularización percutánea, que tiene un


porcentaje de éxito del 90% con una mortalidad inferior al 1%. El principal inconveniente de la
angioplastia es la reestenosis, que se produce en los seis meses siguientes al procedimiento. El
índice de reestenosis es del 30 al 45% cuando se realiza angioplastia simple, reduciéndose al 15-
25% cuando se implantan endoprótesis (stents). En los últimos tiempos, se están empezando a
implantar stents recubiertos con fármacos inmunosupresores o antiproliferativos (rapamicina, taxol,
etc.) que reducen significativamente la tasa de reestenosis a corto y largo plazo. Cuando se
produce una reestenosis, el porcentaje de éxitos del procedimiento posterior es igual que en el
procedimiento inicial.(R4)

127. El paciente A presenta una serología de sífilis con los siguientes resultados: FTA
positivo; RPR positivo 1/2. El paciente B presenta una serología con los siguientes
resultados: TPHA negativo; VDRL positivo 1/64. Ninguno de los dos pacientes presenta
clínica sugestiva de ninguna de las fases de la sífilis. Señale la opción que considera
CORRECTA:

1. 1. Paciente A: sífilis tratada y curada; Paciente B: falso negativo de la prueba treponémica.


2. 2. Paciente A: sífilis tratada y curada; Paciente B: falso positivo de la prueba reagínica.
3. 3. Paciente A: sífilis latente; Paciente B: sífilis tratada y curada.
4. 4. Paciente A: ambas pruebas falsamente positivas; Paciente B: sífilis primaria.
Gráfico de respuestas
Comentario
Las pruebas TPHA y FTA son treponémicas y las pruebas RPR y VDRL son reagínicas. Las
primeras son cualitativas (+/- ) y pueden permanecer positivas durante toda la vida a pesar de
tratamiento correcto. Las segundas son cuantitativas (se consideran habitualmente positivas a
partir de la dilución 1/16) y se negativizan al cabo del tiempo si el tratamiento es correcto. Las
reagínicas se pueden positivizar en el contexto de cualquier situación en la que se active el sistema
inmunológico y por eso presentan una tasa mucho mayor de falsos positivos (por ejemplo en
sujetos infectados por VIH, en pacientes con enfermedades autoinmunes, en mujeres
embarazadas, después de la administración de una vacuna, etc). En la paciente A la prueba
treponémica es positiva y la reagínica negativa (la dilución es menos de 1/16), por lo que lo más
razonable es que se trate de un paciente que en su momento recibió tratamiento frente a T
pallidum: al cabo del tiempo FTA sigue siendo positiva pero la reagínica – RPR – se ha
negativizado. En el caso del paciente B, la negatividad de la prueba treponémica indica que la
positividad de la reagínica se trata de un falso positivo.(R2)

128. Señale cuál de los siguientes compuestos aumenta la producción de bicarbonato


gástrico:

1. 1. Ácido acetilsalicílico.
2. 2. Ranitidina.
3. 3. Acetazolamida.

 
 
 
 
4. 4. Prostaglandinas.
Gráfico de respuestas
Comentario

La homeostasis gástrica depende fundamentalmente de las prostaglandinas. Esto explica que el


empleo de AINES condicione una reducción de las mismas y, por tanto, menos bicarbonato
disponible, mayor acidez, una barrera mucosa gástrica de peor calidad y, por tanto, riesgo de
ulceración.

En cuanto a la ranitidina, precisamente tendría un efecto protector, ya que disminuye la secreción


ácida gástrica por ser antagonista de los receptores histamínicos H2. Por último, el alcohol produce
una lesión directa sobre la pared gástrica, que es lo que justifica la gastritis alcohólica.(R4)

129. El aporte calórico mínimo para un desnutrido severo en la fase I o transicional, es


de:

1. 1. 150 cal/Kg/Día.
2. 2. 100 cal/Kg/Día.
3. 3. 80 cal/Kg/ Día.
4. 4. 50 cal/Kg/Día.
Gráfico de respuestas
Comentario

Pregunta memorística, no se preocupe si la ha fallado. El aporte necesario es de 100, respuesta


correcta 2.(R2)

130. Paciente de 4 años de edad, es llevado a urgencias por tos y polipnea. Antecedente
de síndrome obstructivo bronquial a repetición, rinitis y urticaria. Al exámen físico: FR 64
por minuto, FC 150 por minuto, presenta tirajes, sibilancias y estertores. ¿Cuál sería su
presunción diagnóstica?

1. 1. Bronquiolitis.
2. 2. Insuficiencia cardíaca congestiva.
3. 3. Asma.
4. 4. Reflujo gastroesofágico.
Gráfico de respuestas
Comentario

Debe dominar el Asma para el ENARM. El cuadro que nos describen es el típico de asma (tos,
taquicardia, tirajes, sibilancias...). Los antecedentes de rinitis y urticaria apoyan este diagnóstico.
Respuesta 3 correcta.(R3)

131. Señale lo INCORRECTO en relación a los pseudoquistes pancreáticos:

1. 1. El mejor método diagnóstico es la TC.


2. 2. Surgen 1 semana después del episodio agudo de pancreatitis.
3. 3. Pueden resolverse espontáneamente.
4. 4. Son las lesiones quísticas más frecuentes del páncreas.
Gráfico de respuestas
Comentario

 
 
 
 
Los pseudoquistes son la complicación más frecuente de una pancreatitis aguda, sin embargo la
causa más frecuente de pseudoquistes es la pancreatitis crónica cuya etiología es principalmente
el alcoholismo. El mejor método diagnóstico para las lesiones quísticas o sólidas del páncreas es el
TAC Abdominal. El momento de aparición de los pseudoquistes en relación con el episodio agudo
de pancreatitis es variable entre la 1 y la 4 semana. Los pseudoquistes aparecidos en el contexto
de una pancreatitis aguda deben dejarse evolucionar salvo complicaciones durante al menos 6
semanas pues en este periodo de tiempo un alto porcentaje de ellos desaparecen. Los
pseudoquistes son las lesiones quísticas más frecuentes del páncreas (hasta el 75 % de todas las
lesiones quísticas del páncreas).(R2)

132. Una niña de un mes y 8 días de vida es remitida a su consulta por presentar estrías
de sangre en las heces desde hace 3 días. Presenta entre 3 a 5 evacuaciones al día,
amarillentas y de consistencia normal. Es regurgitadora habitual y ocasionalmente
presenta algún vómito postprandial. Refieren apetito conservado, que ha presentado
algún episodio de llanto aislado por la noche en la última semana y que ha estado afebril
en todo momento. Está alimentada con lactancia materna exclusiva desde que nació y ha
presentado una buena ganancia ponderal. La exploración física no revela ningún dato de
interés. AP: Recién nacida a término (38+, peso al nacimiento 3,460 gr, Apgar 8/9. Precisó
fototerapia durante 24 horas al tercer día por hiperbilirrubinemia (BRT 18.5 mg/dl).
Pruebas metabólicas y cribado auditivo normal. Ha recibido una dosis de la vacuna de la
hepatitis B. AF: padre 28 años intervenido de apendicitis a los 8 años. Madre 27 años (G1
A0 V1) con antecedente de dermatitis atópica. Respecto a la enfermedad que sospecha,
es CIERTO que:

El Helicobacter pylori es el agente infeccioso implicado con mayor frecuencia, y el


1. 1.
tratamiento antibiótico ha cambiado radicalmente el curso de esta enfermedad.
El hecho de recibir lactancia materna exclusiva no descarta una intolerancia a proteínas
2. 2. de leche de vaca cuyo tratamiento sería sustituir la alimentación por una fórmula sin estas
proteínas, por ejemplo la leche de soya.
La colitis es una manifestación muy rara de la intolerancia/alergia a proteínas de leche de
3. 3.
vaca.
El pronóstico vital y funcional es realmente muy bueno, con una resolución completa de
4. 4.
los síntomas en los próximos años de su vida.
Gráfico de respuestas
Comentario

De todas las formas clínicas de intolerancia/alergia a proteínas de leche de vaca ha aumentado


mucho la incidencia de la colitis cuyo síntoma principal es la presencia de hilos/hebras de sangre
en las heces en un niño por lo demás normal.

La edad de aparición suele ser entre las 2-8 semanas de vida. No siempre estarán alimentados
con fórmula artificial (hasta un 60% reciben lactancia materna exclusiva) pero el diagnóstico se
establece al desparecer la clínica tras exclusión de las proteínas de leche de vaca: ya sea por
sustitución con fórmula hidrolizada o por retirada de lácteos de la dieta materna mientras dure la
lactancia.

No es necesario hacer biopsia salvo en casos refractarios. La fórmula de soya tiene un 30% de
reactividad cruzada con las proteínas de leche de vaca, por lo que la clínica puede persistir. 25%
suele presentar antecedentes familiares de atopia. Las pruebas cutáneas de alergia pueden o no
ser positivas. La gran mayoría serán tolerantes hacia los 2-3 años de vida.(R4)

 
 
 
 

133. En los exámenes de


laboratorio de un masculino de 47 años, en seguimiento en urología por impotencia, se
detectan niveles de testosterona de 1 ng/ml (normal 3-10 ng/ml), siendo remitido a
endocrinología para estudio. El reso de laboratorios destaca PRL de 1,759 ng/ml,
solicitándose a continuación una RMN (ver imagen). Dada la patología que presenta el
paciente señale lo CORRECTO:

Si el paciente estuviera tomando un fármaco que produjera hiperprolactinemia, sería


1. 1.
recomendable retirarlo y reevaluar antes de emitir el diagnóstico.
Dado el tamaño de la lesión y los niveles hormonales, lo más probable es que el tumor
2. 2.
produzca hiperprolactinemia por compresión.
Si el paciente presentara ginecomastia, se debería a la acción de la prolactina elevada
3. 3.
sobre el tejido glandular mamario.
Aunque no existan déficit visuales en la anamnesis, será necesario derivarlo a
4. 4.
oftalmología para realización de campimetría.
Gráfico de respuestas
Comentario

Dada la clínica del paciente (impotencia), las cifras de prolactina y la imagen radiológica, habría
que considerar un macroprolactinoma como primer diagnóstico. Recuerde que los prolactinomas
son más frecuentes en mujeres, pero cuando aparecen en varones suelen ser mayores, ya que
tardan más tiempo en producir síntomas, por lo que tienden a consultar más tarde.

Dado el enorme tamaño de la lesión, habría que descartar afectación del campo visual (respuesta
correcta 4). Es posible que el paciente no las refiera, pero eso no quiere decir que no existan, ya
que la compresión del quiasma óptico suele producir una hemianopsia heterónima bitemporal, es
decir, una pérdida de campo periférico. En muchos casos, estas alteraciones campimétricas pasan
desapercibidas o son muy sutiles, al menos de inicio, por lo que debemos descartarlas mediante
una campimetría.(R4)

134. Respecto a la patología del paciente anterior, señale lo FALSO:

Siempre se deben tratar aquellos tumores mayores de 1 cm, independientemente de las


1. 1.
manifestaciones clínicas.
2. 2. La radioterapia puede ser un tratamiento coadyuvante a la cirugía.

 
 
 
 
Recientemente se ha descrito la presencia de insuficiencia valvular cardiaca en pacientes
3. 3. tratados con dosis altas de cabergolina, aunque su aparición en el caso del tratamiento de
la hiperprolactinemia no está clara.
En aquellos pacientes con respuesta al tratamiento médico es rara la recurrencia de la
4. 4.
enfermedad.
Gráfico de respuestas
Comentario

El tratamiento de elección de los prolactinomas es inicialmente médico, mediante análogos de la


dopamina. Recuerde que la dopamina era el enigmático PIF (factor inhibidor de la prolactina), del
que se hablaba antiguamente, y que después resultó ser esta sustancia. Lo habitual es que el
tratamiento médico sea suficiente para controlar la enfermedad, pero eso no significa que las
recurrencias no sean posibles. De hecho, son relativamente frecuentes y, por ello, debemos
realizar determinaciones periódicas de prolactina, para detectar precozmente una posible recidiva.

El resto de las opciones son ciertas y representan principios básicos a tener en cuenta en el
tratamiento del prolactinoma. No se preocupe si no sabía lo que dice la respuesta 3. No obstante,
ya sabe que, en Medicina, un “puede”, “se ha descrito”, o “algunos autores consideran” suelen ser
ciertas.(R4)

135. Respecto a las diferencias entre la leche de vaca y la leche humana, es FALSO que:

1. 1. La leche de vaca es más rica que la leche humana en sales minerales.


2. 2. La leche de vaca es más pobre que la leche humana en cuanto a cantidad de vitamina K.
La composición proteica de la leche de vaca y humana difiere, estando en esta última
3. 3.
compuesta hasta en un 70% de seroproteínas.
4. 4. La leche humana tiene menos calcio que la leche de vaca.
Gráfico de respuestas
Comentario

Debes conocer las principales diferencias de composición entre la leche humana y de vaca:

La leche de vaca en comparación con la leche humana contiene más: Sales minerales (iones),
vitamina B y K, calcio y proteínas.

La leche humana tiene más: vitaminas C y D, ácidos grasos de cadena larga.

El calostro (2- 4 primeros días) tiene más proteínas, sales minerales y factores inmunitarios.

La leche de inicio (para los 6 primeros meses) es más pobre que la de continuación (6 meses en
adelante) en calcio, hierro y proteínas.(R2)

136. A un niño febril de 16 meses de edad se le realizó una punción lumbar por sospecha
de meningitis. ¿Qué característica se considera anormal en el líquido cefaloraquídeo?

1. 1. Proteínas de 15 mg/dl.
2. 2. Xantocromía.
3. 3. Glucorraquia equivalente a la mitad de la glicemia.
4. 4. Dos leucocitos.
Gráfico de respuestas
Comentario
 
 
 
 
En la sospecha de meningitis, uno de los hallazgos de sospecha de origen bacteriano es el
consumo de glucosa por las bacterias en el LCR, que suele encontrarse por debajo del 50% del
valor sanguíneo al momento de la punción.(R3)

137. ¿Qué haría si un recién nacido no presenta movimientos respiratorios luego de


aspirarlo, colocarlo en posición adecuada y estimularlo táctilmente?.

1. 1. Obtener puntaje de APGAR.


2. 2. Iniciar ventilación con presión positiva con ambú.
3. 3. Ofrecer oxigeno a flujo libre.
4. 4. Evaluar la frecuencia cardiaca.
Gráfico de respuestas
Comentario(R2)

 
 
 
 

138. Señale la afirmación FALSA, de entre las siguientes, respecto a la isoinmunización


Rh:

1. 1. La enfermedad hemolítica grave suele afectar al segundo hijo.


2. 2. Suele cursar con ictericia, que aparece en las primeras 24 horas de vida.
Debe realizarse profilaxis con gammaglobulina específica anti-D antes de que pasen 72
3. 3.
horas del parto, cuando el test de Coombs directo es positivo.
4. 4. El 90% de los casos de incompatibilidad se deben al antígeno D.
Gráfico de respuestas
Comentario

 
 
 
 
La prevención de la isoinmunización anti-D se hará sólo si la madre no está sensibilizada, esto es,
si su test de Coombs indirecto es NEGATIVO. Respuesta 3 falsa.(R3)

139. A butcher 47 year-old


woman comes to the physician because of a three-year history of hand weakness. She
also experiences numbness associated with the handling of cold meat. She also refers
four casual fallen over the last year. Physical examination reveals eyelid ptosis, neck
muscle and feet weakness. Osteotendinous reflexes are normal. Asking the patient to
open her hands after a few seconds closed she do it very slowly with difficulties, as it is
shown in the picture. Which of the following is the most likely diagnosis?

1. 1. Duchenne muscular dystrophy.


2. 2. Myotonic dystrophy.
3. 3. Congenital myopathy.
4. 4. Polymyositis.
Gráfico de respuestas
Comentario

En la pregunta se nos presenta un caso típico de distrofia miotónica tipo I o de Steinert.

Dicha entidad constituye un trastorno neuromuscular caracterizado por atrofia muscular y debilidad,
asociado a miotonía y alteraciones sistémicas.

El cuadro más característico es el de una enfermedad que comienza en la adolecencia o juventud


con debilidad importante de las manos y, frecuentemente, pie caído. Es una de las infrecuentes
formas de distrofia muscular con afectación predominantemente distal, así como de los músculos
del cuello.

Los pacientes muestran una facies característicamente alargada y con expresión de tristeza,
acentuada por el hundimiento de las sienes secundario a la atrofia de maseteros y temporales,
siendo común la luxación de mandíbula. Es común la ptosis bilateral y según avanza la
enfermedad la debilidad se extiende a regiones proximales. En individuos de mediana edad, la
debilidad de caderas y piernas suele ser importante, condicionando frecuentes caídas. Es
característica la asociación con el fenómeno de miotonía, o dificultad para la relajación muscular.
Es especialmente ostensible en las manos (dificultad para relajar el apretón de manos) y se
demuestra por percusión muscular con el martillo (clásicamente en la eminecia tenar). La imagen a
la que se refiere la pregunta muestra dicho fenómeno.

 
 
 
 
La debilidad y la miotonía empeoran con el frío. La miotonía congénita (opción 3) ocurre en el
primer año de vida de hijos de madres afectas de distrofia miotónica, y cursa con retraso mental,
pie zambo, alteraciones faciales e infecciones respiratorias de repetición. El resto de opciones no
cursan con miotonía.(R2)

140. A 10-day-old infant is brought to the doctor's office presenting with persistent vomits
and diarrhea. He had an uncomplicated vaginal delivery at 40 weeks' gestation. Physical
examination reveals yellow skin and sclera and an enlargement of his liver and spleen.
His mother is quite anxious because the child had a seizure that morning. The crystalline
lenses of his eyes seem opaque. Which of the following is more consistent with these
findings?

1. 1. It is a case of galactokinase deficiency


2. 2. It is a case of galactose 1 phosphate uridyl transferase deficiency
3. 3. Speech and languaje problems are uncommon
4. 4. Cataracts are irreversible even when an appropriate treatment is given
Gráfico de respuestas
Comentario
It is a case of galactose-1-phosphate uridyl transferase deficiency. The child in this case suffers
from galactosemia, a congenital defect of galactose metabolism due to a deficiency in the enzyme
Galactose-1-phosphate uridylyltransferase. General symptoms of the newborn may be unespecific
(jaundice, diarrhea, loss of appetite…) but there is an associated pathognomonic sign: congenital
cataracts, which are reversible after galactose restriction. Galactosemia when caused by
galactokinase deficiency presents with congenital cataracts as an only symptom, with no systemic
manifestations.(R2)

141. ¿Cuál de los siguientes perfiles analíticos puede corresponder a una cetoacidosis
diabética? (G: glucosa, Na: sodio, B: bicarbonato, Cet: cetonuria):

1. 1. G:420 mg/dl, Na: 133 mEq/l, B: 10 mEq/l, pH: 7,15, Cet: positiva.
2. 2. G: 850 mg/dl, Na: 148 mEq/l, B: 15 mEq/l, pH: 7,2, Cet: negativa.
3. 3. G: 300 mg/dl, Na: 138 mEq/l, B: 25 mEq/l, pH:7,4, Cet: positiva.
4. 4. G: 400 mg/dl, Na: 148 mEq/l, B: 25 mEq/l, pH: 7,41, Cet: negativa.
Gráfico de respuestas
Comentario

Una pregunta sencilla, que además puede resolverse por exclusión. La cetoacidosis diabética es
una complicación típica de la diabetes tipo 1. Para que aparezca, debe existir un déficit de insulina,
con aumento de las hormonas contrarreguladoras, sobre todo el glucagón. Bioquímicamente, se
define como: glucemia mayor de 300 mg/dl, cuerpos cetónicos positivos en orina, acidosis
metabólica con anión gap elevado y disminución del bicarbonato plasmático. Conociendo esto,
analicemos las posibles respuestas:

Las opciones 2, 4 y 5 quedan descartadas. Sin cetonuria, sería muy osado hablar de una
cetoacidosis diabética... Con esto, sólo ofrecerían duda la 1 y la 3.

La opción 3 es incorrecta, porque un pH de 7.4 con un bicarbonato normal (25) descarta la


presencia de acidosis metabólica. Recuerda que el concepto de acidosis metabólica EXIGE
disminución del bicarbonato sérico.

Con esto, vemos que sólo la respuesta 1 entraría en la definición.(R1)

 
 
 
 
142. Señale la FALSA respecto a la fiebre tifoidea:

El norfloxacino es capaz de erradicar el estado de portador, incluso si existen litiasis


1. 1.
biliares.
2. 2. La enfermedad suele contraerse por la ingesta de alimentos o agua contaminados.
3. 3. La leucopenia es más común que la leucocitosis.
4. 4. Las "manchas rosadas" suelen manifestarse cuando empieza la fiebre.
Gráfico de respuestas
Comentario

Las infeciones del tracto gastro intestinal son muy preguntas.

Recuerda que en la fiebre tifoidea suelen ser mas los síntomas sistémicos que la propia diarea.

Puesto que hace referencia a un pequeño detalle que es difícil apreciar. Las “manchas rosadas”
aparecen en coincidencia con el pico máximo de fiebre, no al comienzo de ésta (respuesta 4 falsa).
Un exantema parecido, también rosado y coincidente con los picos febriles, es el que aparece en la
enfermedad de Still del adulto.(R4)

143. Respecto a la ictericia neonatal, todas las siguientes afirmaciones son ciertas,
EXCEPTO:

1. 1. En los prematuros, la ictericia fisiológica aparece en las primeras 24 horas.


A los 15 días de vida, en un RN tanto si fue a término como pretérmino, la bilirrubina
2. 2.
indirecta debe ser <1 mg/dl.
3. 3. La ictericia por incompatibilidad ABO puede ocurrir en el primer hijo.
4. 4. La hipoglucemia y la acidosis aumentan el riesgo de kernicterus.
Gráfico de respuestas
Comentario

No debe de terminar de leer la primera respuesta para saber que es la mala (aún así debe leer
todas). La hiperbilirrubinemia es un método que, según teorías recientes, sirve para proteger al
neonato frente al estrés provocado por los radicales libres. Por este motivo, en mayor o menor
medida, como una defensa natural, todos los neonatos presentan ictericia fisiológicamente. Ésta
comienza en los RNT el 2º día y en los RNPT el tercero. Se prolonga aproximadamente hasta el
día 7 de vida en los RNT y un par de días más en los RNPT. A partir de entonces, la cifra de
bilirrubina se normaliza. Las otras opciones son correctas: la isoinmunización ABO no precisa
sensibilización previa (se da por anticuerpos naturales tipo IgG), y la permeabilidad de la barrera
hematoencefálica es mayor en el neonato gravemente enfermo (séptico, acidótico,...).(R1)

 
 
 
 

144. Paciente masculino de 45


años remitido a las consultas de endocrinología por hallazgo en analítica de colesterol
total de 670 mg/dl con cifras de triglicéridos normales. Entre sus antecedentes familiares
destaca fallecimiento de su padre a los 40 años de edad por infarto agudo de miocardio.
A la exploración física, presenta las lesiones representadas en la imagen. Respecto a la
patología del paciente, señale la respuesta falsa.

Se trata de una enfermedad con herencia AD, por lo que existirán varios familiares
1. 1.
afectos.
2. 2. El diagnóstico definitivo puede realizarse mediante estudio genético del receptor de LDL.
Existe un aumento del riesgo de padecer enfermedades cardiovasculares en las personas
3. 3.
afectas.
Algunos pacientes pueden presentar un arco corneal grisáceo, que aparecerá típicamente
4. 4.
por encima de los 60 años.
Gráfico de respuestas
Comentario
El paciente de la pregunta presenta una Hipercolesterolemia Familiar. Llegamos a este diagnóstico
por la presencia de cifras altas de colesterol total = 290 mg/dl o LDL = 190 mg/dl, especialmente si
cuenta con historia familiar de enfermedad cardiovascular precoz. Además la presencia de
xantomas y xantelasmas “manchas induradas blanquecinas en codos, rodillas y párpados” apoya
el diagnóstico. Es típica también la presencia de un arco corneal senil prematuro (antes de los 40
años). Esta enfermedad se produce cuando el paciente hereda de alguno de sus progenitores una
mutación en el receptor de LDL. Debido a que la enfermedad tiene herencia autosómica
dominante, es frecuente que los pacientes presenten varios familiares de primer y segundo grado
afectos. La ausencia de receptores de LDL altera el aclaramiento de las lipoproteínas que se basan
en el receptor de LDL, estas incluyen LDL e IDL. Esto provoca un incremente de 2- 3 veces
superior de colesterol en individuos heterocigotos y de 3- 6 veces en homocigotos. No se produce
aumento de la cifra de triglicéridos.(R4)

145. Respecto al tratamiento del caso anterior:

1. 1. No es preciso el inicio del tratamiento médico hasta alcanzados los 35 años.


La modificación de la dieta de forma aislada, provoca grandes disminuciones de las cifras
2. 2.
de colesterol (entre 70-80% del total).
El tratamiento farmacológico de elección son las estatinas, aunque en algunos de los
3. 3. casos se necesitará la adición de otros fármacos para conseguir un control adecuado en las
cifras de colesterol.
4. 4. El ácido nicotínico, nunca está indicado por sus efectos secundarios.
Gráfico de respuestas
 
 
 
 
Comentario

El tratamiento de la hipercolesterolemia familiar consiste en una dieta baja en grasas totales,


saturadas y baja en colesterol. Las modificaciones de la dieta provocan normalmente sólo leves
disminuciones en los niveles plasmáticos de colesterol (5- 15%), por lo que la adición de fármacos
va a ser obligatorio. Los de elección en la hipercolesterolemia familiar son las estatinas, que en
algunas ocasiones consiguen objetivos de control como fármaco utilizado. Sin embargo, las
combinaciones de dos o tres fármacos son en muchos casos necesarias para conseguir objetivos,
dada la alta concentración de LDL que presentan estos pacientes. Por lo tanto se pueden emplear
también el ezetimibe, secuestradores de ácidos biliares y el ácido nicotínico asociado a estatinas
para conseguir control. El desarrollo de aterosclerosis es un largo proceso que se inicia en etapas
muy precoces de la vida. Por ello, la edad de inicio de tratamiento es muy controvertida, pero un
enfoque racional sería utilizar fármacos con pocos efectos adversos en niños y comenzar fármacos
más potentes y combinaciones a partir de la edad adulta.(R3)

146. Varón de 55 años, fumador, que consulta por fiebre de 6 semanas de evolución sin
otra sintomatología. En la analítica se demuestra leucocitosis y anemia de trastorno
crónico. En la TAC abdominal presenta una masa renal derecha de 7 cm. ¿Cuál de los
siguientes diagnósticos le parece más probable?

1. 1. Tuberculosis diseminada.
2. 2. Angiomiolipoma infectado.
3. 3. Kala-azar.
4. 4. Hipernefroma.
Gráfico de respuestas
Comentario

Ante una masa renal sólida, el diagnóstico más probable sería el de hipernefroma. Por otra parte,
si además nos hablan de fiebre y anemia de trastorno crónico, tenemos una razón añadida para
pensar en este tipo de cáncer. Recuerda que este tumor también puede producir policitemia.
Aunque es un dato bastante característico, la anemia es bastante más frecuente.(R4)

147. Interprete la siguiente gasometría arterial: pH = 7.29, PaO2 = 50 mm Hg, PaCO2 = 71


mm Hg, Bicarbonato sérico = 42 mEq/l:

1. 1. Acidosis metabólica parcialmente compensada.


2. 2. Acidosis metabólica crónica.
3. 3. Acidosis respiratoria parcialmente compensada.
4. 4. Acidosis respiratoria crónica.
Gráfico de respuestas
Comentario
Se trata de un cuadro de acidosis por el pH de 7,29. Además la PaCO2 tan alta esta indicando un
proceso de hipoventilación, por lo que la causa de acidosis es respiratoria. Sin embargo el riñon ya
esta comenzando a compensar la acidosis, puesto que el riñon ya retiene bicarbonato, como se
desprende de las cifras tan altas de bicarbonato serico. Se trata pues de una acidosis respiratoria
parcialmente compensada, y si el riñon no estuviera compensado las cifras de pH serían aun
mucho menores.(R3)

148. Niño de 3 meses de edad que acude a urgencias porque en los últimos días ha
presentado clínica respiratoria. La madre comenta que presenta episodios frecuentes de
tos, en accesos, que a veces finalizan en cierta cianosis labial y se siguen de un vómito.

 
 
 
 
La madre lleva varias semanas con tos. La radiografía de tórax no muestra alteraciones
significativas. En los exámenes sanguíneos se observa leucocitosis a expensas de
linfocitosis. Ante la entidad clínica que sospecha, ¿cuál sería el tratamiento más
apropiado?

1. 1. Azitromicina.
2. 2. Aciclovir.
3. 3. Amoxicilina-clavulánico.
4. 4. Rifampicina.
Gráfico de respuestas
Comentario

Si se fija bien la pregunta describe el cuadro de tosferina clásico.

Recuerde que se produce por Bordetella pertussis, generalmente en niños menores de 1 año
donde la vacuna no ofrece aún buena inmunidad, aunque en los últimos años existe un repunte
entre adolescentes y adultos jóvenes, por atenuación del efecto vacunal.

Se precede de una clínica catarral inespecífica para dar lugar a la fase paroxística en forma de
accesos de tos emetizante, en ocasiones finalizados en apneas o gallo.

En la biometría hemática existe unalinfocitosis característica.

El tratamiento de elección son losmacrólidos, que se administrarán al paciente y a todos los


contactos como quimioprofilaxis (opción 1 correcta).(R1)

149. Señale lo que le parece ERRONEO sobre la leishmaniasis cutánea:

Es característico el "signo del rastrillo", que consiste en encontrar unos espigones gruesos
1. 1.
y profundos al desprenderse la costra.
Hay casos crónicos con persistencia de la lesión durante años sin remisión espontánea y
2. 2.
resistencia al tratamiento.
3. 3. El método más útil para un diagnóstico seguro es la serología.
Hay casos diseminados, en individuos inmunodeprimidos, con lesiones nodulares,
4. 4.
crónicas, sin tendencia a la ulceración.
Gráfico de respuestas
Comentario
Esta pregunta se puede sacar basándose en conocimientos teóricos o con simple picardía. En
general, en infecciosas, la serología NO será un método diagnóstico óptimo y mucho menos de
certeza, por lo tanto deberíamos haber marcado la opción 4 como falsa, aún sin tener idea del
resto de las opciones. De la misma manera podríamos haber descartado a priori las opciones 3 y 5,
pues dicen "hay casos...", y en el MIR, en general, hay que pensar que "hay descrito" de todo.
Recuerda que el diagnóstico de certeza de la leishmaniasis cutánea lo haremos con histología de
la lesión; en la visceral lo haremos también con histología pero obtendremos la biopsia de la
médula ósea por lo general.(R3)

150. A 7-month-old girl is being evaluated at a routinely check up for vaccination. She has
no medical history and lives with her mother because her parents are divorced. Physical
examination reveals a bluish pigmented area near the base of the spine. What should be
done in this case?

 
 
 
 
1. 1. Reassure the mother and follow routine visits as scheduled.
2. 2. Betablockers have demonstrated to reduce the appearence of this lesion in some cases.
3. 3. Laser therapy.
4. 4. Coagulation tests to rule out a coagulopathy.
Gráfico de respuestas
Comentario
Reassure the mother and follow routine visits as scheduled. The Mongolian spot (congenital dermal
melanocytosis) is a bluish spot located in the lumbar area, asymptomatic, presenting in up to 10%
of newborns. This stain is more common in Latin and Eastern countries (up 95%). It has no
pathological significance and usually disappears within three to five years of age.(R1)

151. Paciente femenino de 78 años de edad con estenosis aórtica que presenta episodios
de hemorragia digestiva baja recurrente sin otra sintomatología, la causa más probable
del sangrado es:

1. 1. Angiodisplasia de colon.
2. 2. Colitis isquémica.
3. 3. Isquemia mesentérica crónica.
4. 4. Enfermedad de Crohn.
Gráfico de respuestas
Comentario

La causa más frecuente de hemorragia digestiva baja recidivante en el anciano es precisamente


ésta, la angiodisplasia de colon. Recuerde que su confirmación suele realizarse mediante
colonoscopía. Existe una asociación con la estenosis aórtica, como sucede en este caso, lo que se
conoce como síndrome de Heyde.(R1)

152. Varón de 60 años fumador y bebedor acude porque desde hace 6 meses tiene unas
lesiones en la punta de los dedos de las manos y de los pies, y además en la nariz y
pabellones auriculares. Estas lesiones son placas eritematoescamosas muy
queratósicas que sangran al intentar desprenderlas. Ante esta clínica, ¿qué exploración
realizaría tras una buena anamnesis?:

1. 1. Enema opaco.
2. 2. Esofagogastroscopia.
3. 3. Ecografía abdominal.
4. 4. Exploración urológica.
Gráfico de respuestas
Comentario
Las dermatosis paraneoplásicas no son un tema importante en el MIR. Debes conocer solamente
el aspecto clínico de las lesiones y a qué tumor se asocian. Ten en cuenta que pueden inspirar
casos clínicos bastante peculiares, como es el caso, así que ten cuidado. El paciente que nos
presentan padece una acroqueratosis de Bazex. Este síndrome paraneoplásico se asocia
principalmente a carcinomas epidermoides de vías aerodigestivas altas (respuesta 2 correcta). Es
absolutamente extraordinario que aparezca esta dermatosis sin asociarse a algún cáncer.
Clínicamente, cursa con importantes alteraciones ungueales y una erupción de placas
eritematosas en zonas acras (pulpejos de los dedos, nariz y orejas), cubiertas por escamas muy
adherentes que producen sangrado al ser desprendidas.(R2)

153. El ectima contagioso es un cuadro causado por:

 
 
 
 
1. 1. Streptococcus pyogenes.
2. 2. Pseudomonas aeruginosa.
3. 3. S. aureus.
4. 4. Poxvirus.
Gráfico de respuestas
Comentario

Pregunta de escasa dificultad perteneciente al tema de infecciones víricas dermatológicas. Hay


que saberse bien el nombre de los cuadros causados por todas las enfermedades infecciosas. El
ectima contagioso o también llamado nódulo de los ordeñadores o nódulo de Orf, se trata de
lesiones nodulares que acontecen en los dedos de las manos de personas en contacto con el
ganado bovino y ovino, y que se produce por un Poxvrus. No se debe confundir con el ectima
gangrenoso que se produce en la infección por pseudomona, tratándose de una lesión ulcerada
dolorosa y hemorrágica, rodeada de edema y eritema.(R4)

154. En el tratamiento empírico inicial de la meningitis en el periodo neonatal se emplean


dos antibióticos asociados (generalmente las cefalosporinas de tercera generación
asociadas a la ampicilina) para cubrir los gérmenes más frecuentes en este período.
Señale, de ellos, el que suele ser resistente a las primeras y sensible a la segunda:

1. 1. Escherichia coli.
2. 2. Neumococo.
3. 3. Listeria monocytogenes.
4. 4. Pseudomonas aeruginosa.
Gráfico de respuestas
Comentario

La listeriosis durante el embarazo puede ser asintomática o causar una enfermedad febril, que
puede confundirse con una gripe o síndrome gripal. Habitualmente, el diagnóstico de certeza sólo
es posible mediante el hemocultivo, donde crecerá un bacilo aerobio, móvil y grampositivo. La
infección, ya sea oculta o manifiesta, puede estimular el trabajo de parto. La listeriemia materna
produce una infección fetal, que provoca característicamente lesiones granulomatosas
diseminadas con microabscesos. La eliminación de meconio es frecuente cuando existe infección
fetal.

El neonato es particularmente susceptible a la infección por Listeria (recuerde que los ancianos
también lo son, como a veces se pregunta en Neurología). La tasa de mortalidad de la listeriosis
neonatal es cercana al 50%. En estos casos, es usual la sepsis neonatal precoz, mientras que la
listeriosis de inicio tardío se manifiesta como una meningitis a las 3-4 semanas de edad.

Estas infecciones son muy parecidas a las causadas por estreptococo beta-hemolítico del grupo B.
Hay evidencia que el tratamiento antimicrobiano materno puede ser eficaz en caso de infección
fetal. Como tratamiento, se recomienda la combinación de ampicilina y gentamicina.(R3)

155. ¿En qué tipo de meningitis la ampicilina sigue siendo el antibiótico de elección?:

1. 1. Estafilococo coagulasa-negativo.
2. 2. Listeria monocytogenes.
3. 3. Haemophilus influenzae.
4. 4. Acinetobacter sp.
Gráfico de respuestas

 
 
 
 
Comentario
Cuestión básica y muy preguntada desde distintos puntos de vista, que no se debe fallar.
Actualmente el uso de la ampicilina en las meningitis ha quedado reducido a la Listeria. Se debe
utilizar en asociación a cefalosporinas de tercera generación o bien a penicilina más gentamicina
para asegurar una buena cobertura antibiótica de otros posibles patógenos asociados. De esta
bacteria es fundamental conocer que es un bacilo grampositivo, que afecta a edades extremas de
la vida (niños y ancianos), alcohólicos, inmunodeprimidos y embarazadas, que se adquiere por vía
digestiva ya que forma parte de la flora gastrointestinal habitual en personas sanas. En LCR
provoca un predominio linfocitario con glucosa disminuida.(R2)

156. Un RNPT de 29 semanas comienza con Silverman de 6 posterior al nacimiento, se le


intuba y se conecta a un respirador. En la Rx tórax se observa un patrón reticulogranular
y broncograma aéreo, más llamativo en el lóbulo inferior izquierdo. Se le administra una
dosis de surfactante y se consigue disminuir la asistencia respiratoria. A las 72 horas
comienza a retener CO2 y se detecta un precordio hiperactivo, pulsos saltones y un soplo
sistólico infraclavicular. En Rx tórax se objetiva cardiomegalia y aumento de las marcas
vasculares pulmonares. ¿Cuál es el diagnóstico más probable?:

1. 1. Enfermedad de membrana hialina (EMH) y persistencia del conducto arterioso (PCA).


2. 2. Enfermedad de membrana hialina (EMH) y comunicación interventricular (CIV).
3. 3. Sepsis por Estreptococo B y endocarditis.
4. 4. Enfermedad de membrana hialina y persistencia de la circulación fetal.
Gráfico de respuestas
Comentario

Por antonomasia, la modalidad de distrés respiratorio típico de la prematuridad es la enfermedad


de membrana hialina, debida a una insuficiente síntesis de surfactante. Además de este dato
epidemiológico, hemos de tener muy en cuenta que las alteraciones radiológicas que se producen
incluyen un infiltrado retículo-granular con broncograma aéreo (expresión radiológica indistinguible
de la sepsis con bronconeumonía, por lo cual hay de cubrir esta posibilidad con antibióticos) y
atelectasias de mayor o menor extensión. En la historia natural de la hialina cabe reseñar que la
fase de mejoría coincide con una fase poliúrica, alrededor del tercer día. Cuando en un neonato
con antecedentes de hialina que había mejorado, vuelve a aparecer distrés y retención de CO2, y
los pulsos se tornan saltones, hemos de descartar la posibilidad de PCA. Para ello, auscultemos
con el estetoscopio (se oye un soplo continuo), y confírmemos por ecocardiografía.(R1)

157. Which of the following statements is INCORRECT regarding severe aortic stenosis?

1. 1. The development of symptoms provides a clear indication for sugical intervention.


A significant increase in mean aortic valve gradient estimation may be an indication for
2. 2.
replacement surgery in asymptomatic patients.
3. 3. Exercise treadmill testing is absolutely contraindicated in this cardiac valvulopathy.
Stress echocardiogram with dobutamine may be useful to assess the indication for valve
4. 4.
replacement.
Gráfico de respuestas
Comentario

La estenosis aórtica severa debe intervenirse quirúrgicamente cuando el paciente esté sintomático
o bien exista disfunción sistólica ventricular izquierda. También serían candidatos a cirugía
aquellos pacientes que, pese a estar asintomáticos y mantener fracción de eyección normal,
muestran intensa calcificación de la válvula o incremento rápido del gradiente transvalvular. Existe

 
 
 
 
un grupo de pacientes con disfunción ventricular en los que existe discrepancia en cuanto a
severidad de la estenosis aórtica por mostrar área valvular aórtica menor (AVA) de 0.6 cm2/m2 de
superficie corporal (criterio de severidad) y sin embargo el gradiente medio está por debajo de 40
mmHg (propio de estenosis aórtica ligera o moderada). En esta circunstancia, la realización de una
ecocardiografía con dobutamina puede ayudar a discernir entre una verdadera estenosis aórtica
severa (incremento del gradiente sin modificar el AVA) de una estenosis pseudosevera, en la que
existirá aumento del AVA con un gradiente final transaórtico menor de 50 mmHg. Recientemente
se ha desarrollado el implante de una prótesis aórtica por vía transapical o transarterial femoral,
con resultados prometedores.(R3)

158. Masculinol de 25 años presenta máculas eritematosas con descamación en cuero


cabelludo, áreas superciliares y surcos nasogenianos desde hace 1 mes. Refiere prurito
sobre todo en cuero cabelludo. ¿Cuál sería su tratamiento tópico?

1. 1. Lindano.
2. 2. Mupirocina.
3. 3. Ketoconazol.
4. 4. Acido retinoico.
Gráfico de respuestas
Comentario

Se trata de un caso clínico muy sencillo. Recuerde siempre que debe fijarse en cómo describen la
lesión y su localización típica. En este caso son lesiones eritemato-descamativas (psoriasis, el
liquen ruber plano y la dermatitis seborreica como las más importantes). La distribución de las
lesiones sigue zonas del cuerpo con abundantes glándulas sebáceas: cuero cabelludo, región
centrofacial, línea media del tronco, axilas e ingles. Con todo esto, deberemos sospechar una
dermatitis seborreica, cuyo tratamiento puede hacerse con antifúngicos tópicos (opción 3) o con
corticoides tópicos suaves, aunque actualmente el tratamiento de elección es el pimecrolimus en
crema.(R3)

159. Gestante de 32 semanas con sangrado vaginal oscuro y escaso. A la palpación


abdominal se aprecia una marcada hipertonía uterina. No se aprecia latido fetal. El estado
materno es malo, con signos de choque y los exámenes de laboratorio de Urgencias son
compatibles con el inicio de una coagulopatía. ¿Qué diagnóstico le sugeriría?

1. 1. Placenta previa sangrante.


2. 2. Abdomen agudo de causa no obstétrica.
3. 3. Desprendimiento prematuro de placenta.
4. 4. Amenaza de parto pretérmino.
Gráfico de respuestas
Comentario

Pregunta muy importante acerca de las hemorragias de tercer trimestre. De las que no se deben
fallar.

Nos presentan un caso clínico compatible con desprendimiento prematuro de placenta


normoinserta: comienzo brusco, sangrado escaso y oscuro, mal estar general de la paciente
(choque y coagulopatía), riesgo de sufrimiento fetal, dolo e hipertonía uterina.(R3)

 
 
 
 
160. Mujer de 37 años que acude a su consulta solicitando método anticonceptivo (no
desea esterilización ). Antecedentes personales: enfermedades propias de la infancia. No
refiere intervenciones quirúrgicas. No alergias medicamentosas conocidas. Fumadora de
20 cigarrillos/ día. No consumo de alcohol ni de drogas. Antecedentes ginecoobstétricos:
3 partos eutócicos; lactancia materna. Tiempo medio de menstruación: 4 / 28 días. No
dismenorrea. Revisiones periódicas anuales. Ultima hace 3 meses con citología normal.
Exploración: genitales externos y vagina normales; cérvix de multípara bien epitelizado;
útero de tamaño y forma normal; anexos no se palpan patológicos. USG: útero en
anteversión, regular, histerometría 75mm; anexos normales. ¿Qué método
anticonceptivo aconsejaría?

1. 1. Anticonceptivos orales combinados.


2. 2. DIU.
3. 3. Diafragma.
4. 4. Minipíldora de gestágenos.
Gráfico de respuestas
Comentario

En una paciente fumadora de más de 35 años, la anticoncepción hormonal sería una


contraindicación absoluta, debido al riesgo cardiovascular, con lo que las opciones 1 y 4 no serían
planteables. En cuando al diafragma, recuerde que debe utilizarse junto con una crema
espermicida y su eficacia está por debajo de la del DIU. Por lo tanto, tampoco sería la mejor opción
para esta paciente. En caso de que éste no pueda utilizarse, tal vez sería planteable, pero no como
primera opción.

Con respecto al DIU, sería interesante que recordase las siguientes ideas:

- Mecanismo de acción: reacción inflamatoria endometrial que evita la implantación.

- Está contraindicado ante la sospecha de embarazo, infección pélvica aguda o reciente,


sangrado uterino anormal, tratamiento con anticoagulantes o tumores cervicales o uterinos de
cualquier tipo.

- El momento habitual de colocación es durante la menstruación.

- El DIU previene mejor el embarazo normal que el ectópico, por lo que aumenta la frecuencia
relativa de éste último.

- El DIU es un factor de riesgo para enfermedad inflamatoria pélvica, al poner en contacto una
cavidad séptica (la vagina) con el interior del útero.(R2)

161. Una paciente caquéctica, consulta por deterioro progresivo. Tiene anemia, ascitis, y
nódulos metastáticos en áreas supraclaviculares y zona umbilical. ¿Cuál considera el
origen más probable del tumor?

1. 1. Estómago.
2. 2. Colon.
3. 3. Hígado.
4. 4. Páncreas.
Gráfico de respuestas

 
 
 
 
Comentario

Paciente con síndrome constitucional y anemia que presenta ascitis y nódulos metastáticos en
áreas supraclaviculares y zona umbilical. Es el típico caso clínico de adenocarcinoma gástrico que
ha metastatizado produciendo adenopatías metastásicas supraclaviculares (conocidas como
ganglio de Virchow) y diseminación por la superficie peritoneal, que ocasiona ascitis y nódulos
periumbilicales (conocidos como el nódulo de la hermana María José).(R1)

162. En una paciente sometida a tratamiento de inducción de la ovulación con citrato de


clomifeno, ¿en caso de que no funcione dicho fármaco, cuál se podría utilizar?:

1. 1. Metformina.
2. 2. Danazol.
3. 3. Progestágenos.
4. 4. Gonadotropinas.
Gráfico de respuestas
Comentario

Respuesta 4 correcta. En caso que el clomifeno no funcione, está indicado el uso de


gonadotropinas.

Citrato de clomifeno: fue el primer fármaco utilizad o para inducir la ovulación. Actúa por unión a los
receptores estrogénicos del hipotálamo bloqueando la retroalimentación negativa del estradiol y
activa el mecanismo neuroendocrino para la secreción de GnRH . Se emplea para la inducción de
la ovulación en pacientes con oligoanovulación y niveles de estrógenos normales (ovario
poliquístico).

Gonadotropinas: obtenidas por purificación a partir de la orina de mujeres postmenopáusicas (hM


G ) o por recombinación genética (rFSH, rLH), se utilizan para la estimulación ovárica en técnicas
de reproducción asistida ya que recluían folículos y estimulan su crecimiento al mismo tiempo que
facilitan su maduración.(R4)

163. Niño de 2 meses que acude a Urgencias por tos intensa. Dos semanas antes había
comenzado con congestión respiratoria superior y tos leve, etiquetándose el cuadro por
un pediatra como un cuadro catarral. La tos empeoró hasta causar vómitos y episodios
de apnea con cianosis. Su madre también ha presentado tos durante varias semanas.
Durante el examen físico el niño comenzó a toser, persistiendo ésta durante más de 30
segundos, con estridor inspiratorio, y cierto grado de cianosis. Después de dejar de
toser, el niño estaba agotado, con llanto débil. En los exámenes de laboratorio se observó
una leucocitosis de 34,000 con un 85% de linfocitos. ¿Cuál es el agente etiológico más
probable de este cuadro?

1. 1. Adenovirus.
2. 2. Bordetella pertussis.
3. 3. Bordetella parapertussis.
4. 4. Rinovirus.
Gráfico de respuestas
Comentario

 
 
 
 
El cuadro que nos describen corresponde a una tos ferina. Esta enfermedad es una infección
producida principalmente por Bordetella pertussis.

Afecta preferentemente a menores de un año. Clínicamente produce un pródromo inespecífico, de


tipo catarral, y luego aparece la fase que caracteriza a la enfermedad, que es la de tos paroxística.
Origina accesos repentinos de tos, con series repetitivas de múltiples toses enérgicas,
acompañándose después de un ruido inspiratorio que recibe el nombre de “gallo”, al pasar aire a
través de una glotis cerrada. La tos, en ocasiones, se asocia con vómitos (tos emetizante).

Las complicaciones de la tos ferina son:

•   Neumonía: la más frecuente. Suele ser por sobreinfección bacteriana secundaria.


•   Convulsiones, cuya causa no está bien establecida.
•   Otras: prolapso rectal, hemorragia subconjuntival, hernia umbilical, epistaxis. Estas
complicaciones se deben a la intensa presión positiva intratorácica que se produce en
relación con la tos.

A pesar de tratarse de una infección bacteriana, es característica la presencia de linfocitosis en la


BH, como podemos ver en este caso. El tratamiento antibiótico de la tos ferina es la eritromicina
durante dos semanas.(R2)

164. Un paciente previamente sano, no fumador, de 38 años, con tos y expectoración


mucoide, desde hace 3 meses acude a consulta con una radiografía reciente que muestra
un infiltrado cavitado en lóbulo superior derecho. Sin datos de dificultad respiratoria, no
tiene fiebre ni dolor alguno, aunque refiere sudoración nocturna ocasional. El examen
físico es normal. Se recogen muestras de esputo y la baciloscopia resulta positiva.
Convive con una hija de 10 años, a la que estudiamos y resulta ser Mantoux negativa, con
Rx tórax normal y asintomática. ¿Qué actitud adoptaremos respecto a ella?:

1. 1. Dar isoniacida durante 3 meses, y entonces repetiremos el Mantoux.


2. 2. Dar pirazinamida al menos seis meses.
3. 3. No dar medicación y repetir la Rx tórax a los 3 meses.
4. 4. Tratamiento con tres fármacos antituberculosos.
Gráfico de respuestas
Comentario

Antes de valorar si es necesaria la profilaxis para tuberculosis hay que investigar si el paciente
presenta enfermedad activa. En este caso, no es así: Rx de tórax negativa y asintomática. El
segundo paso es valorar si cumple criterios para recibir profilaxis, menor de 35 años, mantoux
negativo que convive con un paciente bacilífero es indicación de iniciar tratamiento con isoniacida.
A los 3 meses, se repetirá el Mantoux. Si es negativo, se retira la isoniacida, si es positivo habrá
que reevaluar si existe enfermedad activa.(R1)

 
 
 
 

165. ¿Cuál es el
diagnóstico más probable del paciente del caso anterior?

1. 1. Oncocitoma.
2. 2. Carcinoma de células claras.
3. 3. Angiomiolipoma.
4. 4. Pielonefritis xantogranulomatosa.
Gráfico de respuestas
Comentario

Esta pregunta podría contestarse de dos formas: la primera de ellas, más sencilla, es sabiendo que
el tumor renal más frecuente es el carcinoma de células claras (respuesta 2), también llamado
hipernefroma, adenocarcinoma renal o tumor de Grawitz. La segunda de ellas requiere conocer
rasgos diferenciales (tanto clínicos como epidemiológicos y de radiodiagnóstico) de cada uno de
los tipos histológicos citados.

•   Oncocitoma (respuesta 1): tumor benigno muy infrecuente que no suele causar clínica
(hallazgo ecográfico) y que no presenta hemorragia ni necrosis intratumoral, mostrándose
homogéneo en la TC o con una característica cicatriz central.
•   Angiomiolipoma (respuesta 3): tumor benigno de forma redonda u ovalada con presencia
característica de grasa intratumoral, ofreciendo una imagen típica en la TC (densidad grasa
en el seno del tumor).
•   Pielonefritis xantogranulomatosa (respuesta 4): forma infrecuente de una pielonefritis
bacteriana crónica que puede confundirse con un carcinoma renal. La mayoría de los
pacientes refieren infecciones urinarias de repetición (frecuentemente complicadas con
litiasis) y tienen un patrón ecográfico con zonas hipoecoicas e hiperecoicas. La TC
demuestra aumento difuso (y no focal) del tamaño renal, hallándose múltiples imágenes
redondeadas hipodensas sustituyendo el parénquima renal (corresponden a abscesos y
cálices dilatados).
•   Carcinoma de células claras (respuesta 2): es la que mejor se ajusta al cuadro clínico
(varón hipertenso de mediana edad) y a la imagen tomográfica (masa renal de bordes
irregulares y densidad heterogénea).

(R2)

 
 
 
 
166. Paciente de 53 años de edad, hipertenso sin otros antecedentes patológicos de
interés. A partir del estudio por su hipertensión se realiza un estudio con TC abdominal
cuyas imágenes corresponden a la imagen 12. El paciente no presenta ninguna
sintomatología. Ante este hallazgo ¿cuál es la opción adecuada a seguir?

Remitir al paciente a Urgencias por riesgo de sangrado agudo y realizar una embolización
1. 1.
selectiva del riñón derecho.
2. 2. Solicitar un estudio con renograma con diurético para valorar la funcionalidad renal.
Realizar una punción-biopsia de la lesión para poder diagnosticar al paciente y decidir el
3. 3.
tratamiento adecuado.
4. 4. Programar una nefrectomía radical tras realizar un estudio de extensión.
Gráfico de respuestas
Comentario

Se trata de una pregunta sencilla sobre tumores renales.

Ante el caso clínico descrito, es esencial realizar el correcto diagnóstico de tumor renal, del que se
ha preguntado en numerosas ocasiones el manejo diagnóstico-terapéutico. Ante una masa renal,
debe realizarse, en primer lugar, una ecografía (para valorar si es un quiste simple -actitud
conservadora- o un quiste complicado/masa sólida).

En el caso de hallarnos ante un quiste complejo o una masa sólida, debe realizarse una tomografía
computarizada (TC), que es el mejor método para evaluar una masa renal (y no la RM, que sólo es
de elección cuando se sospecha afectación vascular).

Una vez realizada la TC, pueden plantearse exploraciones complementarias para descartar
afectación metastásica (radiografía de tórax, analítica hepática y, en ocasiones, gammagrafía
ósea). En cambio, no son necesarios estudios funcionales, ni histológicos, pues su resultado no
modifica la actitud terapéutica a adoptar. Una vez descartada la afectación metastásica, el
tratamiento de elección de un tumor renal es la nefrectomía radical (que incluye la fascia de Gerota
y la glándula suprarrenal).

No se considera adecuado, un manejo expectante, ni la embolización selectiva del riñón afectado,


pues el tratamiento de una masa renal sólida debe ser quirúrgico.(R4)

Algoritmo diagnóstico de las masas renales

 
 
 
 

167. A newborn is being evaluated in the delivery room. His mother had a complicated
pregnancy: she did not follow the scheduled controls and she has a history of drug abuse.
Physical examination of the child shows almost complete absence of subcutaneous fat
and low weight, but height and cranial perimeter are within normal limits for gestational
age. What would be an expected complication in this case?

1. 1. Hyperglycemia.
2. 2. Hypercalcemia.
3. 3. Hyperkalemia.
4. 4. Polycythemia.
Gráfico de respuestas
Comentario

 
 
 
 
Polycythemia. Polycythemia occurs in 0.4-12% of newborns. It is more common in large-for-
gestational-age and small-for-gestational-age infants than in AGA. It is a condition that is more
common if a loss of fetal well-being has occurred. In this case we have a mother who has not had
adequate care during her pregnancy. Sustained fetal hypoxia stimulates bone marrow production of
red blood cells.(R4)

168. Paciente de 31 años que acude a urgencias porque siente MEG, y dolor difuso en
ambas fosas renales. Refiere que tras un partido de fútbol hace 3 días, ha estado tomando
pastillas cuyo nombre no recuerda porque le dolían las rodillas. Comenta que ha estado
orinando más que habitualmente. En la analítica destaca Cr 3.1 mg/dL, Urea 120 mg/dL,
Na 139 mEq/L, K 4.9 mEq/L. Leucocitos 10.000 (N 75%, L 20%, E 5%). En la orina presenta
Na 80 y en el sedimento leucocitos 30-40/campo, sin bacteriuria. ¿Primera sospecha?

1. 1. FRA prerrenal
2. 2. Afección glomerular
3. 3. Nefritis inmunoalérgica aguda
4. 4. Impresiona de FRA obstructivo.
Gráfico de respuestas
Comentario
Por lo que parece es un paciente sin antecedentes que tras jugar un partido de fútbol ha podido
tomar un AINE para los dolores musculares o articulares. Destaca un deterioro de función renal,
que no parece prerrenal (sodio en orina alto), ni obstructivo (demasiado joven y sin antecedentes).
Además presenta eosinofilia y en el sedimentos leucocitos (recordemos que en el sedimento en
fresco no se puede determinar qué tipo de células son). La combinación de NUEVO
FÁRMACO+eosinofilia+dolor lumbar+deterioro de función renal = NIIA. Otros datos a favor son
fiebre, rash cutáneo….(R3)

169. Un recién nacido de sexo femenino presenta edema de manos y pies, talla baja para
su edad gestacional y coartación de aorta. Señale la anomalía cromosómica que puede
asociarse a este fenotipo:

1. 1. 45 XO.
2. 2. Trisomía 13.
3. 3. Trisomía 18.
4. 4. 47 XXY.
Gráfico de respuestas
Comentario

Es importante conocer en el ENARM, ciertos síndromes por sus características más


representativas que le permitan reconocer un caso clínico como este que nos habla de un
síndrome de Turner. Tiene cariotipos 45XO, 46XX y mosaicos. Presentan talla baja y frecuentes
malformaciones extragenitales: pliegue cervical (pterigon colli), cubitus valgus, alteraciones
renales, cardiacas (coartación aórtica en los cariotipos 45 XO)...Son causa de abortos y a veces se
detectan en vida embrionaria por presentar higromas quísticos, que son tumoraciones linfáticas
visibles en la ecografía desde el primer trimestre. Son frecuentes los gonadoblastomas.(R1)

170. Una mujer de 37 años consulta por diarrea sanguinolenta de 7 meses de evolución.
En la colonoscopia se comprueba inflamación y ulceraciones en los 30 cm distales del
intestino grueso. Las biopsias son compatibles con colitis ulcerosa. ¿Qué tratamiento le
parece más adecuado, sabiendo que ha presentado alergia a la sulfasalazina?:

 
 
 
 
1. 1. Azatioprina 2 mg/Kg/día.
2. 2. Uno o dos enemas de 5-ASA cada día.
3. 3. Prednisona 60 mg i.v.
4. 4. Metronidazol 250 mg/8h v.o.
Gráfico de respuestas
Comentario
Los síntomas de esta paciente, aunque son prolongados en el tiempo, son leves (no refiere fiebre,
ni dolor abdominal). Tiene predominio de afectación distal, en los 30 cm últimos del colon, es decir,
proctitis. La proctitis es una forma muy frecuente de presentación de la colitis ulcerosa. En estos
casos, es de elección el uso de 5'ASA tópicos, administrados en forma de enemas.(R2)

171. An 80-year-old man presents to the office with a history of cough, undigested food
regurgitation and low grade fever for several months. His wife has noticed a disgusting
smell in her husband's breath during this time. He has recently been diagnosed with initial
Alzheimer's disease and he suffers from chronic atrial fibrillation, hypertension and
diabetes. His nutritional state is not altered. His blood pressure is 148/96mmHg, pulse is
98/min and respiratory rate is 18/min. An esophageal manometry shows normal pressure
and relaxation of the lower esophageal sphincter. Which of these is the most likely
diagnosis for this patient?

1. 1. Achalasia.
2. 2. Esophageal spasms.
3. 3. Scleroderma.
4. 4. Zenker's diverticulum.
Gráfico de respuestas
Comentario
Zenker's diverticulum. Zenker's diverticulum is a false diverticulum caused by traction and pulsion
mechanisms that force the mucosa to herniate through the pharyngeal wall. It can be aymptomatic
or cause dysphagia, sense of lump in the throat and food regurgitation, typically with bad smell.(R4)

172.La anemia ferropénica del niño se caracteriza por los siguientes hechos EXCEPTO
uno:

1. 1. Es la causa más frecuente de anemia nutricional de la infancia.


2. 2. La lactancia materna prolongada evita su aparición.
3. 3. Su desarrollo está favorecido por las infecciones repetidas.
4. 4. El diagnóstico se basa en la determinación de la ferritina sérica.
Gráfico de respuestas

(R2)

173. ¿Cuál de las siguientes es la actitud del brazo en la luxación escapulohumeral


anterior?:

1. 1. Aproximación y rotación externa.


2. 2. Separación y rotación externa.
3. 3. Aproximación y rotación interna.
4. 4. Unicamente en separación.
Gráfico de respuestas
Comentario

 
 
 
 
Cuando el hombro sufre una luxación anteroinferior (la más frecuente) la cabeza humeral "sale"
hacia anterior, inferior, y rota hacia externo. Todo ello condiciona la actitud del brazo en separación
(abducción) y rotación externa.(R3)

174. Paciente varón de 45 años que desde hace unos meses está inquieto y angustiado,
con actitud de sospecha y desconfianza ante los compañeros de trabajo. Según dicen los
familiares, "ve enemigos por todas partes" y tiene tendencia a sentirse ofendido
fácilmente. ¿Cuál es el diagnóstico más probable?

1. 1. Esquizofrenia paranoide.
2. 2. Trastorno delirante.
3. 3. Psicosis maniacodepresiva.
4. 4. Fobia social.
Gráfico de respuestas
Comentario

Este caso clínico nos presenta un paciente con una interpretación paranoide de la
realidad(sospecha y desconfianza de los demás, sensación de ser vigilado o perseguido…). Por su
edad y la evolución (lleva así unos meses), lo más probable es que sea un trastorno delirante. La
esquizofrenia queda descartada por la ausencia de alucinaciones y de otros síntomas típicos, y por
la edad algo tardía (aunque la esquizofrenia se pueda dar a cualquier edad).

No se nos dice nada de consumo de alcohol (ni de alucinaciones) ni de síntomas afectivos,


maníacos o depresivos; tampoco se describen síntomas de una fobia social (miedo a quedar en
ridículo en actividades y situaciones de exposición pública). Los síntomas paranoides se pueden
ver en numerosas enfermedades psiquiátricas; prácticamente cualquier trastorno en el que se
puedan dar síntomas psicóticos puede presentar un perfil paranoide, que se convierte así en el
más frecuente de todos los temas delirantes.(R2)

175. ¿Cuál de los siguientes marcadores tumorales puede ser útil para el diagnóstico y
seguimiento del cáncer de mama?

1. 1. CA 19.9.
2. 2. CA 125.
3. 3. Alfa-fetoproteína.
4. 4. CA 15.3.
Gráfico de respuestas
Comentario

De los marcadores que le ofrecen, el único relacionado con el Ca de mama es el CA15.3. Sin
embargo, es poco sensible como diagnóstico y, aunque parece que sí es un marcador pronóstico
hasta el momento su presencia no modifica el tratamiento. En las recomendaciones ASCO 2007
sólo se recomendaba monitorizar el CA 15.3 y CA 27.29 para seguimiento de enfermedad
metastásica.

Recuerde que el CA 125 es para cáncer de ovario.(R4)

176. An 11-year-old child is brought to the pediatrician's office presenting with yellowish
skin colour and pain in the upper right abdomen. She reports no fever. Physical
examination shows generalized jaundice and a tender abdomen to palpation on the right

 
 
 
 
upper part of the abdomen, with a palpable mass. Blood laboratory test results show
cholestasis pattern with normal amylase and lipase. A cystic extrahepatic mass is
visualized in abdominal ultrasonography. What is the most likely diagnosis?

1. 1. Choledochal cyst
2. 2. Pancreatic cancer
3. 3. Billiary atresia
4. 4. Cystic torsion
Gráfico de respuestas
Comentario
Choledochal cyst. Choledochal cysts represent congenital cystic dilations of the biliary tree.
Diagnosis relies on the exclusion of other possible causes of biliary duct dilation. Laboratory
findings are consistent with a pattern of cholestasis with elevated direct bilirubin. Anular pancreas
produces abdominal pain but no cholestasis. Pancreatic cancer is highly infrequent at this age.
Biliary atresia manifestations start at birth.(R1)

177. Referente a embriología del aparato genital femenino, marque el enunciado


incorrecto

1. 1. Los labios mayores se originan de la cloaca ectodérmica.


2. 2. El útero deriva de la diferenciación de los conductos paramesonéfricos (Müller).
3. 3. La diferenciación gonadal depende del factor inhibidor de Müller.
4. 4. Las gónadas controlan la dirección del desarrollo de los conductos sexuales.
Gráfico de respuestas
Comentario

Pregunta muy difícil de Embriología de un tema poco rentabe para el nacional.

Genitales externos: Durante la quinta semana de vida embrionaria se forman pliegues tisulares a
cada lado de la cloaca que se unen por delante en la linea media para formar el tubérculo genital.
Con la división de la cloaca por el tabique urorrectal y la formación consecuente del perineo, estos
pliegues cloacales se conocen por delante como pliegues urogenitales y por detrás como pliegues
anales.

El tubérculo genital empieza a aumentar de tamaño, pero en el embrión del sexo femenino, su
crecimiento va haciendose mas lento de manera gradual hasta que se convierte en el clítoris, y los
pliegues urogenitales forman los labios menores.

En el varon, el tubérculo genital sigue creciendo para formar el pene, y se cree que los pliegues
urogenitales se fusionan para encerrar a la uretra del pene. Lateralmente a los pliegues
urogenitales, se desarrolla otro par de eminencias, que se conocen en la etapa indiferente como
eminencias labioscrotales.

En ausencia de andrógenos se quedan prácticamente sin fusionar y se convierten en labios


mayores. Respuesta 3 incorrecta.(R3)

178. Paciente joven que, tras participar en la extinción de un incendio, ingresa con
quemaduras de tercer grado en un 15% de su superficie corporal, y durante el ingreso
presenta un episodio de melenas con disminución de su tensión arterial. Señale la
respuesta correcta:

 
 
 
 
1. 1. Lo más probable es que tenga una única úlcera gastroduodenal que está sangrando.
2. 2. Lo más probable es que se trate de múltiples ulceraciones superficiales gastroduodenales.
El tratamiento de elección será un tratamiento médico, aunque se trate de una hemorragia
3. 3.
importante que mantenga al paciente en choque hipovolémico.
4. 4. Estas hemorragias no suelen ser abundantes y no suelen precisar transfusiones.
Gráfico de respuestas
Comentario

Lo que nos están describiendo es un sangrado por úlceras de estrés, llamadas de Curling cuando
se trata de pacientes quemados. Aunque se llamen úlceras, en realidad son mucho más
superficiales y deberían llamarse erosiones, por lo que la opción 2 es correcta. Recuerda que,
desde el punto de vista fisiopatológico, obedecen a cierto grado de isquemia en la mucosa
gástrica.(R2)

179. Respecto a los efectos secundarios de los anticonceptivos orales, señale cuál de las
siguientes afirmaciones es FALSA:

1. 1. Están en relación con la aparición de adenomas hepáticos.


2. 2. Parecen aumentar el riesgo de sangrado de la hiperplasia nodular focal hepática.
3. 3. Aumentan el riesgo de mastopatía fibroquística.
Se ha observado un aumento de incidencia de TVP y TEP en mujeres jóvenes que han
4. 4.
recibido anticoncepción hormonal durante más de un año.
Gráfico de respuestas
Comentario

Es importante que tenga un conocimiento de los modos de actuación, los efectos y las
contraindicaciones de los anticonceptivos orales pues si son preguntados en el ENARM.

La anticoncepción hormonal disminuye la incidencia de patología ovárica y mamaria benigna


(mastopatía fibroquística, adenomas mamarios) aunque parece aumentar ligeramente la incidencia
precoz de cáncer de mama.

También disminuyen la incidencia de carcinoma epitelial de ovario y adenocarcinoma de


endometrio.

La anticoncepción hormonal se asocia con adenomas hepáticos (tumoraciones quísticas


hemorrágicas habitualmente asintomáticas que pueden estallar y producir choque hipovolémico).

Es un factor de riesgo para la enfermedad tromboembólica y el infarto de miocardio.(R3)

180. Mujer de 42 años que consulta por menstruaciones de 12 días de duración cada 24
dias desde hace 3 años. Pruebas complementarias: USG aparato genital interno normal
con endometrio de 11 mm de espesor, Histeroscopía diagnóstica: sin hallazgos
patológicos, Biopsia endometrial: endometrio proliferativo simple, BH: Hb=10.1g/dl
Hcto=30%. Actualmente en tratamiento con hierro oral. ¿Qué tratamiento indicaría?:

1. 1. Antifibrinolíticos durante todo el ciclo, durante 6 meses.


2. 2. Realizaría un legrado endometrial para contener la hemorragia de forma rápida.
3. 3. Análogos GnRH previa a la realización de histerectomía total simple.
4. 4. DIU de Levonorgestrel durante 5 años.
Gráfico de respuestas

 
 
 
 
Comentario

Los progestágenos son el tratamiento indicado en metrorragias disfuncionales con endometrios


proliferativos, provocados sobre todo por ciclos anovulatorios en la perimenopausia. Además
recuerde que es importante descartar patología orgánica como se ha realizado en este caso. Entre
las opciones terapéuticas, el DIU-Levonorgestrel proporciona tratamiento médico eficaz y de larga
duración por lo que es una opción válida para la paciente.(R4)

181. El coriocarcinoma es un tumor constituido por:

1. 1. Elementos del sincitio y citotrofoblasto.


2. 2. Degeneración hidrópica de la vellosidad corial.
3. 3. Hiperplasia del sincitiotrofoblasto.
4. 4. Trofoblasto de origen materno.
Gráfico de respuestas
Comentario

Esta pregunta sobre el coriocarcinoma, tumor muy raro, no tiene especial importancia.
Simplemente debe recordar que se desarrolla a partir de las células germinales. La mayoría
aparecen combinados con otros tipos tumores germinales. Son idénticos a los placentarios pero
mucho menos frecuentes, y al igual que éste elabora gonadotropina corionica. Son muy malignos y
mucho más resistentes a la quimioterapia que sus homólogos placentarios. Histológicamente
consiste en proliferación anormal de elementos trofoblásticos: sincitio y citotrofoblasto, que invade
vasos y metastatiza. La degeneración hidrópica de la vellosidad corial se da en la enfermedad
trofoblástica porque la ausencia de linfáticos hace que se acumule liquido en el espacio
extracelular.(R1)

182. A propósito del tratamiento de las crisis hipertensivas podemos afirmar:

En la disección aórtica no debe reducirse la presión por debajo de 180 mmHg, para evitar
1. 1.
disminuciones bruscas del riego cerebral.
2. 2. No debe usarse tratamiento por vía oral, por la tardanza en comenzar a actuar.
Sólo está indicado tratamiento por vía parenteral en las denominadas emergencias
3. 3.
hipertensivas.
4. 4. El tratamiento de elección en la eclampsia es propranolol.
Gráfico de respuestas
Comentario

Se trata de una pregunta difícil que debes sacar por descarte. Para descartar la opción 1, debe
saber que el principal objetivo del tratamiento de una disección aórtica es la reducción de la
presión arterial, junto con la de la contractilidad cardiaca. Recuerde que en el manejo de una crisis
hipertensiva se debe evitar el uso de nifedipino de acción rápida, ya que puede producir
hipotensión o desencadenar una isquemia miocárdica. La opción 4 la puede descartar porque debe
saber que el tratamiento antihipertensivo de elección en la eclampsia es la metildopa o la
hidralazina. Por último, en la opción 3 la palabra "sólo" le debe hacer sospechar que es falsa.
Efectivamente en la "emergencia hipertensiva" (aquellos casos de crisis hipertensiva en los que la
elevación de la presión arterial requiere una reducción en el plazo de una hora para evitar
morbilidad severa o muerte) el tratamiento debe hacerse via parenteral pero también hay casos de
"urgencia hipertensiva" (en los que debe reducirse la PA en la siguientes 24 horas) en los que
también puede ser necesario el tratamiento parenteral.(R2)

 
 
 
 
183. Señale la relación alteración iónica - alteración electrocardiográfica correcta:

1. 1. Hipercalcemia - acortamiento del QT.


2. 2. Hipocalcemia - acortamiento del QT.
3. 3. Hipocalcemia - onda T positiva.
4. 4. Hipercalcemia - bloqueo de rama.
Gráfico de respuestas
Comentario

Los trastornos del metabolismo del calcio constituyen un tema bastante preguntado en el ENARM.
Recuerda que las cifras normales de Ca en sangre son de 9 a 10,5 y que la hipercalcemia suele
ser asintomática hasta cifras de 11,5- 12,5. Sobre el ECG produce acortamiento del QT, pudiendo
llegar a producir arritmias mortales. La hipocalcemia crónica, en cambio, no suele ser asintomática
sino que da síntomas neuromusculares y neurológicos. En el ECG produce prolongación del QT e
inversión de la T, y disminuye la eficacia de los digitálicos.(R1)

184. Señale la opción FALSA respecto a las diferencias entre la leche materna y la fórmula
artificial:

La fórmula artificial presenta mayor carga proteica, fundamentalmente en forma de


1. 1.
caseína.
El contenido en grasas es similar en ambas leches, aunque la leche materna presenta un
2. 2.
perfil más apropiado debido a los LC-PUFA.
La lactosa, principal hidrato de carbono de la leche, es mayoritario en la fórmula
3. 3.
artificial.
4. 4. El contenido calórico es similar entre ambos tipos de leche.
Gráfico de respuestas
Comentario

Los diferentes tipos de leche y sus diferencias es importante dentro del tema desarrollo y nutrición.
Recuerde que las principales diferencias entre la leche materna y la de fórmula artificial son el
contenido proteico, sales minerales, vitaminas B y K y calcio que son mayoritarias en la fórmula
artificial (opciones 1 correcta).

La lactosa se encuentra en mayor proporción en la leche materna (opción 3 falsa), y el contenido


en grasas y calorías es similar, aunque es más idóneo el materno (opción 2 correcta).(R3)

185. An 8-year-old child is brought to the emergency room presenting with edema,
redness and swelling of her right eye. She has a fever and visual acuity is markedly lower
in the affected eye with intense pain triggered by eye movements. Funduscopy and
neurological examination are normal. Which is the most likely diagnosis?

1. 1. Orbital cellulitis
2. 2. Acute bacterial conjunctivitis
3. 3. Uveitis
4. 4. Preseptal cellulitis
Gráfico de respuestas
Comentario
Orbital cellulitis. Orbital cellulitis is an inflammation of the tissues behind the orbital septum.
Common signs and symptoms of orbital cellulitis include pain with eye movement, vision loss,
bulging of the infected eye, and limited eye movement. Along with these symptoms, patients

 
 
 
 
typically have redness and swelling of the eyelid, pain, discharge, inability to open the eye and
systemic signs of infection, such as fever and malaise. Preseptal cellulitis is excluded in this case,
because extra ocular muscles are not involved, so there should be no pain with eye movement.(R1)

186. El síntoma inicial en la apendicitis aguda es casi de manera invariable:

1. 1. Distensión abdominal.
2. 2. Diarrea.
3. 3. Dolor.
4. 4. Escalofríos.
Gráfico de respuestas
Comentario
Debes conocer la evolución clínica de la apendicitis que se correlaciona con los hallazgos
fisiopatológicos que se van sucediendo. La inflamación y el aumento de presión en la luz
apendicular son percibidos por el paciente como dolor visceral mal localizado en la región
periumbilical o epigástrico. Luego aparecen los signos de irritación peritoneal y más tarde cuando
la invasión bacteriana se extiende a la pared la serosa inflamada del apéndice entra en contacto
con el peritoneo parietal originando un dolor localizado en fosa ilíaca derecha.(R3)

187. Género microsporum afecta, salvo raras excepciones a:

1. 1. Piel, uñas y pelo.


2. 2. Piel y uñas, pelo no.
3. 3. Piel y pelo, uñas no.
4. 4. Sólo uñas.
Gráfico de respuestas
Comentario
Pregunta bastante difícil y que considero que no resulta nada rentable el saberse la predilección
por cada estructura que tiene cada género de los dermatofitos. Los hongos dermatofíticos,
constituyen un grupo de agentes queratinofílicos, o capaces de colonizar los tejidos que poseen
queratina (piel, uñas y pelos), dando lugar a las denominadas tiñas. Los dermatofitos, se agrupan
en tres géneros imperfectos: Microsporum, Tricosporum, y Epidermophiton. Se ha observado que
el género Microsporum tiene una mayor tendencia a la colonización de pelos y piel, siendo mucho
más infrecuente la afectación ungueal. Bibliografía: Manual de dermatología Iglesias. 1ª Ed. Pag.
258.(R3)

188. En 1988, el Colegio Americano de Reumatología estableció unos criterios de


clasificación, que son muy útiles para el diagnóstico de la artritis reumatoide. Señale cuál
es INCORRECTO en la lista que se enumera:

1. 1. Artritis de tres o más áreas articulares.


2. 2. Simetría de las manifestaciones articulares.
3. 3. Rigidez matutina de, al menos, un cuarto de hora de duración.
4. 4. Factor reumatoide positivo.
Gráfico de respuestas
Comentario

Existen dos tipos de crecimiento intrauterino retardado (CIR):

 
 
 
 
- CIR tipo I o CIR simétrico: Se produce desde el comienzo de la gestación. El crecimiento
longitudinal, los diámetros abdominales y otras medidas guardan entre sí las proporciones
adecuadas. Entre sus causas, destacan las cromosomopatías y las infecciones connatales.

- CIR tipo II o CIR asimétrico: Es mucho más frecuente que el tipo I. Consiste en una reducción
de los diámetros abdominales, quedando preservados los parámetros óseos de medida (diámetro
biparietal y longitud femoral). La causa más frecuente de CIR II son las enfermedades placentarias,
como la preeclampsia u otras condiciones que puedan producir una disminución del intercambio
útero- placentario. En definitiva, el peso de la placenta no guarda la debida proporción con el del
feto.(R3)

189. ¿Cuál de las siguientes medidas fetales es la más fiable para el diagnóstico
ultrasonográfico de la edad gestacional en el tercer trimestre?

1. 1. Longitud cefalo-nalgas.
2. 2. Diámetro biparietal.
3. 3. Diámetro transverso abdominal.
4. 4. Longitud femoral.
Gráfico de respuestas
Comentario

Una pregunta difícil sobre un aspecto muy concreto del ultrasonido obstétrico. Respecto a este
tema, debemos recordar lo siguiente:

- En el primer trimestre, el parámetro más útil para calcular la edad gestacional sería la longitud
cráneo-caudal.

- En el segundo y tercer trimestre, el parámetro más fiable y con menos dispersión es la longitud
femoral (respuesta 4 correcta).

Es posible que hayan dudado con el diámetro biparietal, que también es una medida útil. Sin
embargo, como su dispersión es mayor que la de la longitud femoral, ésta sería preferible.(R4)

190. Con respecto a la refracción de los distintos medios ópticos del ojo, señale la
respuesta correcta:

1. 1. La potencia dióptrica de la córnea en el ser humano es despreciable.


La córnea es la lente más potente del ojo. Su potencia es modulable gracias a la acción
2. 2.
del músculo ciliar.
3. 3. Cristalino y córnea tienen una potencia similar.
La córnea es más potente que el cristalino. Sin embargo, la potencia del cristalino es
4. 4.
modulable gracias a la acción del músculo ciliar.
Gráfico de respuestas
Comentario
Desde el punto de vista refractivo, el globo ocular es un dioptrio, constituido por dos lentes, córnea
y cristalino. La córnea (43 D) es más potente que el cristalino (17 D). Sin embargo la potencia del
cristalino es “regulable” gracias a la acción del músculo ciliar, a través de un mecanismo que recibe
el nombre de acomodación. Ello permite que los sujetos jóvenes puedan ver bien de cerca sin
necesidad de utilizar gafas. A partir de los 40 años este mecanismo pierde competencia,
determinando la aparición de presbicia.(R4)

 
 
 
 
191. Un varón de 23 años sufre una caída mientras conducía una moto de elevada
cilindrada. Refiere que cayó sobre el muñón del hombro derecho, presentando dolor en
esta localización. En la radiografía anteroposterior se observa un desplazamiento vertical
mínimo de la articulación acromioclavicular. ¿Cuál de las siguientes afirmaciones es
FALSA en este paciente?

1. 1. El tratamiento será conservador.


2. 2. El signo de la tecla es negativo.
3. 3. La clavícula puede elevarse ligeramente sobre el acromion.
4. 4. Los ligamentos acromioclaviculares están íntegros.
Gráfico de respuestas
Comentario

El traumatismo directo sobre el hombro puede provocar lesiones en la articulación


acromioclavicular. Cuando observamos un desplazamiento vertical de la clavícula sobre el
acromion, y una pérdida de la congruencia acromioclavicular en la radiología simple, debemos
sospechar una rotura de los ligamentos acromioclaviculares que permiten la luxación
acromioclavicular. Clínicamente, se manifiesta por el signo de la tecla cuando además se rompen
los coracoclaviculares (grado III) y es una lesión típica de adultos jóvenes que sufren accidentes de
tráfico o deportivos.(R4)

192. Femenino de 25 años de edad, con pareja estable desde hace 2 años, con
antecedentes ginecoosbtétricos: G1-A1-P0, un USG reciente con resultado de quistes en
ovario derecho, antecedentes personales de alergia a betalactámicos y a sulfamidas,
antecedentes familiares de cáncer de endometrio, consulta por el método anticonceptivo
más adecuado para ella. Se le recomienda utilización de anticonceptivos orales
combinados (estrógenos + progesterona). De las siguientes opciones, todas son
contraindicaciones absolutas para el uso de anticonceptivos orales, EXCEPTO una.
Señálela:

1. 1. Dismenorrea.
2. 2. Historia de tromboembolismo previo.
3. 3. Antecedentes personales de cáncer de mama.
4. 4. Antecedentes de colestasis durante un embarazo.
Gráfico de respuestas
Comentario

Pregunta sencilla sobre los anticonceptivos orales. Realmente está clara la indicación en casos de
dismenorrea, ya que pueden controlar y regulan el ciclo menstrual de la paciente. En el resto de
casos es clara la contraindicación. Recuerde especialmente la relación de los ACO con el
tromboembolismo, ya que este es un aspecto importante.(R1)

193. De las siguientes afirmaciones sobre la isquemia mesentérica aguda, señale la


INCORRECTA:

1. 1. La embolia suele ser distal al origen de la arteria mesentérica superior.


2. 2. La trombosis venosa es de difícil diagnóstico precoz.
3. 3. La isquemia no oclusiva tiene el mejor pronóstico.
4. 4. La cirugía precoz ofrece la mejor oportunidad de tratamiento eficaz.
Gráfico de respuestas
Comentario

 
 
 
 
La causa más usual es un émbolo cardiógeno, normalmente en un paciente anciano con fibrilación
auricular.

La técnica diagnóstica de elección es la arteriografía, que mostrará alteraciones distalmente al


origen de la mesentérica superior (opción 1).

Como dice la opción 2, la trombosis venosa es de diagnóstico más tardío y difícil, puesto que es
más larvada en su presentación.

La opción incorrecta es la 3, dado que los pacientes con isquemia no oclusiva suelen ser muy
ancianos, con pluripatología y múltiples factores de riesgo cardiovascular asociados
(arteriosclerosis extensa en mesentérica superior).

El tratamiento ha de instaurarse de la forma más precoz posible, por ejemplo, mediante


embolectomía, que es la técnica más habitual, para una mayor viabilidad del intestino afecto (si se
demora, puede necrosarse).(R3)

194. Una de las siguientes afirmaciones sobre la patogenia de la preeclampsia es FALSA;


¿cuál?:

El hematócrito aumenta con la gravedad y la duración de la hipertensión inducida por el


1. 1.
embarazo.
2. 2. El péptido natriurético auricular aumenta significativamente en la preeclampsia.
3. 3. Existe una disminución de la síntesis de tromboxano A2.
4. 4. La perfusión renal y la filtración glomerular están disminuidas.
Gráfico de respuestas
Comentario

Lo que ocurre en la preeclampsia es un aumento de la producción de endotelina y tromboxano, un


aumento de la sensibilidad vascular a la angiotesnina II y una disminución en la formación ed
agentes vasodilatadores (oxido nitrico y prostaciclinas).

Todas estas alteraciones provocan un aumento de las resistencias vasculares, mayor


agregabilidad plaquetaria, activación del sistema de la coagulaión y disfunción endotelial que se
traducen en los síntomas y signos de la enfermedad.(R3)

195. ¿Cuál de las siguientes NO es una alteración circunscrita sino global del
conocimiento?:

1. 1. Asterognosia.
2. 2. Prosopagnosia.
3. 3. Despersonalización.
4. 4. Delirium.
Gráfico de respuestas
Comentario
Pregunta intuitiva y de rápida contestación si nos damos cuenta de que todas la respuesta son
alteraciones con un único síntoma, excepto la R5, delirium que es un síndrome, el cuadro
confusional agudo, conjunto de alteraciones, en los que se ven afectadas las diversas memorias, la
atención, el lenguaje, síntomas vegetativos, estado anímico... Bibliografía: Manual Psiquiatría CTO-
Medicina 4ªEd, pág 32.(R4)

 
 
 
 
196. Señale qué germen se aísla con MENOS frecuencia en las secreciones bronquiales
de niños con fibrosis quística:

1. 1. Mycoplasma pneumoniae.
2. 2. Haemophilus influenzae.
3. 3. Staphylococcus aureus.
4. 4. Pseudomonas aeruginosa.
Gráfico de respuestas
Comentario

La microbiología de la fibrosis quística es muy importante ya que ha sido preguntada varias veces.
La mayoría de los pacientes están colonizados por H. influenziae, S. aureus o Pseudomonas
aeruginosa. El germen que coloniza con más frecuencia de forma crónica las vias respiratorias en
los pacientes con FQ es Pseudomonas aeruginosa variedad mucoide, el que primero lo hace es S
aureus, .y el de peor pronóstico Burkholderia cepacea, que es particularmente difícil de tratar y se
asocia a un deterioro clínico acelerado. No es frecuente aislar el Mycoplasma pneumoniae.(R1)

197. Neonato que presenta un episodio, posterior a nacer, de dificultad respiratoria


severa, con cianosis generalizada que mejora con el llanto. Señale la respuesta FALSA
acerca de la patología que sospecha:

1. 1. Para su diagnóstico ha de utilizarse la radiografía lateral de cráneo con contraste nasal.


2. 2. La alteración es bilateral en la mayoría de los casos.
3. 3. Una medida paliativa útil es el uso de las tetinas de McGovern.
4. 4. El tratamiento debe ser quirúrgico y urgente.
Gráfico de respuestas
Comentario

Los neonatos respiran por el sitio ideal (las fosas nasales), siempre y cuando no haya factores
mecánico-obstructivos que lo impidan. Sospeche la presencia de éstos cuando le describan un
neonato con dificultad para respirar en reposo y al mamar que mejora con el llanto. Tenga en
cuenta que, cuando el neonato llora, coge aire por la nariz y por la boca. Su tratamiento es
quirúrgico. El diagnóstico es una combinación de sosepcha clínica y la ausencia de paso inmediato
de una sonda nasogástrica por ambas fosas nasales.(R1)

198. Respecto a H. ducreyi, señala cuál de las siguientes afirmaciones es FALSA:

1. 1. Provoca una enfermedad de transmisión sexual.


2. 2. Es responsable de conjuntivitis contagiosa.
Un tratamiento recomendado para el tratamiento de esta infección es eritromicina 500 mg
3. 3.
cada 6 horas durante 7-10 días.
4. 4. Las lesiones, de tipo ulceroso no induradas, son dolorosas.
Gráfico de respuestas
Comentario

Con esta pregunta, podemos repasar los conceptos más preguntados sobre esta infección. El
chancro blando o chancroide es una enfermedad de transmisión sexual causada por
el Haemophilus ducreyi. Tiene un período de incubación de 1- 3 días, tras el que aparece un
chancro blando y doloroso, múltiple, de fondo sucio, acompañado de adenopatías dolorosas
unilaterales, con tendencia a ulcerarse. El tratamiento consiste en 250 mg im de ceftriaxona en

 
 
 
 
monodosis, o eritromicina durante 7 días. La opción 3 es falsa, pues este agente no es
responsable de conjuntivitis contagiosa.(R2)

199. Acerca de los requerimientos nutricionales, marque lo INCORRECTO:

1. 1. El RN a término el requerimiento hídrico del primer día es de 60–70 mL/Kg/d.


2. 2. Los requerimientos de glucosa en el RN son de 4 a 8 g/Kg/d.
3. 3. Los requerimientos calóricos en el primer trimestre es de 100 a 120 Kcal/Kg/d.
4. 4. Los requerimientos proteicos es de 2 a 3 g/Kg/d.
Gráfico de respuestas
Comentario

Requerimientos hidratos de carbono: RNT 10-17 g/kg/dia, Prematuro 13-15 g/kg/dia.

Lípidos: RNT 4-6 g/kg/dia y prematuro hasta 9 g/kg/dia.

Hídricos: 1-2 días: 6080 ml/kg/dia, 3-7 día 100-150 ml/kg/dia, 8-30 día 120-180 ml/kg/dia

Proteinas: RNT 2.25-2.5 g/kgdia, prematuro 3-4.5 g/kg/dia.

Por lo que la respuesta que debió elegir es la 2.(R2)

200. En relación a las complicaciones del infarto agudo al miocardio NO es cierto:

La rotura aguda de pared libre suele manifestarse en forma de disociación


1. 1.
electromecánica.
La presencia de derrame pericárdico en la fase aguda del IAM es patognomónica de
2. 2.
rotura cardiaca.
La presencia de un salto oximétrico en el ventrículo derecho es indicativa de rotura septal
3. 3.
interventricular.
4. 4. El RIVA no precisa de tratamiento antiarrítmico.
Gráfico de respuestas
Comentario
Aunque no es excepcional observar ligero derrame pericárdico en un infarto agudo de miocardio,
no siempre se debe a una rotura, que en caso que el paciente no tenga repercusión
hemodinámica, sería contenida. La propia reacción inflamatoria puede dar ligero derrame. La
presencia de salto oximétrico en ventrículo derecho indica comunicación interventricular. El RIVA
es una arritmia que indica reperfusión, y aunque puede afectar ligeramente el estado
hemodinámica, suele ser transitorio y no precisa de tratamiento pues suele estar bien tolerado. El
bloqueo AV que aparece en el infarto inferior suele responder a atropina. Sin embargo si parece un
BAV en un infarto anterior hay que pensar que se ha necrosado parte del septo que contiene el
sisinfrahisiano de conducción, lo que oscurece el pronóstico dado que probablemente no responda
a atropina y precise de estimulación interna o externa.(R2)

201. El diafragma pélvico está formado por los siguientes músculos:

1. 1. El elevador del ano y coccígeo.


2. 2. Transverso superficial del periné y elevador del ano.
3. 3. Elevador del ano y puborectal.
4. 4. Bulbocavernosos y coccígeo.

 
 
 
 
Gráfico de respuestas
Comentario

Pregunta muy dificil sobre anatomía del suelo pélvico. La respuesta es la 1 elevador del ano y
coccígeo.(R1)

202. Un paciente diagnosticado de enfermedad de Hodgkin en estadio IIIB recibió


quimioterapia tipo MOPP, alcanzando remisión completa. A los 18 meses de remisión
completa se objetivó recidiva diseminada. ¿Cuál sería el tratamiento correcto?:

1. 1. MOPP.
2. 2. ABVD.
3. 3. ABVD más radioterapia ganglionar total.
4. 4. Radioterapia ganglionar total.
Gráfico de respuestas
Comentario
En la actualidad se considera al ABUD como el mejor tratamiento poliQT de la EH.(R1)

203. Una paciente de 19 años es traída a la urgencia por su familia, quien sospecha la
ingestión de alguna sustancia; la chica se muestra desinhibida, con severa inquietud
psicomotora y sin tolerar los límites que se le imponen; algo recelosa del personal
sanitario, se agita al ver aparecer a los de seguridad, emprendiéndola a golpes contra
ellos, por lo que precisa contención mecánica y sedación farmacológica; ¿qué fármaco
de los siguientes preferiremos para tranquilizarla?

1. 1. Diacepam.
2. 2. Fentanilo.
3. 3. Haloperidol.
4. 4. Clormetiazol.
Gráfico de respuestas
Comentario

El cuadro clínico parece una intoxicación por cocaína, o tal vez por otra droga, pero en cualquier
caso una sustancia activadora del sistema nervioso central. El cuadro de agitación probablemente
sea debido a una psicosis tóxica. Ante un cuadro así, lo que debe hacerse es emplear un
neuroléptico incisivo, de acción rápida y contundente. Entre las opciones que nos ofrecen, el único
que se adapta a este perfil es el haloperidol (respuesta 4 correcta). La clorpromacina, como el
haloperidol, es un neuroléptico, pero su perfil es más sedativo y su efecto es menos contundente.

El diacepam es una benzodiacepina y, por tanto, tiene acción sedante y ansiolítica. El problema
que plantea es que, en este caso, deberíamos emplear la vía intramuscular (la intravenosa sería
difícil, en un paciente agitado…). La absorción de las benzodiacepinas por vía intramuscular es
errática, por lo que no nos lo podríamos permitir si pretendemos que se absorba de forma
inmediata.

El clometiazol (opción 5) se utiliza para el síndrome de abstinencia a alcohol, como ansiolítico y


sedante. Por último, el fentanilo (opción 3) es un opiáceo muy potente, pero en este caso no
necesitamos un analgésico.(R3)

 
 
 
 
204. Una mujer de 45 años acude a consulta por presentar un cuadro de fiebre alta,
escalofríos, mialgias y cefalea. Tiene una temperatura de 39ºC. La exploración
cardiopulmonar es normal a excepción de la existencia de una bradicardia relativa con
respecto a la intensidad de la fiebre (Fc 75 l.p.m.-39ºC). La exploración abdominal
evidencia dolor difuso a la palpación profunda, no se palpan organomegalias y se
aprecian máculas pequeñas de color rojo pálido que desaparecen con la presión.
Analíticamente destaca una leucopenia de 4.500/mm3. Su diagnóstico de sospecha es:

1. 1. Shigelosis.
2. 2. Sarampión.
3. 3. Tularemia.
4. 4. Fiebre tifoidea.
Gráfico de respuestas
Comentario
Es conveniente memorizar esta pregunta, pues representa la clínica característica de una fiebre
tifoidea, causada por Salmonella typhi: fiebre, cefalea, bradicardia relativa, dolor abdominal difuso,
leucopenia y roséola tifoidea, apareciendo ésta última sobre todo al inicio de la segunda semana.
Penetra en el organismo por la ingesta de agua y verduras contaminadas. El diagnóstico precoz se
basa en el hemocultivo (en las 2 primeras semanas del cuadro es el procedimiento más rentable) o
en el coprocultivo ( a partir de la tercera semana). Respecto al tratamiento, debido a la aparición de
cepas de S. Typhi resistentes a diversos antibióticos, el tratamiento actualmente recomendado son
las quinolonas o cefalosporinas de 3ª generación. Opción 5 correcta.(R4)

205. Respecto a los estrógenos, señale cuál de las siguientes afirmaciones es FALSA:

1. 1. Tienen una curva de liberación unimodal.


2. 2. Bloquean la acción de la prolactina sobre la mama.
3. 3. Se sintetizan en la granulosa.
4. 4. Existe un pico de estrógenos 24-36 horas antes de la ovulación.
Gráfico de respuestas
Comentario

Debe recordar los efectos de las hormonas en el ciclo genital femenino. Los estrógenos tiene una
liberación bimodal: crecen hasta el pico preovulatorio (24-36 h antes de la ovulación) y tiene otro
pico menor en la fase lútea. Son tróficos para todo el aparato genital. A nivel local inducen
receptores de FSH. Niveles bajos y moderados inhiben la secreción de FSH y niveles altos tiene un
efecto gatillo sobre LH. Estimulan el crecimiento y la proliferación de los órganos sexuales
femeninos. Bloquean a la PRL en la excreción de la leche. La disminución de los niveles
circulantes de estrógenos tiene que ver en la causa de la osteoporosis postmenopáusica.(R1)

206. A 30-year-old pregnant woman at 29 weeks' gestation goes to see her obstetrician
complaining of a one-week history of lack of fetal movements. Ultrasonography shows
absence of fetal vital signs. Blood test results are the following: Fibrinogen 180 mg/dL,
PT 17 sec, APTT 30 sec, Platelets 170,000, Hemoglobin 13, Leukocytes 4500. Which of the
following is the most appropriate management?

1. 1. Labor induction.
2. 2. Cesarean delivery.
3. 3. Periodic observation.
4. 4. Blood transfusion.
Gráfico de respuestas

 
 
 
 
Comentario
Labor induction. This case report describes an intrauterine fetal death, confirmed by analytical and
ultrasound parameters. The retention of a dead fetus can be life-threatening to the mother, resulting
in a consumption coagulopathy. Fetal extraction must be immediate, in a physiological way if
possible.(R1)

207. El tratamiento sustitutivo con diálisis de la insuficiencia renal crónica está indicado
generalmente cuando el filtrado glomerular es inferior a 10 ml/min, pero hay situaciones
que condicionan un inicio más precoz. Señale cuál de las siguientes respuestas NO
corresponde a uno de estos casos:

1. 1. Diabetes mellitus.
2. 2. Insuficiencia cardiaca congestiva.
3. 3. Polineuropatía.
4. 4. Cáncer avanzado.
Gráfico de respuestas
Comentario
Es el momento de mirarse tanto las indicaciones como las contraindicaciones de la diálisis,
sabiéndote cualquiera de las dos prodrías contestar la pregunta. Según la Sociedad Española de
Nefrología, las indicaciones de diálisis son: 1. Síntomas urémicos: náuseas, vómitos? 2.
Pericarditis urémica 3. Hiperpotasemia no por exceso dietético o tratamiento con IECA 3. Acidosis
metabólica severa no controlable con tratamiento médico 4. Sobrecarga de volumen que no sea
por transgresión dietética (sal) 5. Edema pulmonar no debido a insuficiencia cardíaca
descompensada y que no responda a medidas terapéuticas simples 6. Desnutrición: albúmina
inferior a 4 g% que no sea por pérdidas urinarias 7. Encefalopatía urémica: desorientación,
confusión, flapping, mioclonías o crisis convulsivas 8. Tendencia hemorrágica con sangrado
atribuible a uremia 9. HTA que no responda bien al tratamiento 10. Aclaramiento de creatinina 5
ml/min o concentración de creatinina superior a 12 mg% con peso > 70 kg (10 ml/min para
pacientes diabéticos). Pero ojo hay situaciones que aunque no son contraindicaciones hacen que
los pacientes no se incluyan en diálisis: Demencia grave, estados vegetativos persistentes,
tumores muy avanzados, enfermedad pulmonar o hepática terminal?.(R4)

208. ¿Cuál es el síntoma clínico predominante en las fibrosis pulmonares?:

1. 1. Expectoración.
2. 2. Hemoptisis.
3. 3. Disnea de esfuerzo.
4. 4. Dolor costal.
Gráfico de respuestas
Comentario

Tema poco preguntado en el ENARM, por lo que debe repasar lo más importante. Pregunta de
dificultad moderada sobre la fibrosis pulmonar.

En la fibrosis pulmonar se produce una alteración en el intersticio pulmonar que da lugar a una
alteración en el intercambio gaseoso por trastornos en la difusión, y esto se traduce en disnea.

En condiciones normales, el intercambio gaseoso tiene lugar en el tercio inicial del recorrido del
capilar por el alveolo, de forma que en los restantes 2/3 no hay difusión de gases porque ya no hay
gradiente de presión. Esto sería una especie de reserva.

 
 
 
 
Las enfermedades en las que hay alteración de la difusión no suelen producir disnea en reposo
porque se utiliza este espacio de “reserva”, pero durante el ejercicio aumenta la velocidad de la
sangre a su paso por el pulmón, de forma que esa “reserva” se vuelve insuficiente y aparece
disnea durante el ejercicio.(R3)

209. Paciente de 26 años, fumadora, sin hijos ni pareja


estable, es remitida a su consulta de ginecología por citología positiva para HSIL (lesión
intraepitelial de alto grado). Usted realiza una colposcopia ante semejante hallazgo,
obteniéndose la imagen que se muestra. ¿Cuál es la conducta más indicada a
continuación?:

1. 1. Tratamiento con antiinflamatorios locales.


2. 2. Estudio de extensión de la enfermedad.
3. 3. Biopsia de la lesión.
4. 4. Conización cervical.
Gráfico de respuestas
Comentario

En la imagen se ve ina lesión en "mosaico", con áreas blancas separadas por vasos, dando ese
aspecto característico. Es un patrón sugestivo de lesión neoplásica o preneoplásica de cérvix; por
tanto, el primer paso es biopsiarlo para confirmar su naturaleza maligna.(R3)

210. Con respecto a la patología que presenta la paciente, ¿cuál de los siguientes
hallazgos NO sería sugestivo de patología cervical?:

1. 1. Eritroplasia cervical.
2. 2. Áreas de neoformación vascular.
3. 3. Superficie ulcerada.
4. 4. Leucoplasia.
Gráfico de respuestas
Comentario

A continuación, te recordamos los hallazgos colposcópicos anormales:

1.- Epitelio acetoblanco.

 
 
 
 
2.- Epitelio yodonegativo.

3.- Cambios de coloración (leucoplasia).

4.- Neoformación vascular.

5.- Superficie irregular, con pérdida del epitelio normal, con ulceración.

La ectopia o eritroplasia cervical consiste en la presencia de epitelio cilíndrico por debajo del orificio
cervical externo, visible en la colposcopia desde la vagina, quedando en contacto con el medio
vaginal hostil. Aunque la mayor parte de las veces son lesiones asintomáticas, pueden originar
leucorrea y hemorragia postcoital. El diagnóstico se realiza de visu mediante colposcopía, y
pueden eliminarse si producen clínica (a veces produce sangrado). No obstante, no se relaciona
con patología cervical maligna (respuesta 1 correcta).(R1)

211. Una mujer de 60 años ha sido intervenida mediante tumorectomía de un carcinoma


de mama. Se trataba de un tumor de 2 cm con ganglios libres y receptores hormonales
negativos. ¿Cuál de las siguientes opciones de tratamiento recomendaría?

1. 1. Quimioterapia con un taxano.


2. 2. Radioterapia adyuvante.
3. 3. Tamoxifén adyuvante.
4. 4. Está curada, puede realizarse reconstucción mamaria.
Gráfico de respuestas
Comentario

Cuando se trata de un tumor de pequeño tamaño, ha de saber que la mastectomía radical ofrece
los mismos resultados que la tumorectomía, siempre que ésta se acompañe de radioterapia
(respuesta 2 correcta). La clave está en que el tumor que nos describen es de 2 cm. En este caso,
no estaría indicado emplear quimioterapia, puesto que los ganglios axilares son negativos. Por otra
parte, la negatividad de los receptores hormonales hace que no necesitemos añadir
tamoxifeno.(R2)

212. De las técnicas serológicas y microbiológicas que se mencionan, ¿cuál elegiría para
la comprobación diagnóstica de la sífilis en período primario?

1. 1. TPI (Prueba de inmovilización de Nelson).


2. 2. FTA (Prueba de inmunofluorescencia indirecta).
3. 3. VDRL.
4. 4. Examen del exudado al campo oscuro.
Gráfico de respuestas
Comentario

Dentro de la sífilis, lo más importante son las pruebas serológicas.

La sífilis primaria aparece tras un periodo de incubación de 21 días, y su lesión característica es el


chancro duro, que aparece en el lugar de inoculación. Es una lesión sobreelevada, de consistencia
cartilaginosa, no dolorosa, de fondo limpio, sin exudado y normalmente única. Se acompaña de
adenopatías regionales, normalmente inguinales, bilaterales que, al igual que el chancro, son de
consistencia dura, no dolorosas y no supuran.

 
 
 
 
El diagnóstico de elección, en este momento, es la visualización del treponema en microscopio de
campo oscuro (R- 5). Ten en cuenta que, en el período primario, hay pacientes que todavía no han
realizado la seroconversión y, por tanto, las pruebas serológicas podrían dar falsos negativos.(R4)

213. En una paciente de 70 años con arteritis de células gigantes. ¿Cuál de los siguientes
afirmaciones consideraría FALSA?

1. 1. El parámetro analítico que mejor se correlaciona con la evolución clínica es la anemia.


2. 2. El síntoma más habitual es la cefalea.
3. 3. La claudicación mandibular es un dato típico.
4. 4. Una biopsia normal no excluye definitivamente el diagnóstico.
Gráfico de respuestas
Comentario

Aunque puede acompañarse de anemia, el parámetro analítico más útil tanto en el diagnóstico
como en el seguimiento de la evolución posterior es la VSG. La anemia, cuando aparece, sería de
tipo trastorno crónico, y la correlación con la evolución no es tan valorable como la VSG.(R1)

214. Varón de 31 años consulta por episodios repetidos de artritis inflamatoria


oligoarticular asimétrica de medianas articulaciones desde hace tres años, cuya duración
es de 2-4 semanas. Actualmente presenta unas úlceras de aspecto sucio en la boca, tiene
fiebre, ligero dolor abdominal, cefalea intensa y signos de tromboflebitis en la pantorrilla
derecha. ¿Cuál es el diagnóstico más probable?

1. 1. Enfermedad de Whipple.
2. 2. LES.
3. 3. Síndrome de Reiter.
4. 4. Síndrome de Behçet.
Gráfico de respuestas
Comentario

Recuerda los criterios diagnósticos de la enfermedad de Behçet:

- Aftas orales (imprescindible)

- Aftas genitales

- Lesiones cutáneas

- Inflamación ocular (uveítis anterior)

- Patergia

Otros síntomas característicos son:

- Artritis y artralgias

- Clínica neurológica

 
 
 
 
- Inflamación del tubo digestivo

- Trombosis venosa profunda ó tromboflebitis superficial.

(R4)

215. A 5-month-old girl is brought to the pediatrician's consultation for presenting delayed
psychomotor development. This morning her mother found an abdominal mass while she
was bathing him in the bathtub. The doctor realizes that it affects the liver and the spleen,
and maybe also both kidneys. Her belly is markedly protruded. She has a fatty rounded
'doll's face'. Her development is very delayed in relation to other kids (lower than
percentile 3). Laboratory blood test results show markedly elevated levels of cholesterol,
uric acid, triglycerides and lactic acid, but no hypertransaminasemia. What is the most
likely diagnosis?

1. 1. Gaucher's disease
2. 2. Down's syndrome
3. 3. Von-gierke's disease
4. 4. Congenital biliary atresia
Gráfico de respuestas
Comentario
Von-Gierke's disease. Glycogen storage disease type I or von Gierke's disease, is the most
common of the glycogen storage diseases. This genetic disease results from deficiency of the
enzyme glucose-6-phosphatase. There is an impairment of liver gluconeogenesis, so this child has
predisposition to hypoglycemia (that may be fatal) and lactic acidosis. Non-metabolized glycogen
accumulates in abdominal organs causing visceromegaly. Chronic hypoglycemia leads to lack of
insulin action so triglycerides are increased. In VHB infection, we would expect to to find abnormal
elevation of liver enzymes and no visceromegaly unless cirrhosis occurs.(R3)

216. Una mujer de 47 años de edad acude a la consulta refiriendo sequedad vaginal de
varios meses de evolución, con relaciones sexuales dolorosas, a lo que se suma la
reciente aparición de sofocos, así como un estado de irritabilidad que, según ella, se ve
empeorado por las dificultades que presenta para conciliar el sueño. En relación a las
modificaciones endocrinas que caracterizan este período, señale la respuesta
INCORRECTA:

1. 1. Durante la menopausia existen niveles elevados de FSH.


2. 2. Durante la menopausia existen niveles elevados de LH.
Durante la menopausia, los niveles de testosterona de origen ovárico disminuyen
3. 3.
drásticamente.
4. 4. Durante la menopausia, las obesas tienen mayor producción de estrógenos periféricos.
Gráfico de respuestas
Comentario

Pregunta fácil sobre menopausia que podemos sacar por descarte si no sabemos la correcta.

Se sabe que las gonadotropinas aumentan en la menopausia, con mayor aumento de FSH
(opciones 1 y 2). Durante la menopausia, las mujeres obesas tienen mayor producción de
estrógenos periféricos ya que éstos se sintetizan en la grasa, y el estrógeno periférico más
importante es la estrona, que en la menopausia es el estrógeno principal (opción 4).

 
 
 
 
La opción a elegir por descarte será la 3, pero lo explicaremos a continuación. Durante la
menopausia la testosterona plasmática global disminuye muy poco, habiéndo demostrado un
aumento de testosterona de origen ovárico, ya que la LH estimularía la síntesis de andrógenos en
el estroma ovárico. Esto justifica que, en algunas mujeres ancianas, aparezca mayor vello facial
que en sus años jóvenes.(R3)

217. NO es un objetivo del ultrasonido de tercer trimestre:

1. 1. Identificar la vitalidad y la estática fetal.


2. 2. Diagnostico de anomalías estructurales y marcadores de cromosomopatía.
Diagnosticar anomalías de la localización placentaria y del volumen del líquido
3. 3.
amniótico.
4. 4. En casos indicados, estudios de flujo feto-placentarios.
Gráfico de respuestas
Comentario

Pregunta muy importante y sencilla sobre el papel del ultrasonido. En el primer trimestre nos sirve
para identificar el número de embriones, en el caso de gestación múltiple, diagnostico de
cigosidad, identificación del latido cardiaco embrionario, estimación de la edad de gestación,
detección y medida de la translucencia nucal (marcador de cromosomopatía fetal), observación de
la morfología embrionaria y por último identificar la existencia de patología uterina y de los anejos.
En el segundo trimestre el objetivo es el diagnostico de anomalías estructurales y marcadores de
cromosomopatías y si no se ha realizado la ecografía de nivel básico del primer trimestre, incluye
sus objetivos. En el tercer trimestre los objetivos son identificar la vitalidad y la estática fetal,
estimar el crecimiento fetal, diagnostico de anomalías de la localización placentaria (placenta
previa), diagnosticar anomalías del volumen del liquido amniótico y en casos indicados, estudios de
flujo feto-placentarios.(R2)

218. Un RNT, sin antecedentes de interés al que se le realizó profilaxis ocular y


antihemorrágica a los pocos minutos del nacimiento, presenta a las 48 horas de vida
sangrado por el cordón umbilical y sangre fresca en heces. Señale la INCORRECTA:

1. 1. No estará indicado administrar vitamina K, porque ya ha recibido una dosis de profilaxis.


Un factor de riesgo para la aparición de este cuadro es el tratamiento materno con
2. 2.
fenitoína.
3. 3. Estaría indicado realizar un estudio de coagulación.
La enfermedad hemorrágica por déficit de vitamina K de comienzo tardío (>1 semana)
4. 4.
puede estar asociada a una hepatitis neonatal o atresia biliar.
Gráfico de respuestas
Comentario

Concepto importante. La enfermedad hemorrágica del recien nacido suele ser por déficit de
vitamina K, por lo tanto no está contrainicada su administración aunque ya se haya recibido una
dosis. Respuesta 1 incorrecta.(R1)

219. Un hombre de 49 años, con antecedentes de ingesta de etanol de 60 g/día,


asintomático, presenta en una revisión rutinaria: Hb 16 g/dl, Hto. 42%, ASAT 80 U/L, ALAT
65/U/L, GGT 72 U/L, F alcalina y LDH normales. Serología para virus de hepatitis A y C
negativos, virus B: AntiHbc positivo; Hbs Ag, Ac HBc Ig M y Hbe Ag: negativos, DNA del
virus B; negativo. Fe 210 microgr/dl, Ferritina 1,050 ng/ml. Colesterol 225 mg/dl.
Endoscopia alta: sin alteraciones hasta segunda porción duodenal. El USG abdominal

 
 
 
 
muestra un hígado con aumento de tamaño y ecogenicidad discretamente aumentada. Si
realizase un estudio genético, ¿qué esperaría encontrar?

1. 1. Alteraciones del gen APC, en cromosoma 5.


2. 2. Alteraciones del oncogén supresor p53.
3. 3. Mutación del gen C282Y.
4. 4. Una alteración en el cromosoma 6, de herencia recesiva ligada al sexo.
Gráfico de respuestas
Comentario

En este paciente con hígado aumentado de tamaño y ecogenicidad aumentada, con niveles de
hiero y ferritina elevados debe sospechar una hemocromatosis. La hemocromatosis se asocia a
mutación en el gen HFE y en la mayoría de los casos existe una mutación del gen C282Y.(R3)

220. Un día en que la presión atmosférica es de 705 mmHg, un paciente de 40 años se


presenta en la sala de Urgencias con una presión arterial de oxígeno de 37 mmHg, una
presión de anhídrido carbónico de 82 mmHg y un pH de 7,22. Calculamos la presión
alveolar de oxígeno, que resulta ser 39 mmHg. Entre las causas de insuficiencia
respiratoria mencionadas más abajo, ¿cuál será la más probable?

1. 1. Una embolia de pulmón.


2. 2. Tiene una crisis asmática grave.
3. 3. Una neumonía extensa.
4. 4. Una sobredosis de morfina.
Gráfico de respuestas
Comentario

Pregunta fácil (por ser ampliamente explicada en clase) de un tema constante en el MIR.

Una vez más en el MIR nos preguntan sobre las causas de hipoxemia y el modo de diferenciarlas.
Se considera hipoxemia cuando la PaO2 es menor de 80 mmHg e hipercapnia cuando la PaCO2
es mayor de 45 mmHg. La insuficiencia respiratoria se define con una PaO2 menor de 60 mmHg y
es global si, además, añade una PaCO2 mayor de 50 mmHg. Existen varios mecanismos que
ocasionan disminución del O2 en la sangre, que pueden diferenciarse según el valor de la PaCO2,
de la D(A-a) y la respuesta al tratamiento con oxígeno suplementario. (ver los esquemas de la
página siguiente).

En nuestra pregunta, los datos que nos sirven para llegar a la causa más probable de hipoxemia
del paciente son:

•   PaO2 de 37 mmHg (por tanto, está en insuficiencia respiratoria).


•   PaCO2 de 82 mmHg (es decir, tiene retención de carbónico, así que la insuficiencia
respiratoria es global).
•   D(A-a)O2 = 39 mmHg – 37 mmHg = 2 mmHg (normal <15 mmHg).

Tenemos, en consecuencia, un paciente que está hipoventilando y cuya D(A-a)O2 es normal (la
causa no está en el pulmón). La única opción de las que nos ofrecen que concuerda con estos
datos es la 5: sobredosis de morfina (produciría hipoventilación por alteración del centro
respiratorio). En el caso de un EPOC reagudizado (opción 2) o una crisis asmática grave (opción 3)
podríamos encontrar retención de carbónico, pero la D(A-a)O2 estaría elevada (el mecanismo
responsable sería la alteración en la relación V/Q).

 
 
 
 
En el caso de la embolia de pulmón (opción 1), el mecanismo responsable también es la alteración
V/Q, pero la PCO2 estaría disminuida en lugar de aumentada por la hiperventilación del paciente.
Por último, en la neumonía (opción 4) lo que da lugar a hipoxemia es un mecanismo de shunt y
encontraríamos una PCO2 disminuida por la hiperventilación y una D(A-a)O2 aumentada.(R4)

Algoritmo diagnóstico de la hipoxemia

221. Femenino de 72 años de edad, con 7 dias de dolor en el cuadrante superior derecho,
ictericia progresiva y temperatura de 39 ºC recibió tratamiento con cefalexina vía oraL,
sin mejoría. Ingresa a urgencias hipotensa, con compromiso del sensorio. Leucocitos:
19,000 x mm2, bilirrubinas, transaminasas y fosfatasa alcalina elevadas, concentración
de amilasa sérica normal. ¿Cuál es su diagnostico?

1. 1. Colelitiasis.
2. 2. Enfermedad de Wilson.
3. 3. Colecistitis aguda alitiásica.
4. 4. Colangitis supurativa aguda.
Gráfico de respuestas
Comentario

Cuadro clínico típico de libro compatible con colangitis.

El paciente presenta la triada de Charcot: fiebre, dolor abdominal e ictericia.

Pentada de Reynolds: hipotensión y cambios en el estado mental + triada de Charcot.

 
 
 
 
Respuesta 4 correcta.(R4)

222. Respecto a la hemorragia postparto precoz (HPP), ¿cuál de las siguientes


afirmaciones es INCORRECTA?:

1. 1. La HPP complica del 5-15% de todos los partos.


2. 2. Actualmente supone una causa rara de mortalidad materna.
3. 3. Suponen el 50% de las hemorragias postparto.
4. 4. La hemorragia posparto tiende a repetirse en sucesivos partos.
Gráfico de respuestas
Comentario

La HPP complica del 5-15% de todos los partos. Es la causa aislada más importante de mortalidad
materna. La mitad de las HPP son precoces (las que ocurren en las primeras 24 horas posparto).
La OMS estima en 150,000 el número de muertes anuales por HPP en países en vías de
desarrollo. La hemorragia posparto tiende a repetirse en sucesivos partos. La medida más efectiva
debe radicar en su prevención.(R2)

223. ¿Cómo se denomina la prueba de laboratorio que se basa en la capacidad de la IgM


para aglutinar hematíes de carnero sensibilizados con IgG de conejo dirigida contra los
hematíes del carnero?

1. 1. Waaler-Rose.
2. 2. Singer-Plotz.
3. 3. ANCA.
4. 4. AC anti-fosfolípidos.
Gráfico de respuestas
Comentario
Recuerda que el factor reumatoide es un anticuerpo dirigido contra epítopes localizados en el
fragmento Fc de la inmunoglobulina G, que suelen ser anticuerpos de clase IgM. Hasta hace
algunos años, los FR se detectaban por pruebas de aglutinación como la prueba de Waaler- Rose,
que se basa en la capacidad que tienen los FR para aglutinar hematíes de carnero sensibilizados
con suero de conejo anticarnero. La prueba de látex es similar, en vez de hematíes de carnero se
utilizan partículas de látex recubiertas con Ig G humana purificada. Las pruebas de aglutinación
resultan muy poco específicas.(R1)

224. Un RN se ha intubado en la sala de partos por presentar apnea desde el nacimiento.


La madre había recibido un derivado opiáceo. ¿Cuál de los siguientes fármacos y qué vía
de administración utilizaría de 1ª elección?:

1. 1. Naloxona por vía intratraqueal.


2. 2. Adrenalina por vía intratraqueal.
3. 3. Bicarbonato sódico por vía intravenosa.
4. 4. Teofilina por sonda nasogástrica.
Gráfico de respuestas
Comentario

Una causa de depresión respiratoria neonatal es la administración de opiáceos como anestésico a


la madre. La depresión respiratoria provocada se reversibilizará a través de la administración al
recién nacido de naloxona. La naloxona, antagonista opiáceo, se puede administrar por vía
intratraqueal aprovechando la intubación del niño (respuesta correcta 1) es e tratamiento de

 
 
 
 
elección no solo en RN también en depresiones respiratorias en adultos esto es muy preguntado
en el ENARM.(R1)

225. Una paciente de 67 años acude a la consulta por fugas de orina a lo largo del día que
le incomodan muchísimo y condicionan terriblemente su vida ya que se producen con
mínimos esfuerzos. Se trata de una paciente muy activa y limita sus clases de baile e
incluso hacer la compra. Tiene sobrepeso moderado ya que después de sus 4 partos
nunca volvió a su IMC habitual. Varios cultivos de orina han resultado negativos. Ha
realizado ejercicios de suelo pélvico e incluso ha utilizado conos vaginales, sin
resultados hasta el momento. Niega urgencia miccional en ninguna situación.

Se trata de una incontinencia urinaria de esfuerzo pura, y probablemente la paciente se


1. 1.
beneficiará de la colocación de un sling suburetral.
Se trata de una incontinencia urinaria de esfuerzo pura, y deberá ser tratada con
2. 2.
anticolinérgicos.
Se trata de una incontinencia urinaria de esfuerzo, hecho que deberá ser comprobado
3. 3.
mediante un estudio urodinámico antes de proponer tratamiento alguno.
Se trata de una incontinencia urinaria mixta y deberá ser tratada con anticolinérgicos más
4. 4.
sling ureteral.
Gráfico de respuestas
Comentario
Se trata de una incontinencia pura de esfuerzo en una paciente con varios factores de riesgo:
multípara, edad, sobrepeso. En principio no precisará estudio urodinámico para su diagnóstico, ya
que según la ICS (International Continence Society) sólo será necesario en aquellas incontinencias
mixtas o de urgencia (no a las de esfuerzo) que no respondan a tratamiento conservador, vayan a
ser tratadas mediante procedimientos invasivos o vayan a ser incluidas en algún estudio.(R1)

226. An 85-year-old man with a history of hypertension, diabetes mellitus, ischemic heart
disease and COPD comes to his general practitioner for progressive weakness.
Hemogram shows: - Leukocyte count 6,500/microL - Hemoglobin 7 g/dL - MCV 105 fl -
Platelet count 180,000/microL - Reticulocyte count 25,000/microL Blood smear study
reveals neutrophils with hyposegmented nucleus. Which of the following is the most
appropriate treatment for this patient’s illness? man with a history of hypertension,
diabetes mellitus, ischemic heart disease and COPD comes to his general practitioner for
progressive weakness. Hemogram shows:

1. 1. Iron, folate and cobalamin.


2. 2. Folate and cobalamin.
3. 3. Steroids.
4. 4. Periodic transfusion of packed red cells.
Gráfico de respuestas
Comentario
El frotis y la anemia arregenerativa con ligera macrocitosis en un anciano hacen que la
mielodisplasia sea el diagnóstico más probable. En pacientes como el referido, de edad muy
avanzada y pluripatología, las transfusiones periódicas son el tratamiento habitual.(R4)

227. En una laparotomía por un tumor ovárico, encontramos afectación de ambos


ovarios. La biopsia intraoperatoria informa de la existencia de numerosas células en
anillo de sello, con lo que se confirma su naturaleza metastásica. ¿Dónde buscaría usted
el tumor primario?:

 
 
 
 
1. 1. En el riñón
2. 2. En el pulmón.
3. 3. En el tubo digestivo.
4. 4. En las glándulas salivares.
Gráfico de respuestas
Comentario

Lo que nos describe este caso clínico es un tumor de Krukenberg. Acerca del mismo, debe
recordar las siguientes características:

- Origen más frecuente: estómago, aunque puede proceder de otras regiones del tubo
digestivo.

- Histología: células en anillo de sello.

- Llegada al ovario por vía hematógena, es decir, se trata de una metástasis.


(R3)

228. Una de las siguientes manifestaciones clínicas NO es propia de la sífilis tardía:

1. 1. Insuficiencia valvular aórtica.


2. 2. Lesiones cutáneas.
3. 3. Adenopatías generalizadas.
4. 4. Lesiones óseas.
Gráfico de respuestas
Comentario
Las ETS son un tema bastante preguntado en el examen MIR; cuando hablamos de sífilis tardía
nos referimos a aquella que acontece a partir de un año postinfección, y dentro de ella englobamos
la sífilis terciaria y cuaternaria. La lesión característica de la sífilis terciaria es el goma, lesión
granulomatosa que con frecuencia afecta piel, mucosas y sistema musculoesquelético, sobre todo
huesos largos ( opción 2 y 5 ). También pertenecen a esta fase otros cuadros: cardiovasculares-
recordar que típicamente se afecta la aorta ascendente ( pudiendo producir insuficiencia aórtica,
opción 1 )- , neurológicos ( neurosífilis asintomática, meningitis crónica, ACVA) y oftalmológicos (
coriorretinitis, opción 4 ) . Hay dos cuadros de neurosífilis que se incluyen dentro de la sífilis
cuaternaria: tabes dorsal ( desmielinización de cordones posteriores ), y parálisis general
progresiva ( con las características pupilas de Argyll- Robertson). Las adenopatías generalizadas
son más propias de la sífilis secundaria, que acontece antes del primer año postinfección ( por lo
tanto, la respuesta correcta es la opción 3 ).(R3)

229. El protocolo del cáncer de ovario incluye todos, EXCEPTO:

1. 1. Apendicectomía.
2. 2. Omentectomía.
3. 3. Linfadenectomía paraaórtica.
4. 4. Linfadenectomía inguinal.
Gráfico de respuestas
Comentario

El tratamiento quirúrgico en el cáncer de ovario tiene una doble utilidad: diagnóstica al permitir
estadificar adecuadamente a las pacientes, y terapéutica, extirpando la mayor cantidad de masa
tumoral posible. La cirugía consiste en:

 
 
 
 
Lavado y aspiración de líquido peritoneal.

Histerectomía total con anexectomía bilateral.

Linfadenectomía pélvica y paraaórtica.

Omentectomía (extirpación del epiplón).

Exploración de la superficie peritoneal y biopsia de lesiones sospechosas, además de biopsiar el


peritoneo vesical, el fondo de saco de Douglas, los espacios parietocólicos y la cúpula
diafragmática.

Apendicectomía, especialmente en los tumores mucinosos.

Entre estos no se incluye la linfadenectomía inguinal. Respuesta 4.(R4)

230. Las hemorroides se consideran de tercer grado cuando:

1. 1. Son sangrantes.
2. 2. Se prolapsan temporalmente en el momento de la defecación.
3. 3. Se prolapsan indefinidamente y han de reducirse manualmente.
4. 4. Se trombosan.
Gráfico de respuestas
Comentario

Las hemorroides internas se producen por dilatación del plexo venoso hemorroidal interno formado
por venas rectales superior y media.

Se clasifican en IV grados según la intensidad del prolapso.

El grado I permanece en recto y su tratamiento es conservador.

El grado II prolapsa a través del ano cuando el paciente puja, reduciéndose espontáneamente y se
trata con ligadura de vaina de caucho o bien esclerosis.

El grado III prolapsa cuando el paciente puja pero es necesaria la restitución manual y el
tratamiento es con ligadura de vaina de caucho.

El grado IV es el prolapso persistente cuyo tratamiento es la hemorroidectomía.(R3)

231. ¿Cuál de las siguientes NO es un componente del denominado síndrome CREST,


manifestación típica de la esclerosis sistémica con afectación cutánea limitada?:

1. 1. Calcinosis cutánea.
2. 2. Fenómeno de Raynaud.
3. 3. Sindesmofitos.
4. 4. Telangiectasias.
Gráfico de respuestas
Comentario

 
 
 
 
Las manifestaciones del síndrome de CREST son Calcinosis, fenómeno de Raynadu, afectación
Esofágica (típicamente hipomotilidad de 2/3 inferiores, y disfunción de EEI), eSclerodactilia y
Telangiectasias.(R3)

232. Un paciente de 33 años, agricultor, no fumador, consulta porque en las últimas


semanas presenta episodios de disnea, febrícula, tos seca, astenia y anorexia. En la
radiografía de tórax se observa un patrón alveolo-intersticial. ¿Qué diagnóstico debe
sospechar en primer lugar?:

1. 1. Histiocitosis de células de Langerhans.


2. 2. Fibrosis pulmonar idiopática.
3. 3. Silicosis.
4. 4. Neumonitis por hipersensibilidad.
Gráfico de respuestas
Comentario
Se trata de un cuadro subagudo de neumonitis por hipersensibilidad, que es un conjunto do
enfermedades producidas por la inhalación de polvos orgánicos. La clave para el diagnóstico es la
profesión del paciente, pues la exposición a agentes orgánicos tiene lugar habitualmente en el
medio laboral. Las entidades más frecuentes son el pulmón del granjero, que sería este caso,
producido por inhalación de esporas de gérmenes que crecen en el grano almacenado
(termoactinomices, aspergilus), el pulmón del cuidador de aves y el pulmón del humidificador.(R4)

233. Mujer de 24 años en diálisis por hialinosis segmentaria y focal. Recibió un trasplante
renal de cadáver y a los 30 días de evolución, presentó proteinuria >4.5 gr/día. ¿Cuál es
el diagnóstico más probable?:

1. 1. Glomerulopatía del traplante.


2. 2. Rechazo agudo.
3. 3. Recidiva de la enfermedad original.
4. 4. Enfemedad de injerto contra huésped.
Gráfico de respuestas
Comentario
Pregunta de dificultad moderada que consiste en un caso clínico en el que aparece proteinuria tras
un trasplante, que nos debe hacer sospechar una recidiva de la HSF (RESPUESTA 2
CORRECTA). En relación con el trasplante renal y la glomérulopatías debemos conocer
principalmente cuáles son las GN que recidivan tras el trasplante: GNRP tipo III (pauciinmune),
GNMP tipo II (enfermedad por depósitos densos en la MBG) y GNFS. Con menor frecuencia, la GN
membranosa también puede hacerlo o aparecer de novo.(R3)

234. En relación con la edad gestacional (EG), marque la CORRECTA:

1. 1. El ultrasonido prenatal tiene mayor dispersión y confiabilidad a mayor EG.


2. 2. La valoración neuromuscular es suficiente para determinar la EG.
3. 3. Los periodos de adaptación podrían alterar al Balllard y al Dubowitz.
4. 4. Siempre hay concordancia con los datos obstétricos.
Gráfico de respuestas
Comentario

 
 
 
 
Respuesta sencilla de la edad gestacional. La respuesta correcta es la 3, ya que por obvias
razones los periodos de adaptación pueden alterar la escala de Ballard y de Dubowitz.

El ultrasonido prenatal tiene una mayor confiabilidad a menor EG. La valoración neuromuscular no
es suficiente para determinar la EG, existen otras determinaciones más sensibiles y específicas y
no todos los datos obtenidos serán concordantes entres sí.(R3)

235. Paciente de 50 años con cirrosis grado


B de Child, presenta en la TAC lo que muestra la imagen nº ##. ¿Cuál entre los siguientes
sería el tratamiento de elección?

1. 1. Quimioembolización.
2. 2. Sorafenib.
3. 3. Alcoholización.
4. 4. Trasplante hepático.
Gráfico de respuestas
Comentario

Se observa una única masa hepática que refuerza con contraste en fase arterial, además de tener
un paciente en Child B, sin más comorbilidades.

Para diagnosticar el cáncer hepático no es necesario una biopsai si las pruebas de imagen son
altamente sugestivas de carcinoma hepatocelular (TAC en fase arterial y venosa).(R4)

 
 
 
 

236. Niña de 1 año con fiebre de hasta 39


ºC de 2 días de evolución. No presenta en la exploración foco claro para la fiebre, salvo
faringe algo hiperémica. Se toma muestra de orina mediante bolsa colectora,
detectándose nitritos y leucocitos mediante tira reactiva, por lo que se procede a toma
de orina estéril mediante punción suprapúbica y la tinción de Gram de dicha orina
muestra más de 1 bacteria por campo de gran aumento. En los exámes de laboratorio
sanguíneos destaca leucocitosis con neutrofilia, así como elevación importante de la
PCR. Tras el diagnóstico y el tratamiento adecuado para este caso, la niña es estudiada
en la consulta de nefrología, donde se le decide practicar una cistoureterografía
miccional seriada (CUMS) que se muestra en la imagen. Respecto al cuadro clínico citado,
señale la respuesta FALSA:

1. 1. La infección urinaria es la causa más frecuente de fiebre sin foco en menores de 1 año.
La presencia de nitritos y leucocitos en la orina no es diagnóstico de infección urinaria ya
2. 2.
que existen causas de falsos positivos.
3. 3. Para el diagnóstico de infección urinaria es necesario un cultivo positivo de orina.
El tratamiento de elección en estos casos de sospecha de infección urinaria con fiebre
4. 4.
debe ser siempre con antibióticos intravenosos de entrada.
Gráfico de respuestas
Comentario

La sola sospecha de infección urinaria no justifica la administración de antibióticos en todos los


casos. Cuando éstos se utilizan, tampoco tienen por qué ser necesariamente intravenosos
(depende de la gravedad y de la edad del paciente). Una respuesta como la 4, aunque sólo fuese
por lo radical de su expresión, debería hacerle desconfiar muy seriamente. La antibioticoterapia
intravenosa queda reservada para los casos más graves, aparte de que lo más adecuado es
confirmar la infección, como se explica en otra de las opciones.(R4)

237. La niña de la pregunta anterior fue diagnosticada finalmente de infección urinaria al


presentar un urocultivo con >1000 UFC/ml de E. coli que respondió bien al tratamiento
antibiótico. En relación al manejo adecuado de este tipo de pacientes tras la resolución
del proceso infeccioso, señale la opción FALSA:

 
 
 
 
Se recomienda realizar ultrasonido renal y vesical tras un episodio de IVU febril para
1. 1.
descartar hidronefrosis y abscesos renales.
La cistoureterografía miccional seriada no sólo permite el diagnóstico del reflujo vesico-
2. 2. ureteral sino que lo clasifica en grados que pueden determinar la actitud terapéutica más
adecuada.
El reflujo vesico-ureteral primario se debe a una anomalía congénita en la unión urétero-
3. 3. vesical, mientras que el secundario, mucho más frecuente que el primario, puede tener
múltiples etiologías.
El pilar básico del manejo de niños con reflujo vésico-ureteral es la profilaxis antibiótica
4. 4. con el objetivo de prevenir pielonefritis de repetición que puedan dejar cicatrices renales
y afectar a la función renal (nefropatía por reflujo).
Gráfico de respuestas
Comentario

Para el diagnóstico de IVU, se precisan más de 100 000 UFC, pero en este caso nos bastaría con
una sola, ya que el método de muestra ha sido la punción suprapúbica. Una vez resuelto el caso,
debemos plantearnos si existe una anomalía morfológica que lo justifique, por lo que realizaremos
un ultrasonido renal y vesical, para empezar el estudio. También se utilizará la cistoureterografía
miccional seriada (CUMS) para ver el grado de reflujo y elegir el tratamiento más adecuado. La
respuesta 3 es falsa por un pequeño detalle: el RVU es más frecuente en su forma primaria que el
secundario.(R3)

238. ¿Qué fármaco elegiría para tratar una neumonía en un niño de 5 años de edad, que
se acompaña de cefalea, malestar general, tos llamativa, radiografía con patrón
intersticial y que en los análisis de sangre presenta trombocitopenia y crioaglutininas?:

1. 1. Eritromicina.
2. 2. Penicilina G sódica.
3. 3. Amoxicilina.
4. 4. Trimetroprim-Sulfametoxazol.
Gráfico de respuestas
Comentario

Pregunta concreta que necesita de buen olfato clínico. Estamos ante una neumonía por
Mycoplasma pneumonae siendo el tratamiento de elección en este caso la eritromicina.(R1)

239. Niño de 8 años que acude a su consulta por presentar rágades labiales (queilitis
angular) de forma crónica. En la exploración, usted nota que el niño tiene un cráneo con
numerosos abultamientos en regiones fronto-parietales y un engrosamiento de la cara
anterior de la tibia. Tiene la nariz en silla de montar y un pequeño derrame simétrico y no
doloroso en ambas rodillas, que es fluctuante en el tiempo. ¿Sobre qué incidiría en la
anamnesis para intentar llegar a un diagnóstico?

1. 1. Dieta del niño, fundamentalmente ingesta de lácteos.


2. 2. Si ha tenido fracturas de repetición ante traumatismos mínimos.
3. 3. Si la madre padeció alguna enfermedad infecciosa durante el embarazo.
4. 4. Consumo de tóxicos durante el embarazo.
Gráfico de respuestas
Comentario

La sífilis congéntia tardía produce un cuadro clínico muy típico: queratitis intersticial + hipoacusia
neurosensorial +

 
 
 
 
deformidades óseas. Éstas pueden producirse a distintos niveles (ensanchamiento de la clavícula,
frente, tibia, deformidades en las piezas dentarias, alteración de la pirámide nasal). Por otra parte,
es frecuente que también asocien patología neurológica (tabes dorsal, paresia juvenil). Por ello, el
cuadro que nos describen debemos enfocarlo hacia una posible sífilis materna. Respuesta 3
correcta.(R3)

240. En cuanto a la ictericia neonatal, marque lo CORRECTO:

1. 1. El momento de inicio no ayuda en el diagnóstico.


2. 2. El riesgo de Kernicterus se presenta con valores de bilirrubina > 30 mg/dl.
3. 3. La reabsorción de hematomas puede causar ictericia.
4. 4. La ictericia fisiológica aparece en las primeras 24 horas.
Gráfico de respuestas
Comentario

Las características de la ictericia no fisiológica son las siguientes:

• Inicio en las primeras 24 horas de vida.

• Duración superior a 10-15 días.

• Bilirrubina total mayor de 12 mg/dl en RNT o mayor de 1 5 mg/dl en

RNPT.

Incremento de la bilirrubina superior a 5 mg/dl/24 horas.

• Bilirrubina directa mayor a 1 mg/dl o superior al 20 % de la bilirrubina

total.

La ictericia fisiológica nunca comienza el primer día de vida, ni se prolonga más allá de los días 10-
15. Cursa con buen estado general. La billrrubina no supera los 12 mg/dl en los RNT ni los 14
mg/dl en los RNPT.

En los RNPT, la ictericia fisiológica suele ser de inicio algo más tardío que en el RNT, y también
habitualmente es algo más prolongada, así como también puede normalmente alcanzar niveles
más altos.(R3)

241. Un paciente de 63 años, con antecedentes de hipertensión arterial severa, presenta


un cuadro de inicio súbito caracterizado por disfagia, disfonía, vértigo, ataxia en
extremidades derechas y pérdida de la sensibilidad termoalgésica en hemicara derecha
y miembros izquierdos. Una TC craneal excluye la presencia de sangrados
intracraneales. Señale cuál es la causa más frecuente de este cuadro:

1. 1. Oclusión de la arteria vertebral derecha


2. 2. Oclusión distal de la arteria basilar.
3. 3. Oclusión proximal de la arteria basilar.
4. 4. Oclusión de la arteria cerebelosa posteroinferior izquierda.
Gráfico de respuestas

 
 
 
 
Comentario
Pregunta de dificultad moderada sobre un tema importante para el MIR como son los síndromes
troncoencefálicos. El síndrome bulbar lateral o síndrome de Wallenberg es secundario a la
obstrucción de la arteria vertebral en la mayor parte de los casos y a la de la cerebelosa
posteroinferior (PICA) en el resto. Clínicamente se caracteriza por: 1)síndrome vertiginoso con
naúseas y vómitos por afectación de los núcleos vestibulares, 2)disartria y disfagia por lesión del
núcleo ambiguo, 3)diplopía, por extensión a la protuberancia y afectación del VI par, 4)hipoestesia
corporal contralateral por afectación del núcleo espinotalámico, 5) hipoestesia facial ipsilateral por
afectación del núcleo del V par, 6)ataxia cerebelosa ipsilateral secundaria a afectación del cerebelo
y pedúnculo cerebeloso inferior, 7)síndrome de Horner ipsilateral.(R1)

242. Escolar de 7 años de edad, asmático, acude a Urgencias por presentar tos y
sibílancias. Al evaluarlo se obtiene una puntuado de 6 en el score de Bierman - Pearson.
Seleccione el manejo inicial:

1. 1. Dexametasona y ambroxol.
2. 2. Salbutamol y aminofilina.
3. 3. Salbutamol y dexametasona.
4. 4. Aminofilina y cromoglicato.
Gráfico de respuestas
Comentario

ASMA es un tema muypreguntado debes domianarlo en todos sus aspectos se pregunta muy
frecuente en el ENARM. La Escala de Bierman y Pierson se usa para ver la gravedad del cuadro
clínico de las bronquitis agudas, en el que se puntúan la frecuencia respiratoria, las sibilancias, la
cianosis y la retracción costal, clasificándose según la puntuación en:

-Leve: menores de 5 puntos.

-Moderada: de 6-8 puntos.

-Grave: 9-12 puntos.

En el tratamiento se incluye el uso de salbutamol y corticoides. Los mucolíticos no se utilizan, con


lo que la 1 se elimina. Las xantinas se reservan para casos refractarios, por su poco riesgo
beneficio, cuando las otras medidas hayan fracasado, con lo que la 2 y 4 se eliminan.(R3)

243. Uno de los siguientes fármacos NO está indicado en la angina de pecho estable.
Señale cuál:

1. 1. Aspirina.
2. 2. Heparina sódica.
3. 3. Nitratos.
4. 4. Antagonistas del calcio.
Gráfico de respuestas
Comentario
Pregunta de baja dificultad sobre un tema muy frecuente en el MIR, como es el tratamiento de la
angina estable. Este tratamiento consiste en administrar en primer lugar AAS, que disminuye la
incidencia del síndrome coronario agudo. También se administran nitratos cuando aparece un
episodio de dolor y además como tratamiento crónico para aumentar la tolerancia al esfuerzo. Se

 
 
 
 
utilizan los beta- bloqueantes para disminuir la demanda de oxígeno, al disminuir la contractilidad
miocárdica y la frecuencia cardíaca. Los antagonistas del calcio también son muy útiles en la
angina de pecho, por dos razones. El verapamil y el diltiacem disminuye la función del VI, la
frecuencia cardíaca y la contractilidad, por lo que no deben asociarse a los betabloqueantes. El
amlodipino y el nifedipino, en cambio, tienen un efecto vasodilatador, que es responsable de la
taquicardia refleja, por lo que sí se pueden asociar a los betabloqueantes. La heparina en principio
no está indicada en la angina estable, aunque sí en la inestable.(R2)

244. Señale el enunciado FALSO:

1. 1. Las crisis parciales simples no cursan con deterioro del nivel de conciencia.
2. 2. Las ausencias típicas se siguen de confusión postcrítica.
Las ausencias atípicas con frecuencia ocurren en niños con otros trastornos neurológicos
3. 3.
subyacentes.
4. 4. La crisis jacksoniana es una forma de crisis parcial simple.
Gráfico de respuestas
Comentario

Pregunta asequible acerca de la epilepsia. Dentro de la clasificación de las crisis epilépticas hay
que tener claras dos divisiones: Crisis parciales - generalizadas (según afecten a una parte o a
toda la corteza) y crisis simples - complejas (según se afecte o no el nivel de conciencia). Por
tanto, la respuesta 1 es verdadera ya que las crisis simples mantienen el nivel de conciencia y las
complejas no. La respuesta 4 también es verdadera ya que la crisis Jacksoniana es una crisis
parcial simple motora. El enunciado 3 también es cierto. Por tanto, la falsa es la respuesta 2 ya que
las ausencias típicas no se siguen de confusión posterior y esto precisamente es una de las
características que las diferencia de las crisis parciales complejas.(R2)

245. En una de las siguientes pacientes está totalmente CONTRAINDICADA la


anticoncepción hormonal:

1. 1. Paciente con miomas uterinos.


2. 2. Paciente con colelitiasis.
3. 3. Antecedentes de ictericia colestásica del embarazo.
4. 4. Hemangioma hepático.
Gráfico de respuestas
Comentario

Es muy importante que conozca el mecanismo de acción, los efectos secundarios y las
contraindicaciones de los anticonceptivos orales de cara al ENARM pues han sido preguntados en
diversas ocasiones.

Las contraindicaciones absolutas son pacientes con riesgo cardiovascular (fumadoras mayores de
35 o no fumadoras mayores de 40), antecedentes de enfermedad tromboembólica, HTA mal
controlada, diabetes con afectación vascular, vasculopatía inflamatoria, cardiopatías graves,
pacientes con afectación hepática importante (adenoma hepático o hepatopatías activas), pofiria
aguda intermitente, antecedentes de ictericia durante el embarazo (colestasis intrahepática),
cáncer de mama u otros tumores hormonodependientes, discrasia sanguínea y sangrado genital
anormal no filiado.(R3)

246. Para medir el flujo plasmático renal se puede utilizar el aclaramiento de:

 
 
 
 
1. 1. Inulina.
2. 2. Acido paraminohipúrico.
3. 3. Acido fosfórico.
4. 4. Iodotalamato marcado con I-125.
Gráfico de respuestas
Comentario

Debe conocer algunos conceptos de fisiología renal.

El aclaramiento es el volumen de sangre de una sustancia que se depura en el riñón por unidad de
tiempo.

El ácido paraminopúrico (PAH) se usa para medir el flujo plasmático renal (RC- 2). El PAH, a su
paso por el riñón, es filtrado y secretado al túbulo de tal modo que, en un solo paso, es totalmente
eliminado del plasma. Por lo tanto, la medida del aclaramiento del PAH es igual al volumen total de
plasma que llega al riñón por unidad de tiempo, es decir, al flujo plasmático renal.(R2)

247. Un piloto de carreras, presenta un accidente en unos entrenamientos. Tras comentar


alegremente con sus compañeros que sólo ha sido un susto y que sólo le duele
ligeramente el costado izquierdo, se empieza a marear y finalmente se desvanece. Señale
lo INCORRECTO en relación a la patología que usted sospecha:

1. 1. El bazo es el órgano más frecuentemente lesionado en traumatismos cerrados.


2. 2. El mejor método diagnóstico es la TC.
3. 3. Es característico el signo de Kehr.
4. 4. En la mitad de los casos puede ocurrir una rotura esplénica diferida.
Gráfico de respuestas
Comentario

Es una pregunta que cae algunas veces ya que es el bazo es el órgano del abdomen que más se
lesiona. Clínicamente se observan signos generales de hemorragia y locales de irritación peritoneal
en el área esplénica. En raros casos (<5%) puede haber una rotura esplénica diferida por un
hematoma eventualmente contenido por la cápsula, manifestándose generalmente dentro de la
primera semana tras el trauma (opción 4 falsa). El signo de Kehr es un dolor intenso en hombro
izquierdo que se presenta en algunos casos de rotura de bazo. El diagnóstico se establece por
ultrasonido o TC. Si el paciente está hemodinámicamente inestable, la punción- lavado peritoneal
es indicación de cirugía inmediatamente. El tratamiento en ausencia de lesiones significativas y de
hemorragia persistente puede ser consevador (esplenorrafia), reservando la esplenectomía para
lesiones extensas del parénquima.(R4)

248. Mujer de 40 años de edad, multípara con 4 partos vaginales, que como método
anticonceptivo actual utiliza anticonceptivos orales (desde hace 8 años) y tabaquismo de
20 cigarrillos al día. Acude a nuestra consulta por presentar manchado hemático (goteo)
en los períodos intermenstruales. Se realiza citología cervical, en el que se reportan
células con núcleos vesiculares atípicos, compatible con adenocarcinoma. Se realizó una
biopsia que nos permitió tipificar el tumor como un adenocarcinoma seroso papilar.
Tanto el estudio radiográfico del tórax, como el TC abdomino-pélvico resultaron
normales. Señale, entre los siguientes elementos, cuál sería de utilidad para establecer
el estadiaje TNM del carcinoma invasor de cuello uterino:

1. 1. Metrorragia.

 
 
 
 
2. 2. Dolor pélvico.
3. 3. Pérdida de peso.
4. 4. Afectación de los parametrios.
Gráfico de respuestas
Comentario

Recuerde la importancia de la afectación de los parametrios en el cáncer de cervix. El estadio II A


tiene afectados los 2/3 superiores de la vagina sin afectar a los parametrios. Esto hace que sea
susceptible de ser tratado mediante la intervención de Werthein- Meigs. Sin embargo, la afectación
de los parametrios, nos pone directamente en un estadio II B que es susceptible de tratamiento con
radioterapia.(R4)

249. A newborn infant undergoes a routine checkup. Physical examination is


unremarkable, except for a scrotal mass that transilluminates with light. His mother is
very concerned because she does not want surgery. What should you do next?

1. 1. Reassurance and observation


2. 2. Testicular ultrasound
3. 3. Aspiration of fluid
4. 4. Surgery

(R1)

250. Paciente de 55 años, diagnosticado de artritis reumatoide desde hace 5 años, que
sigue tratamiento crónico con prednisona (10 mg diarios) y metotrexate. Consulta por un
cuadro de 10 días de evolución consistente en cefalea progresiva, tendencia a la
somnolencia y fiebre. A la exploración física presenta una escala de Glasgow de 13
puntos, nistagmo horizontal, paresia de los pares craneales VI, IX y XII derechos y ataxia.
Tras descartar mediante un TC craneal la existencia de signos de hipertensión
intracraneal, se practica una punción lumbar. Señale cuál de las siguientes
combinaciones de hallazgos en el LCR y agente etiológico considera más probable:

Pleocitosis polimorfonuclear, discreta hipoglucorraquia e hiperproteinorraquia.


1. 1.
Streptococcus pneumoniae.
Pleocitosis mononuclear, glucorraquia y proteinorraquia normales. Virus herpes simple
2. 2.
tipo 2.
3. 3. Pleocitosis linfocitaria, hipoglucorraquia e hiperproteinorraquia. Aspergillus fumigatus.
Pleocitosis linfocitaria, hipoglucorraquia marcada e hiperproteinorraquia. Listeria
4. 4.
monocytogenes.
Gráfico de respuestas
Comentario

Esta pregunta aborda el diagnóstico diferencial de un síndrome meníngeo de evolución subaguda


(10 días) en un sujeto inmunodeprimido (consumo crónico de corticoides y metotrexate).

La simple evolución clínica del cuadro permite descartar con facilidad las causas de meningitis
aguda, como el neumococo (respuesta 1) o los bacilos gramnegativos. Tampoco es compatible con
un cuadro de meningitis vírica (respuesta 2).

Dentro de las etiologías de las meningitis subagudas o crónicas, el enunciado presenta ciertos
detalles que orientan hacia un cuadro de romboencefalitis (ataxia, nistagmo y alteración de pares

 
 
 
 
craneales bajos) que, en el contexto de un paciente sometido a tratamiento inmunosupresor,
resultan muy sugerentes de Listeria monocytogenes (respuesta 4).

Por último, la infección fúngica del SNC es excepcional y se presenta habitualmente en sujetos
profundamente inmunodeprimidos (neutropénicos) o sometidos a procedimientos neuroquirúrgicos,
por lo que no parece la etiología más probable (respuesta 3).(R4)

251. A 71-year-old woman presents to her physician with a 4-month history of asthenia
and arthralgia and a 2-month history of nasal congestion and a 2-week history of cough.
She is especially worried now because she has been coughing up blood-tinged sputum
for the last 4 days. These symptoms are not accompanied by fever, nausea or vomiting.
She has hypertension and migraine. Over the past 4 weeks, she has been taking over-the-
counter NSAIDs for her arthralgia. Her pulse is 90/min, breathing frequency are 12/min
and blood pressure is 140/83 mm Hg. On physical examination, the patient appears
fatigued, with diffuse tenderness in her joints which are not warm or erythematous. A
clear nasal discharge is also noted, with no other significant findings. Lab tests show:
Hematocrit 32%, Mean corpuscular volume 82 μm3, Leukocyte count 15,100/mm3,
Segmented neutrophils 71%, Eosinophils 2%, Lymphocytes 15%, Monocytes 12%,
Platelet count 325,000/mm3, Serum Urea nitrogen 32 mg/dL, Creatinine 3.4 mg/dL,
Antinuclear antibodies 1:256, Rheumatoid factor negative, Antineutrophil cytoplasmic
antibodies positive. Urinalysis shows: Blood 3+, Protein 3+, RBC 15–19/hpf, WBC 1–5/hpf,
RBC casts rare. Which of the following is the most likely causative mechanism of this
patient's renal failure?

1. 1. Tuberculosis.
2. 2. Interstitial nephritis.
3. 3. Hypertension.
4. 4. Vasculitis.
Gráfico de respuestas
Comentario
Vasculitis. The combination of pulmonary (blood-tinged sputum), upper airway, arthralgia and
kidney failure in a patient with anti-neutrophil cytoplasm antibodies (ANCAs) is suggestive of
VASCULITIS, particulary GRANULOMATOSIS WITH POLYANGIITIS a.k.a WEGENER'S DISEASE
(if the antibodies react against proteinase= c-ANCA) or CHURG STRAUSS (if the antibodies are p-
ANCA). Remember that the presence of such antibodies is not always detected in patients with
these vasculitides.(R4)

252. Acude a valorar a un recién nacido niña a las pocas horas del nacimiento.
Antecedentes perinatales: 38 semanas de edad gestacional, embarazo controlado y
normal. Cultivo recto-vaginal negativo. Presentación cefálica. pH en sangre de cordón
7.21. Peso al nacimiento 3,500 gramos. Parto vaginal, distócico. Antecedentes familiares:
sin interés. Exploración física: Buen estado general, buena coloración, sin signos de
alteración respiratoria, pulsos palpables y simétricos. AP: buena ventilación bilateral,
ruidos cardiacos rítmicos sin soplos. Abdomen: blando, depresible, no masas no
megalias significativas. Cordón: 3 vasos. Caderas: normal. Genitales femeninos
normales. Cabeza y cuello normales. Clavícula normal. Buen tono muscular, fontanela
normotensa. La extremidad superior derecha se encuentra en adducción y rotación
interna y el antebrazo en pronación. El reflejo de Moro es asimétrico (ausente en el lado
derecho), los reflejos de prensión, Galant y succión están conservados. Respecto al
cuadro que presenta es FALSO que:

 
 
 
 
1. 1. En algunos casos se aprecia una elevación del hemidiafragma del lado afectado.
2. 2. A veces la sensibilidad de la parte externa del brazo puede estar alterada.
3. 3. Es más frecuente en la distocia de nalgas, por ser esta habitualmente más traumática.
Inicialmente el tratamiento es conservador, inmovilizando la extremidad superior de
4. 4. forma intermitente y en abducción + rotación externa del brazo y antebrazo en
supinación.
Gráfico de respuestas
Comentario

Tema preguntado en algunas ocasiones en el nacional.

La parálisis braquial de Erb-Duchenne (afecta a los nervios superiores del plexo braquial C5-C6) es
la más frecuente. Se asocia con distocia de hombros y su clínica es: RN incapaz de realizar los
movimientos de abducción y rotación externa del brazo y de supinación del antebrazo. Faltan el
reflejo bicipital y el Moro en el lado afecto. En ocasiones afecta a la sensibilidad de la cara externa
del brazo. Si se afecta también el nervio frénico (C4) puede producirse una hemiparálisis del
diafragma (elevación del diafragma en el lado afecto).

La parálisis de Dejerine-Klumpke es rara, afecta a los nervios inferiores del plexo braquial (C7 y
C8) y afecta al antebrazo y a la mano (pérdida del reflejo de prensión). Si afecta también a la raíz
T1 produce síndrome Horner (ptosis+ miosis +enoftalmos). En general la recuperación es
completa. Depende de si hay sección (neurotmesis) del nervio o solo interrupción de las fibras con
vaina de mielina integra (axonotmesis) o edema perineural (neuroapraxia). Las parálisis superiores
tienen mejor pronóstico que las que asocian daño en los nervios inferiores. El tratamiento consiste
en inmovilizar de forma intermitente el brazo con férulas en la postura contraria a la provocada por
la lesión para evitar contracturas. Si persiste más allá de 3- 6 meses se considera la cirugía.(R3)

253. ¿Cuál de las siguientes opciones acerca de las miocarditis es INCORRECTA?:

La miocarditis vírica produce clínica de insuficiencia cardíaca, así como arritmias que
1. 1.
pueden llegar a producir la muerte.
En la analítica puede existir aumento de la CPK y CPK-MB que no indica que haya
2. 2.
necrosis cardíaca necesariamente.
La carditis por enfermedad de Lyme tiene como manifestación cardíaca más típica las
3. 3.
arritmias, que pueden requerir la implantación de un marcapasos.
4. 4. La miocarditis vírica puede recidivar y progresar hacia una miocardiopatía hipertrófica.
Gráfico de respuestas
Comentario

La miocarditis es un tema poco preguntado, pero debes conocer lo básico.

La miocarditis vírica produce clínica de IC, así como arritmias que pueden llegar a producir la
muerte. Sin embargo, es un cuadro que, en general, suele ser autolimitado, si bien a veces puede
recidivar y progresar a una miocardiopatía crónica dilatada (NO hipertrófica).

En los exámenes de laboratorio suele aparecer una elevación de la CPK y la CPK- MB (el
isoenzima cardíaco), aunque no indica necesariamente que haya necrosis miocárdica.

La enfermedad de Lyme cursa característicamente con afectación cardíaca en la fase intermedia,


que consiste principalmente en alteraciones de la conducción que pueden requerir la implantación
de un marcapasos.

 
 
 
 
La miocarditis de células gigantes es una enfermedad inflamatoria del miocardio que puede cursar
con IC rápidamente mortal y arritmias, y en la que aparecen en el miocardio células
multinucleadas. La etiología es desconocida.(R4)

254. Recién nacido pretérmino ingresado en la UCIN por enfermedad de membrana


hialina, permanece intubado durante las primeras 24 horas, presentando notable mejoría
en los 2 días posteriores. Al cuarto día se incrementan sus necesidades de oxígeno y en
la gasometría se detecta mayor retención de CO2. Usted sospecha una persistencia de
conducto arterioso. Señale lo CORRECTO:

1. 1. En la exploración es frecuente encontrar pulsos saltones y fuertes.


2. 2. Suelen mejorar con administración generosa de líquidos.
3. 3. Las prostaglandinas son el tratamiento de elección para los RNPT con clínica de ductus.
Suele aparecer el ductus coincidiendo con la mejoría de EMH porque aumentan las
4. 4.
resistencias vasculares pulmonares.
Gráfico de respuestas
Comentario

La dificultad de esta pregunta está en reconocer el caso clínico. Se trata de una PCA, que siempre
tiene que sospechar en un contexto como éste: enfermedad de la membrana hialina que
súbitamente empeora, cuando ya comenzaba a estabilizarse, y esto es debido a la caída de las
resistencias pulmonares cuando se instaura el tratamiento con la oxigenoterapia. Es entonces
cuando se pone de manifiesto el conducto arterioso, que semiológicamente se caracteriza por los
pulsos saltones y por un soplo continuo muy típico (soplo de Gibson) o en "maquina de vapor".(R1)

255. El tumor más frecuente a nivel del ángulo pontocerebeloso es el:

1. 1. Glioma de tronco cerebral.


2. 2. Colesteatoma del poro acústico.
3. 3. Meningioma de punta de peñasco.
4. 4. Neurinoma del acústico.
Gráfico de respuestas
Comentario

Uno de los aspectos más importantes en el estudio de los tumores craneales es conocer cuál es el
más frecuente en función de la región, la edad y la patología acompañante.

En el ángulo pontocerebeloso, el tumor más frecuente es el neurinoma del acústico (80-89%),


seguido por el meningioma (5-10%) y el colesteatoma (5%). Recuerda que en el niño, el tumor más
frecuente de fosa posterior es el astrocitoma pilocítico, mientras que en el adulto sería el
hemangioblastoma.

Respecto al neurinoma del acústico, es necesario recordar que realmente es un schwannoma


vestibular. Anatomopatológicamente, es relevante conocer los dos subtipos: Antoni A (células
alargadas) y Antoni B (tejido laxo reticular). Sintomatológicamente, es importante recordar la
hipoacusia con acúfenos y el vértigo, seguidos de hipoestesia y debilidad facial. El método
diagnóstico de elección sería la RM, de cara a conocer el tamaño del tumor y afectación de
estructuras adyacentes y planificar la cirugía (tratamiento de elección).(R4)

256. ¿Cuál de los siguientes patrones histológicos corresponde a la dermatitis


herpetiforme?:

 
 
 
 
1. 1. Ampolla intraepidérmica con eosinófilos.
2. 2. Ampolla subepidérmica con eosinófilos.
3. 3. Ampolla subepidérmica con neutrófilos.
4. 4. Ampolla intraepidérmica y disqueratosis.
Gráfico de respuestas
Comentario

Pregunta acerca de la dermatitis herpetiforme, tema de muy poca importancia para fines del
ENARM. Sin embargo, no suelen preguntar conceptos histológicos por lo que esta pregunta no es
de las más relevantes.La dermatitis herpetiforme se trata de una enfermedad benigna y crónica
caracterizada por una erupción papulovesiculosa muy pruriginosa, localizada en áreas extensoras
de manera simétrica en pacientes con una enteropatía sensible al gluten, habitualmente
asintomática. Suele comenzar entre la 2ª- 4ª décadas. Las lesiones son polimorfas, con pápulas,
placas urticariformes y pequeñas vesículas excoriadas agrupadas sin afectación mucosa.
Histológicamente se trata de una ampolla subepidérmica con microabscesos de polimorfonucleares
en las papilas dérmicas e infiltado neutrofílico en la dermis. Depósito granular de IgA en lainmuno
fluorecencia. Se trata con sulfona y dieta exenta de gluten. Recuerda que las ampollas
intraepidérmicas son flácidas y el Nikolsly es positivo, mientras que las subepidérmicas son tensas
y el Nikolsly es negativo.(R3)

257. ¿En qué grupo de edades es más frecuente la hemorragia uterina disfuncional (HUD)
anovulatoría?:

1. 1. Niñas y premenopáusicas.
2. 2. Adolescentes y perimenopáusicas.
3. 3. Adultas y menopáusicas.
4. 4. Niñas y adolescentes.
Gráfico de respuestas
Comentario

Las edades en que la HUD es más frecuente corresponden a los extremos de la vida fértil; esto es,
en la adolescencia y la perimenopausia; pues en estas edades son frecuentes los ciclos
anovulatorios.

Las niñas y las menopáusicas (respuestas 1 y 3) no tienen menstruación.(R2)

258. Masculino de 66 años con antecedentes de bocio multinodular normofuncionante


que acude al servicio de Urgencias por nerviosismo, temblor distal y palpitaciones, con
intolerancia al calor desde hace 24 horas, 36 horas antes se había sometido a una
exploración radiológica para la que había precisado contraste intravenoso. Señale el
diagnóstico y la actitud a seguir con el paciente:

Progresión del bocio multinodular a un bocio multinodular tóxico, y el tratamiento de


1. 1.
elección es el I 131.
Tirotoxicosis inducida por ioduro (Jod-Basedow). El tratamiento consistiría en beta-
2. 2.
bloqueantes hasta que remitiese la sintomatología.
3. 3. Efecto Wolff-Chaikoff. El tratamiento consistiría en antitiroideos.
Desarrollo de una enfermedad de Graves en el contexto de un bocio multinodular de larga
4. 4.
evolución. El tratamiento es con antitiroideos y beta-bloqueantes.
Gráfico de respuestas
Comentario

 
 
 
 
El yodo es necesario para la formación de las hormonas tiroideas, y su metabolismo está muy
estrechamente controlado, hasta el punto de que la yoduria refleja la cantidad de yodo de que
dispone una persona. Sin embargo, el yodo en situaciones y dosis determinadas puede producir
efectos distintos a los fisiológicos. Un ejemplo es el fenómeno de Jod- Basedow que ocurre en
pacientes con bocio ante la administración de dosis elevadas de yodo y cuyo efecto es la aparición
de un hipertiroidismo que en el momento del diagnóstico tiene una gammagrafía hipocaptante. Otro
ejemplo es el Wolf- Chaikoff, que consiste en la inhibición de la organificación ante la
administración de dosis altas de yodo en pacientes con hipertiroidismo. Otros efectos son la
inducción de bocio o hipotiroidismo en pacientes con enfermedades autoinmunes de tiroides o la
fibrosis preoperatoria que se consigue con la administración e lugol.(R2)

259. Una paciente de 83 años de edad, en tratamiento con digoxina por una insuficiencia
cardíaca congestiva con fibrilación auricular crónica, acude a urgencias con un
incremento de su disnea, así como sensación de desvanecimiento. La presión arterial es
de 100/60 mmHg. En el ECG aparece un ritmo aproximadamente de 30 latidos por minuto
y no se observan ondas P. En la radiografía de tórax aparecen signos de insuficiencia
cardíaca. ¿Cuál de los siguientes tratamientos sería el más adecuado para esta paciente?

Administración intravenosa de líquidos, a ser posible en forma de suero fisiológico, para


1. 1.
remontar la hipotensión arterial.
2. 2. Administración intravenosa de inotropos positivos, como dopamina y dobutamina.
Interrumpir la digoxina, administrar diuréticos moderadamente e implantar un
3. 3.
marcapasos transitorio.
4. 4. Implantar un marcapasos definitivo.
Gráfico de respuestas
Comentario

Han preguntado varias veces sobre la digoxina en el ENARM, por lo tanto es importante conocerla
bien.

Debe tener algunos conocimientos sobre su intoxicación. Hay muchos factores que incrementan la
posibilidad de que aparezcan efectos tóxicos por la digital. Por una parte, algunas situaciones
incrementan los niveles sanguíneos de digoxina, como la insuficiencia renal y la administración de
ciertos fármacos (quinidina, amiodarona, verapamil...). Por otro lado, algunas situaciones
incrementan la toxicidad por digoxina, como la hipokalemia, la hipercalcemia, la hipomagnesemia y
el hipotiroidismo.

La intoxicación digitálica tiene clínica cardíaca (puede producir arritmias, siendo las más frecuentes
las extrasístoles ventriculares) y extracardíaca (náuseas, vómitos, diarrea, alteraciones visuales,
etc.).

El tratamiento consiste en retirar el fármaco, implantar un marcapasos provisional cuando las


alteraciones de la conducción sean graves, y tratar las arritmias. A veces se administran
anticuerpos Fab antidigoxina. Recuerde que la hemodiálisis es poco eficaz.(R3)

260. Señale la opción FALSA acerca del granuloma umbilical:

La existencia de microorganismos saprofitos sobre el cordón retrasa la caída del mismo y


1. 1.
aumenta el riesgo de infección.
2. 2. Mediante la limpieza de la zona con alcohol conseguimos la desaparición de dicho tejido.

 
 
 
 
La persistencia del tejido de granulación abundante se manifiesta como una zona
3. 3.
vascular, granular, blanda, rosada o blanquecina.
El tratamiento consiste en cauterización de la zona con nitrato de plata más antibioterapia
4. 4.
sistémica.
Gráfico de respuestas
Comentario
El tratamiento del granuloma (pequeña masa blanda de color rojo pálido) umbilical implica el uso
de nitrato de plata. No son necesarios los antibióticos. El uso de alcohol no provoca su
desaparición. La perionfalitis a veces genera la aparición de una pequeña granulación
umbilical.(R4)

261. Mujer de 32 años, embarazada de gemelos y diagnosticada de polihidramnios,


comienza después del parto con una importante hemorragia. A la palpación se detecta la
ausencia de retracción uterina. Debemos pensar en:

1. 1. Placenta acreta.
2. 2. Desgarro de partes blandas.
3. 3. Atonía uterina.
4. 4. Rotura uterina.
Gráfico de respuestas
Comentario

La causa más frecuente de las hemorragias del alumbramiento es la atonía uterina. Además, esta
paciente que nos presentan tiene un riesgo aumentado, por la sobredistensión uterina mantenida
por el polihidramnios y el embarazo gemelar. La clínica de un útero que no se contrae nos apoya
en nuestra sospecha, y lo único que tenemos que descartar es la presencia de una rotura uterina
como causa de atonía uterina. Para ello, se revisaría la cavidad uterina y, si no existe solución de
continuidad, comenzaría el tratamiento de la atonía con masaje uterino, fármacos uterotónicos,
taponamiento uterino… Si las medidas conservadoras fracasan, la histerectomía sería el
tratamiento definitivo.(R3)

262. Indique el diagnóstico más probable en un paciente varón de 32 años con hemoptisis
severa, infiltrados alveolares y afectación renal:

1. 1. Lupus eritematoso diseminado.


2. 2. Granulomatosis de Wegener.
3. 3. Enfermedad de Good Pasture.
4. 4. Crioglobulinemia.
Gráfico de respuestas
Comentario
Esta pregunta es de dificultad media. La enfermedad de Goodpasture o enfermedad antimembrana
basal glomerular consiste en la asociación de hemorragia alveolar difusa (se ven en la radiografía
de tórax como infiltrados alveolares bilaterales con broncograma aéreo) y glomerulonefritis, que se
acompaña de anticuerpos antimembrana basal en el suero, de causa desconocida. La
manifestación renal es la hematuria que suele aparecer 2 ó 3 semanas después de la hemoptisis.
El síndrome de hemorragia reno- pulmonar (hemoptisis y hematuria) se define como síndrome de
Goodpasture y tanto las opciones 1, 3, 4 y 5 entran dentro del diagnóstico diferencial pero su causa
más frecuente es la enfermedad de Goodpasture.(R3)

263. Si se sospecha el diagnóstico de artritis por cristales de pirofosfato con


condrocalcinosis, ¿cuáles son las radiografías más idóneas para confirmarlo?:

 
 
 
 
1. 1. Hombros, pelvis y manos.
2. 2. Manos, rodillas y pelvis.
3. 3. Sacroilíacas y rodillas.
4. 4. Hombros, pelvis y pies.
Gráfico de respuestas
Comentario
La rodilla es la articulación más afectada en la artropatía por pirofosfato cálcico dihidratado
(PPCD). Otras localizaciones son las muñecas, sínfisis del pubis, hombros, tobillos, codos y
manos. La mejor respuesta es la tercera ya que indica entre otras localizaciones la más frecuente
que es la rodilla. Los datos clínicos y radiológicos indican que el depósito de PPCD es poliarticular
al menos en dos tercios de los pacientes. Cuando el cuadro clínico se parece al de una artrosis
lentamente progresiva, el diagnóstico puede ser más difícil. En tales casos, la distribución de las
lesiones articulares pueden ser una buena pista para identificar la artropatía por PPCD. Por
ejemplo, la artrosis primaria rara vez afecta a lasa articulaciones metacarpofalángicas, las
muñecas, los codos, los hombros o tobillos. Si en la radiografía se observan depósitos densos,
puntiformes o lineales en el cartílago hialino (condrocalcinosis), aún se consolida más el
diagnóstico de artropatía por PPCD.(R2)

264. Gestante de 36
años, primigesta, de 33 semanas, en la que en el USG se diagnostica un CIR. Usted realiza
un Doppler en la arteria umbilical y observa los hallazgos que se muestran en la imagen.
Señale la CORRECTA:

1. 1. Indicaría nuevo control con Doppler en una semana.


2. 2. Finalizaría la gestación a las 37 semanas.
3. 3. Finalizaría la gestación.
4. 4. Realizaría una prueba de Pose.
Gráfico de respuestas
Comentario
Como se ve en la imagen, el flujo en la arteria umbilical es claramente patológico, con un flujo
reverso en la diástole. Es un signo ominoso que se asocia a muerte fetal en el 50% de los casos,
por lo que no esperaría otra semana ni hasta la semana 37, sino que finalizaría ya la gestación,
incluso a pesar de tratarse de una gestación de 33 semanas.(R3)

265. El tratamiento del fibroadenoma de mama usualmente es:

1. 1. Extirpación con anestesia local en paciente ambulatorio.


2. 2. Mastectomía simple.

 
 
 
 
3. 3. Cuadrantectomía.
4. 4. Extirpación quirúrgica seguida de radioterapia.
Gráfico de respuestas
Comentario

Es una patología benigna, que en el caso de que se decida operar, no suele requerir más que
extirpación ambulatoria.(R1)

266. Un prematuro de 900 g de peso y 27 semanas de gestación sufre distrés respiratorio


desde el nacimiento. A las 36 horas de vida presenta hipotensión, bradicardia, cianosis y
tensión aumentada de la fontanela. ¿Qué exploración diagnóstica le parece más oportuna
entre las que se muestran?:

1. 1. Análisis de sangre con serie roja y plaquetas.


2. 2. Exploración cerebral con ultrasonido.
3. 3. Estudio del tiempo de protrombina y del tiempo parcial de tromboplastina.
4. 4. Punción lumbar y cultivo de LCR.
Gráfico de respuestas
Comentario

En un RNPT con alta probabilidad de enfermedad de membrana hialina, otra de las complicaciones
que debería pensar es la hemorragia intraventricular.

Al contarte que presenta hipotensión, bradicardia, cianosis y aumento de la tensión de la fontanela


deberá pensar en dicho diagnóstico. La exploración diagnóstica que debe realizar es el ultrasonido
cerebral. Respuesta 2 correcta.(R2)

267. Lactante de 6 meses, acude a Urgencias por tos intensa en accesos que acaban en
gallo inspiratorio, congestión facial, lagrimeo, salivación y vómitos intensos que impiden
su alimentación en la última semana. Señale la respuesta FALSA:

1. 1. En la biometría hemática lo más frecuente es encontrar leucopenia con neutrofilia.


2. 2. La complicación más frecuente es la neumonía por sobreinfección bacteriana.
3. 3. El tratamiento adecuado sería eritromicina durante 14 días.
4. 4. El agente responsable es un cocobacilo gramnegativo.
Gráfico de respuestas
Comentario

Pregunta de dificultad media en la que debemos reconocer en el enunciado el dato clave de la tos
ferina, que son los accesos de tos que terminan en "gallo" inspiratorio.

Sabiendo la enfermedad que es, ya podemos contestar que la correcta es la 1, puesto que es dato
básico para conocer de esta enfermedad que se presenta como leucocitosis con linfocitosis
absoluta debido a que esta bacteria libera un factor estimulante del crecimiento de los linfocitos.

La determinación de IgG antifactor estimulante es la prueba más sensible y específica para su


diagnóstico.(R1)

268. A 65-year-old female presents to the emergency room with abdominal pain and fever.
Physical exam shows tenderness and pain of palpation in the lower left quadrant.

 
 
 
 
Temperature is 38.4ºC. Lab test reveals WBC of 23,000. Which of the following is the best
next step in the management of this patient?

1. 1. Urgent abdominal surgery.


2. 2. Exploratory laparotomy.
3. 3. Ambultory pain control.
4. 4. Start iv antibiotics and perform a CT-scan.
Gráfico de respuestas
Comentario
Start iv antibiotics and perform a CT-scan. Our main suspicion is acute diverticulitis, therefore we
immediately start intravenous antibiotic and analgesic treatment. To confirm diagnosis a CT-scan
should be performed.(R4)

269. Femenino de 44 años que acude a la consulta de ginecología por menorragias, sin
otra sintomatología acompañante. La exploración ginecológica muestra vagina y cérvix
normales, mamas normales y tacto vagino-abdominal con útero aumentado de volumen,
de consistencia dura y superficie irregular. El USG confirma el diagnóstico de tres
miomas de entre 3-4 cm de diámetro, de localización submucosa. En los exámenes de
laboratorio se detecte una anemia ferropénica moderada. ¿Qué tipo de tratamiento
propondría a esta paciente?

1. 1. Miomectomía abdominal.
2. 2. Análogos de GnRH.
3. 3. Tratamiento médico de la anemia ferropénica y observación de los miomas.
4. 4. Tratamiento de la anemia ferropénica e histerectomía abdominal.
Gráfico de respuestas
Comentario

Nos presentan el caso de una paciente con miomas, en la que éstos tienen repercusión clínica:
menorragias y anemia ferropénica. Si tratamos la anemia exclusivamente con hierro, lo más
probable es que reaparezca pasado un tiempo, porque la causa persistiría. Por ello, hay que
ofrecerle una solución para el problema de base, que son los miomas.

Los análogos de la GnRH no son el tratamiento definitivo de los miomas. Se utilizan previamente a
la cirugía, porque disminuyen su tamaño, pero no de forma aislada. Por otra parte, el tipo de
cirugía preferible en esta paciente es la histerectomía, puesto que ya no se encuentra en edad
reproductiva. La miomectomía se reserva para pacientes jóvenes que todavía no han cumplido sus
deseos genésicos.(R4)

270. ¿Cuál de las siguientes le parece una buena opción para tratar el síndrome de
Cushing?:

Extirpación del tumor primario en casos de secreción ectópica de ACTH, aunque esté
1. 1.
muy avanzado y no sea curativo.
En caso de adenoma suprarrenal, extirpación quirúrgica del tumor, con tratamiento
2. 2.
esteroideo sustitutivo posterior a la intervención por la posible atrofia contralateral.
Extirpación quirúrgica, en caso de carcinoma suprarrenal, y después tratamiento de
3. 3.
mantenimiento con aminoglutetimida.
Hipofisectomía en caso de adenoma secretor de ACTH, completando el tratamiento con
4. 4.
radioterapia si se trata de un tumor mayor de 1 cm.
Gráfico de respuestas

 
 
 
 
Comentario
En cuanto al tratamiento del síndrome de Cushing, en los casos de adenoma o carcinoma
suprarrenal, se debe realizar extirpación quirúrgica, siendo opcional el tratamiento previo con
inhibidores de la esteroidogénesis pero no para evitar los niveles excesivos de cortisol en la cirugía
(hecho que no influye en la misma), sino en los casos en que exista clínica importante derivada del
hipercortisolismo. En los casos en que exista un tumor ectópico muy avanzado, no se realiza
cirugía, sino tratamiento farmacológico paliativo. En los casos de carcinoma suprarrenal se realiza
extirpación quirúrgica, y si existen restos, el fármaco de elección es el mitotane. En los tumores
hipofisarios, se realiza cirugía transesfenoidal, y si existen restos se administraría radioterapia. La
indicación de ésta última no es que se trate de un tumor mayor de 1 cm, sino que existan restos
tras la cirugía. Cuendo se opera un tumor suprarrenal que secreta cortisol, se debe administrar
tratamiento corticoideo para evitar la atrofia contralateral y una posible insuficiencia
suprarrenal.(R2)

271. Mujer de 20 años de edad, sexualmente activa, presenta fiebre, dolor abdominal
inferior, náuseas y vómitos. Posteriormente, dolor en hipocondrio derecho. Al examen:
cervicitis mucopurulenta, leucocitosis y aumento de la velocidad de sedimentación
globular. ¿Cuál del siguiente microorganismo causa con más frecuencia este cuadro
clínico?:

1. 1. Treponema pallidum.
2. 2. Chlamydia trachomatis.
3. 3. Mycoplasma hominis.
4. 4. Ureaplasma urealyticum.
Gráfico de respuestas
Comentario

Pregunta clásica en el ENARM. Describen un caso típico de enfermedad pélvica inflamatoria, que
puede estar complicada por una perihepatitis (síndrome de Fitz-Hugh-Curtis). En nuestro país,
México, las etiologías más frecuentes son Chlamydia trachomatis y Neisseria gonorrhoae
(gonococo).

Luego entonces, la respuesta correcta es la 2.(R2)

272. Nos avisan por un paciente de 84 años ingresado en geriatría, tras sufrir una caída,
por importante deterioro funcional. Desde hace 24 horas ha comenzado con distensión
abdominal y vómitos. Última defecación hace 48 horas. A la exploración presenta un
abdomen distendido, timpánico y con ruidos hidroaereos disminuidos. No parece dolerle
a la palpación profunda. El tacto rectal es limpio y sin masas. No presenta laparotomías
ni hernias palpables. Se realiza una Rx de abdomen y un TAC abdominal donde se
observa dilatación de colon transverso y derecho sin claro cambio de calibre. En cuanto
a la patología que sospecha en este paciente, indique cuál es el tratamiento inicial que
considera más adecuado:

1. 1. Sonda nasogástrica (SNG) y sueros.


2. 2. Descompresión con enemas.
3. 3. Cirugía urgente.
4. 4. Gastrografín y repetir Rx en 24 horas.
Gráfico de respuestas
Comentario

 
 
 
 
Pregunta de un tema repetitivo e importante. El cuadro que presenta el paciente es característico
de un Síndrome de Ogilvie o pseudoobstrucción aguda de colon. El tratamiento inicial consiste en
la descompresión mediante enemas (la descompresión con SNG en este caso no suele ser útil, al
tratarse de un problema funcional y distal del colon) y medidas de soporte. Si fracasa debe optarse
por la colonoscopia, a no ser que la dilatación sea muy importante (superior a 12 cm), en la que el
riesgo de perforación es alto e indicaríamos tratamiento quirúrgico. El gastrografín sólo se emplea
en casos de obstrucción intestinal de delgado por bridas.(R2)

273. Mujer de 30 años, madre de tres hijos, acude a urgencias por dolor cólico en
hipocondrio derecho, náuseas, vómitos, y escalofríos. Antecedente de dispepsia a
grasas. Al examen: T 38.8ºC., no ictericia, Murphy (+). El estudio diagnóstico que
recomendaría es:

1. 1. Ultrasonido abdominal.
2. 2. Radiografía de abdomen.
3. 3. Tomografía abdominal.
4. 4. Sólo requiere evaluación clínica.
Gráfico de respuestas
Comentario

Todo paciente con sospecha de patología de vía biliar, deberá realizarse ultrasonido abdominal, ya
que es el mejor estudio para su estudio... No debe confundirse con la TC, ya que esta última no es
tan sensible, ni especfiica para valorar la patología de las vías biliares.(R1)

274. Paciente de 65 años de edad, diagnosticado de leucemia linfática crónica, presenta


adenopatías palpables cervicales y axilares bilaterales. Leucocitos 85x109/l con 85% de
linfocitos, Hto 40% y Hb 13 g/dl y plaquetas 50x109l. ¿En qué estadio clínico de Rai y de
Binet se encuentra?

1. 1. Estadio III de Rai y estadio A de Binet.


2. 2. Estadio III de Rai y estadio C de Binet.
3. 3. Estadio IV de Rai y estadio C de Binet.
4. 4. Estadio II de Rai y estadio B de Binet.
Gráfico de respuestas
Comentario

Esta pregunta solamente requiere conocer ambos sistemas de estadiaje.

Sistema de Rai

•   Estadio 0: linfocitosis absoluta en sangre periférica superior a 15.000 por milímetro cúbico.
•   Estadio I: linfocitosis más adenopatías.
•   Estadio II: linfocitosis más hepato y/o esplenomegalia (con o sin adenopatías).
•   Estadio III: linfocitosis más anemia inferior a 11 g/dl de hemoglobina en varones y 10 en
mujeres.
•   Estadio IV: linfocitosis más trombopenia inferior a 100.000 por milímetro cúbico.

Sistema internacional (Binet)

 
 
 
 
•   Estadio A: leucemia sin anemia ni trombopenia, y con menos de tres áreas linfoides
afectas.
•   Estadio B: leucemia sin anemia ni trombopenia, pero con tres o más áreas linfoides
afectas.
•   Estadio C: leucemia con anemia y/o trombopenia.

Se consideran áreas linfoides (unilaterales o bilaterales) las cinco siguientes: adenopatías


cervicales o supraclaviculares, axilares, inguinales, hígado y bazo.(R3)

275. El agente más frecuente de sepsis nosocomial en el recién nacido es:

1. 1. Estreptococo del grupo B.


2. 2. Candida albicans.
3. 3. Staphylococcus epidermidis.
4. 4. Haemophilus influenzae.
Gráfico de respuestas
Comentario

Los principales factores de riesgo son la prematuridad y los procedimientos invasivos.

El agente etiológico más frecuente son los estafilococos coagulasa negativos, como el
Staphylococcus epidermidis.

El diagnóstico de confirmación es a través de hemocultivos positivos y además podremos obtener


el antibiograma.(R3)

276. A 2-day-old preterm infant (27 weeks' gestation) is brought to the emergency
department presenting with sleepiness and breast refusal. His mother claims that the
child is lethargic and that he cries persistently. Temperature is 38ºC. Physical
examination shows a distended abdomen. During examination, the doctor notices blood
in the baby's diaper. What is the most likely diagnosis?

1. 1. Salmonella neonatal infection


2. 2. Biliary atresia
3. 3. Necrotizing enterocolitis
4. 4. Meconium ileus
Gráfico de respuestas
Comentario
Necrotizing enterocolitis. Necrotizing enterocolitis is a life-threatening condition that appears more
frequently in preterm births. Intestinal ischemia leads to bowel mucosa injury and risk of perforation
and peritonitis. The typical signs are abdominal distension and bloody diapers. Meconium ileus may
cause abdominal distension but bloody stools are not present. Bowel volvulus could be a possible
diagnosis, but the history of a premature birth should point towards Necrotizing Enterocolitis.(R3)

277. ¿Cuál de éstas NO sería una indicación directa para la realización de un ciclo de
fecundación in vitro?:

1. 1. Obstrucción tubárica bilateral.


2. 2. Endometriosis severa.
3. 3. Cinco ciclos fallidos de inseminación intraconyugal.
4. 4. Síndrome de ovario poliquístico.

 
 
 
 
Gráfico de respuestas
Comentario

El SOP (Síndrome del ovario poliquístico) NO es indicación directa para la fecundación in vitro sin
haber intentado antes la inducción a la ovulación.

La oligoastenoteratozoospermia consiste en la presencia de una baja concentración de


espermatozoides en el semen, nomalmente debida a una escasa fabricación de los mismos en los
testículos, por lo tanto es una de las indicaciones de FIV (fecundación in vitro).(R4)

278. A 32-year-old homeless man is brought to the ER by the police. He was found in poor
condition, sweating and drooling. Physical examination reveals: BP 130/70 mm Hg, pulse
110/min and temperature 38.5ºC. His mouth is swollen and red, and there is a tender,
indurated swelling with palpable crepitus in the submandibular area. What is the most
likely diagnosis?

1. 1. Acute sialadenitis.
2. 2. Diphtheria.
3. 3. Ludwig's angina.
4. 4. Parotid gland cancer.
Gráfico de respuestas
Comentario

Pregunta sencilla sobre la angina de Ludwig, respuesta 3 correcta.

Los datos clave de esta pregunta son los siguientes: vagabundo, en malas condiciones generales,
con diaforesis y babeando. Los signos vitales con fiebre y taquicardia. La boca edematosa y roja,
asociado a crepitación subamndibular.

Es un tipo de celulitis que compromete el piso de la boca, debajo de la lengua. A menudo ocurre
después de una infección de las raíces de los dientes (como absceso dental) o una lesión en la
boca.

El tratamiento consiste en antibioterapia parenteral cubriendo anaerobios y Gram negativos y en


ocasiones es necesario el drenaje quirúrgico.(R3)

279. Entre los medicamentos que disminuyen la eficacia de los antinceptivos hormonales
orales de forma importante, se encuentra:

1. 1. Ampicilina.
2. 2. Fenilbutazona.
3. 3. Rifampicina.
4. 4. Furosemide.
Gráfico de respuestas
Comentario

Es importante que conozca las interacciones farmacológicas de ciertos fármacos, sobre todo que
los tenga clasificados como inductores o inhibidores enzimáticos.

 
 
 
 
La rifampicina es un inductor del citocromo P-450, por lo tanto disminuye la eficacia de aquellos
fármacos que utilizan este sistema enzimático para ser metabolizados, como los anticoagulantes
orales como la warfarina y los anticonceptivos orales. Durante la toma de rifampicina los
anticonceptivos orales no serán eficaces por lo tanto se necesitaran métodos de barrera durante
ese tiempo.(R3)

280. A 5-year-old boy is brought to consultation for presenting a sudden loss of vision in
the left eye. The boy has a history of mental retardation but has a normal physical
development according to growth curves. He has a skinny appearance, with long fingers,
arms and legs. Rest of physical examination shows pectus excavatum, moderate
scoliosis, pes cavus and knocked knees. What is the most likely diagnosis?

1. 1. Marfan's syndrome
2. 2. Ehler Danlos Syndrome
3. 3. Pseudoxanthoma elasticum
4. 4. Homocystinuria
Gráfico de respuestas
Comentario
Homocystinuria. The boy in this case has an abnormal phenotype associated with a thrombotic
complication. The diagnosis is homocystinuria. Homocystinuria is an autosomal recessive disorder
caused by deficiency of the enzyme cystathionine synthetase. The lack of this enzyme results in
increased levels of homocysteine, which is highly thrombogenic. Patients with this disease have a
marfanoid appearance with tall height and hypermobility. There is often mental retardation.(R4)

281. Respecto al screening del cáncer de cérvix, señale lo FALSO:

Existe evidencia de que el cribado del cáncer de cérvix uterino mediante citología permite
1. 1.
reducir la mortalidad por dicha neoplasia.
2. 2. Se aconseja repetirla al año, continuando en los casos negativos los controles cada 3 años.
La mayoría de los programas finalizan a los 65 años, aunque algunos datos aconsejan su
3. 3.
prolongación hasta los 70.
No debe efectuarse cribado en pacientes a las que se les haya practicado una
4. 4.
histerectomía.
Gráfico de respuestas
Comentario

El cribado del cáncer de cérvix uterino se realiza mediante citología y permite reducir la mortalidad
por dicha neoplasia. Debe realizarse de manera sistemática a todas las mujeres desde el inicio de
las relaciones sexuales pudiéndose iniciar el cribado a los 3 años del inicio de las relaciones
sexuales dado que el riesgo de H-SIL o cáncer en los primeros años tras la exposición al HPV es
bajo. Dado que la sensibilidad de una única citología es baja se aconseja repetirla al año,
continuando en los casos negativos controles cada 3 años. La mayoría de los programas finalizan
a los 65 años, aunque algunos datos aconsejan su prolongación hasta los 70. No debe efectuarse
cribado tras la histerectomía total si ésta se realizó por patología benigna. En pacientes con
histerectomía por CIN o carcinoma debe continuarse el cribado por el riesgo de lesiones
vaginales.(R4)

282. La presencia de placas hialinas pleurales, calcificadas en forma de semiluna sobre


las cúpulas diafragmáticas:

1. 1. Indica necesariamente la existencia previa de exposición al asbesto.

 
 
 
 
2. 2. Es sugestiva de exposición al asbesto.
3. 3. Es indicación de pleurectomía.
4. 4. Degenera posteriormente a mesotelioma maligno difuso de la pleura.
Gráfico de respuestas
Comentario

La placas pleurales (calcificaciones de la pleura parietal) no son específicas de exposición al


asbesto; también aparecen en la TB, hemorragias o conectivopatías. Sugieren exposición al
asbesto en pacientes con antecedentes epidemiológicos compatibles. No producen síntomas, no
aumentan el riesgo de mesotelioma y no requieren tratamiento.(R2)

283. Mujer de 28 años, primípara; parto eutócico hace 10 días. Puerperio afebril. Lactancia
materna. Acude a urgencias refiriendo cuadro de dolor en mama derecha de 48 horas de
evolución, Tª de 38º C, enrojecimiento y calor local. Dice tener algunas dificultades para
vaciarla al amamantar. Exploración: mamas asimétricas (derecha de mayor tamaño); no
alteraciones a nivel de areola-pezón; secreción láctea espontánea. Mama derecha
ingurgitada, eritematosa a nivel de CSE y caliente; no alteraciones tróficas en piel. A la
palpación se detecta una zona indurada, dolorosa, pero sin zonas fluctuantes evidentes
en CSE. Señale la respuesta INCORRECTA:

Los gérmenes más frecuentemente implicados en este proceso son estafilococos,


1. 1.
estreptococos y colibacilos.
2. 2. Es necesario retirar la lactancia.
3. 3. El tratamiento incluye la administración de antibióticos y analgésicos antiinflamatorios.
4. 4. Es posible la formación de un absceso que requiere drenaje quirúrgico.
Gráfico de respuestas
Comentario

El caso clínico que nos presentan es el de una mastitis. Se trata de una infección muy frecuente en
mujeres con lactancia materna. La producen gérmenes que pasan de la piel a la glándula mamaria
a través de grietas y heridas que se forman secundariamente a la lactancia. Ante una mastitis,
debe pensar en microorganismos grampositivos, comoS. aureus.

El tratamiento de la mastitis se realiza con antibióticos que cubra S. aureus y analgésicos,


evacuación periódica del pecho y aplicación de calor local. En el caso de que se forme un absceso,
será necesario completar el tratamiento anterior con el drenaje quirúrgico. No es necesario inhibir
la lactancia, salvo que sea una mastitis bilateral.(R2)

284. Paciente de 27 años, aficionado al parapente, tiene un accidente que le hace caer
desde una altura de aproximadamente 50 metros. Ingresa en el hospital en coma con una
valoración en la escala de Glasgow de 4 puntos. Padece algunas fracturas de miembros
inferiores, que se estabilizan y fijan quirúrgicamente sin problemas, y se realizaron
diversos TAC cerebrales, que fueron informados repetidamente como normales. Una
RMN, realizada una semana después del accidente, detectó hemorragias puntiformes en
varias zonas del cuerpo calloso, unión corticosubcortical cerebral y porción dorsolateral
del tronco del encéfalo. Tres semanas después del traumatismo, a pesar de que existe
cierta mejoría en las pruebas de imagen, la situación del paciente persistía inmodificada,
con una puntuación de 4 puntos de Glasgow. Señale la afirmación FALSA en relación con
este cuadro:

1. 1. Es improbable que la situación de este paciente obedezca a causas no neurológicas.

 
 
 
 
Dada la resolución progresiva de las hemorragias en las pruebas de imagen, el pronóstico
2. 2.
de recuperación no es malo.
3. 3. Este tipo de cuadros suelen producirse en el contexto de un TCE de alta energía.
Histológicamente, probablemente exista mayor afectación en sustancia blanca que en
4. 4.
sustancia gris.
Gráfico de respuestas
Comentario
No hay comentario.(R2)

285. Mujer de 25 años con un embarazo con un parto eutócico hace 5 años. Lactancia
materna durante 4 meses e inicio de menstruaciones regulares a los 7 meses postparto.
Acude refiriendo amenorrea de 8 meses de evolución. La paciente no toma ninguna
medicación habitual y no refiere hábitos tóxicos. Exploración física, incluyendo
ginecológica, dentro de límites normales. En la evaluación de esta paciente se seguirán
los siguientes pasos, EXCEPTO:

1. 1. Realización de niveles de prolactina sérica.


2. 2. Realización de niveles de TSH y prolactina.
3. 3. Laparoscopia diagnóstica.
4. 4. Evaluación de los niveles de gonadotropinas.
Gráfico de respuestas
Comentario

Ante una amenorrea en primer lugar se realiza un test de embarazo, si es negativo seguimos el
estudio con TSH y PRL, en caso de que sean normales damos una pequeña cantidad de
progesterona (5 días) si la paciente menstrua es porque existía una anovulación que impedía el
aumento de la progesterona. Si no menstrua administramos estrógenos y progestágenos durante 3
meses, si no menstrua a pesar de una adecuada secuencia hormonal debemos pensar en
alteraciones genitales estructurales. Si la presenta tras la administración hormonal combinada
debemos pensar en un problema ovárico o hipotálamo-hipofisario y para ello ya determinaremos
gonadotropinas. Otras pruebas si fueran necesarias serán realizadas en pasos posteriores una vez
tengamos orientada la causa de la amenorrea.(R1)

286. ¿Cuál es la indicación quirúrgica más frecuente en pacientes con pancreatitis


crónica?

1. 1. Esteatorrea.
2. 2. Pseudoquiste.
3. 3. Absceso.
4. 4. Dolor resistente a analgésicos.
Gráfico de respuestas
Comentario

El tratamiento fundamental de la pancreatitis crónica es el del dolor. No obstante, no sólo se


emplean analgésicos, sino que existen otras medidas que pueden servir como coadyuvantes. Por
ejemplo, la administración de enzimas pancreáticas puede aliviarlo de forma indirecta, al disminuir
la secreción pancreática. En caso de dolor intratable, puede ser necesario recurrir a la cirugía,
siendo el motivo más frecuente de intervención quirúrgica en esta enfermedad. La intervención
debe aliviar el dolor y preservar las funciones pancreáticas endocrina y exocrina. Las distintas
técnicas quirúrgicas son:

 
 
 
 
- Esfinteroplastia: Útil en caso de obstrucción focal en el orificio ampular.

- Técnica de Puestow: De elección cuando existe un conducto de Wirsung dilatado de forma difusa.

- Cirugía resectiva: En pancreatitis limitadas a una zona concreta del páncreas.

- Ablación nerviosa del ganglio celíaco y nervios esplácnicos.(R4)

287. After initial management, which of the following is


the most appropriate next step with this patient?

1. 1. Urgent MRI.
2. 2. Vascular exploration by a surgeon.
3. 3. Monitoring the patient by intensive care unit.
4. 4. Assessment of common peroneal nerve motor function by a neurosurgeon.
Gráfico de respuestas
Comentario
La complicación más temible de una luxación de rodilla es la lesión inmediata (laceración,
desgarro) o diferida (trombosis tras lesión de la íntima arterial) de la arteria poplítea. Es
imprescindible la realización de un estudio vascular (habitualmente con Eco-Doppler), y la
valoración por Cirugía Vascular de la pierna del paciente. Un porcentaje alto de casos de luxación
de rodilla tiene lesión arterial, y un porcentaje alto de lesiones arteriales no se recupera a pesar de
una cirugía de revascularización, necesitándose finalmente una amputación de la pierna.(R2)

288. Niño de 5 años diagnosticado de infección por VIH de transmisión vertical. Presenta
un cuadro de tos de unas semanas de evolución. La auscultación pulmonar y cardíaca es
normal, al igual que el resto de la exploración excepto linfadenopatías generalizadas. TA
90/50 mmHg, FC 98 lpm, FR 40 rpm, Tª 36.3 ºC, Saturación periférica de oxígeno: 90%. Es
FALSO que:

Las infecciones oportunistas en el niño son más frecuentes que en al adulto, siendo de las
1. 1. no oportunistas las más incidentes las infecciones bacterianas, especialmente la neumonía
por Streptococcus pneumoniae.

 
 
 
 
La neumonía por Pneumocystis jirovecii en niños tiene una incidencia máxima entre los 3
2. 2. y los 6 meses de edad, y una gran mortalidad sobre todo por encima del año de edad,
aunque ha disminuido con las nuevas pautas de tratamiento intensivo.
El cuadro que más probablemente presenta este paciente es causado por la propia
3. 3.
infección del VIH.
El tratamiento de la complicación que más probablemente presenta el paciente será con
4. 4.
corticoides orales.
Gráfico de respuestas
Comentario

De las complicaciones respiratorias del VIH en la infancia hay que saber el diagnóstico diferencial
entre la neumonía por Pneumocystis jirovecii, la neumonía intersticial linfoide y otros cuadros
comunes.

La neumonía por Pneumocystis jirovecii es un cuadro agudo de fiebre, taquipnea, disnea e


hipoxemia severa, con infiltrados intersticiales bilaterales rápidamente progresiva. El diagnóstico se
confirme detectando el germen en una muestra obtenida mediante lavado broncoalveolar.
Tratamiento con Trimetoprim-Sulfametoxazol.

La neumonía intersticial linfoide es un cuadro crónico que ocurre en un 25% de los pacientes.
Cursa con taquipnea y tos insidiosas e hipoxemia leve-moderada con acropaquias en los casos
avanzados. En la radiografía se aprecia un patrón reticulonodular difuso. Se asocia a la infección
por el propio VIH y la hiperplasia linfoide que ocasiona en el epitelio bronquiolar. Se trata con
corticoides orales y broncodilatadores.

Son muy frecuentes las infecciones por gérmenes comunes sobre todo el neumococo, pero
también virus respiratorios, Haemophylus, Staphylococcus aureus, Pseudomonas y Legionella. La
tuberculosis con frecuencia tiene extensión extrapulmonar.

En la infección por VIH infantil son más frecuentes las infecciones de repetición por bacterias
comunes que las infecciones oportunistas típicas aunque éstas tienden a ser más graves que en el
adulto.(R1)

289. Señale la afirmación CORRECTA en relación a las gastritis de estrés:

Hay una pérdida de la integridad de la mucosa gástrica con erosiones que llegan hasta la
1. 1.
submucosa sin sobrepasarla.
2. 2. Las erosiones se observan más frecuentemente en el antro.
3. 3. La forma más frecuente de presentación clínica es la hemorragia digestiva.
La mayoría de los pacientes con úlcera de estrés requieren cirugía debido a la gravedad
4. 4.
del sangrado.
Gráfico de respuestas
Comentario
Las úlceras y erosiones agudas son provocadas por la hipoperfusión tisular, que ocurre en
enfermos en situación crítica, por sepsis, insuficiencia respiratoria o grandes quemados. En esta
última situación, se denominan úlceras de Curling. Cuando la causa es un traumatismo que afecta
al sistema nervioso central, más que por hipoperfusión lo que se produce es úlceras agudas por
hiperacidez y reciben el nombre de úlceras de Cushing. Las úlceras agudas o de estrés son
múltiples, poco profundas y, por tanto, causan poco dolor y se expresan clínicamente por sangrado
múltiple. Suelen afectar a cuerpo y antro gástrico. Las gastritis agudas, con úlceras sangrantes,
pueden controlarse con medicación antisecretora, radiología intervencionista y, con menor
frecuencia, requerir cirugía.(R3)

 
 
 
 
290. En un paciente que consulta por astenia y debilidad progresivas, se encuentra en la
exploración física esplenomegalia y el hemograma es el siguiente: leucocitos 1.500
/microL, hemoglobina 6.5 g/dL, plaquetas 40.000 /microL. ¿Cuál es el diagnóstico menos
probable?:

1. 1. Aplasia de médula ósea.


2. 2. Cirrosis hepática.
3. 3. Leishmaniasis visceral.
4. 4. Linfoma no hodgkiniano.
Gráfico de respuestas
Comentario
Las cinco opciones son causa de pancitopenia, pero sólo las cuatro últimas presentan
esplenomegalia. En la aplasia no hay tejido hematopoyético, ni en la médula ni extramedular que
compense el fallo medular.(R1)

300. Un paciente anciano acude al servicio de urgencia por deterioro general. Entre otros
síntomas, el paciente refiere incontinencia urinaria insensible en los últimos días. Entre
los datos analíticos obtenidos destaca una creatinina 4 veces mayor que los niveles de
referencia. La causa más probable de su situación sería:

1. 1. Glomerulonefritis aguda postinfecciosa.


Síndrome prerrenal por deshidratación y pérdida del control de esfínteres secundaria a
2. 2.
disminución de conciencia.
3. 3. Retención urinaria crónica con incontinencia por rebosamiento.
4. 4. Oligoanuria por obstrucción litiásica y pérdida urinaria por urgencia.
Gráfico de respuestas
Comentario

La incontinencia urinaria insensible (pérdidas de orina que no van precedidas de tenesmo) puede
verse en algunos casos de disfunción del sistema nervioso autónomo (p.e. neuropatía diabética) o
también como consecuencia de una obstrucción de las vías urinarias, cuando la presión
intraluminal es suficiente como para vencerla momentáneamente. En este caso, las opciones nos
ofrecen pocas dudas en este sentido, ya que solamente se menciona uno de los grupos de causas
que hemos citado (respuesta 3 correcta). Por otra parte, tampoco aparecen signos o síntomas
sugerentes de otras opciones (faringitis u otra infección previas, deshidratación, dolor en fosa renal
y elevación de la LDH, dolor cólico, etc.).(R3)

301. A previously healthy 5-year-old boy is brought to the pediatrician's office presenting
with a new onset rash two weeks after returning from camp. He has been feeling asthenia
and occasional headaches since then. His father says that a tick bit him and he had to
remove the insect with tweezers. The rash is 5 cm in diameter and centered around the
tick bite, with a bull's eye appearance. Topic steroids have not improved the lesion. What
is the best treatment option in this case?

1. 1. Oral ciprofloxacin.
2. 2. Oral terbinafine.
3. 3. Intravenous penicillin.
4. 4. Oral amoxicillin.
Gráfico de respuestas
Comentario

 
 
 
 
Oral amoxicillin. The sequence described in this case is typical of Lyme disease, caused by Borrelia
species. The first stage is erythema migrans, that appears at the site of the inoculation within a
week of the tick bite. Other early symptoms may include fever and general malaise. The treatment
of choice is oral amoxicillin or doxycycline. Doxycycline is contraindicated in children.(R4)

302. A 47-year-old female is concerned about her risk of ovarian cancer because her aunt
died of it and her mother has just been diagnosed. She is asymptomatic and her physical
examination is completely normal. She smokes one pack of cigarettes per day and she is
diabetic. Which of the following should be consider for ovarian cancer screening in this
patient?

1. 1. Biannual pelvic examination.


2. 2. Yearly transvaginal ultrasound.
3. 3. CA125 serum test if tested positive for BRCA1.
4. 4. Yearly transabdominal ultrasound if tested positive for BRCA2.
Gráfico de respuestas
Comentario

Las prueas de detección precoz de cáncer, existen exclusivamente para determinadas patologías
como el de mama, cervico-uterino, próstata, colon y pulmón. En el caso de cáncer de ovario en
general no existe, solo existiría en el caso que se comprobara alguna mutación en BRCA1, como
explica la respuesta 3.(R3)

303. Los síntomas fundamentales del llamado síndrome del túnel del carpo se deben a la
compresión del nervio:

1. 1. Mediano.
2. 2. Cubital.
3. 3. Radial.
4. 4. Palmar mayor.
Gráfico de respuestas
Comentario
Es una pregunta muy sencilla que lo único que hay que recordar es que en el síndrome del túnel
carpiano se afecta el nervio mediano. El síndrome del túnel carpiano es la neuropatía de
compresión más frecuente de todo el organismo. Aparece a mujeres de edad media. Aunque se
afecta más frecuentemente a la mano dominante. Clínicamente cursa con parestesias en el
territorio del mediano, (región volar de primer, segundo, tercer dedo y mitad radial del cuarto) de
predominio nocturno nocturnas. Para su exploración se usan las maniobras de Tinel (percutir sobre
la zona comprimida lo que reproduce las parestesias) y la maniobra de Phalen (mantener las
muñecas del paciente en flexión máxima durante un minuto y también reproduce las
parestesias).(R1)

304. Paciente de 15 años que consulta por haber presentado en el último año 3-4
episodios de pérdida brusca de conocimiento con caída al suelo, movimientos tónico-
clónicos en las cuatro extremidades de 4-5 minutos de duración, mordedura de lengua e
incontinencia urinaria. A continuación presentaba somnolencia y cefalea durante 30
minutos. Además, refería frecuentes sacudidas musculares bruscas y momentáneas,
aisladas, en ambos brazos y, a veces, en las piernas; esto le había provocado algunas
caídas y que se le cayeran objetos de las manos. Tanto unos como otros ocurrían en las
primeras horas de la mañana, generalmente en el desayuno. El paciente no tenía
antecedentes personales de interés, pero sí historia familiar positiva de crisis

 
 
 
 
convulsivas. La exploración general y neurológica fue normal. La TC no mostraba
alteraciones. El EEG intercrítico mostraba descargas de punta-onda o polipunta-onda
lenta generalizadas. ¿Qué síndrome epiléptico padece el paciente?

1. 1. Epilepsia mioclónica benigna de la infancia.


2. 2. Epilepsia mioclónica juvenil (Síndrome de Janz).
3. 3. Síndrome de Lennox-Gastaut.
4. 4. Epilepsia mioclono-astática.
Gráfico de respuestas
Comentario

La epilepsia mioclónica juvenil es el prototipo de epilepsia generalizada idiopática. Supone el 10%


de todas las epilepsias y es la epilepsia mioclónica más frecuente. La edad de inicio es entre los 8
y 25 años de edad. La mayoría de los pacientes presentan distintos tipos de crisis, además de las
mioclónicas: 90 % asocian crisis tónico-clónicas y el 30% ausencias típicas.

Las crisis se presentan como sacudidas musculares breves, habitualmente en miembros


superiores, característicamente al despertar, y favorecidas por la privación previa de sueño y el
consumo de alcohol. Se mantiene el nivel de conciencia, excepto en las crisis severas.

El EEG muestra actividad paroxística punta-onda en la mayoría de los casos, y siempre es


patológico durante el sueño. Un tercio de los pacientes presenta actividad paroxística fotosensible,
aunque no es frecuente que tengan un correlato clínico.

El tratamiento de elección de este tipo de crisis es el ácido valproico. Del resto de las opciones, no
olvide que la carbamacepina se emplea sobre todo para crisis parciales (también en el dolor
neuropático, como la neuralgia del trigémino), y la etosuximida se reserva para las crisis de
ausencia típicas.(R2)

305. En la patogenia de la colecistitis aguda litiásica el factor mas importante es:

1. 1. Inflamación.
2. 2. Infección.
3. 3. Obstrucción.
4. 4. Hemólisis.
Gráfico de respuestas
Comentario

El factor desencadenante más importante de colecistitis aguda litiásica es la obstrucción de la


vesícula biliar, que desencadenará el resto de procesos inflamatorios que desembocan en la
colecistitis por litiasis.(R3)

306. En cuanto al manejo del recién nacido prematuro, señale lo INCORRECTO:

El uso de bicarbonato en prematuros extremos condiciona la presentación de hemorragia


1. 1.
intracerebral.
Los recién nacidos prematuros requieren menores volúmenes de líquidos para infusión
2. 2.
intravenosa que los recién nacidos a termino.
3. 3. Una natremia de 150 mEq/L sugiere disminuir la infusión de líquidos.
4. 4. Todos los recién nacidos de bajo peso son prematuros.
Gráfico de respuestas

 
 
 
 
Comentario

No todos los RN con bajo peso son prematuros, respuesta incorrecta número 4.

Se ha asociado a la hemorragia intraventricular con el uso de bicarbonato, por lo que la opción 1


no la debe de seleccionar. Por obvias raones en una hipernatremia el tratamiento es diinuir la
infusión de suero fisiológico e iniciar glucosado o al 0.45%.(R4)

307. Un paciente de 80 años, con historia de hipertensión y en tratamiento con enalapril


y espironolactona, acude al hospital por astenia y debilidad muscular severa. La presión
arterial es de 110/70 mm Hg. En el ECG destacan ondas T picudas y elevadas,
extrasístoles ventriculares y QT corto. ¿Cuál es el diagnóstico más probable?:

1. 1. Hiperpotasemia.
2. 2. Hipomagnesemia.
3. 3. Hipocalcemia.
4. 4. Hipernatremia.
Gráfico de respuestas
Comentario
Pregunta muy sencilla sobre el diagnóstico de la hiperpotasemia. El paciente está en tratamiento
con dos fármacos que elevan la cifra de k sérico, un IECA (Enalapril) y un diurético ahorrador de k
(Espironolactona), se describe la clínica típica (Debilidad muscular de predominio en miembros
inferiores) y presenta alteraciones electrocardiográficas compatibles (Recuerda que lo más precoz
y característico es la presencia de T picudas).(R1)

308. Respecto a las respuestas de inmunidad celular, indique la afirmación CORRECTA:

Las células NK siempre expresan, constitutivamente, el receptor de afinidad media para


1. 1.
IL-2.
Los linfocitos colaboradores TH1 frenan las reacciones de inmunidad celular una vez
2. 2.
vencida la infección.
Los linfocitos colaboradores TH2 controlan y potencian las reacciones de inmunidad
3. 3.
celular.
En su proceso de maduración en el timo, los timocitos que solo interaccionan con el
4. 4.
HLA-I se convierten en LT-CD4.
Gráfico de respuestas
Comentario

Una pregunta potencialmente anulable, ya que ninguna de las opciones son correctas, por las
razones que se explican en el vídeo asociado a este simulacro.(R3)

309. Un hombre de 72 años de edad con sólo antecedentes de hipercolesterolemia bien


controlada, leve depresión y enfermedad por reflujo gastroesofágico controlada con
omeprazol, acude al servicio de urgencias por presentar desde hace 12 horas dolor
intenso en fosa ilíaca izquierda y aumento del número de deposiciones (hasta 4/día),
líquidas pero sin productos patológicos. A la exploración se observa dolor a la palpación
superficial y profunda en fosa ilíaca izquierda, ruidos presentes y no signos de irritación
peritoneal. Los resultados de laboratorio urgente son normales excepto leucocitosis
(16,3 103/μl) con desviación izquierda. Se le realiza una TAC abdominal urgente en el que
se describen imágenes sugerentes de diverticulosis con signos de diverticulitis aguda
sin datos de perforación. ¿Qué actitud terapéutica considera más correcta?

 
 
 
 
1. 1. Realización de colonoscopia urgente.
2. 2. Colocación urgente de un drenaje percutáneo, dirigido por TC, en área inflamada.
Alta domiciliaria, recomendando dieta rica en fibra y que acuda nuevamente a urgencias
3. 3.
si empeora la situación del paciente.
Observación hospitalaria con dieta absoluta y administración de ciprofloxacino y
4. 4.
metronidazol por vía i.v.
Gráfico de respuestas
Comentario

El tratamiento de una diverticulitis se basa en reposo intestinal, reposición hidroelectrolítica y


antibióticos. Los antimicrobianos han de garantizar una buena cobertura frente a gramnegativos y
anaerobios (en este caso, la combinación de ciprofloxacino y metronidazol es una posible opción
válida, respuesta 4 correcta). No obstante, si se forma un absceso de cierto tamaño o el paciente
presenta afectación del estado general, sería recomendable la punción del absceso guiada por
TAC (que no es nuestro caso). La laparotomía urgente se aconseja en caso de peritonitis purulenta
o fecaloidea.(R4)

310. ¿Cuál de los siguientes hallazgos no forma parte del diagnóstico del síndrome de
HELLP en la embarazada?

1. 1. Elevación de enzimas hepáticas.


2. 2. Trombocitopenia.
3. 3. Hipertensión arterial.
4. 4. Anemia hemolítica.
Gráfico de respuestas
Comentario

Tema básico para el ENARM, debe dominar el tema en todos sus aspectos. El síndrome HELLP
es la asociación de hemólisis, elevación de las enzimas hepáticas y plaquetopenia. A nivel
cerebral, se manifiesta el vasoespasmo como alteraciones occipitales tipo cefalea frontal, fotopsias,
escotomas y ceguera cortical transitoria y no como una verdadera encefalopatía hipertensiva, sin
que el grado de vasoespasmo se correlacione directamente con la gravedad (hasta 20% de
eclampsias debutan con TA normal o límite).(R3)

311. Una mujer de 76 años, sin otros antecedentes que hipertensión arterial, consulta por
ictericia indolora y prurito, con anorexia. En los exámenes de laboratorio destaca una
bilirrubina total de 12 mg/dl (con 9.5 mg/dl de bilirrubina directa). El ultrasonido
abdominal muestra dilatación biliar intra y extrahepática, junto con un nódulo hepático
único menor de 2 cm, localizado periféricamente, en la cara anterior del lóbulo izquierdo.
La TC confirma estos hallazgos, demostrando además la presencia de una masa de 3.5
cm en la cabeza pancreática. La punción aspirativa con aguja fina del nódulo hepático
resulta concluyente para adenocarcinoma. Señale la mejor opción terapéutica:

Quimio/radioterapia neoadyuvante condicionando la opción de cirugía radical a la


1. 1.
respuesta inicial.
Drenaje biliar externo percutáneo con carácter paliativo, con eventual reconversión a
2. 2.
drenaje interno en caso de intolerancia o complicaciones.
Derivación biliar quirúrgica paliativa, con o sin gastro-yeyunostomía profiláctica en
3. 3.
función de los hallazgos intraoperatorios.
Prótesis biliar metálica mediante colangiopancratografía retrógrada endoscópica, con
4. 4.
opción a quimioterapia paliativa.
Gráfico de respuestas

 
 
 
 
Comentario

La paciente consulta por un cuadro de obstrucción de la porción biliar extrahepática, sugestivo de


una neoplasia pancreática. En las pruebas complementarias, se observa una masa en hígado que
parece ser una metástasis. Nos encontraríamos entonces ante en un estadio IV según la
clasificación TNM y sería irresecable, por lo que la paciente sólo se beneficiaría de tratamiento
paliativo. De las opciones restantes, la respuesta 4 sería de elección por presentar menos
complicaciones.(R4)

312. Para una paiente femenino de 38 años de edad, fumadora crónica, con
hipermenorrea y presencia de varices en miembros inferiores, ¿Cuál de los siguientes
métodos anticonceptivos es el MENOS indicado?

1. 1. Dispositivo intrauterino de cobre.


2. 2. Anticonceptivos orales combinados.
3. 3. Dispositivo intrauterino activo con progesterona.
4. 4. Esterilización quirúrgica.
Gráfico de respuestas
Comentario

Contraindicaciones absolutas para el uso de ACO que hemos repasado en varias ocasiones.

Pacientes con riesgo cardiovascular: fumadora mayores de 35 años o no fumadoras mayores de


40 años.

Antecedentes de trombosis venosa profunda o TEP.

Cirugía mayor con previsibles periodos prolongados de inmovilización.

Mutaciones de genes con carácter trombogénico.

HTA mal controlada.

DM con afectación vascular: vasculopatía, nefropatía, retinopatía y neuropatía.

Vasculopatía inflamatoria.

Cardiopatías graves.

Pacientes con afectación hepática importante: adenoma hepático, hepatopatías activas como
cirrosis activa.

Porfiria aguda intermitente.

Antecedentes de ictericia durante la gestación, colestasis intrahepática.

Embarazo, confirmado o sospechado.

Cáncer de mama y otros tumores hormonodependientes.

 
 
 
 
Discrasia sanguínea, anemia de células falciformes, por le riesgo de trombosis.

Sangrado genital anormal no filiado.(R2)

313. A 56 year-old patient comes to the


physician because of hypertension, edema and hematuria. Proteinuria and ANCA are
detected. A renal biopsy is performed and an image is shown. No deposits are seen on
immunofluorescence test. Which of the following is the most likely diagnosis?

1. 1. Post-infectious glomerulonephritis.
2. 2. Pauciimmune rapidly progressive glomerulonephritis.
3. 3. Type 2 membranoproliferative glomerulonephritis.
4. 4. Goodpasture syndrome.
Gráfico de respuestas
Comentario
En la microscopía óptica se observa proliferación extracapilar en forma de semiluna epitelial.
Aunque en la imagen del ovillo no se puede saber a qué tipo de glomerulonefritis extracapilar
corresponde en el enunciado nos informan de que el paciente tiene ANCA positivos (Compatible
con extracapilar III), ausencia de inmunocomplejos (no es una extracapilar II) y ausencia de
anticuerpos antimembrana basal glomerular (No es una extracapilar I).(R2)

314. Masculino de 63 años, diabético, acude a Urgencias por presentar desde hace 2 días
diarrea y dolor abdominal difuso. En las últimas 24 horas se ha añadido fiebre de 39ºC,
malestar general y astenia. Finalmente se aisló en el coprocultivo un Campilobacter.
¿Cuál le parece ahora el tratamiento antibiótico de elección?:

1. 1. Ampicilina.
2. 2. Eritromicina.
3. 3. Norfloxacino.
4. 4. Amoxicilina-clavulánico.
Gráfico de respuestas
Comentario

Resumimos a continuación algunos aspectos de interés sobre Campylobacter; dicha enterobacteria


produce diarrea a través de la producción de citotoxinas, las cuales destruyen las células,
invadiendo los enterocitos, y produciendo una diarrea con moco, pus y sangre en heces y
presencia de leucocitos al microscopio. Se transmite a través de alimentos crudos o poco
cocinados o tras contacto directo con animales infectados, y es más frecuente en personas donde

 
 
 
 
los mecanismos de defensa del tracto digestivo están disminuídos, como ocurre en las
hipogammaglobulinemias. El tratamiento de elección consiste en eritromicina.

No olvides que esta relacionado con el desarrollo Guillian Barré, no es el caso de nuestro paciente
pero es preguntado contastanteente en el ENARM.(R2)

315. A 15-year-old girl is brought to the emergency room with a few-hour history of severe
lower abdominal pain and swelling. The pain is dysmenorrhea-like and she has had
similar episodes with increasing intensity over the past year. She does not refer changes
in bowel habits, metrorrhagia or leucorrhea. She has not experienced menarche yet, but
she has normal secondary sexual characteristics. Physical examination reveals a painful
hypogastrium, with a palpable mass located 3 cm above the pubic symphysis. Inspection
of the external genitalia shows an introitus with a tense membrane, behind of which you
can see a dark-colored fluid material. Rectal exam confirms a mass occupying the minor
pelvis. Which of the following answers is WRONG?

1. 1. Ultrasound will probably detect hematocolpos and / or hematometra.


2. 2. This syndrome is included in the group of cryptomenorrheas.
3. 3. Surgical correction requires extensive and complex procedures.
Patients with this syndrome have an increased risk of developing endometriosis in the
4. 4.
future.
Gráfico de respuestas
Comentario

Puede parecerle una pregunta difícil pero si aplica el sentido común de cómo resolvería la causa
seguro que da con la respuesta. Lo más importante de las amenorreas es conocer su diagnóstico
diferencial, además debe conocer unos datos básicos de las causas más frecuentes o más
características. El diagnóstico del himen imperforado se basa en la exploración genital que se debe
realizar a toda niña prepúber en la primera visita al pediatra. El acúmulo menstrual retenido puede
producir dolor abdominal. Cura con la incisión y evacuación del contenido vagina, es decir una
intervención muy sencilla..(R3)

316.
Paciente tercigesta, con antecedente de dos partos eutócicos en las semanas 35 y 34,
respectivamente, acude en la semana 31 a urgencias del hospital por percibir
contracciones. Exploración: cérvix permeable 1 dedo, borrado 50%, consistencia media,
posición media. Feto longitudinal podálica. Se realiza RCTG que se muestra en la imagen.
Longitud cervical por USGtransvaginal 20 mm. Test de la fibronectina positivo. ¿Cuál
sería la actitud más correcta en este caso?:

 
 
 
 
1. 1. Mantener a la paciente en observación y reevaluar en 2 horas.
2. 2. Iniciar tocólisis con betamiméticos y maduración pulmonar con betametasona.
3. 3. Iniciar tocólisis con atosibán y maduración pulmonar con betametasona.
4. 4. Ya se ha iniciado el parto; realizar una cesárea, ya que la presentación es podálica.
Gráfico de respuestas
Comentario

A pesar de que la dinámica uterina es de parto, el cérvix todavía no lo es. Teniendo en cuenta que
el embarazo todavía no ha llegado a su semana 34, debemos tratar de frenar el parto. A partir de la
semana 34, no sería necesario utilizar corticoides para la maduración pulmonar fetal, pero antes de
este momento sí lo es, como en este caso. Por último, entre los betamiméticos y el atosibán, en la
actualidad se prefiere este último.

No olvide que el atosibán es un inhibidor competitivo de la oxitocina, de ahí su carácter tocolítico.


Es tan eficaz como la ritodrina (betamimético clásicamente utilizado), pero sus efectos adversos
son menores, por lo que resulta preferible.(R3)

317. Femenino de 33 años consulta por un nódulo mamario que se diagnostica de


carcinoma ductal. Su madre a los 50 años y su hermana a los 40 han tenido cáncer de
mama. Su tía materna a los 45 y su abuela a los 55 han tenido cáncer de ovario. ¿Qué
estudio solicitaría?

1. 1. BCRA1.
2. 2. Her2.
3. 3. P53.
4. 4. Receptor de estrógeno.
Gráfico de respuestas
Comentario

En el cáncer de mama, el componente hereditario es muy importante. Existen dos genes, BRCA1 y
BRCA2, que son los responsables de más de la mitad de casos de cáncer de mama hereditario
(respuesta 1 correcta).(R1)

318. La presion arterial de O2 (PaO2) depende de los siguientes factores EXCEPTO:

1. 1. Concentración de Hb en la sangre que pasa por los capilares pulmonares.


2. 2. Presión barométrica.
3. 3. Cociente respiratorio (R).
4. 4. Presión del vapor de agua.
Gráfico de respuestas
Comentario
La presion arterial de O2 va a depender de la presión alveolar de O2 que depende de la FIO2, del
cociente respiratorio(que indica la pCO2), de la presion barométrica, y de la presión del vapor de
agua. La paO2, si no hay patología parenquimatosa, es entre 10 y 15 mmHg menor que la PAO2,
pero no depende de la concentración de Hemoglobina. La concentración de hemoglobina si varia la
cantidad total de O2 que pasa a sangre pero no la presión parcial.(R1)

319. Respecto a la consulta del adolescente, marque lo CORRECTO:

1. 1. Muchos son forzados a ir a la consulta médica.


2. 2. Suelen ser inseguros de cómo expresar sus problemas.

 
 
 
 
3. 3. Les preocupa cualquier diferencia física con sus similares.
4. 4. Todas son correctas.
Gráfico de respuestas
Comentario

Por sentido común, la respuesta correcta sería la 4.(R4)

320. Con relación al Test de Apgar, señale la respuesta INCORRECTA:

Mide cinco parámetros: Respiración, frecuencia cardiaca, color, tono e irritabilidad


1. 1.
refleja.
2. 2. Se realiza al 1 er. y 5 to. minuto después del parto.
3. 3. La valoración del Apgar no interfiere con la reanimación neonatal.
Un puntaje menor a 7 requiere que al neonato se le de asistencia probablemente con
4. 4.
oxigenoterapia.
Gráfico de respuestas
Comentario

Debes dominar el APGAR para el día del ENARM.

Recuerde(R4)

El test de Apgar valora cinco parámetros: esfuerzo respiratorio, respuesta a la estimulación (o a la


introducción de una SNG), frecuencia cardíaca, tono muscular y color.

Cada parámetro se valora entre 0 y 2 puntos. La situación óptima de cada parámetro se evalúa con
dos puntos; la situación más desfavorable ("ausencia de"), con cero puntos. La situación intermedia
se valora con 1 punto.

El test de Apgar se realiza al minuto de vida y a los 5 minutos (y cada 5 minutos hasta los 20
minutos de nacido si la puntuación a los 5 minutos es inferior a 7).

Un test de Apgar bajo en los primeros minutos de vida no sirve para establecer valoraciones
pronósticas. No obstante, una puntuación inferior a 3, mantenida más allá de los 20 minutos de
vida, sí puede predecir una elevada morbi-mortalidad.

 
 
 
 
321. ¿Cuál de los siguientes psicofármacos debe iniciarse a dosis muy bajas y ascender
lentamente para minimizar el riesgo de una reacción cutánea grave?:

1. 1. Trazodona.
2. 2. Amitriptilina.
3. 3. Lamotrigina.
4. 4. Clozapina.
Gráfico de respuestas
Comentario
Todos los fármacos mencionados en la pregunta se deben iniciar a dosis bajas y su ascenso tiene
que ser lento para evitar efectos adversos y mejorar la tolerabilidad. En el caso de la lamotrigina
(utilizado como estabilizador a la hora de prevenir recaídas depresivas en el trastorno bipolar) el
problema es el desarrollo de reacciones cutáneas graves (síndrome de Stevens- Johnson,
necrolisis epidérmica tóxica, etc.), son más frecuentes en niños y en pacientes que toman otros
fármacos con similar toxicidad cutánea (ácido valproico), disminuyéndose el riesgo si el ascenso se
realiza muy lentamente.La clozapina se asciende despacio por la aparición de sedación e
hipotensión, algo parecido a lo que sucede con la trazodona o la amitriptilina (esta última también
tiene importantes efectos anticolinérgicos). En el caso del litio van a plantear problemas los efectos
gastrointestinales, el temblor o la poliuria, realizándose el ajuste en función de la tolerancia a los
efectos adversos y el ascenso de los niveles plasmáticos.(R3)

322. Varón de 56 años que acude a urgencias por dolor en epigastrio que irradia hacia
hipocondrio derecho, náuseas y vómitos alimentario-biliosos e ictericia leve. La analítica
realizada en urgencias fue la siguiente: Hb 14.2 g/dl, leucocitos 11.000/mm3 con 80%
neutrófilos, plaquetas 200.000/mm3, glucosa 123 mg/dl, urea 45 mg/dl, creatinina 1.0
mg/dl, albúmina 5.5 g/dl, GOT 87 U/l, GPT 89 U/l, GGT 88 U/L, bilirrubina total 2.5 mg/dl,
fosfatasa alcalina 400 U/l, LDH 120 U/l, amilasa 15 U/ml, sodio 136, potasio 4.8. Se realiza
una ecografía abdominal donde se observa una vesícula con una imagen hiperecogénica
en el infundíbulo en íntimo contacto con colédoco, colédoco distal de 5 mm con
dilatación proximal desde hepático común y de vía biliar intrahepática. ¿Cuál, de entre
las siguientes, es el diagnóstico más probable?:

1. 1. Ampuloma.
2. 2. Coledocolitiasis.
3. 3. Síndrome de Mirizzi.
4. 4. Enfermedad de Caroli.
Gráfico de respuestas
Comentario
El Síndrome de Mirizzi es una entidad caracterizada por la compresión por una litiasis vesicular en
el infundíbulo generalmente sobre la vía biliar y por tanto la compresión provoca dilatación desde
ese nivel con colédoco distal de calibre normal.(R3)

323. 36 hours after birth, a newborn presents with weak crying, hypotonia and seizures.
He has slight macrocephaly with microphthalmus and bilateral choriorretinitis. Physical
examination at birth showed hepatosplenomegaly. When he was 12 hours old, he was
found to have jaundice and petechiae, especially located in skin folds. Which finding
would you expect to find in a brain CT scan of this patient?

1. 1. Periventricular calcifications.
2. 2. Cerebellar hypoplasia.

 
 
 
 
3. 3. Diffuse cerebral edema.
4. 4. Diffuse calcifications.
Gráfico de respuestas
Comentario

Cuando nos hablan de macrocefalia, debe considerar una posible toxoplasmosis cerebral,
diagnóstico que además es compatible con la coriorretinitis y la hepatosplenomegalia que nos
describen. Desde el punto de vista de la imagen cerebral, nos encontraríamos con calcificaciones
dispersas. La infección connatal de las calcificaciones periventriculares es el CMV.(R4)

324. Señale la INCORRECTA en relación a la pseudoobstrucción intestinal idiopática:

1. 1. Se afecta sobre todo el intestino delgado.


2. 2. Puede haber niveles elevados de prostaglandina E.
3. 3. El único tratamiento efectivo en los brotes agudos es la cirugía.
4. 4. Pueden aparecen alteraciones en la motilidad de la vejiga.
Gráfico de respuestas
Comentario

La Pseudoobstrucción intestinal no es un tema muy importante pero si debe saber en que consiste
y cual es su manejo, puesto que ha sido preguntado en alguna ocasión. Esta pregunta la puede
sacar si sabs el tratamiento de esta patología aunque no domine los aspectos de su fisiopatología.

El manejo de la pseudoobstrucción intestinal debe ser escalonado: inicialmente se usan enemas


de descompresión y tratamiento de apoyo; si estos fracasan y existe dilatación colónica esta
indicada la colonoscopia descompresiva. Solo en caso de que exista riesgo de perforación está
indicada la cirugía con resección del tramo afecto. La neostigmina actúa inhibiendo la
acetilcolinesterasa favoreciendo la motilidad intestinal y puede ser útil en el manejo de estos
pacientes evitando la colonoscopia descompresiva y la cirugía.(R3)

325. Hombre de 3 meses que consulta por vómitos desde hace 1 mes. Vomita después
de todas las tomas, sin fuerza; come siempre con buen apetito y presenta adecuado
desarrollo pondero estatural, sin pérdida de peso. Realiza dos evacuaciones diarias con
heces de consistencia, aspecto y volumen normal. Entre sus antecedentes destaca estar
alimentado con fórmula artificial desde el mes de vida. Su actitud sería:

1. 1. Hacer endoscopia para descartar esofagitis.


2. 2. Solicitar US abdominal y/o tránsito digestivo.
3. 3. Realizar manometría esofágica.
4. 4. Indicar a la madre medidas posturales.
Gráfico de respuestas
Comentario

Se trata de un niño con reflujo no patológico, es decir reflujo que no afecta al desarrollo
pondoestatural y no produce más clínica que las regurgitaciones. Por lo tanto no estaría indicada
ninguna prueba complementària ni ningún tratamiento farmacológico. Se tendría que indicar a la
familia la posición erecta después de las tomas unos 30 minutos, el reposo con cama inclinada y
informarles que se trata de algo madurativo que mejorará con la edad, la posición de sedestación y
la comida sólida o más consistente.(R4)

 
 
 
 
326. Un varón de 70 años con artritis reumatoide de 30 años de evolución consulta por
astenia, anorexia, disnea de pequeños esfuerzos, hinchazón en ambas piernas y dolor y
tumefacción en rodillas y codos. En la exploración destaca presión venosa yugular
elevada, ruidos cardíacos tenues, crepitantes en bases pulmonares y
hepatoesplenomegalia. En la analítica destaca: Hb de 10 g/dl, VSG 60, albúmina de 2 g/l
y proteinuria de 10 g en 24 horas. ECG con ritmo sinusal a 70 lpm y bajo voltaje. ¿Qué
prueba estaría más indicada para establecer el diagnóstico?

1. 1. Ecocardiograma.
2. 2. Biopsia renal.
3. 3. Biopsia de grasa subcutánea abdominal.
4. 4. Biopsia de médula ósea.
Gráfico de respuestas
Comentario

Es un caso clínico preguntado con bastante frecuencia en el MIR, así que si no la has acertado
repásalo. Nos presentan un paciente con AR de 30 años de evolución (pista muy importante) con
afectación principalmente cardiaca (el paciente presenta clínica de insuficiencia cardiaca: edemas
generalizados, hepatoesplenomegalia, crepitantes bilaterales y aumento de la presión yugular) y
renal (proteinuria importante). Ante todo esto debemos sospechar en primer lugar una
AMILOIDOSIS. El amiloide que se deposita en las formas secundarias es de tipo AA y el
diagnostico se hace mediante aspiración de grasa abdominal (menos cruento y por tanto de
primera elección), biopsia rectal o biopsia renal.(R3)

327. Niño de 12 años con una talla para la edad en el percentil 10 y cuya velocidad de
crecimiento está por encima del percentil 25. Existen antecedentes de baja talla familiar
en la rama materna y de pubertad retrasada en la paterna. La exploración física es normal,
siendo el volumen testicular de 3 cc. Señale la respuesta CORRECTA:

1. 1. La opción terapéutica de elección es el empleo de hormona del crecimiento.


El indicador clínico que mejor refleja la normalidad del proceso es la velocidad de
2. 2.
crecimiento.
3. 3. Es aconsejable estudiar el eje de la hormona del crecimiento.
4. 4. Se debe realizar control periódico de la talla cada 2 meses.
Gráfico de respuestas
Comentario

Una pregunta de dificultad media que se responde de forma directa a partir del Manual CTO.

Tal como explicamos en el manual de la Sección de Pediatría, el indicador más sensible para
detectar precozmente este tipo de alteraciones es la velocidad de crecimiento (respuesta 2
correcta). Ésta es máxima durante los cuatro primeros años de la vida, produciéndose un segundo
incremento (aunque menor) niños.

Durante la pubertad, siendo más precoz en niñas que en Los datos que hay que tener en cuenta
para evaluar una talla baja son la velocidad de crecimiento y la correlación con la talla de los
padres. En este caso clínico se trata de un niño con una velocidad de crecimiento normal. Por otra
parte, la talla se puede considerar baja pero dentro de los límites normales y, sobre todo, acorde
con la talla materna.

Además, nos explican que en la rama paterna, existen antecedentes de pubertad retrasada.

 
 
 
 
Por consiguiente, no está indicado utilizar hormona del crecimiento ni realizar estudios del eje
hipotálamo-hipofisario. En otras palabras, no estamos ante un caso en el que haya que pensar en
patología de ningún tipo, al menos con estos datos, por lo que no precisa que tomemos medidas
extraordinarias.(R2)

328. ¿Cuál sería el tratamiento más adecuado para una paciente con un carcinoma de
cérvix estadio IB2?:

1. 1. Histerectomía total simple.


2. 2. Histerectomía con doble anexectomía y linfadenectomía.
3. 3. Histerectomía radical tipo Wertheim-Meigs.
4. 4. Quimioterapia y radioterapia.
Gráfico de respuestas
Comentario
Al ser un tumor de >4 cm, de entrada es inoperable, por tanto se administra quimioterapia
radiosensibilizante y después radioterapia. En el estadio IB2, la cirugía+RT no ha demostrado más
eficacia que quimioterapia + radioterapia, y además la cirugía tiene mayor tasa de
complicaciones.(R4)

329. ¿En cuál de los siguientes pacientes está más indicada la administración de
inmunoglobulina antivaricela-zoster?:

1. 1. Lactante sano de 3 meses, cuya hermana de 4 años acaba de presentar varicela.


Una niña de 2 años con leucemia, cuya hermana de 6 años comenzó con varicela el día
2. 2.
anterior.
3. 3. Hombre de 4 años con leucemia, que tiene fiebre y acaba de presentar varicela.
4. 4. Hombre de 7 años, que recibe corticoides por asma y presenta varicela.
Gráfico de respuestas
Comentario

Las indicaciones de profilaxis pasiva con gammaglobulina antivaricela zoster son:

En las primeras 72 horas postexposición en:

- Immunodeprimidos sin el antecedente de vacunación o infección previa de varicela.

- Recién nacidos de una madre cuya varicela comenzó entre los 5 días previos al parto y 48 horas
después.

- Prematuro de mayor o igual a 28 semanas cuya madre carece de antecedentes de varicela o


serología negativa.

- Prematuros menores de 28 semanas o de 1000 g de PRN sea cual sea el estado serológico
materno.
(R2)

330. En relación con los pólipos del colon, indique la afirmación que le parece CIERTA:

1. 1. Hasta el 25% de los pólipos malignizan.


2. 2. La mayoría de los cánceres colorrectales derivan de un pólipo adenomatoso.

 
 
 
 
El síndrome de Gardner asocia los pólipos del colon a neoplasias del sistema nervioso
3. 3.
central.
4. 4. Los pólipos adenomatosos tubulares son los que más tienden a malignizarse.
Gráfico de respuestas
Comentario

La opción correcta es la 2, ya que la mayoría de los cánceres colorrectales provienen de un pólipo


previo (90% de los casos). Sin embargo, ante un pólipo, la probabilidad de malignización es menor
del 1%, siendo los que más riesgo tienen los adenomatosos con componente velloso.

En caso de carcinoma in situ, como el tumor no sobrepasa la membrana basal, la polipectomía


endoscópica es más que suficiente. El síndrome que asocia tumores colónicos derivados de
pólipos y tumores del SNC es el síndrome de Turcot.(R2)

331. Una gestante de 10 semanas que acude a recoger los resultados de laboratorio de
primer trimestre y en el cultivo de orina se encuentran >100,000 UFC de E. coli. La
paciente está asintomática. ¿Qué tratamiento debe recibir?:

1. 1. Ninguno, dado que está asintomática.


2. 2. Quinolonas.
3. 3. Fosfomicina.
4. 4. Tetraciclinas.
Gráfico de respuestas
Comentario

La bacteriuria asintomática en la gestante es necesario tratarla, ya que tiene más riesgo de


progresar a ITU y a pielonefritis aguda y a provocar amenaza de parto pretérmino. Todos los
antibióticos anteriores son teratogénicos excepto la fosfomicina así lo indica la guia de práctica
clínica.(R3)

332. Con respecto al desarrollo pondero-estatural infantil NO es cierto que:

Es normal que los recién nacidos pierdan hasta un 15% de su peso al nacer en los
1. 1.
primeros 7 días de vida.
2. 2. Los niños duplican su peso al nacimiento cuando cumplen 5 meses de vida.
3. 3. Un peso menor de 3 kilos al nacimiento es bajo peso en los recién nacidos a término.
La velocidad de crecimiento estatural hasta los 4 años es la más alta después de
4. 4.
nacimiento.
Gráfico de respuestas
Comentario

Del crecimiento en peso y talla del niño debe saber:

Antropometría neonatal: peso (2.500- 3.500 gramos a término), talla (48- 53 cm) y perímetro
cefálico (32- 37 cm).

Se considera pequeño para la edad gestacional aquel por debajo del percentil 10 y elevado lo que
esté por encima del p90.

Peso: Un niño duplica su peso al 5º mes, lo triplica al año y lo cuadriplica a los 2 años.

 
 
 
 
Talla: Un niño dobla su altura a los 4 años.

La pérdida de peso en un RN hasta del 10%.(R1)

333. La enfermedad de membrana hialina afecta principalmente al recién nacido:

1. 1. A termino.
2. 2. De madre hipertensa.
3. 3. De madre adicta a opiáceos.
4. 4. Prematuro.
Gráfico de respuestas
Comentario

La enfermedad de membrana hialina es la causa más frecuente de distrés respiratorio en el recién


nacido pretermino. Se debe a un déficit de surfactante, que no alcanza la superficie pulmonar hasta
la semana 34-35. Este déficit causa un aumento de tensión superficial y una tendencia al colpso
pulmonar.

En el bebé a término, hijo de madre hipertensa e hijo de madre adicta a los opiaceos, ocurre lo
contrario, se reduce el riesgo de sufrir enfermedad de la membrana hialina.(R4)

334. La reducción del flujo sanguíneo que puede tolerar el intestino sin sufrir lesión es:
del:

1. 1. 15%.
2. 2. 30-50%.
3. 3. 50-75%.
4. 4. Más del 75%.
Gráfico de respuestas
Comentario
El intestino es capaz de soportar una reducción del flujo sanguíneo del 50- 75% sin llegar a
lesionarse.(R3)

335. El hermafroditismo verdadero se caracteriza por lo siguiente, EXCEPTO:

1. 1. Ovotestes.
2. 2. Cariotipo 46 XX.
3. 3. Genitales externos ambiguos.
4. 4. Gónadas funcionales.
Gráfico de respuestas
Comentario

Hermafroditismo Verdadero. Se trata de individuos que presentan simultáneamente tejido ovárico y


testicular en la conformación interna de su aparato reproductor. El diagnóstico correcto debe
establecerse histológicamente, no dependiendo de la apariencia física de los órganos sexuales
externos.

El hermafroditismo verdadero puede producirse por: mosaicismo de los cromosomas sexuales


debido a errores en la división mitótica o meiótica; quimerismo por doble fertilización o por fusión
de dos óvulos fertilizados normalmente; translocación de genes del cromosoma Y a un autosoma o

 
 
 
 
al cromosoma X, y por mutación de genes los autosómicos o del cromosoma X que participan en la
diferenciación sexual.

La mayoría de los hermafroditas 46, XX son SRY negativo y la presencia de tejido testicular se
explica de forma similar que en el hombre 46, XX; es decir, mosaicismo críptico o con línea 46, XY
no detectada, translocación de los genes determinantes del testículo al cromosoma X o a un
autosoma y mutación de los genes autosómicos o del cromosoma X que participan en la formación
de los testículos. Los pacientes SRY positivo y cariotipo 46, XX pueden originar hombres 46, XX; o
hermafroditas verdaderos si se inactiva el cromosoma X que porta el gen SRY.

Por su parte, los hermafroditas 46, XY pueden tener un mosaicismo gonadal con línea celular 46,
XX no detectada, una mutación del gen SRY o una mutación en algún gen autosómico de los que
participan en la cascada de la diferenciación sexual masculina.

El diagnóstico de hermafroditismo verdadero es posible sólo si se demuestra la existencia de tejido


testicular y ovárico. Para confirmar la presencia de tejido ovárico no es suficiente demostrar la
presencia de estroma ovárico, sino que es necesario detectar la existencia de ovocitos. Este
hallazgo lo diferencia de la disgenesia gonadal mixta, donde sólo se demuestra el componente
estromal ovárico en la gónada acintada.(R4)

336. El síndrome de Zieve consiste en:

1. 1. Esteatosis hepática alcohólica, anemia hemolítica e hipertrigliceridemia.


2. 2. Hiperesplenismo y esteatosis hepática.
3. 3. Cirrosis biliar primaria, hipergammaglobulinemia y plaquetopenia.
4. 4. Anemia microangiopática, esplenomegalia y hepatitis crónica.
Gráfico de respuestas
Comentario

El síndrome de Zieve es una complicación extremadamente infrecuente de la esteatosis hepática


secundaria al alcohol (hígado graso alcohólico). Se define por la asociación de hipertrigliceridemia,
anemia hemolítica e ictericia en un paciente con esteatosis hepática alcohólica. Los pacientes
suelen tener además dolor abdominal. La trombocitopenia, cirrosis, hiperesplenismo e
hipergammaglobulinemia también pueden manifestarse como parte de este síndrome, aunque no
forman parte de la tríada que lo define.(R1)

337. Neonato de 7 días de vida que acude a Urgencias por fiebre cuantificada en 38ºC,
irritabilidad, con succión conservada. Al examen físico ictérico y se observa disminución
del 12% del peso con respecto al de su nacimiento. Qué actitud tomaría usted:

1. 1. Tomaría BH y examen de orina y dar de alta de ser normales.


2. 2. Hemocultivos y urinocultivo, con posterior evaluación a las 24 horas.
Sospecharía mala técnica de lactancia y daría de alta con apoyo de formula y evaluaría a
3. 3.
las 48 horas.
Sospecharía mala técnica de lactancia pero de todas maneras buscaría infección, hasta
4. 4.
realizando punción lumbar.
Gráfico de respuestas
Comentario

Muy bien, nos plantean el caso de un neonato con fiebre y pérdida ponderal.

 
 
 
 
Entonces, si bien la pérdida ponderal podemos achacarla a mala técnica de alimentación, la fiebre
no suele ser producida por este fenómeno, así que tendriamos que buscar un foco infeccioso.

No podemos quedarnos tranquilos sin una búsqueda diagnóstica exhaustiva, como plantean las
opciones 1, 2 y 3, incluso si hay que realizar una PL. La ictericia, puede estar en el contexto
fisiológio del recién nacido.(R4)

338. Entre las manifestaciones clínicas de sífilis congénita precoz, señale cuál NO suele
aparecer en el recién nacido:

1. 1. Coriza.
2. 2. Hepatoesplenomegalia.
3. 3. Signos de infección del S.N.C.
4. 4. Ictericia.
Gráfico de respuestas
Comentario

En la sífilis congénita precoz, la manifestación más frecuente es la hepatomegalia, sin duda


alguna. Acompañan a ésta un exantema polimorfo (máculas, pápulas, ampollas) con repercusión
sobre la mucosa nasal (coriza o rinorrea) y lesiones óseas con periostitis, que derivan en lo que se
conoce como falsa parálisis (antiálgica) de Parrot. La afectación del SNC es más típica de la sífilis
congénita tardía.(R3)

339. A 72-year-old male comes to the emergency room complaining of increased urinary
frequency, dysuria and scrotum pain that radiates to the left flank. He denies urethral
discharge. His temperature is 101ºF (38,2ºC), his blood pressure is 130/85 and his pulse
is 90/min. Rectal exam shows tenderness of the prostate. Urinalysis shows bacteriuria
and pyuria. Which of the following is the most likely cause of this patient's infection?

1. 1. Escherichia coli
2. 2. Ureaplasma urealyticus
3. 3. Staphylococcus aureus
4. 4. Chlamydia trachomatis
Gráfico de respuestas
Comentario
Escherichia coli. Acute epididymitis causes fever and scrotum pain. It can be associated to an UTI
or to sexually transmitted urethritis. In this case, the absence of urethral discharge and other
symptoms of urethritis suggest a non-sexually transmitted infection. The most frequent cause ,as in
any UTI, is E. Coli.(R1)

340. En relación a los divertículos colónicos, señala el enunciado FALSO:

1. 1. Su incidencia aumenta con la edad.


2. 2. Son más frecuentes a nivel de sigma.
3. 3. Son protusiones en fondo de saco de toda la pared intestinal.
4. 4. La hemorragia ocurre sólo en un pequeño porcentaje de pacientes.
Gráfico de respuestas
Comentario
Los divertículos de colon son herniaciones de la mucosa colónica a través de las capas musculares
de la pared del colon. La enfermedad diverticular, o diverticulosis, es más frecuente en personas
ancianas. Se localizan fundamentalmente en colon descendente. Suelen ser asintomáticos. Pero

 
 
 
 
cuando dan clínica, lo más frecuente es la hemorragia (que generalmente es de divertículos de
colon ascendente).(R3)

341. Un niño de 10 meses presenta estatura baja, orejas prominentes de implantación


baja, disgenesia gonadal y cariotipo 46 XY. Su madre nos cuenta que poco después de
nacer fue sometido a una intervención por presentar una patología cardíaca congénita en
la que "le metieron un tubito por la pierna". Desde entonces se había encontrado bien,
pero ahora le encuentra decaído, "como cansado" y presenta a la exploración síntomas
de congestión venosa. ¿Qué patología cardíaca cree que padece sólo con estos datos y
cuál sería su tratamiento?:

1. 1. Estenosis pulmonar, valvuloplastia pulmonar.


2. 2. Estenosis pulmonar, valvulotomía pulmonar.
3. 3. Estenosis aórtica, prótesis valvular.
4. 4. Coartación de aorta postductal, técnica de Crafoord.
Gráfico de respuestas
Comentario

Pregunta muy difícil y de poca relevancia.

Ante el antecedente de patología cardíaca congénita, que fue tratada probablemente con
valvuloplastia percutánea con balón, debemos sospechar una Estenosis Pulmonar. Y como ha
vuelto a reaparecer, el tratamiento de elección sería en esta ocasión la valvulotomía pulmonar. Si
vamos más allá, con el cuadro descrito, podríamos sospechar un síndrome de Noonan, una
enfermedad autonómica dominante, con un fenotipo similar al síndrome de Turner (anteriormente
fue llamada síndrome de Turner masculino), pero con cariotipo normal. Los rasgos cardinales son
talla baja, dismorfias craneofaciales, pectum excavatum, cardiopatía congénita y criptorquídea. Las
malformaciones cardiovasculares ocurren en 50 a 80% de los sujetos con síndrome de Noonan. La
estenosis de la válvula pulmonar, asociada a una válvula pulmonar displásica es la cardiopatía más
común (20 a 50% de los pacientes). Virtualmente todos los tipos de cardiopatías congénitas han
sido descritos en ellos. La frecuencia de síndrome de Noonan en pacientes con estenosis
pulmonar es del 7%. La miocardiopatía hipertrófica, tanto obstructiva como no obstructiva, se
describe en el 20 al 30% de estos pacientes, pudiendo estar presente ya en el período de RN o
bien aparecer en el período de lactante o niñez. Por ello se debe realizar un seguimiento
ecocardiográfico.(R2)

342. Respecto al derrame pleural, es FALSO:

1. 1. El agente más típico causante de derrame pleural paraneumónico es S. pneumoniae.


El derrame pleural tuberculoso es la causa más frecuente de exudado en países
2. 2.
subdesarrollados, suele ser unilateral y con glucosa menor de 50 mg/dl.
3. 3. La causa tumoral más frecuente de quilotórax son los linfomas.
En el derrame pleural paraneumónico predominan los neutrófilos, en el tuberculoso los
4. 4.
linfocitos.
Gráfico de respuestas
Comentario

Tanto el derrame tuberculoso como el paraneumónico reúnen características de exudado. Sin


embargo, en el primero predominan los linfocitos y en el segundo los polimorfonucleares. En el
caso concreto de la tuberculosis, muchas características son compartidas con el derrame de la
artritis reumatoide (ADA+, linfocitos, glucosa baja…). Sin embargo, en el caso del derrame

 
 
 
 
reumatoideo el descenso de la glucosa es más pronunciado que en el tuberculoso, tal como
menciona la respuesta 3. Por otra parte, es correcto que, en muchos países subdesarrollados, la
causa más frecuente de exudado pleural es la tuberculosis.

La respuesta incorrecta es la 1, puesto que la aparición de derrame paraneumónico está más


relacionada con las neumonías estafilocócicas y las producidas por H. influenzae.(R1)

343. Las afirmaciones siguientes acerca de la atelectasia son correctas, EXCEPTO:

1. 1. Abarca 50% de las complicaciones pulmonares postoperatorias.


Es más frecuente después de las operaciones de la porción superior del abdomen que
2. 2.
después de las de la porción inferior.
3. 3. Puede resultar de obstrucción bronquial o hipoventilación.
4. 4. Es usual que se manifieste en las primeras 24 horas del postoperatorio.
Gráfico de respuestas
Comentario
La aparición de una atelectasia en el postoperatorio inmediato es una complicación cada vez
menos frecuente debido al adecuado uso de los analgésicos en esta fase del postoperatorio que
permiten al paciente mantener una adecuada mecánica ventilatoria que estaría dificultada en casos
de no tener un adecuado control del dolor y que ha llevado incluso al uso de catéteres epidurales
de forma casi rutinaria en algunos tipos de cirugía. También el empleo de espirómetros
incentivados y en general de una adecuada fisioterapia respiratoria han contribuido en este hecho,
de forma que su frecuencia no suele superar el 10 % de los casos.(R1)

344. De los métodos terapéuticos citados a continuación, ¿cuál considera que resulta
más eficaz para prevenir la fractura de cadera?

1. 1. Estrógenos.
2. 2. Bifosfonatos.
3. 3. Actividad física.
4. 4. Calcitonina.
Gráfico de respuestas
Comentario

Esta pregunta hace referencia a la prevención de la osteoporosis y, por tanto, de la fractura de


cadera. Dentro de las medidas no farmacológicas mantener una actividad física adecuada aumenta
la formación de hueso, y es la respuesta que eligen como medida más eficaz para prevenir
fracturas futuras. Además, favorece la agilidad del paciente y evita la aparición de caídas.

Como medidas farmacológicas han demostrado su utilidad en la reducción de fracturas de cadera


tanto los bifosfonatos, como los estrógenos. Al haber dos medidas farmacológicas eficaces, lo
lógico es pensar que la respuesta correcta es el ejercicio físico.(R3)

345. Médico interno de pregrado de 22 años que se punciona de forma accidental con una
aguja que acaba de usar para extraer sangre de un drogadicto para hacer unos análisis.
El drogadicto es VIH negativo, pero VHB positivo; el MIP está vacunada contra el VHB. La
actitud adecuada en este caso es:

1. 1. Administrar Ig y vacunar lo antes posible.


2. 2. Vacunación completa nuevamente.
3. 3. Si el título de anti-HBs es mayor de 10 UI, no hay que hacer nada.

 
 
 
 
4. 4. Si el título de anti-HBs es menor de 10 UI, se da la pauta completa de vacuna.
Gráfico de respuestas
Comentario

Tras una punción accidental en un sujeto vacunado, la actitud sería determinar el título de
Anti- HBs, de forma que si es mayor de 10 UI no habría que hacer nada. Si fuera menor de 10 UI,
sería conveniente realizar profilaxis con inmunoglobulina.(R3)

346.A 18-year-old male is brought to the physician's office presenting with marked
aggressiveness and poor school performance during the past two months. He spends
five hours a day at the gym. He denies drug consumption. Physical examination shows
incipient frontal alopecia, acne and good muscle development. Which substance is this
patient most likely abusing?

1. 1. Anabolic steroids
2. 2. Marihuana
3. 3. Alcohol
4. 4. Sedatives
Gráfico de respuestas
Comentario
Anabolic steroids. Aggressiveness and accelerated androgenic alopecia suggest anabolic steroid
consumption. The physical changes associated with this substance might also include baldness,
gynecomastia, hepatic function impairment and testicular atrophy. Women present hirsutism and
clitoris enlargement.(R1)

347. Paciente de 25 años que acude a


su consulta por amenorrea secundaria. Peso 55 kg, talla 165 cm. Caracteres sexuales
secundarios normales. Solicita estudios de laboratorio que demuestran lo siguiente:
beta-HCG en orina negativa, en sangre: TSH 1.5 mUI/l, T4 libre 1.6 ng/dl, prolactina 35
ng/ml, estradiol 55 pg/ml, FSH 5 mUI/ml, LH 8 mUI/ml, progesterona 0.8 ng/ml. Usted le
practica un ultrasonido vaginal con la imagen ovárica que se muestra. Señale la opción
CORRECTA:

1. 1. Se trata de un quiste dermoide, sobre todo al ser joven y asintomática.

 
 
 
 
Se trata de un endometrioma que produce destrucción del parénquima ovárico y, por
2. 2.
tanto, amenorreas secundarias.
3. 3. Se trata de un síndrome de ovarios poliquísticos.
4. 4. Ya que la imagen ecográfica es normal, podríamos pensar en un microprolactinoma.
Gráfico de respuestas
Comentario

Estamos ante un caso clínico de amenorrea secundaria en la que en el estudio hormonal tenemos
únicamente la prolactina ligeramente elevada y una progesterona anovulatoria. Aunque la LH/FSH
no es mayor a 2, sí hay más LH que FSH, y con todo este contexto podemos tener en mente que
se trata de un síndrome de ovarios poliquísticos (amenorrea, aumento de PRL leve, LH elevada), lo
cual queda corroborado por la imagen del USG, en la cual se ve un ovario con zona central
ecodensa y en la periferia la presencia de 12 o más folículos con diámetros entre 2 y 9 mm y/o un
volumen ovárico mayor de 10 ml.(R3)

348. ¿Qué prueba realizaría para confirmar su diagnóstico y qué tratamiento propondría
para la paciente del caso clínico anterior?

Solicitar marcador tumoral Ca 19.9, alfa-fetoproteína y laparoscopía diagnóstico-


1. 1.
terapéutica.
2. 2. Solicitar marcador tumoral Ca-125 y laparoscopía diagnóstico-terapéutica.
Prueba de 5 mg diarios de acetato de medroxiprogesterona durante 10 días, y después
3. 3.
pautar un anticonceptivo hormonal.
4. 4. Solicitar RM de silla turca y pautar cabergolina 1 mg semanal.
Gráfico de respuestas
Comentario

La opción 1 sería válida si la paciente tuviera un quiste dermoide ovárico y la 2 si fuera una
endometriosis. La 3 es la opción correcta, ya que estamos ante un síndrome de ovarios
poliquísticos. Con el gestágeno trasformamos el endometrio proliferativo en secretor y, al
suspenderlo, provocamos una hemorragia por deprivación, lo cual es indicativo de anovulación. El
tratamiento más correcto es un anticonceptivo hormonal que minimice los efectos androgénicos,
regule el ciclo endometrial e inhiba la hipófisis, prefiriendo aquellos cuyo gestágeno sea
antiandrogénico como el acetato de ciproterona, acetato de clormadinona o drospirenona. La RM
de silla turca (respuesta 4) la solicitaríamos si sospechásemos un prolactinoma, ten en cuenta que
este nivel de prolactina es una elevación muy leve debida al síndrome de ovario poliquístico.(R3)

349. Mujer de 25 años con nódulo mamario palpable de aparición brusca. El ultrasonido
revela un nódulo anecogénico, de límites muy precisos, morfología regular y refuerzo
posterior, único, de 3 cm de diámetro. ¿Cuál de los que se relacionan es el diagnóstico
más probable?

1. 1. Quiste.
2. 2. Displasia fibrosa.
3. 3. Cáncer.
4. 4. Hamartoma.
Gráfico de respuestas
Comentario

 
 
 
 
Un nódulo anecogénico, de límites muy precisos, morfología regular y refuerzo posterior, único, de
3 cm de diámetro es compatible con un quiste. El resto de patologías no presentan refuerzo
posterior y en el caso de carcinoma no hay presencia de límites precisos.(R1)

350. Si realizamos una tomografía computarizada craneal a un chico de 21 años que,


desde hace 3 años, presenta retraimiento social, abandono de sus actividades
académicas, soliloquios, descuido en el aseo personal y mantenimiento de posturas
extrañas durante largo tiempo, podremos encontrar:

1. 1. Atrofia de la cabeza del núcleo caudado.


2. 2. Hiperintensidad en la sustancia blanca cerebral.
3. 3. Disminución de los cuerpos mamilares.
4. 4. Dilatación ventricular.
Gráfico de respuestas
Comentario

El caso clínico nos describe una posible esquizofrenia, en la que se combinan síntomas negativos
(retraimiento social, descenso del rendimiento académico, abandono del aseo) y positivos
(soliloquios, posturas extrañas). El hallazgo radiológico mediante técnicas de imagen estructural
(TC y RM) más frecuente en estos pacientes es el aumento del tamaño de los ventrículos
cerebrales, aunque se describen más de 100 anomalías asociadas a la enfermedad.

La atrofia del núcleo caudado es propia de la enfermedad de Huntington. El hallazgo de imágenes


hiperintensas en la sustancia blanca (leucoaraiosis) es muy inespecífico y se ha relacionado con la
enfermedad vascular cerebral (sobre todo lesiones de pequeño vaso). La disminución del tamaño
de los cuerpos mamilares se describe en la encefalopatía de Wernicke, sobre todo cuando
progresa hasta producir un trastorno amnésico de Korsakov (también aparecen lesiones en
núcleos talámicos).

Actualmente, se está investigando mediante técnicas de imagen funcional (SPECT, PET, RM-
funcional con espectroscopia); cuando se evidencian síntomas psicóticos, se suele producir un
aumento de la actividad (vascular, metabólica) en los ganglios basales (debida a un aumento de
actividad dopaminérgica); cuando existen síntomas negativos o depresivos intensos, se produce
una reducción de la actividad en regiones frontales.(R4)

351. A 5-year-old boy with no previous medical history presents to the emergency
department due to progressive fatigue and cough accompanied by fever (38ºC) for the
past three days. He is tachycardic and hypotensive. A systolic murmur is found on
cardiac auscultation. Further significant findings include: hypoperfusion signs (oliguria
and acral cyanosis) and painful hepatomegaly. What is the most likely diagnosis?

1. 1. Acute myocarditis.
2. 2. Rheumatic fever.
3. 3. Kawasaki disease.
4. 4. Acute pericarditis.
Gráfico de respuestas
Comentario

En un paciente sin factores de riesgo caridiovascular, con datos clínicos de insuficiencia cardiaca,
asociado a antecedente de síndrome febril, se deberá descartar el diagnóstico de miocarditis
vírica.(R1)

 
 
 
 
352. Los niños con enfermedad de Ebstein presentan todo lo siguiente, EXCEPTO:

1. 1. Desplazamiento inferior de la válvula tricúspide anormal.


2. 2. Soplo sistólico y diastólico.
3. 3. Aurícula derecha pequeña y ventrículo derecho muy hipertrofiado.
4. 4. Valva tricuspídea anterior agrandada y abombada.
Gráfico de respuestas
Comentario

La anomalía de Ebstein se caracteriza por un desplazamiento hacia el VD del velo septal y a veces
el posterior de la válvula tricúspide, que además suele tener cierto grado de malformación. El velo
anterior no acostumbra a estar desplazado, pero no es raro que esté parcialmente restringida su
movilidad por bandas musculares o cuerdas tendinosas. La insuficiencia tricuspídea es frecuente, y
una parte variable del VD está atrializado con la pared adelgazada. Esta enfermedad se asocia al
consumo de litio durante el embarazo.

La mayoría de los pacientes sobrevive al menos hasta el tercer decenio de la vida. Este defecto se
asocia a CIA en la mitad de los casos, con frecuente cianosis, ya que la presión en la cámara
auricular derecha está elevada por la insuficiencia tricuspídea.

Las manifestaciones clínicas son variables, lo más característica es la existencia de cianosis


progresiva, cuando asocia cortocircuito D-I. Los síntomas derivados de la insuficiencia tricuspídea,
la disfunción del VD y las taquiarrítmias paroxísticas, generalmente asociadas a la frecuente 25%
presencia de vías accesorias derechas y síndrome de Wolff-Parkinson-White y taquicardias por
reentrada antidrómica.

La radiografía se observa cardiomegalia progresiva a expensas de AD. El ECG sobrecarga de la


AD con frecuente bloqueo de rama derecha y preexcitación si hay WPW.

En la exploración física puede presentarse con soplo sistólico y diastólico.(R3)

353. Hombre de 5 años presenta de forma súbita dolor en testículo derecho, sin
antecedente de traumatismo previo. Exploración: afebril, testículo tumefacto, doloroso a
la exploración, con ausencia del reflejo cremastérico. Señale la afirmación FALSA:

1. 1. En el Ultrasonido-Doppler se observaría disminución del flujo sanguíneo en ese testículo.


El tratamiento consiste en antibióticos, antiinflamatorios, y después de la fase aguda,
2. 2.
cirugía.
3. 3. En mayores de 13 años, se debe hacer diagnóstico diferencial con una epididimitis.
4. 4. En el periodo neonatal, generalmente tiene mal pronóstico.
Gráfico de respuestas
Comentario

Se trata de un caso de torsión testicular, por lo que en el USG doppler se obsrvará disminución del
flujo sanguíneo en ese testículo. El tratamiento es quirúrgico y no con antibióticos e inflamatorios,
por lo que la respuesta que debió elegir es la número 2.(R2)

354. A 27-year-old woman goes to her gynecologist's office with a history of pelvic pain
after sexual intercourse and during premenstrual days. She denies unprotected sexual
relations. Her blood pressure is within normal limits. Vaginal posterior fornix is tender at
palpation and uterus is painful with motion. Temperature is 37.5ºC. Regarding the

 
 
 
 
previously described disease, which of the following is the main complication you should
alert your patient about?

1. 1. Infertility
2. 2. Ovarian cancer
3. 3. Abortions
4. 4. Acute peritonitis
Gráfico de respuestas
Comentario
Infertility. The most frequent and serious complication of endometriosis is infertility due to the
formation of peritoneal adhesions and tubal obstruction, which prevent adequate transportation of
the oocytes into the uterine cavity.(R1)

355. Un hombre de 52 años, fumador e hipertenso, acude a urgencias con un infarto


agudo de miocardio de localización inferior. La fase aguda transcurre sin complicaciones.
El siguiente paso más razonable a llevar a cabo sería:

Tratamiento con aspirina y antianginosos, control de los factores de riesgo y hacer la


1. 1.
próxima revisión a los 6-12 meses.
Realizar una ergometría a los 7-14 días del infarto y según su resultado hacer o no una
2. 2.
coronariografía.
Tratar al paciente con antiarrítmicos (por ejemplo, procainamida) durante al menos tres
3. 3.
meses, para prevenir la aparición de arritmias mortales.
Sólo control de los factores de riesgo y, si vuelve a tener síntomas, realizar una
4. 4.
coronariografía.
Gráfico de respuestas
Comentario

Después de un infarto de miocardio, se debe realizar un ecocardiograma para valorar la función


ventricular. Si tiene datos de mal pronóstico, como disfunción ventricular, angor post-IAM o
arritmias ventriculares secundarias, se realiza directamente una coronariografía, para conocer la
anatomía coronaria y evaluar la posibilidad de angioplastia o cirugía de by-pass.

Si, como en este caso, no existen datos de mal pronóstico, se realiza una prueba de detección de
isquemia, como la ergometría, a partir de los 5-7 días del infarto (respuesta 3 correcta). Observe
que nos dicen que “la fase aguda transcurre sin complicaciones”, en este dato está la clave de la
pregunta.

En los casos en los que se utilizó angioplastia primaria con revascularización como técnica de
reperfusión, no suele ser necesaria la realización de la ergometría.(R2)

356. En relación al examen físico del recién nacido marque el enunciado FALSO:

El perímetro cefálico debe valorarse al tercer día cuando el edema y el moldeado de la


1. 1.
cabeza ha terminado.
2. 2. EI reflejo rojo del ojo, es indicio precoz de patología en la retina.
La distancia entre los cantos internos de los ojos, en un recién nacido a termino, varia
3. 3.
entre 1.5 a 2.5 cm.
4. 4. La fontanela posterior, normalmente, puede no palparse en el recién nacido a término.
Gráfico de respuestas
Comentario

 
 
 
 
Si el paciente permanece estable a partir de las tres horas(no tercer día como dice el enunciado
número 1) de vida, una vez realizado el proceso de adaptación extrauterino, se puede realizar una
exploración física completa en la que se incluye la medición de los parametros antropométricos del
sujeto entre los que se incluye el perímetro cefálico que debe oscilar entre los 32-37cm.(R1)

357. Which of the following statements is FALSE regarding alopecia areata?

1. 1. It's a scarring and irreversible alopecia.


2. 2. It's associated with autoimmune diseases.
3. 3. It can affect any hair-bearing area.
4. 4. It can affect any age group.
Gráfico de respuestas
Comentario

Pregunta de cierta dificultad. No hace falta que es estudie con detenimiento la alopecia areata.
Basta que sepa que la alopecia que produce NO es CICATRIZAL, así pues la respuesta falsa sería
la 1. Como sabe, se trata de una enfermedad autoinmune, por lo que no es sorprendente que se
una a otras similares. Las otras opciones aunque no lo sepa, no debería haberls dado por falsas.
Cuando una opción dice PUEDE, si no lo sabe, asume que lo más probable es que sea cierta (hay
que ser muy atrevido para, sin saber, negar algo como posible).(R1)

Alopecias cicatrizales y no cicatrizales

358. De los siguientes signos ultrasonográficos, ¿cuál NO se relaciona con colecistitis


aguda?:

1. 1. Dilatación de vesícula biliar.


2. 2. Engrosamiento de pared vesicular.
3. 3. Dilatación de vías biliares.
4. 4. Acumulación de líquido perivesicular.
Gráfico de respuestas
Comentario

 
 
 
 
Pregunta sencilla que no puede fallar. Le mencionan los signos de la COLECISTITIS AGUDA, no
de la coledocolitiasis, por lo que la respuesta incorrecta es la 3, ya que en la colecistitis aguda no
debería haber dilatación de vías biliares.(R3)

359. Varón de 60 años, en el paro desde hace 6 años, bebedor


moderado y con mala higiene dentaria, que presenta desde hace dos años dificultad para
la deglución de sólidos y pérdida de peso progresiva. Acude por vómitos y regurgitación
de comida; halitosis de un mes de evolución. ¿Cuál es la técnica de elección en este
paciente?:

1. 1. Endoscopía, debido la edad y la los factores de riesgo y a la clínica actual.


2. 2. Primero realizaría un enema de bario, dado que es una técnica menos agresiva.
Lleva mucho tiempo con la clínica, así que le mandaría a consulta de digestivo para
3. 3.
realización de manometría.
Probablemente toda la clínica esté en relación a su consumo excesivo de alcohol, que
4. 4. produce relajación esfínter esofágico inferior con clínica de reflujo y una mala higiene
dentaria que produce halitosis.
Gráfico de respuestas
Comentario

Un cuadro de disfagia con pérdida de peso puede ser compatible con varios diagnósticos. No es
obligado que se trate de un cáncer de esófago (por ejemplo, la acalasia podría producir estos
mismos síntomas), pero es una de las posibilidades y, por lo tanto, deberíamos realizar una
endoscopía. Es improbable que se trate de una neoplasia maligna porque el tiempo de evolución
es muy prolongado (dos años) y el cáncer de esófago es muy agresivo. A pesar de ello, la
endoscopía tiene un papel fundamental en el estudio de la disfagia.(R1)

360. Su médico de atención primaria decide, tras la negativa del paciente a hacerse una
endoscopía, la realización de un esofagograma con bario (ver imagen). ¿Cuál es, por
tanto, la causa de la clínica del paciente?:

1. 1. Divertículo de Zenker.
2. 2. Carcinoma de esófago.
3. 3. Enfermedad de Chagas.

 
 
 
 
4. 4. Cuerpo extraño.
Gráfico de respuestas
Comentario

Dado que nuestro paciente se niega a someterse a la endoscopía, no queda más remedio que
respetar su decisión y optar por otra prueba que pueda darnos información sobre su enfermedad
esofágica. En este caso, el esofagograma con bario resulta de gran ayuda, poniendo de manifiesto
un divertículo de Zenker (mitad superior de la imagen). El diagnóstico es evidente, ya que parte del
contraste se ha quedado en una cavidad accesoria, separada del esófago, donde incluso se forma
un nivel. Probablemente, este divertículo sea la causa del cuadro de halitosis por el que consulta el
paciente, por los fenómenos de putrefacción que sufren los alimentos en el interior de este
divertículo, al verse aislados del tránsito normal.(R1)

361. La malformación congénita de oído externo más frecuente es:

1. 1. Microtia.
2. 2. Anotia.
3. 3. Orejas de soplillo.
4. 4. Agenesia del conducto auditivo externo.
Gráfico de respuestas
Comentario
Una de las anomalías más frecuentes del oído externo es la oreja en asa o de soplillo. Se trata de
una hipertrofia o de un excesivo abombamiento del cartílago de la concha o de una deficiente
plegadura del cartílago del pabellón (hipoplasia o ausencia de antehélix). Normalmente se realiza
corrección quirúrgica en la edad preescolar. La anotia es la ausencia de pabellón auricular, la
microtia es el pabellón pequeño o deformado. La agenesia del CAE suele asociarse a la microtia y
a alteraciones del oído medio.(R3)

362. ¿Cuál de los siguientes NO es una hernia de la pared abdominal?:

1. 1. Hernia de Spiegel.
2. 2. Hernia incisional.
3. 3. Hernia epigástrica.
4. 4. Hernia de Hesselbach.
Gráfico de respuestas
Comentario

Es una pregunta difícil.

La hernia de Spiegel se produce en el punto de unión del borde lateral del recto abdominal con la
línea semilunar de Douglas.

La hernia incisional se produce después de una intervención en la que se haya realizado una
incisión en la pared abdominal.

La hernia epigástrica aparece en la línea alba superior al ombligo.

La hernia de Hesselbach es una profusión de divertículos a través de la vaina femoral, usualmente


asociada a una hernia inguinal oblicua.(R4)

 
 
 
 
363. Entre las siguientes aseveraciones referentes al divertículo de Zenker hay una
FALSA. Señálela:

1. 1. Es más frecuente en varones.


El estudio patogénico demuestra que está causado por la incoordinación en la relajación
2. 2.
del músculo cricofaríngeo.
3. 3. Se suele asociar a hernia de hiato.
El tratamiento quirirrgico sólo debe realizarse en presencia de complicaciones o de
4. 4.
clínica importante.
Gráfico de respuestas
Comentario

El divertículo de Zenker se desarrolla en la región posterior de la hipofaringe en la unión con el


esófago superior. Se acompaña de disfagia tipo faríngeo por disfunción de la musculatura
cricofaríngea. Además, el acúmulo de alimento condiciona halitosis, e incluso en ocasiones
regurgitación maloliente. Sucede más en hombres y se suele asociar a hernia de hiato. Si es
sintomático es ya indicación de miotomía del cricofaríngeo y, si fuera necesario, mejor aún,
acompañado de diverticulotomía.(R4)

364. Mujer de 52 años, con amenorrea desde hace 4 meses, que consulta por la presencia
de artromialgias, cefaleas, sofocos e irritabilidad en los últimos meses. Entre sus
antecedentes destaca el ser fumadora de un paquete de tabaco al día y el ser hipertensa
en tratamiento. Entre las recomendaciones que usted le haría a esta paciente NO se
encuentra:

1. 1. Disminuir el consumo de sal.


2. 2. Tratamiento hormonal sustitutivo.
3. 3. Ejercicio físico moderado.
4. 4. Aumentar los productos lácteos de la dieta.
Gráfico de respuestas
Comentario

La administración de terapia hormonal sustitutiva está contraindicada en pacientes con


antecedentes de cáncer de mama o endometrio, tromboembolismo venoso activo, hepatopatía
grave o neoplasias hepáticas, LES, melanoma y vasculopatía diabética. Tampoco estaría indicado
en pacientes con riesgos cardiovasculares (en este caso mujer fumadora e hipertensa) por el
riesgo que asocia.(R2)

365. Indique en cuál de los ejemplos que se citan a continuación el riesgo de resistencia
cruzada entre los antibióticos que se mencionan y, por tanto, de fracaso del tratamiento
es mayor:

Si ante el fracaso del tratamiento con amoxicilina se utiliza posteriormente la


1. 1.
combinación amoxicilina/ácido clavulánico.
Si ante el fracaso del tratamiento con claritromicina se utiliza posteriormente
2. 2.
clindamicina.
3. 3. Si ante el fracaso del tratamiento con cloxacilina se utiliza posteriormente vancomicina.
4. 4. Si ante el fracaso del tratamiento con clindamicina se utiliza posteriormente daptomicina.
Gráfico de respuestas
Comentario

 
 
 
 
Nos están preguntando cuál de los siguientes antibióticos no deben darse conjuntamente. Dado
que macrólidos (claritromicina) y lincosánidos (clindamicina) son dos grupos de antibióticos de
estructura química diferente, pero con muchas similitudes en su acción (ambos producen una
enzima que metila el ARN ribosómico), es lógico que también compartan mecanismos de
resistencia (resistencia cruzada) (respuesta 2 correcta).

En la respuesta 1, el ácido clavulánico es inhibidor enzimático de las β-lactamasas, que impide la


destrucción de la amoxicilina. Las respuestas 3 y 4 son antibióticos con diferente mecanismo de
acción, no produciendo en este caso resistencia cruzada.(R2)

366. ¿Cuál de los siguientes elementos anatómicos NO es necesario para proporcionar


la estabilidad al tobillo?

1. 1. Integridad ósea del maléolo tibial y peroneal.


2. 2. Músculo sóleo y tibial anterior.
3. 3. Ligamento deltoideo.
4. 4. Ligamento peroneoastragalino anterior y posterior.
Gráfico de respuestas
Comentario

Los músculos sóleo y tibial anterior sirven para MOVER la articulación del tobillo (flexión plantar y
flexión dorsal, respectivamente), pero no para darle estabilidad. Este aspecto depende de las
estructuras osteoligamentosas que se citan en el resto de las opciones.(R2)

367. Señale en qué supuesto hay que estudiar siempre la causa de la ictericia:

1. 1. La ictericia que aparece a las 48 horas de vida.


2. 2. La bilirrubina alcanza una cifra superior a 10 mg/dL.
3. 3. La bilirrubina directa supera el 20% de la total en cualquier momento.
4. 4. En el prematuro alcanza un valor superior a 12 mg/dL al 7º día de vida.
Gráfico de respuestas
Comentario

Esta pregunta puede parecer tediosa y compleja, pero no lo es tanto si tiene las nociones claras.
Ha de saber que las hiperbilirrubinemias pueden ser de dos tipos en función de la cifra de
bilirrubina directa: colestáticas (si BID excede el 20%) y no colestáticas (si BID menor del 20%). La
opción 3 designa una ictericia colestática, tributaria por tanto de estudio. El resto de las opciones
forma parte del espectro de la ictericia fisiológica.(R3)

368. Varón de 35 años, enolismo moderado, consulta por hiperpigmentación ocular, así
como hipertricosis malar, ampollas y erosiones en dorso de manos a mínimos
traumatismos, sensaciones punzantes de la piel con la exposición solar de reciente
aparición. ¿Qué orientación diagnóstica le sugiere?

1. 1. Porfiria aguda intermitente.


2. 2. Porfiria cutánea tarda.
3. 3. Cloasma.
4. 4. Toxicodermia.
Gráfico de respuestas
Comentario

 
 
 
 
Pregunta muy importante, ya que ha caído en varias convocatorias.

En el MIR, debes ser capaz de reconocer la porfiria cutánea tarda, porque es una enfermedad muy
frecuente. Esta enfermedad se hereda de forma AD o bien es adquirida esporádicamente (la
mayoría de los casos). Se debe a déficit de URO- descarboxilasa, y la presencia de uroporfirina y
coproporfirina en sangre, orina y heces confirma el diagnóstico.

Las manifestaciones comienzan a los 30- 40 años, y es más frecuente en varones. Hay
hiperfragilidad cutánea en el dorso de las manos más hiperpigmentación facial e hipertricosis
malar, así como una marcada fotosensibilidad.

El tratamiento es eliminar el alcohol, flebotomías periódicas o cloroquina.

Recuerda que la forma más grave de porfiria es la llamada porfiria eritropoyética congénita o
enfermedad de Gunther, cuyas manifestaciones son mucho más precoces e intensas, como se
muestra en la fotografía que vemos a continuación.(R2)

 
 
 
 
369. Habitualmente la fecundación tiene lugar en:

1. 1. Superficie del ovario.


2. 2. Porción ampular de la trompa.
3. 3. Porción ístmica de la trompa.
4. 4. Porción intersticial de la trompa.
Gráfico de respuestas
Comentario

Pregunta fácil sobre la fisiología de la fecundación. El óvulo se fecunda en la trompa, con lo que
descartamos la opción 1. Las demás opciones nos indican distintas partes de la trompa. La
correcta es la opción 2, ya que es en la zona ampular donde se produce la fecundación, y tardará 7
días en recorrer su camino hasta implantarse en el endometrio. Recuerde que éste es el
fundamento por el cual la gran mayoría de los embarazos ectópicos ocurren en la zona ampular de
la trompa.(R2)

370. La edad recomendada para la retirada de la lactancia materna es:

1. 1. Entre los 15 días y los 2 meses.


2. 2. A los 6 meses.
3. 3. A los 8 meses.
4. 4. A los 4 meses.
Gráfico de respuestas
Comentario

La edad recomendada a la cual se deja de administrar la lactancia materna sería a los 6 meses. Es
una pregunta que no pude fallar.(R2)

371. ¿En qué infección vulvovaginal está indicado el tratamiento de la pareja


sistemáticamente?

1. 1. Candida albicans.
2. 2. Candida glabrata.
3. 3. Virus del papiloma humano.
4. 4. Trichomona vaginalis.
Gráfico de respuestas
Comentario
En las candidiasis sólo se hace tratamiento de pareja si ésta tiene síntomas o en casos de recidiva,
al igual que para la Gardnerella. En el HPV se hace tratamiento médico o quirúrgico si aparecen
condilomas mientras que la infección por Trichomona se considera una ETS y sistemáticamente
tratamos a la pareja.(R4)

372. Un paciente de 25 años acude al hospital con un síndrome febril, alteraciones de la


conducta, confusión, desorientación y alucinaciones. Poco después tiene varios
episodios convulsivos, tras lo cual entra en coma. En la exploración se aprecia rigidez de
nuca y hemiparesia izquierda. El estudio del LCR se caracterizó por tener un aspecto
ligeramente hemorrágico, 450 leucocitos/mm3, glucosa 60 mg/dl y proteínas 560 mg/dl.
El EEG mostró actividad difusa de ondas lentas y en la TAC se evidenció la presencia de
lesiones hemorrágicas con edema cerebral a nivel del lóbulo temporal. ¿Cuál sería su
tratamiento de elección?

 
 
 
 
1. 1. Ribavirina.
2. 2. Aciclovir.
3. 3. Ganciclovir.
4. 4. Zidovudina.
Gráfico de respuestas
Comentario

La encefalitis por VHS ha sido preguntada en múltiples ocasiones. Las ideas más importantes al
respecto son:

- Es la causa más frecuente de encefalitis vírica.

- Si se acompaña de focalidad neurológica, es de predominio temporal.

- El diagnóstico es inicialmente clínico. Actualmente, la confirmación se realiza mediante la


demostración del ADN del virus en LCR. En el pasado, era precisa una biopsia cerebral.

- Dada la lentitud de las pruebas diagnósticas, está justificado el tratamiento empírico ante la
sospecha de esta entidad (aciclovir i.v.).(R2)

373. Un niño de 5 días de edad, nacido con 1,200 g de peso, muestra mal aspecto general,
gran distensión abdominal, retiene en el estómago las tomas administradas por sonda y
tiene algún vómito. En la radiografía simple hay distensión de asas abdominales, y se ve
aire en la pared del intestino, pero no en el peritoneo. ¿Qué diagnóstico le sugiere este
cuadro clínico?:

1. 1. Ileo meconial por fibrosis quística.


2. 2. Estenosis o atresia de duodeno.
3. 3. Enterocolitis necrotizante
4. 4. Atresia de ano.
Gráfico de respuestas
Comentario

El caso clínico es muy sugestivo de enterocolitis necrotizante. Es frecuente esta entidad en RNPT.
Además las radiografía muestra neumatosis intestinal (aire en la pared) lo que nos refuerza la
sospecha de que el neonato esté sufriendo una enterocolitis necrotizante.(R3)

374. A 63-year-old male comes to the ER due to sudden onset of severe retrosternal and
upper abdominal pain. He has been vomiting for the past few hours after consuming
alcohol. He smokes 40 cigarettes a day, and has a history of alcoholic pancreatitis. His
temperature is 37.8ºC, BP is 127/75, pulse is 110/min, and respirations are 30/min.
Physical examination reveals palpable crepitus in the suprasternal notch. What is the
most likely diagnosis?

1. 1. Acute pancreatitis.
2. 2. Boerhaave's syndrome.
3. 3. Acute gastroenteritis.
4. 4. Lung perforation.
Gráfico de respuestas
Comentario

 
 
 
 
Boerhaave's syndrome. Esophageal rupture is a rupture of the esophageal wall. Iatrogenic causes
account for approximately 50-55% of esophageal perforations, usually due to medical
instrumentation such as endoscopy or paraesophageal surgery. In contrast, the term Boerhaave's
syndrome is reserved for the 10% of esophageal perforations which occur due to vomiting. The
classic history of esophageal rupture is one of severe retching and vomiting followed by
excruciating retrosternal chest and upper abdominal pain. Odynophagia, tachypnea, dyspnea,
cyanosis, fever, and shock develop rapidly thereafter. Subcutaneous emphysema (crepitation) is an
important diagnostic finding but is not very sensitive, and a pleural effusion may be detected.(R2)

375. ¿Cuál de las siguientes afirmaciones es FALSA, en relación con la meningitis


neonatal?:

1. 1. El periodo neonatal es la época de la vida con mayor incidencia de meningitis.


2. 2. La meningitis ocurre al menos en el 30% de la sepsis neonatales.
3. 3. Las meningitis causadas por Haemophilus son relativamente frecuentes en el neonato.
4. 4. Tienen peor pronóstico las meningitis por microorganismos Gram negativos.
Gráfico de respuestas
Comentario

Entre los microorganismos más frecuentes como causales de la meningitis neonatal se encuentran
los Gram negativos o Listeria entre otros, pero no el Haemophilus.(R3)

376. Recién nacido a término (38+5) de peso


adecuado para la edad gestacional. Presenta al nacimiento dificultad respiratoria severa
con cianosis y baja saturación periférica de oxígeno. A la exploración destaca abdomen
excavado y latido cardíaco rítmico sin soplos en hemitórax derecho. Tras reanimación
neonatal con intubación orotraqueal y ventilación mecánica, el paciente es trasladado a
la unidad de cuidados intensivos neonatales donde se obtiene la siguiente radiografía.
Señale la afirmación CORRECTA respecto a lo observado en la prueba de imagen:

1. 1. La presencia de la dextrocardia evidenciada se suele asociar con anomalías esplénicas.


2. 2. Lo más probable es que se trate de una malformación adenomatoidea quística.
3. 3. El paciente no está intubado.
4. 4. Se evidencia una marcada hipoplasia pulmonar.
Gráfico de respuestas
Comentario

 
 
 
 
Se comenta en la pregunta siguiente.(R4)

377. Respecto a esta patología, señala la opción CORRECTA:

1. 1. En una mayoría de los casos hay participación hepática y esto empeora el pronóstico.
El tratamiento es una emergencia quirúrgica, debiendo ser intervenido en las primeras 24
2. 2.
horas de vida.
La localización más frecuente es un defecto a nivel de los pilares diafragmáticos
3. 3.
posteriores, sobretodo en el lado derecho.
4. 4. En su manejo el uso del óxido nítrico inhalado no tiene ningún papel.
Gráfico de respuestas
Comentario

Hay distintos tipos de hernias (defectos) a nivel diafragmático.

La hernia de Morgagni es de localización anteromedial (retroesternal), más frecuente en el lado


derecho, generalmente asintomática al nacimiento se suele diagnosticar como un hallazgo
incidental en una radiografía de tórax realizada por otro motivo.

En la hernia de hiato el contenido abdominal asciende a su través.

En esta pregunta hacen referencia a la hernia diafragmática congénita o de Bochdaleck. Se


produce por un defecto a nivel posterior, generalmente izquierdo (85%), aunque puede ser incluso
bilateral (<5%). La malrotación intestinal y la hipoplasia pulmonar forman parte de la malformación,
siendo esta última la causa de muerte en los peores casos. En muchos de ellos a la dificultad para
una adecuada función de intercambio gaseoso se añade una hipertensión pulmonar. La
hipertensión pulmonar se puede tratar con óxido nítrico inhalado al ser este un potente
vasodilatador pulmonar.

El cuadro clínico es el de un recién nacido con dificultad respiratoria que aparece en las primeras
48 horas de vida, frecuentemente con cianosis. El abdomen aparecerá excavado y es posible
escuchar ruidos hidroaereos, correspondientes a borborigmos intestinales. En el caso más
corriente (herniación por el lado izquierdo) el corazón se ve desplazado a la derecha. La
auscultación sugerirá dextrocardia aunque no es una dextrocardia real, las cuales efectivamente se
asocian a anomalías a nivel del bazo (asplenia, poliesplenia). Con mucha frecuencia se hernian
también vísceras sólidas, sobretodo el hígado, lo cual empeora el pronóstico. El tratamiento inicial
consiste en garantizar una adecuada ventilación, oxigenación y circulación, siendo la cirugía un
paso posterior (como mínimo se suele retrasar 48 horas).(R1)

378. La causa más frecuente de incompatibilidad sanguínea en el recién nacido es:

1. 1. Rh.
2. 2. ABO.
3. 3. Kell.
4. 4. Duffy.
Gráfico de respuestas
Comentario

La causa más frecuente de incompatibilidad sanguínea en el RN es la debida al sistema ABO,


siendo menos grave que la asociada al sistema Rh. Respuesta correcta 2.(R2)

 
 
 
 
379. Todas EXCEPTO una de las siguientes pueden causar un síndrome de ojo seco:

1. 1. Síndrome de Sjögren secundario.


2. 2. Parálisis facial.
3. 3. Obstrucción congénita de la vía lagrimal.
4. 4. Hipovitaminosis A.
Gráfico de respuestas
Comentario
El ojo seco puede obedecer a muy diversas causas, pudiendo fallar la producción de cualquiera de
los componentes de la lágrima. Efectivamente la inflamación de la glándula lagrimal compromete
su función y por eso una dacrioadenitis crónica puede producir un cuadro de ojo seco. Está
inflamación puede además tener carácter autoinmune como sucede en el S de Sjögren. En el caso
del déficit de vitamina A, lo que falla es la producción de mucina, y por ello la lagrima, aún siendo
cuantitativamente normal no permanece suficiente tiempo en contacto con la superficie ocular. Por
eso en este caso el test de Schirmer no tiene porque estar necesariamente alterado, pero el tiempo
de ruptura de la película lagrimal estará disminuido. En el caso e la parálisis facial se produce un
ojp seco por un mecanismo distinto, en este caso el ojo seco es de carácter evaporativo. Por eso
los pacientes afectos de una parálisis facial deben usar lágrimas artificiales y lubricantes, y en caso
de no producirse recuperación espontánea es aconsejable la realización de algún procedimiento
quirúrgico que estreche la hendidura palpebral y reduzca la superficie evaporativa. Sin embargo la
obstrucción de la vía lagrimal no produce ojo seco, sino por el contrario un cuadro clínico
protagonizado por la epífora, pues al no poder ser conducida la lagrima hacia las fosas nasales
rebosa por el párpado.(R2)

380. El cuadro de hemorraga uterina disfuncional se asocia frecuentemente con:

1. 1. Hiperplasia endometrial.
2. 2. Hiperplasia secretora.
3. 3. Endometrio atrófico.
4. 4. Endometrio normal.
Gráfico de respuestas
Comentario

La respuesta correcta es la 1 hiperplasia endometrial, ya que se la HUD se puede presentar con


dicho dato.(R1)

381. El tratamiento de elección para NIC III es:

1. 1. Histerectomía total.
2. 2. Radioterapia.
3. 3. Conización.
4. 4. Conducta expectante con control cada 3-6 meses.
Gráfico de respuestas
Comentario

Tema primordial en el ENARM. El tratamiento de las neoplasias intraepiteliales de cérvix varia


según su grado:

NIC I: podemos optar por la observación en el caso que se trate de pacientes jóvenes que
sabemos que vamos a poder seguir con controles periódicos. Si no es así, procederemos a
tratamiento con crioterapia, láser o electrocoagulación.

 
 
 
 
NIC II o III: Conización.

Carcinoma: Histerectomía.(R3)

382. Paciente de 2 meses de edad acude por presentar convulsiones tónico clónicas
generalizadas de 2 minutos de duración, durante la evaluación no se determina fiebre.
Los exámenes de laboratorio y la punción lumbar son normales, los estudios de
imágenes no muestran mayor alteración. Durante la hospitalización en los días siguientes
se presentaron convulsiones de corta duración. El diagnóstico probable será?

1. 1. Convulsión neonatal.
2. 2. Epilepsia.
3. 3. Convulsión asociada a fiebre.
4. 4. Convulsión febril compleja.
Gráfico de respuestas
Comentario

Si le dicen que el paciente no ha presentado fiebre nunca, debe de descartar inmediatamente la


opción 3 y la 4; y si le dicen que tiene 2 meses de edad, por lo tanto es un lactante menor, deberá
eliminar también la opción 1, por lo que la opción correcta es la número 2.(R2)

383. Es característico que en la curva flujo-volumen se observe mucho más afectada la


porción inspiratoria que la espiratoria en:

1. 1. La obstrucción intratorácica variable.


2. 2. La obstrucción fija de las vías aéreas superiores.
3. 3. Las neoplasias endobronquiales.
4. 4. La obstrucción extratorácica variable de las vías aéreas superiores.
Gráfico de respuestas
Comentario

La afectación preferente de la inspiración sugiere fuertemente una causa extratorácica, como


ocurre, por ejemplo, en la traqueomalacia (respuesta 5 correcta).

Si se tratase de una obstrucción fija, sea de vías superiores o de inferiores (neoplasia


endobronquial), se afectarían por igual la inspiración y la espiración. El asma bronquial y las
obstrucciones intratorácicas variables afectan más a la espiración.(R4)

384. Un neonato de 21 días presenta ictericia, hepatomegalia y acolia. En los exámenes


de laboratorio destaca una cifra de bilirrubina total de 19 mg/dl con un 50% de bilirrubina
directa, así como GGT de 500 UI/L, con el resto normal. La gammagrafía hepática muestra
captación normal, pero no hay excreción biliar. ¿Qué diagnóstico se corresponde con
este cuadro?

1. 1. Hepatitis neonatal idiopática.


2. 2. Atresia biliar extrahepática.
3. 3. Enfermedad de Byler.
4. 4. Déficit de alfa-1-antitripsina.
Gráfico de respuestas
Comentario

 
 
 
 
Dada la captación normal del contraste por parte del hígado, sin que éste pase a la vía biliar,
debemos considerar una posible atresia biliar extrahepática. Recuerde que esta enfermedad es la
causa más frecuente de trasplante hepático en la infancia.(R2)

385. Mujer de 32 años con historia de tres partos eutócicos, refiere perdida de orina al
toser. Al examen cistouretrocele de grado I, el tratamiento recomendado es:

1. 1. Observación.
2. 2. Ejercicios de Kegel.
3. 3. Colporrafia anterior.
4. 4. Uretropexia.
Gráfico de respuestas
Comentario

Al tratarse de incontinencia de esfuerzo leve, se sugiere como tratamiento ejercicios para


rehabilitar el suelo pélvico... en este caso la respuesta 2 los ejercicios de Kegel.

Para grados más avanzados, el manejo suele ser quirúrgico, mediante la colocación de bandas
suburetrales libres de tensión.(R2)

386. Un neonato prematuro recibe teofilina oral por presentar pausas de apnea. ¿Cuál de
los siguientes antibióticos puede provocar disminución de su metabolismo e
intoxicación?:

1. 1. Ampicilina.
2. 2. Anfotericina.
3. 3. Eritromicina.
4. 4. Vancomicina.
Gráfico de respuestas
Comentario
La verdad es que, hoy por hoy, la cafeína ha desplazado de una forma prácticamente total a la
teofilina en el tratamiento de las pausas de apnea. La razón es que la teofilina tiene un difícil
manejo, hay que tener cautela con las posibles interacciones que genera (por ejemplo, con la
eritromicina), y hay que hacer periódicamente evaluaciones de su nivel en sangre.(R3)

387. Niña de 13 años que acude a su centro de salud por aparición brusca de fiebre alta
y odinofagia intensa. En la exploración presenta aceptable estado general, la
auscultación cardiopulmonar es normal, el abdomen es blando y depresible, presenta
algunas adenopatías laterocervicales bilaterales y en el examen de la orofaringe se
aprecia una faringe eritematosa, con amígdalas hipertróficas con placas y exudados
bilaterales y lengua saburral. En la piel se observa tenue exantema con micropápulas
muy levemente eritematosas que blanquean a la presión. Teniendo en cuenta su
sospecha diagnóstica, señale cuál de las siguientes afirmaciones es INCORRECTA:

1. 1. La infección está producida por el Streptococo pneumoniae.


Es útil la recogida de exudado faríngeo para cultivo y detección rápida de antígenos
2. 2.
estreptocócicos.
3. 3. El exantema suele resolverse en 3-4 días con descamación foliácea.
4. 4. Si el exantema afecta a la cara, suele respetar el triángulo nasolabial.
Gráfico de respuestas
Comentario
 
 
 
 
Estamos ante un caso de escarlatina: faringoamigdalitis con exantema micropapular que se palpa
mejor que se ve y desaparece por descamación en pocos días.

La lengua se recubre los primeros días de una capa blanquecina (lengua en fresa blanca o
saburral) y posteriormente se descama y aparece hiperémica (lengua en fresa roja).

Si se afecta la cara, típicamente respeta el triángulo nasolabial (facies de Filatov). Se trata de una
infección debida a Streptococcus pyogenes.

Para confirmar el diagnóstico es útil la detección rápida de antígenos o cultivo, y el tratamiento de


elección es penicilina oral durante 10 días.(R1)

388. A 55-year-old woman is diagnosed with pancreatic head cancer. A complete


preoperative evaluation is performed, confirming that the tumor is resectable. Surgical
intervention is scheduled. Intraoperative findings are as follows: pancreatic head tumor
measuring 4 cm in diameter, portal vein not invaded, no invasion of adjacent organs,
reactive adenitis seen in the intraoperative lymph node biopsy. Which of the following is
the most appropriate surgical technique?

1. 1. Total pancreatectomy.
2. 2. Pancreaticoduodenectomy (Whipple procedure).
3. 3. Palliative bypass: cholecystojejunostomy.
4. 4. Palliative bypass: gastrojejunostomy and cholecystojejunostomy.
Gráfico de respuestas
Comentario

El tratamiento curativo de esta enfermedad es la intervención de Whipple o


duodenopancreatectomía cefálica, que es la técnica quirúrgica de elección para el cáncer de
cabeza de páncreas.

En la técnica de Whipple, se reseca el antro gástrico, todo el marco duodenal y unos centímetros
del yeyuno proximal en bloque, junto con la cabeza del páncreas, que se secciona a nivel del
llamado cuello pancreático sobre la porta y que incluye la vía biliar principal distal, es decir el
colédoco distal. Como gesto añadido, se debe resecar la vesícula biliar, lo cual es obligado
siempre que se practica una intervención en la vía biliar principal. Así pues, debemos reconstruir
haciendo tres anastomosis:

1) Entre el conducto de Wirsung y el tubo digestivo.

2) Entre la vía biliar y el tubo digestivo.

3) Entre el estómago y el tubo digestivo.

(R2)

 
 
 
 

389. Acude a Urgencias un hombre de 71 años


refiriendo la presencia de sangre en heces desde hace 48 horas. Se realiza una
colonoscopía hasta ciego en la que se observa (ver imagen). ¿Cuál de las siguientes
afirmaciones es CORRECTO?

1. 1. La edad del paciente no es la propia para desarrollar esta lesión.


2. 2. El tratamiento debe ser quirúrgico, incluyendo la resección de la zona enferma.
El tratamiento con estrógenos y progrestágenos resuelve la patología en la mayoría de los
3. 3.
pacientes.
4. 4. El tratamiento endoscópico de elección es la fulguración con argón.
Gráfico de respuestas
Comentario

La causa más frecuente de hemorragia digestiva baja en un anciano es el sangrado diverticular,


seguida de la angiodisplasia de colon. En este caso, el diagnóstico correcto sería la segunda,
teniendo en cuenta la imagen que nos aportan como dato (observe los vasos angiodisplásicos en
los márgenes de la lesión central). El tratamiento puede efectuarse por vía endoscópica,
fulgurando la zona con argón.(R4)

390. Paciente de 32 años con test de embarazo positivo y amenorrea de 9 semanas.


Consulta por hiperemesis gravídica de 6 días de evolución y metrorragia escasa de 3
horas de evolución. En la exploración se comprueba sangrado escaso de cavidad uterina
y útero aumentado como gestación de 14 semanas. En el ultrasonido presenta imagen
intrauterina en “copos de nieve” y ausencia de saco gestacional intraútero. La
determinación de β-HCG es de 110,000 mUI/ml. Tras evacuación por succión y curetaje
se diagnostica de mola hidatiforme completa. En el seguimiento posterior, cuál de las
siguientes es CIERTA:

1. 1. Tratamiento con metotrexate y ácido folínico semanales en cualquier caso.


Determinación semanal de β-HCG durante un mes tras la normalidad de la misma.
2. 2.
Después determinación mensual durante 1 año.
3. 3. Realización de radiografía simple de tórax cada 3 meses durante 1 año.
4. 4. Determinación trimestral de β-HCG, tras la normalidad de la misma, durante 1 año.
Gráfico de respuestas
Comentario

En este caso, se trata de una pregunta de dificultad media sobre un tema prioritario de Obstetricia:
las hemorragias del primer trimestre. Lo más importante es saber hacer el diagnóstico diferencial
entre las tres causas siguientes: aborto, embarazo ectópico y mola hidatiforme.

No obstante, en esta pregunta también es necesario conocer el algoritmo diagnóstico-terapéutico.

 
 
 
 
Ante una mola hidatiforme, lo primero es realizar un legrado por succión-aspiración, o una
histerectomía en mayores de 40 años sin deseo de tener hijos. Después, hay que realizar un
seguimiento mediante determinación semanal de b-hCG.

Tras varias determinaciones normales (un mes), se pasa a realizarlas mensualmente durante un
año para poder establecer la curación (respuesta 2 correcta).

Durante este tiempo, la paciente debe tomar medidas anticonceptivas para evitar que un posible
embarazo falsee los resultados del análisis de b-hCG.

Sobre el resto de opciones, el estudio de extensión y el tratamiento con metrotexato (o


poliquimioterapia) se realizan si la b-hCG está aumentada durante más de 8 semanas. Además:

•   Respuesta 1: "en cualquier caso" ya debería hacernos pensar que es falsa. El metotrexato
se usa cuando es una mola localizada persistente, o bien una mola diseminada sin factores
de mal pronóstico (poliQT si mal pronóstico).

•   Respuesta 3: TC o Rx cada tres meses durante un año debería hacernos pensar que es
excesivo, con un exceso de irradiación para la paciente, cuando disponemos de la b-hCG.

(R2)

Tratamiento de la enfermedad trofoblástica

 
 
 
 

391. Niño de 10 años de edad que presenta desde hace 2 días tumoración dolorosa en el
canto externo del párpado superior. ¿Qué cuadro sistémico estará padeciendo con más
probabilidad?:

1. 1. Púrpura de Schönlein-Henoch.
2. 2. Artritis crónica juvenil.

 
 
 
 
3. 3. Sarampión.
4. 4. Enfermedad celiaca.
Gráfico de respuestas
Comentario
Pregunta difícil. No te preocupes si la has fallado. Esta es la forma de presentación de una
dacrioadenitis aguda, es decir, inflamación de la glándula lagrimal principal. Te debe venir a la
mente este diagnóstico ante toda tumoración dolorosa localizada en el canto externo del párpado
superior, pues es la localización anatómica de la glándula. Generalmente se acompaña de una
adenopatía preauricular. Dentro de las posibles etiologías de este proceso encontramos causas
sistémicas, como parotiditis, sarampión (opción 3) o gripe y causas locales como la erisipela.(R3)

392. Posteriormente, tras el


posible diagnóstico, se convence al paciente para ingreso y realización de endoscopia y
ecoendoscopia (ver imagen). ¿Qué sospecharía en este enfermo y qué técnica se
indicaría después?

Carcinoma epidermoide con metástasis. Ya que la ecoendoscopia me permite ver la


1. 1. infiltración del tumor y adenopatías, solicitaría un TAC para descartar metástasis a
distancia.
2. 2. Realizaría un PET-TAC de inicio.
3. 3. Seguramente sea una recidiva del divertículo de Zenker.
4. 4. Adenocarcinoma esófago. Realizaría un TAC de extensión.
Gráfico de respuestas
Comentario

Siendo coherentes con lo que hemos dicho de la pregunta anterior, la sospecha diagnóstica sería
la misma: carcinoma epidermoide con metástasis (de ahí el dolor óseo). Para realizar un estadiaje
correcto, debemos realizar una TC, que pondrá de manifiesto las adenopatías tumorales y la
afectación a distancia, en caso de existir.

Observe que, mediante ecoendoscopía, podemos intuir la infiltración de la pared esofágica. No


importa si no es capaz de interpretar correctamente la mitad derecha de la imagen. Esta técnica
nos permite valorar el grado de infiltración (aunque, lógicamente, no de forma tan precisa como el
estudio anatomopatológico).(R1)

393. Tras un accidente de tráfico, el conductor del turismo presenta un trauma facial con
epistaxis e imposibilidad para la apertura bucal completa, con desviación de la boca hacia
la izquierda al hacerlo. Respecto a la patología que sospecha, señale la respuesta
correcta:

1. 1. Es la fractura facial más frecuente.


2. 2. La radiografía de senos es la prueba de elección pasa su diagnóstico.

 
 
 
 
3. 3. Con la mordida, habrá un defecto de cierre del lado izquierdo.
4. 4. El TC puede ser necesario en caso de fractura complicada.
Gráfico de respuestas
Comentario
La fractura mandibular es la segunda más frecuente de las fracturas faciales. Cuando afecta al
cóndilo, al abrir la boca ésta se desvía hacia el lado de la lesión, y al cerrarla se produce una
maloclusión del contralateral. Para el diagnóstico la prueba de elección es la ortopantografía, y en
casos complicados es necesario un TC. En el tratamiento, suele ser suficiente una reducción
cerrada de la fractura e inmovilización.(R4)

394. En un servicio de Urgencias ingresa un hombre de 65 años que presenta dolor


abdominal cólico, estreñimiento y leve distensión abdominal. El tacto rectal muestra una
gran masa en recto. Indique cuál de las pruebas siguientes es más útil para indicar cirugía
de urgencias:

1. 1. Enema opaco.
2. 2. Radiografía simple de abdomen.
3. 3. Biopsia de la masa.
4. 4. Sigmoidoscopia.
Gráfico de respuestas
Comentario

Pregunta práctica en relación con el tema de la obstrucción intestinal, que es un tema que de cara
al ENARM no le debe plantear muchos problemas y del que debe saber su etiología más frecuente
y su cuadro clínico.

Recuerde que la causa más frecuente de obstrucción a nivel del intestino grueso es el cáncer
colorrectal (tacto rectal sugerente).

En cuanto al diagnóstico es fundamentalmente clínico y la prueba complementaria más útil para


confirmarlo es la radiografía simple de abdomen tomada en decúbito y en bipedestación. En ella se
observa la distensión gaseosa de las asas de intestino por encima de la obstrucción y en la placa
tomada en bipedestación se pueden ver los característicos niveles hidroaéreos dentro de las asas
intestinales distendidas. Este examen además de confirmar el diagnóstico, da información acerca
del nivel o altura de la obstrucción, si es intestino delgado o de colon. No suelen ser necesarias
otras pruebas de imagen para el diagnóstico de obstrucción intestinal.(R2)

395. Varón de 50 años, hipertenso, en tratamiento con enalapril desde hace 5 años.
Nefrectomía izquierda secundaria a litiasis. Acude a Urgencias refiriendo los siguientes
antecedentes: 2 semanas antes presentó cólico renal expulsivo sin precisar medicación.
Durante los días siguientes la diuresis osciló entre 4 litros durante varios días y
posteriormente, 1 litro. 24 horas antes de la consulta: oliguria severa. A la exploración se
encuentra normohidratado, sin globo vesical ni otros hallazgos llamativos. En el
sedimento de orina existe microhematuria y leucocitos aislados. Proteinuria negativa. El
diagnóstico más probable es:

1. 1. Insuficiencia renal secundaria a nefroangioesclerosis.


2. 2. Proceso renal obstructivo.
3. 3. Insuficiencia renal secundaria a hiperfiltración.
4. 4. Insuficiencia renal secundaria a inhibidores de la enzima de conversión de angiotensina.
Gráfico de respuestas

 
 
 
 
Comentario

Datos que nos hacen sospechar FRA postrenal en este paciente:

- Riñón único

- Antecedentes de litiasis, y cólico renal hace 2 semanas

Oliguria progresiva(R2)

396. La fiebre botonosa mediterránea se manifiesta clínicamente por:

1. 1. Meningitis.
2. 2. Hemorragia digestiva.
3. 3. Exantema maculopapular.
4. 4. Enfermedad crónica febril.
Gráfico de respuestas
Comentario

Pregunta muy fácil sobre una de las dos ricketsiosis más preguntadas: la fiebre botonosa, la otra
es la fiebre Q. La clínica de la fiebre botonosa responde a un cuadro clínico común para muchas
rickettsiosis: fiebre, malestar general, mialgias, cefalea intensa, exantema con compromiso de
palmas y plantas. A esto se agrega el dato típico de la fiebre botonosa, que es sin duda la "mancha
negra", una escara negruzca que aparece en el lugar de inoculación. Es importante que recuerde
que si bien la mayoría de las rickettsiosis cursan con un EXANTEMA, existen excepciones como la
Fiebre Q, que no lo presenta. La opción correcta por tanto es la 3, ya que ninguna de las otras
manifestaciones son típicas ni habituales en esta enfermedad.(R3)

397. Femenino de 40 años de edad, con una tumoración mamaria derecha, poco móvil,
localizada en el cuadrante superoexterno. ¿Cuál de las siguientes pruebas confirma el
diagnóstico?

1. 1. Marcadores tumorales.
2. 2. Ultrasonido mamario 2D.
3. 3. Biopsia del tumor.
4. 4. Ultrasonido Doppler 3D.
Gráfico de respuestas
Comentario

Detecte lo que le están preguntando, ¡¡¡CONFIRME EL TUMOR!!! Debe tener mucho cuidado al
resolver el ENARM, ya que una lectura rápida hará que responda erroneamente.

Obviamente el diagnóstico de neoplasia es histológico, por lo que la única opción que permitiría
tener una muestra de tejido es la biopsia.(R3)

398. A 30-year-old male patient, who received a lung transplantation 3 months ago, comes
to the emergency department with fever for the last week that is not relieved with
acetaminophen. His past medical history is significant for cystic fibrosis with several
admissions to hospital due to severe infections. Blood pressure is 100/60 mmHg, pulse
is 80/min and temperature is 38ºC. Physical examination is unremarkable. Blood cell

 
 
 
 
count shows 1500 leukocytes with abnormal liver profile. What diagnostic test would help
to confirm your suspected diagnosis?

1. 1. PPD (Mantoux)
2. 2. X-ray
3. 3. VZV serology
4. 4. Cytomegalovirus serology
Gráfico de respuestas
(R4)

399. ¿Cuál de las siguientes moléculas HLA es de clase II?

1. 1. A11.
2. 2. B7.
3. 3. Cw3.
4. 4. DR2.
Gráfico de respuestas
Comentario

Pregunta fácil acerca de las moléculas de histocompatibilidad. Son glicoproteínas de membrana


compuestas de dos cadenas que se clasifican en CPH de clase I, que se encuentran en la
membrana de, prácticamente, todas las células nucleadas, y plaquetas; están compuestas por una
cadena pesada (alfa) y una cadena ligera constante, la beta2-microglobulina. No expresan CPH de
clase I: hematíes, sincitiotrofoblasto y algunos escasos timocitos. Las mejor conocidas son HLA-A,
HLA-B y HLA-C. Las moléculas de CPH de clase II presentes en la superficie de las células
presentadoras de antígenos, las del sistema mononuclear fagocítico y los linfocitos B, están
formados por una cadena alfa y otra beta. Las tres moléculas de CPH de clase II más conocidas
son HLA-DR, HLA-DP y HLA-DQ. Fíjate como regla que las HLA tipo I son una letra y las de tipo II
son dos letras.(R4)

Características de los distintos tipos de molécula HLA

400. Un paciente de 27 años, con infección por VIH e inmunodepresión avanzada, refiere
pérdida de visión bilateral, mayor en ojo izquierdo. El diagnóstico más probable a
descartar es:

 
 
 
 
1. 1. Coroiditis toxoplásmica.
2. 2. Retinopatía por VIH.
3. 3. Retinitis por CMV.
4. 4. Degeneración macular por inmunodeficiencia.
Gráfico de respuestas
Comentario

El CMV produce clínica en fases avanzadas, cuando existen recuentos de menos de 50-75
linfocitos TCD4. Puede producir cuadros muy diversos, pero uno de los más típicos es la retinitis
por CMV, donde encontraríamos una imagen oftalmoscópica como la que aquí se describe, con
hemorragias y exudados amarillentos perivasculares (imagen en “queso y tomate” o, aún más
gráfico, imagen en “pizza margarita”, como dicen algunos autores).

Hay que tener especial cuidado con la respuesta 2. De hecho, la retinopatía VIH es más frecuente
que la retinitis por CMV, pero casi nunca es sintomática. Dado que se trata de un caso con pérdida
visual, sería mucho más probable la respuesta 3.

En cuanto a la coriorretinitis por Toxoplasma, debes saber que la toxoplasmosis es la causa más
frecuente de retinitis posterior en el paciente inmunocompetente. En cambio, en un paciente VIH,
es más frecuente por CMV.(R3)

401. Con relación al tratamiento del hipotiroidismo congénito, indique la respuesta


CORRECTA:

Han de normalizarse los niveles de tiroxina de manera progresiva para evitar efectos
1. 1.
secundarios.
2. 2. El retraso en su inicio puede condicionar una lesión cerebral definitiva.
3. 3. Ha de retirarse a los 3 años a todos los pacientes para descartar que sea transitorio.
4. 4. Requiere un seguimiento de laboratorio semestral.
Gráfico de respuestas
Comentario

Tema muy importante en el examen hoy en día, es poco frecuente diagnosticar clínicamente un
hipotiroidismo congénito debido a que, en su mayor parte, son diagnosticados con el tamiz
neonatal, que se realiza determinando los niveles de TSH obtenida entre los dos y los cinco días
de vida (cuyos valores estarían elevados).

Las manifestaciones del hipotiroidismo congénito tardan varias semanas en presentarse


plenamente, estando totalmente establecidas hacia el 3º-6º mes de vida. Aparece una facies
peculiar (cara tosca, párpados y labios tumefactos, nariz corta con base deprimida, hipertelorismo,
boca semiabierta y macroglosia). Otras manifestaciones típicas son el retraso en la maduración
ósea, la hernia umbilical, ictericia prolongada, letargia y estreñimiento.

El principal problema del hipotiroidismo congénito es su repercusión sobre el desarrollo intelectual.


Éste sería más grave cuanto más se demore el diagnóstico (respuesta 2 correcta), por lo que es
importante instaurar el tratamiento cuanto antes. Aunque la causa más frecuente de hipotiroidismo
congénito es la disgenesia tiroidea, independientemente de cuál sea la causa exacta, ante
cualquier caso de hipotiroidismo debemos apresurarnos a establecer el tratamiento sustitutivo lo
antes posible.(R2)

 
 
 
 

402.
Un paciente presenta el ECG que se muestra en la imagen nº 20. ¿En cuál de los
siguientes casos no está indicada la profilaxis antitrombótica?

1. 1. Mujer de 40 años con antecedentes de enfermedad tromboembólica.


2. 2. Varón de 76 años hipertenso y con antecedentes de ingresos hospitalarios por ICC.
3. 3. Mujer de 60 años con antecedentes de infarto de miocardio sin elevación del ST.
4. 4. Varón de 80 años con disfunción ventricular severa (FEV1 20%).
Gráfico de respuestas
Comentario

Pregunta importante para repasar las indicaciones de la anticoagulación oral crónica (ACO) en la
fibrilación auricular, que es el ritmo que nos muestra el ECG de la pregunta. Puede reconocerlo
porque n o existen ondas “p” y porque el ritmo es absolutamente irregular.

Los factores de riesgo de enfermedad embólica se dividen en alto y bajo grado:

- Los factores de alto riesgo embolígeno son: antecedentes de embolia o AIT, estenosis mitral y
prótesis valvulares.

- Los factores de riesgo moderado son: la HTA, diabetes mellitus, edad > de 75 años y FEV1
menor o igual al 35%.

Considerando los factores de riesgo anteriores, estaría indicada la ACO si coexisten dos o más
factores de riesgo moderados o si existe un factor de alto riesgo. En el caso de un solo factor de
riesgo moderado se podría optar entre antiagregar o anticoagular. Teniendo en cuenta lo anterior,
en la opción 4 no estaría indicada la profilaxis antitrombótica porque no reúne ningún factor de
riesgo de los anteriormente mencionados.(R3)

403. La placenta es el órgano de conexión fetomaterna. Tiene función de barrera, función


endocrina e intercambiadora de sustancias. Respecto a esta última, es FALSO que:

1. 1. El agua y los electrolitos utilizan la difusión simple según gradiente.


2. 2. Los aminoácidos necesitan de un transporte activo.
3. 3. Las moléculas grandes, como la IgM, pasan por pinocitosis.
4. 4. El hierro pasa por transporte activo.
Gráfico de respuestas

 
 
 
 
Comentario

Una pregunta fácil de responder.

La respuesta falsa es la 3, ya que:

- La inmunoglobulina que atraviesa la placenta es IgG, no IgM (su tamaño es enorme como para
poder pasar por la placenta) (respuesta 3 falsa).

El resto de las opciones son válidas y hacen referencia a algunos mecanismos de intercambio de
sustancias que debe conocer.(R3)

404. Juan es un paciente de 35 años que es revisado periódicamente en la consulta por


presentar un nódulo tiroideo inicialmente caliente pero que no suprimía al resto del
tiroides y de un tamaño de 3 cm de diámetro, con citología negativa para malignidad. En
un periodo de 2 años el nódulo ha crecido, teniendo actualmente un tamaño de 4.5 cm,
con una gammagrafía compatible con adenoma tóxico, ya que suprime al resto de tiroides
y presenta exámenes de laboratorio compatibles con hipertiroidismo. ¿Cuál sería su
actitud terapéutica?:

1. 1. I131.
2. 2. Betabloqueantes.
3. 3. Hemitiroidectomía que incluya al nódulo, previa normalización función tiroidea.
4. 4. Antitiroideos durante un año.
Gráfico de respuestas
Comentario

En esta pregunta nos presentan el caso clínico de un paciente con un nódulo solitario de 3 cms,
con PAAF benigna, que a lo largo de su evolución presenta un crecimiento de tamaño y además
desarrolla un cuadro de hipertiroidismo franco (adenoma tóxico).

El adenoma tóxico es una de las causas de hipertiroidismo primario, y se caracteriza por presentar
en la gammagrafía un nódulo hipercaptante con supresión del resto de la glándula. No existe un
tratamiento de elección de entrada en esta patología. Sin embargo ante un paciente joven que
desarrolla un adenoma tóxico, que además ha aumentado de tamaño, lo más indicado sería la
hemitiroidectomía con estudio histológico del nódulo. Recuerde que en pacientes con
hipertiroidismo, antes de recibir radioyodo o ser intervenidos, se debe normalizar lo más posible la
concentración de hormonas tiroideas. La administración de radioyodo puede emplearse también en
esta entidad, aunque se suele emplear más en pacientes de edad avanzada.(R3)

405. En el recién nacido, con líquido amniótico meconial o fétido y portador de amnionitis,
la primera acción a realizar, apenas desprendida la cabeza del perineo materno es:

1. 1. Ventilación a presión positiva.


2. 2. Secado enérgico.
3. 3. Aspiración orofaringea.
4. 4. Masaje cardiaco.
Gráfico de respuestas
Comentario

 
 
 
 
En el caso de aguas meconiales se debe evitar estimular al neonato para que llore, puesto que
aspiraría el meconio. Por lo tanto la primera medida sería aspirar el contenido orofaríngeo para
evitar la aspiración.(R3)

406. Paciente de 23 años diagnosticado de enfermedad celíaca y sometido a dieta sin


gluten. Tras ello recuperó peso pero siguió con dos a tres evaucaciones al día de heces
blandas. Las grasas en heces eran de 10 g/día. Los anticuerpos antiendomisio tipo IgA
persistían positivos tras dos años de tratamiento. La causa más probable de esta
insuficiente respuesta al tratamiento es:

1. 1. Intolerancia a la lactosa.
2. 2. Linfoma intestinal.
3. 3. Esprue refractario.
4. 4. Incumplimiento de la dieta sin gluten.
Gráfico de respuestas
Comentario

El tratamiento de la enfermedad celíaca radica en aportar los nutrientes que falten y en realizar una
dieta libre de gluten. Lo más normal es que, en cuestión de semanas, haya una clara mejoría, que
inicialmente es clínica y después histológica.

Cuando el paciente no responde al tratamiento, existen muchas posibilidades, pero la más


frecuente es que no esté cumpliendo la dieta adecuadamente. Para valorar el cumplimiento, se
monitorizan los anticuerpos IgA antitransglutaminasa, que negativizan tras unos meses siguiendo
una dieta sin gluten estricta. También puede deberse a un diagnóstico incorrecto, la existencia de
otra causa concurrente (déficit de lactasa, esprue colágeno, desarrollo de un linfoma…). No
obstante, lo primero es mucho más habitual (respuesta 4 correcta) y, además, es la situación que
mejor explica la persistencia de anticuerpos positivos.(R4)

407. ¿Cuál es el tratamiento de elección del absceso periamigdalino?

1. 1. Amoxicilina/clavulánico.
2. 2. Corticoesteroides.
3. 3. Clindamicina.
4. 4. Quirúrgico.
Gráfico de respuestas
Comentario

El absceso y el flemón periamigdalino son las complicaciones más frecuentes de las


faringoamigdalitis agudas. Son más comunes en adultos que en niños. Debes sospecharlos ante
un caso clínico donde te hablen de una amigdalitis con mala evolución, que no mejora pese al
tratamiento antibiótico, con abombamiento del pilar amigdalino u otros síntomas (trismus, otalgia
refleja, sialorrea...).

En caso de tratarse de un flemón periamigdalino, bastaría tratamiento antibiótico y esteroideo...


Pero en esta pregunta nos matizan que es un ABSCESO. Y el tratamiento de un absceso es ubi
pus, ibi evaqua, es decir, tratamiento quirúrgico (respuesta 4 correcta).(R4)

408. El ciclo anovulatorio es un ciclo:

1. 1. Monofásico.

 
 
 
 
2. 2. Difásico.
3. 3. Corto.
4. 4. Largo.
Gráfico de respuestas
Comentario

El ciclo femenino se divide en dos fases proliferativa y secretora (realmente hay otra tercera fase
que es la hemorrágica). El ciclo anovulatorio, al no existir el pico de LH que provoca la ovulación, el
folículo no se convierte en cuerpo lúteo, que produce progesterona. La progesterona es la que va a
producir la maduración del endometrio (fase secretora) que no existirá en un ciclo anovulatorio, y
produce modificaciones en todo el aparato genital que lo adecuan a la gestación.(R1)

409. En relación con el tratamiento quirúrgico de la estenosis mitral, ¿cuál de las


siguientes afirmaciones es FALSA?:

1. 1. La principal indicación quirúrgica de la estenosis mitral es la presencia de sintomatología.


No se recomienda la valvulotomía en los pacientes totalmente asintomáticos con
2. 2. independencia de los datos hemodinámicos a menos que se haya producido alguna
embolia.
La valvuloplastia percutánea con balón es una alternativa a la valvuloplastia en pacientes
3. 3.
con una anatomía valvular desfavorable.
La sustitución valvular será el procedimiento de elección cuando esté contraindicada la
4. 4. valvuloplastia, o si la anatomía es desfavorable al estar la válvula mitral muy
distorsionada.
Gráfico de respuestas
Comentario
El tratamiento inicial de la estenosis mitral sintomática está indicado el seguimiento periódico y la
profilaxis antibiótica de endocarditis infecciosa ante situaciones de riesgo de bacteriemia, asi como
diuréticos para aliviar la congestión, y si cae en FA algún fármaco que controle la frecuencia
(betabloqueantes, digoxina o verapamil o diltiazem) y anticoagulación oral crónica. Si los síntomas
persisten y/o si el paciente presenta fracaso ventricular derecho o hipertensión pulmonar entonces
estaría indicado el a bordaje mecánico de la válvula que puede ir desde valvuloplastia percutánea
con balón en el caso de válvulas favorables (conservadas sin gran calcificación ni insuficiencia
asociada, ni trombos en la aurícula) hasta sustitución valvular por una prótesis. En casos de
pacientes jóvenes (menores de 70 años o con esperanza de vida superior a 10) sin
contraindicaciones para anticoagulación elegiremos una prótesis metálica (por su mayor duración).
En caso de pacientes ancianos o con contraindicaciones para anticoagulación elegiremos prótesis
valvular biológica (que no precisa anticoagulación per se).(R3)

410. A patient presents with asthenia, orthostatic hypotension, cutaneous


hyperpigmentation and elevated ACTH levels. Which of the following is the most
appropriate diagnostic test?

1. 1. 2-mg Dexamethasone Suppression test (0.5 mg/6h during 2 days).


2. 2. ACTH stimulation test.
3. 3. 8-mg Dexamethasone Suppression test (2 mg/6h during 2 days).
4. 4. Methopirone test.
Gráfico de respuestas
Comentario
Esta es una pregunta directa con una respuesta relativamente sencilla. Un paciente que presenta
hipotensión ortostática, y sobre todo, con hiperpigmentación mucocutánea, nos debe orientar hacia
un déficit de glucocorticoides por una patología de la suprarrenal. El único diagnóstico posible es la

 
 
 
 
insuficiencia suprarrenal primaria (Enfermedad de Addison). El diagnóstico de elección se realiza
con el test de estimulación de ACTH. Con esta prueba se comprobará la ausencia de respuesta del
cortisol, confirmando el diagnóstico.(R2)

411. ¿Cuál de las siguientes etiologías de amenorrea presenta niveles elevados de


gonadotropinas?:

1. 1. Disgenesia gonadal.
2. 2. Craneofaringioma.
3. 3. Síndrome de Sheehan.
4. 4. Síndrome de Kallman.
Gráfico de respuestas
Comentario

En el estudio de amenorreas cuando se encuentran elevadas las gonadotropinas, el problema se


encuentra a nivel ovárico, por lo que la respuesta correcta es la número 1 o disgenesia
gonadal.(R1)

412. ¿Cómo son los niveles hormonales en el día 21 del ciclo genital femenino?:

1. 1. LH: baja, FSH: alta, progesterona: alta, estrógenos: altos.


2. 2. LH: baja, FSH: baja, progesterona: alta, estrógenos: altos.
3. 3. LH: alta, FSH: baja, progesterona: alta, estrógenos: bajos.
4. 4. LH: alta, FSH: alta, progesterona: alta, estrógenos: altos.
Gráfico de respuestas
Comentario

A partir de la ovulación (día 14) el folículo sufre una transformación gracias a la LH, y se convierte
en cuerpo lúteo, cuya misión es establecer las condiciones que favorezcan la gestación: para ello
produce progesterona. Parte de esta progesterona producida por el cuerpo lúteo se transforma en
andrógenos y en estrógenos, lo que motiva el pico de estrógenos y progesterona en la mitad de la
fase secretora del ciclo (día 21). La FSH y la LH tiene un pico en la ovulación día 14 pero en el día
21 ambas están bajas.(R2)

413. Paciente que sufre un episodio de pancreatitis aguda, pero que, tras varios días de
tratamiento médico, persiste con fiebre, dolor abdominal e íleo paralítico junto con
elevación discreta y sostenida de la cifra de amilasa sérica. Se realiza una USG abdominal
en la que se observa un pseudoquiste pancreático de 5 cm de diámetro. Señale la
respuesta INCORRECTA en cuanto a su tratamiento:

Si el estado general del paciente es satisfactorio, deberá observarse durante 4-6 semanas,
1. 1.
y si en este periodo el pseudoquiste ha crecido, se decidirá la intervención quirúrgica.
La extirpación del pseudoquiste es el método ideal de tratamiento definitivo del
2. 2. pseudoquiste, pero esto se reserva para los pseudoquistes localizados en la parte distal del
páncreas.
Si el paciente presenta fiebre mantenida de 38º C en el momento del diagnóstico, está
3. 3. indicada la cirugía urgente del pseudoquiste, ya que esto invariablemente implica que está
infectado.
En los casos de ascitis pancreática, el drenaje interno del pseudoquiste resulta un método
4. 4.
curativo.
Gráfico de respuestas

 
 
 
 
Comentario

Los pseudoquistes son formaciones de contenido líquido no recubiertas del epitelio de los
conductos pancreáticos, que tampoco tienen una cápsula propiamente dicha y que se producen
por el acúmulo de secreciones pancreáticas tras agresiones diversas en el seno del tejido
pancreático, en su mayoría por pancreatitis.

Esta pregunta es relativamente sencilla, aunque no sepas demasiado sobre el manejo del
pseudoquiste pancreático. La respuesta 4 es muy restrictiva al decir que la fiebre,
INVARIABLEMENTE, traduce una infección del pseudoquiste. En un paciente encamado
podríamos encontrar otras muchas razones por las que pueda aparecer fiebre: atelectasias,
infecciones urinarias, neumonías nosocomiales, fiebre medicamentosa…(R3)

414. Paciente varón, de 56 años que presenta pancitopenia, esplenomegalia, y en el que,


al intentar realizar un aspirado de médula ósea, no se consigue material. En la sangre
periférica usted observa unas células que son positivas para la fosfatasa ácida tartrato-
resistente. ¿Qué patología sospecharía con estos datos?

1. 1. Hemoglobinuria paroxística nocturna.


2. 2. Leucemia mielógena aguda.
3. 3. Leucemia de células peludas.
4. 4. Macroglobulinemia de Waldenström.
Gráfico de respuestas
Comentario

La tricoleucemia, o leucemia de células peludas, se ha preguntado alguna vez en el examen MIR.


Los detalles más relevantes de esta enfermedad son:

- En la exploración física, habrá una importante esplenomegalia, pero no adenopatías.

- En el hemograma, no habrá leucocitosis, sino leucopenia.

- Es típico el aspirado seco en médula ósea, por la fibrosis medular.

- Las células de sangre periférica son positivas para la fosfatasa ácida tartrato- resistente.

- Produce un aumento de susceptibilidad a la infección por Legionella.

- El tratamiento actualmente se realiza con 2- CDA (cladribina).(R3)

415. Está usted en la consulta de ginecología funcional, valorando a una mujer de 17 años
por amenorrea primaria. Mide 150 cm (talla mucho menor que la de sus padres y
hermanos) y pesa 50 kilos. El examen físico revela infantilismo sexual sin ninguna otra
anomalía. En la bóveda vaginal se palpa una estructura compatible con cérvix uterino, y
el examen rectal sugiere la presencia de un útero pequeño. Las concentraciones séricas
de FSH y LH son 120 y 100 mIU/ml respectivamente. De los cariotipos siguientes el que
más probablemente encontraremos en esta paciente es:

1. 1. 45 XO.
2. 2. 46 XX / 47 XXY.

 
 
 
 
3. 3. 46 XY / 47 XXY / 45 XO.
4. 4. 46 XX.
Gráfico de respuestas
Comentario

Es importante que conozcas ciertas características básicas del síndrome de Turner porque puede
ser preguntado. Aproximadamente la mitad son 45 XO, el 25 % son mosaico 45X/46 XX y el resto
presentan un cromosoma X con anomalías estructurales. Tiene estrías gonadales que carecen de
folículos con trompa y útero infantiles. Las gonadotropinas están elevadas y el estradiol bajo. Los
genitales externos son femeninos pero permanecen inmaduros con escasez de vello pubiano y
axilar. No tiene desarrollo mamario. Recordemos que es la causa más frecuente de amenorrea
primaria que cursa con talla corta, linfedema de manos y pies, pterigun colli, tórax en coraza,
coartación de aorta, malformaciones renales, nevus pigmentados, alteraciones autoinmunes...Para
el tratamiento se debe realizar femenización con estrógenos a dosis crecientes hasta conseguir
desarrollo mamario. El tratamiento con GH y oxandrolona mejora el pronóstico de talla final.(R1)

416. The eye fundus of a patient who suffers


hypertension shows the next image. Mark the CORRECT answer:

1. 1. At early stages, we should look for microaneurysms.


2. 2. At final stages, we should look for neovascularization.
3. 3. According to the image, this patient does not have visual symptoms.
Hypertensive retinopathy is much more frequent after years of evolution and hypertensive
4. 4.
peaks.
Gráfico de respuestas
Comentario
Dado el antecedente de hipertensión arterial como único dato, aparte de la imagen, no resulta
difícil intuir que se trata de una retinopatía hipertensiva, que es la enfermedad que padece el
paciente. Uno de los datos a su favor es la presencia de arteriolas muy finas, con exudados en
torno a ellas. Dado que la mácula tiene un aspecto normal, lo más probable es que la agudeza
visual no esté afectada, por lo que el paciente difícilmente referirá síntomas visuales. A juzgar por
las opciones que nos ofrecen, nos plantean un posible diagnóstico diferencial entre retinopatía
hipertensiva y diabética. En la retinopatía diabética, los microaneurismas aparecen desde fases
muy precoces, siendo una lesión muy característica de esta patología (se ve como un pequeño
punto rojo, ya que simplemente es un capilar dilatado), y suele haber hemorragias aparte de
exudados. Sin embargo, la retinopatía diabética no suele complicarse con neovasos, y por lo tanto
no suele ser necesario fotocoagular al paciente. La respuesta correcta es la 4. Efectivamente, la

 
 
 
 
presencia de retinopatía hipertensiva es más frecuente en casos muy crónicos y mal
controlados.(R4)

417. En una paciente gestante de 8 semanas, portadora de distrofia muscular de


Duchenne, ¿qué método de diagnóstico prenatal es el indicado?:

1. 1. Determinación de alfafetoproteína sérica materna.


2. 2. Biopsia de corion.
3. 3. Funiculocentesis.
Triple cribado de alfafetoproteína sérica materna, fracción beta de la gonadotropina
4. 4.
coriónica humana y estriol no conjugado plasmático.
Gráfico de respuestas
Comentario

Pregunta de dificultad media sobre los métodos de diagnóstico prenatal. Se nos presenta un caso
clínico de una paciente en el primer trimestre (8 semanas) en la que hay que descartar que el feto
sea portador de distrofia muscular de Duchenne. Se puede sacar esta pregunta por descarte, ya
que todas las opciones excepto la 2 (biopsia corial) se realizan en el segundo trimestre. Así, la
opción correcta es la 2, porque además de ser la única que se realiza a las 8 semanas de
gestación, está indicada para el diagnóstico de trastornos bioquímicos como la enfermedad de
Gaucher, Niemann- Pick, Tay-Sachs, distrofia muscular de Duchenne o de Becker,
cromosomopatías, infecciones fetales (TORCH). En resumen, siempre que necesite un diagnóstico
citogenético prenatal antes de las 12 semanas de gestación, recurra a la biopsia corial.(R2)

418.
Masculino de 62 años, con antecedentes de diabetes de larga evolución e insuficiencia
renal crónica, en tratamiento con hemodiálisis, que es traído a urgencias por disnea y
dolor precordial de 2 horas de evolución. A la llegada a Urgencias se realiza un ECG que
muestra la imagen nº ##. ¿Cuál de los siguientes enunciados es CORRECTO?:

1. 1. Se deben administrar nitratos endovenosos.


2. 2. No se deben administrar antiagregantes plaquetarios.
3. 3. El tratamiento con fibrinolíticos es el de elección en este caso.
4. 4. La angioplastía podría ser la primera opción terapéutica.
Gráfico de respuestas
Comentario

 
 
 
 
Se trata de una pregunta sobre un síndrome coronario agudo con elevación de ST de 4 mm en DII,
DIII y aVF, en el cual el tratamiento de elección es la reperfusión urgente, bien sea mediante
angioplastia o bien mediante fibrinolisis. Ambos tratamientos realizados de forma precoz
disminuyen la progresión del infarto y, por tanto, disminuyen la zona infartada. Respecto a la
fibrinolisis, a diferencia de la angioplastia, recuerde que no es tan efectiva para disminuir la
aparición de arritmias, porque puede no abrir completamente el vaso.(R4)

419. Niño de 8 años que presenta exantema eritematoso localizado en las mejillas,
asociado a exantema macular eritematos dispuesto en encaje principalmente en las
extremidades. Se debe sospechar en:

1. 1. Megaloeritema.
2. 2. Exantema súbito.
3. 3. Escarlatina.
4. 4. Rubéola.
Gráfico de respuestas
Comentario

No puede fallar las enfermedades exantemáticas.

El eritema infeccioso o quinta enfermedad o megaloeritema está causado por Parvovirus B19, es
una enfermedad propia de la edad escolar 5-15 años. Predomina durante los meses de primavera-
verano.

El paciente se encuentra afebril apareciendo un exantema que evoluciona en tres fases.

Fase inicial o del bofetón eritema de ambas mejillas de aparición brusca.

Fase intermedia: elementos maculopapulosos y eritematosos en tronco y extremidades,


generalmente en superficies de extensión

Tercera fase es la más característica: aclaramiento central de las lesiones, dándoles un aspecto de
encaje o reticulado sin descamación.

Desaparece y aparece en un periodo de entre una y cuatro semanas. Es posible que reaparezca
transitoriamente ante el ejercicio, exposición al sol o fricción de la piel.

Complicaciones: A'S: artritis y artralgias de grandes y pequeñas articulaciones, abortos o hydrops y


aplásicas graves.

Tratamiento es sintomático.(R1)

420. Señale la peculiaridad clínica que NO es característica del síndrome alcohólico fetal:

1. 1. Bajo peso al nacimiento.


2. 2. Microcefalia.
3. 3. Retraso mental.
4. 4. Hipoplasia de arco aórtico.
Gráfico de respuestas
Comentario

 
 
 
 
El síndrome alcohólico fetal es un síndrome dismórfico evitable que aparece tras el consumo de
alcohol durante la gestación. Cursa con retraso ponderoestatural, retraso mental, con microcefalia
asociada, facies peculiar (micrognatia, labio superior fino, filtrum nasal largo) y ocasionalmente
también asocia cardiopatía (hipertrofia septal asimétrica, no hipoplasia de arco aórtico).(R4)

421. Una mujer se encuentra en el sexto mes de gestación. La presión arterial es de 105/60
mmHg y el examen general de orina es normal. ¿Qué esperaría encontrar en el estudio
de laboratorio de esta embarzada?

1. 1. Hemoconcentración.
2. 2. Aumento del filtrado glomerular.
3. 3. Aumento del ácido úrico.
4. 4. Ausencia de variación del aclaramiento de creatinina.
Gráfico de respuestas
Comentario

Los cambios fisiológicos en e el embarazo en el ENARM es un tema bastante rentable.

Una pregunta muy sencilla sobre la fisiología de la gestación. El caso que nos presentan tiene una
presión arterial normal y no existen alteraciones patológicas en el sedimento urinario, por lo que
debemos pensar que se trata de una gestante sana. Durante el embarazo, el filtrado glomerular
aumenta de forma fisiológica, por lo que aumenta el aclaramiento de creatinina, eliminándose una
mayor cantidad de esta sustancia.

De todos los cambios que se producen en el organismo de la gestante, uno de los más importantes
es el aumento del filtrado glomerular.(R2)

422. Un niño de 6 años acude porque se ha despertado con intenso dolor testicular hace
un par de horas. Refiere que el dolor ha empezado de repente. No refiere fiebre ni
molestias miccionales en los días previos. En cuanto a la exploración, usted espera
encontrar todo lo siguiente excepto:

1. 1. Signo de Prehn negativo.


2. 2. Signo de Prehn positivo.
3. 3. Abolición del reflejo cremastérico.
4. 4. Teste elevado.
Gráfico de respuestas
Comentario

El signo de Prehn puede ser de ayuda en el diagnóstico diferencial de la torsión testicular. Cuando
éste es positivo consiste en que el dolor disminuye con la elevación del escroto, situación típica de
las orquiepididimitis. En cambio, lo que sí tendría este niño sería la 2 (Prehn negativo), donde el
dolor no disminuye al ejecutar la maniobra elevando el testículo. Como nos preguntan cuál NO
esperaría encontrar, la solución correcta sería la 3, ya que el signo de Prehn positivo aparece en
las orquitis u orquiepididimitis.(R2)

423. Varón de 27 años, sin antecedentes de interés, que consultó por astenia, anorexia,
vómitos y febrícula de 5 días de evolución. En la exploración física se observó buen nivel
de consciencia, ictericia mucocutánea y hepatomegalia. En la analítica destacaba: GOT
830 UI/l, GPT 1.600 UI/l, F. Alcalina 830 UI/l, Bb 12,5 g/lL y protrombina 75%. Las siguientes
solicitudes diagnósticas estarían justificadas EXCEPTO:

 
 
 
 
1. 1. IgM VHA.
2. 2. HBAgS.
3. 3. DNA VHB.
4. 4. IgM VHB.
Gráfico de respuestas
Comentario

El cuadro que nos plantean es una hepatitis aguda, y se trata de diagnosticar la causa. Sobre los
distintos virus, podemos decir:

La IgM anti- VHA, si es positiva, traduce una hepatitis aguda por este virus. Por tanto, la opción 1
estaría justificada. Recuerda que si apareciese una IgG anti- VHA positiva, no sería el autor del
cuadro, porque implicaría infección pasada.

El VHC suele producir cuadros crónicos, no agudos, como sucede en este caso. No obstante,
habría que tenerlo en cuenta entre las posibles etiologías. En este caso, estaría justificado solicitar
el RNA, y no anticuerpos anti- VHC, porque al estar en fase aguda es muy probable que todavía
sean negativos. No olvides que, en el virus C, es posible que la seroconversión se produzca
después de semanas o meses.

En el caso del VHB, tiene interés solicitar el HBsAg, cuya positividad implicaría la presencia del
virus. Aparte, para distinguir si la infección es aguda o crónica, se solicitarían anticuerpos anti- core
(IgM anti- HBc e IgG anti- HBc). Si es positivo el IgM, sería una hepatitis aguda; en caso de serlo el
IgG, crónica.

En cambio, la opción 4 es la que tiene menos interés. Date cuenta de que nos hablan de una
hepatitis aguda, no una crónica. La cuantificación del DNA del VHB se utiliza para estimar el grado
de replicación, dato importante en una hepatitis crónica… Pero ante un cuadro de hepatitis aguda
de causa todavía no filiada, lo que nos interesa es averiguar la causa. Por supuesto, si sale
positivo el DNA del VHB, también sabríamos que está presente, pero hay formas más baratas y
sencillas de detectar su presencia (como el HBsAg). Por tanto, el DNA resérvalo para
circunstancias en las que te interese conocer el estado replicativo (por ejemplo, para
monitorizar la respuesta al tratamiento con interferón, en una hepatitis crónica).(R3)

424. El método utilizado para detectar el cáncer de la mama en etapa preclínica es:

1. 1. Mastografía.
2. 2. Termografía de mama.
3. 3. Ultrasonido mamario.
4. 4. Tomografía de mama.
Gráfico de respuestas
Comentario

Pregunta de un tema muy importante para el ENARM. El método usado para el screening del
cáncer de mama es la mastografía.

En mujeres jóvenes, de menos de 40 años, es más eficaz el ultrasonido. De todos modos, los
programas de screening suelen ir dirigidos a mujeres de más de 40 años; generalmente el
screening consiste en mamografías bianuales a mujeres entre 40 y 70 años.(R1)

425. Una de las siguientes respuestas respecto a la tiflitis es INCORRECTA:

 
 
 
 
1. 1. Se asocia a situaciones de neutropenia.
2. 2. Es causada sobre todo por Staphilococcus aureus.
3. 3. Es un proceso inflamatorio del ciego.
4. 4. Puede precisar cirugía.
Gráfico de respuestas
Comentario
Pregunta difícil sobre un tema poco preguntado en el MIR. La tiflitis, denominada en ocasiones
enterocolitis necrotizante es un síndrome clínico de fiebre y dolor en fosa iliaca derecha en un
paciente inmunodeprimido. Casi siempre se observa en pacientes neutropénicos tras
quimioterapia, siendo más frecuente en niños y pacientes con leucemia aguda. Suele
acompañarse de hemocultivos positivos para BGN aerobios (opción 2 falsa) y diarrea, a menudo
sanguinolenta. El diagnóstico es clínico y se apoya en un engrosamiento de las paredes del ciego,
visualizado bien por ecografía o TC con contraste. El tratamiento es principalmente médico, con
antibióticos de amplio espectro si bien en ocasiones puede ser necesaria la cirugía. Las recidivas
son raras y la mayoría de los pacientes se recuperan sin problemas.(R2)

426. A 28-year-old male patient comes to the office bacause of sporadic diarrhea. He
reports that the episodes occur when he is under stressful situations such as an
important exam or a public speech, but also after drinking a large amount of coffee or
even without apparent cause. Stools are normal in color and they are sometimes
associated to mucus. After ruling out alarm signs and symptoms such as important
weight loss, bloody diarrhea, fever or nightly episodes, which of the following diagnostic
tests is indicated?

1. 1. Colonoscopy
2. 2. Thyroid panel test (TSH, T4, Free T4, Free T3)
3. 3. Upper GI endoscopy
4. 4. Liver function panel test (AST, ALT, GGT, AP, Bilirubin)
(R2)

427. Una paciente de 23 años acude a la consulta porque en un cultivo realizado hace una
semana aparecen >10000 UFC/ml de E. coli multisensible. Refiere encontrarse
completamente asintomática aunque asegura que ya una vez en la adolescencia había
tenido cistitis. El resto de los datos de la analítica muestran una creatinina de 0.7, 9000
leucocitos sin desviación izquierda y 13.2 de hemoglobina. El test de embarazo es
positivo. En este caso:

1. 1. No es necesario realizar tratamiento ya que se trata de una bacteriuria asintomática.


2. 2. Lo más probable es que los síntomas empiecen unos días.
3. 3. Precisa tratamiento por las probabilidades elevadas de progresar a una PNA.
4. 4. Podría tratarse con Ciprofloxacino 500 mg 1c/12h.
Gráfico de respuestas
Comentario
Se trata de una bacteriuria sintomática que debe tratarse en caso de embarazo, inmusupresión,
proteus, previo a manipulación urológica. En ningún caso, estando embarazada, podría tratarse
con quinolonas, ya que éstas dañan el cartílago fetal.(R3)

428. ¿Cuál de las siguientes funciones NO es propia de la progesterona?:

1. 1. Induce la secreción del endometrio.


2. 2. Estimula la contracción uterina.
3. 3. Induce la secreción del epitelio tubárico.

 
 
 
 
4. 4. Tiene una acción anabólica.
Gráfico de respuestas
Comentario

La progesterona se fabrica en el cuerpo lúteo durante la fase secretora, su misión es la maduración


del endometrio y provocar las manifestaciones que beneficien una gestación. Así, preparan las
mamas para la lactancia, deprimen la excitabilidad de las fibras miometriales, puesto que las
contracciones uterinas impedirían la gestación (también relajan el músculo liso digestivo y ureteral),
elevan el metabolismo y la temperatura corporal y disminuyen la cantidad de moco cervical y su
contenido en ácido siálico, aumentando su viscosidad para que no puedan atravesarlo nuevos
espermatozoides.

La oxitocina es la que favorece la contracción uterina.(R2)

429. Mujer de 65 años de edad sin síntomas articulares hasta hace 5 años. Desde
entonces, de forma intermitente, refiere dolor en interfalángicas proximales y distales de
ambas manos con signos inflamatorios de corta duración seguidos de desarrollo de
tumefacción de consistencia ósea y localización asimétrica alrededor de la articulación.
¿Cuál es el diagnóstico más probable?

1. 1. Artrosis.
2. 2. Artritis reumatoide.
3. 3. Hiperparatiroidismo.
4. 4. Hemocromatosis.
Gráfico de respuestas
Comentario
Esta pregunta no podéis fallarla. Recordad simplemente afectación de INTERFALÁNGICAS
DISTALES (además de proximales) con desarrollo de nódulos de Heberden y de Bouchard en
episodios de corta duración en pacientes de avanzada edad.

Normalmente la AR no presenta afectación de IFD distales y la clínica no es compatible con artritis


por microcristales (el hiperparatiroidismo y la hemocromatosis son causas a recordar de la
condrocalcinosis en pacientes menores de 50 años).(R1)

430. La presencia de acidosis hiperclorémica, pH urinario alcalino, hipopotasemia y


nefrocalcinosis en ausencia de infección urinaria, aminoaciduria, fosfaturia y glucosuria,
es muy sugestiva de:

1. 1. Hiperparatiroidismo primario.
2. 2. Acidosis tubular distal completa.
3. 3. Acidosis tubular incompleta.
4. 4. Intoxicación por etilenglicol.
Gráfico de respuestas
Comentario

La duda razonable estaría entre las acidosis tubulares, dado el detalle de la hipercloremia. No
puede tratarse de una proximal, ya que no orina aminoácidos, fosfatos ni glucosa (es decir, no
tiene síndrome de Fanconi, disfunción tubular proximal). La respuesta correcta, por tanto sería la 2,
que además viene respaldada por la nefrocalcinosis (más frecuente en casos con acidosis tubular
distal que en otros tipos de acidosis tubulares).(R2)

 
 
 
 
431. ¿En cuál de los siguientes síndromes hay actividad ovárica normal?:

1. 1. S. de Morris.
2. 2. S. de Rokitansky.
3. 3. S. de Swyer.
4. 4. S. de Klinefelter.
Gráfico de respuestas
Comentario

Debe conocer algunos datos básicos de estos síndromes, que aunque no son muy frecuentes
como preguntas puede caer algún caso clínico en que le describan uno de ellos. En el síndrome de
Rokitansky lo fundamental es una alteración en la permeabilización de los conductos de Müller. El
fenotipo y el cariotipo es femenino normal. Los ovarios son normales. El útero es rudimentario y no
está canalizado. Hay agenesia de los 2/3 superiores de la vagina, por lo que a la inspección se
observa una vagina corta que termina en un fondo de saco ciego. Presenta frecuentes
malformaciones renales y urinarias.(R2)

432. En la UCIN nos avisan porque un RNPT de 28 semanas, con 15 días de vida, actual
portador de una vía central, y con antecedentes de enfermedad de membrana hialina
grave, está desde hace dos días menos reactivo, tiene mal aspecto y controla mal la
temperatura. Tiene distrés respiratorio moderado y una leve distensión abdominal, no
especialmente llamativa. En la BH presenta: leucocitos: 4,700 (900 neutrófilos, múltiples
bandas), Hb: 8 g/dl. Plaquetas: 100,000/mm3. La Rx simple de abdomen es normal. ¿Cuál
es el diagnóstico más probable?:

1. 1. Sepsis nosocomial.
2. 2. Enterocolitis necrotizante.
3. 3. Ileo meconial.
4. 4. Metabolopatía.
Gráfico de respuestas
Comentario

Un neonato con vía central es un candidato al desarrollo de sepsis. Tenga en cuenta que los
criterios que definen sepsis en el adulto no son válidos ni aplicables al período neonatal, donde
esta situación se define a través de alteraciones de la BH. La BH indicativa de sepsis es aquél que
presenta leucopenia menor de 5000, neutropenia menor de 1500 y desviación izquierda (cociente
IT>0.16). El niño del caso clínico cumple rigurosamente lo dicho sobre estas líneas.(R1)

433. De los siguientes enunciados, marque el FALSO.

1. 1. La gravedad del RCIU es mayor cuando la noxa es precoz.


2. 2. El crecimiento intrauterino es expresión del potencial genético del embrión.
3. 3. El hipotiroidismo materno se asocia a taquipnea transitoria en el recién nacido.
4. 4. La cardiomiopatía hipertrófica se asocia a la diabetes gestacional.
Gráfico de respuestas
Comentario

La respuesta falsa es la 3, ya que es la diabetes materna la que se asocia a taquipnea transitoria


en el RN. El resto de respuestas son correctas.(R3)

 
 
 
 
434. ¿La deficiencia de qué vitamina puede producir un eritema simétrico en las
superficies expuestas, similar a una quemadura solar?:

1. 1. Piridoxina (B6).
2. 2. Vitamina C.
3. 3. Niacina.
4. 4. Tiamina.
Gráfico de respuestas
Comentario
Pregunta de importancia relativamente baja porque no es una cuestión preguntada en el MIR.El
déficit de niacina da lugar a la pelagra que se caracteriza por la triada de diarrea, dermatitis
fotosensible y demencia. Las lesiones aparecen, de forma más o menos simétrica, en áreas de la
piel expuesta a la luz del sol, como la cara, el reverso de las manos, el cuello, los antebrazos y
porciones expuestas de las piernas. En los sujetos de raza blanca las lesiones inicialmente
recuerdan al eritema de las quemaduras solares. Esta dermatitis pelagrosa comienza con un
aumento de la pigmentación que posteriormente se vuelve seca, escamosa y eventualmente
agrietada. Casi siempre hay una línea definida de demarcación entre estas lesiones y la piel sana.
El estado de la piel puede permanecer estático, cicatrizar o empeorar e incluso da lugar a
vesículas que contienen un exudado incoloro. Las áreas que han perdido una capa de piel, a veces
son brillantes, delgadas y más bien despigmentadas.(R3)

435. Un joven de 16 años presenta fiebre, astenia, odinofagia y disfagia. En la exploración


física presenta eritema y exudados faringoamigdalares con adenopatías laterocervicales
y en el abdomen existe hepatomegalia. ¿Qué prueba, más específica, solicitaría usted
para confirmar la enfermedad que sospecha en nuestro joven?:

1. 1. Frotis sanguíneo.
2. 2. Anticuerpos IgM anti CMV.
3. 3. Anticuerpos IgM frente antígenos de la cápside viral (ACV).
4. 4. Anticuerpos IgM frente a antígenos precoces difusos (APD).
Gráfico de respuestas
Comentario
Es importante que dominéis el diagnóstico serológico de la MI (el caso clínico es el típico de esta
enfermedad). Para hacer el diagnóstico de mononucleosis infecciosa (VEB) debemos basarnos en
técnicas microbiológicas, un frotis sanguíneo nunca será diagnóstico (eliminamos opción 1). Los
anticuerpos heterófilos positivos apoyan el diagnóstico pero no lo certifican (eliminamos opción 2).
Para CONFIRMAR una primoinfección por el VEB nos valemos de la IgM ACV (anti cápside viral)
y/o la seroconversión de ANEB (anticuerpos anti antígeno nuclear), por esto la correcta es la
opción 4. Los anticuerpos anti APD son útiles para predecir el riesgo de ciertas complicaciones de
la infección por VEB, como el carcinoma de cavum, en poblaciones de riesgo.(R3)

436. Respecto a la infección urinaria en la infancia, señale la opción INCORRECTA de


entre las siguientes:

Es una de las infecciones más frecuentes en la infancia después de la infección de vías


1. 1.
respiratorias y la gastroenteritis.
Es más frecuente en el sexo femenino, excepto en neonatos, en que es igual de frecuente
2. 2.
en ambos sexos.
3. 3. Los gérmenes causantes son gramnegativos.
4. 4. En hombres con fimosis es típica la infección por Proteus mirabilis.
Gráfico de respuestas
Comentario

 
 
 
 
La respuesta correcta es la 2 ya que efectivamente la infección urinaria es más frecuente en el
sexo femenino, pero la segunda parte de la afirmación es falsa ya que en neonatos es más
frecuente en el sexo masculino, en algunas series hasta 5 veces más frecuente.(R2)

437. Qué afirmación es INCORRECTA respecto a las variables pronósticas del cáncer de
mama:

El tamaño del tumor y el estado de los ganglios linfáticos axilares son factores de valor
1. 1.
pronóstico probado.
La recidiva del cáncer de mama es más probable en los tumores que poseen receptores
2. 2.
hormonales de estrógenos y de progesterona.
La sobreexpresión del gen c-erbB-2 (HER- 2/neu) puede ser indicativa de
3. 3.
resistencia/sensibilidad a determinados tratamientos.
4. 4. Elevadas concentraciones de PCNA (Ki67) indican peor pronóstico.
Gráfico de respuestas
Comentario

Pregunta fácil de uno de los temas primordiales de ginecología: el cáncer de mama.

En este caso, se nos pregunta la incorrecta acerca de los factores pronósticos.

El cáncer de mama es el cáncer más frecuente en la mujer. Los factores de riesgo más destacados
para padecerlo son la excesiva estimulación estrogénica (menarquia precoz, menopausia tardía, la
nuliparidad), los antecedentes familiares (BCRA1, BCRA2) y la patología mamaria previa.

Los factores de mal pronóstico más relevantes son:

•   Edad mayor de 35.


•   Tamaño tumoral mayor de 2 cm.
•   Afectación de ganglios axilares: se ha demostrado como el factor único más importante
para predecir la supervivencia global y libre de enfermedad. El número de ganglios afectos
es también pronósticamente muy significativo (respuesta 1 correcta).
•   Tipo histológico y grado de diferenciación: En general, el subtipo histológico de cáncer de
mama invasivo no es pronósticamente importante, pero algunos tipos especiales de
adenocarcinoma ductal infiltrante aparecen unidos a un riesgo muy bajo de recidiva:
carcinomas tubulares puros, carcinomas papilares puros y carcinomas mucinosos puros. El
grado de diferenciación tumoral sí que representa un factor pronóstico destacado.
•   Receptores hormonales negativos: la presencia de receptores hormonales es un factor de
pronóstico favorable. Tenga en cuenta que su presencia nos informa acerca de la
hormonodependencia del tumor y, por consiguiente, de la posibilidad de bloquearlos
reduciendo así la acción trópica hormonal (respuesta 2 incorrecta).
•   C-erb-B2 y p53: la sobreexpresión de c-erb-B2 conlleva peor respuesta a CMV y
tamoxifeno, y mejor respuesta a adriamicina (respuesta 3 correcta). P53 se considera un
regulador negativo del crecimiento celular, la mutación de p53 se considera un factor de
mal pronóstico por determinar pérdida de la función supresora, activación del crecimiento
celular y aumento del riesgo de progresión de la neoplasia.
•   Angiogénesis e invasión vascular: la presencia de mayor número de microvasos indica
peor pronóstico, porque aumenta la capacidad del tumor de producir metástasis a
distancia.
•   Agresividad local.
•   Gestación.

 
 
 
 
•   Otros: el antígeno Ki67 (PCNA) identifica las células proliferantes dentro de un tumor y, por
tanto, cuanto mayor es su presencia, más agresivo es el tumor. La positividad del Ki67 se
correlaciona con el grado de diferenciación tumoral, invasión vascular, metástasis en
ganglios linfáticos, y se relaciona inversamente con la presencia de receptores hormonales
(respuesta 4 correcta).

(R2)

438. Lactante de 13 meses de edad que es llevado a urgencias por fiebre de 39° C de 48
horas de evolución, sin otra sintomatología asociada. Exploración física sin hallazgos
significativos, destacando buen estado general. Usted le iba a entregar el alta, pero el R3
de Pediatría que está de guardia, le pide un EGO y un urocultivo por sondaje. En la orina
destaca leucocituria ++, hematuria + y nitritos ++ y en el Gram de orina se observan
bacilos Gram negativos. La biometría hemática y química sanguínea se reporta sin
leucocitosis y la proteína C reactiva es de 50 mg/L. El R3 le dice que no es necesario el
ingreso hospitalario, pero que le prescriba un antibiótico oral. Señale el tratamiento
empírico menos adecuado en este caso:

1. 1. Amoxicilina.
2. 2. Amoxicilina-clavulánico.
3. 3. Trimetroprim-sulfametoxazol.
4. 4. Cefixima.
Gráfico de respuestas
Comentario

Se trata de una pregunta de dificultad media sobre la infección del vías urinarias en el niño.

Es importante destacar que en lactantes y niños menores de dos años, los síntomas son
inespecíficos, predominando la fiebre, anorexia, estancamiento pondero-estatural o pérdida de
peso, decaimiento y síntomas digestivos.

En esta pregunta nos plantean el caso de un lactante de 13 meses que acude por fiebre alta de
dos días de evolución, sin foco aparente. Si bien es cierto que las infecciones virales, catarros de
vías altas etc, suelen ser causa frecuente de fiebre a esta edad, es importante tener en cuenta la
infección urinaria en el diagnóstico diferencial de la fiebre sin foco en estos niños. Por ello antes de
dar de alta al paciente, se le realiza un EGO en el que destaca la presencia de leucocitos y nitritos.
Los nitritos son producidos por bacterias que reducen los nitratos a nitritos. La presencia de nitritos
es por tanto un indicador indirecto de presencia de bacterias en la orina.

La leucocituria es más sensible pero menos específica que los nitritos. Puede presentar muchos
falsos positivos. La combinación de nitritos y leucocituria aumenta la sensibilidad a un 93%.
Además, se le ha realizado un Gram de orina, que muestra bacilos Gram negativos. La tinción de
Gram consiste en la visualización mediante microscopio óptico de una gota de orina fresca. Si se
detecta una bacteria por campo de gran aumento, se estima la presencia de 100,000 colonias por
milímetro, es decir sugiere infección urinaria.

La BH y QS muestra elevación de la PCR, apoyando la sospecha de infección bacteriana.

Dados los hallazgos en la orina, la presencia de nitritos y leucocitos, así como el resultado del
Gram, debemos sospechar una infección urinaria e iniciar tratamiento empírico hasta el resultado
del urocultivo.

 
 
 
 
La elección del antibiótico es frecuentemente empírica, y el germen más frecuente es E. coli.
Debemos por tanto elegir un antibiótico que cubra bien los Gram negativos. En este caso además
nos informan mediante el Gram de orina que el germen responsable es un Gram negativo.

Entre las opciones que nos ofrecen el único antibiótico que no cubre bien estas bacterias es la
amoxicilina, de ahí que esta sea la respuesta correcta.

No todos los niños con infección urinaria necesitan ingresar para tratamiento parenteral. Los
principales criterios de ingreso son: edad <3 meses, inmunodeprimidos, anomalías nefrourológicas,
mal estado general, dificultad para tolerar el antibiótico o mantener una buena hidratación, así
como problemas de seguimiento adecuado del niño.

Si es posible el antibiótico oral, se recomienda mantener el tratamiento hasta un total de 10-14


días.(R1)

439. Con respecto a las quemaduras, ¿qué manifestaciones clínicas presentan las
quemaduras de segundo grado?:

1. 1. Eritema, dolor intenso y sequedad.


2. 2. Superficie dura y dolor escaso o ausente.
3. 3. Dolor intenso, formación de ampollas y exudado.
4. 4. Dolor escaso o ausente, exudado y ampollas.
Gráfico de respuestas
Comentario

La aparición de ampollas es el hallazgo definitorio de las quemaduras de segundo grado, por lo


que desecharíamos las opciones 1 y 2.

Aquí donde está el problema, es que el segundo grado puede dividirse en superficial (hasta la
dermis papilar) y profundo (hasta la dermis reticular), siendo las únicas diferencias que en el
superficial hay exudado y dolor, mientras en el profundo hay una costra cérea, gruesa, NO hay
exudación o es mínima y el dolor es escaso o ausente, por lo que la opción correcta sería la 3 ya
que en la combinación de la 4 debería ser sin exudado o un exudado mínimo y con costras.(R3)

440. En el cáncer oral, descartando los labios, ¿cuál de estas localizaciones es la más
frecuente?

1. 1. Encía superior.
2. 2. Paladar duro.
3. 3. Lengua móvil.
4. 4. Paladar blando.
Gráfico de respuestas
Comentario

El cáncer oral es un tumor maligno que es más genérico en varones e, histológicamente, la


mayoría son epidermoides. Las localizaciones más habituales son labio (sobre todo inferior) y
lengua móvil (sobre todo en bordes laterales). El factor etiológico más frecuentemente implicado es
el tabaco.(R3)

 
 
 
 
441. Un paciente diagnosticado de esclerosis múltiple acude a un servicio de urgencias
por presentar desde hace 2 días, con carácter progresivo, pérdida de fuerza en MID que
le impide la deambulación ¿Cuál es el tratamiento de elección en este momento?

1. 1. Corticoides en altas dosis vía intravenosa.


2. 2. Plasmaféresis.
3. 3. Interferón beta subcutáneo.
4. 4. Interferón gamma.
Gráfico de respuestas
Comentario

Pregunta en forma de caso clínico sobre el tratamiento de la esclerosis múltiple. Lo primero que
hay que tener muy claro es que nos encontramos ante un paciente con un brote importante de su
enfermedad desmielinizante (clínica motora). El tratamiento de un brote de EM consiste en
corticoides por vía intravenosa, excepto en los casos en los que el brote es leve (brote sensitivo)
donde se utilizarían por vía oral. Los corticoides disminuyen la gravedad y la duración del brote,
pero no modifican el curso de la enfermedad, el número de brotes ni mejoran las secuelas de
brotes previos. La azatioprina y el Interferon beta se usan como tratamiento de base para modificar
el curso de la enfermedad. La plasmaféresis y el interferón gamma no se utilizan en el tratamiento
de la EM.(R1)

442. Un masculino paciente acude a consulta por dolor y derrame en ambas rodillas. La
artrocentesis aporta un líquido sinovial turbio con viscosidad disminuida, 40.000
leucocitos (80% neutrófilos), glucosa normal y proteínas altas. ¿Cuál de estas entidades
NO incluiría en el diagnóstico diferencial?:

1. 1. LES.
2. 2. Gota.
3. 3. Artrosis.
4. 4. Artritis por hongos.
Gráfico de respuestas
Comentario

Nos dan el cuadro de un paciente con una oligoartritis simétrica de miembros inferiores en el que
se identifica un líquido articular de características inflamatorias ( células >3000 con predominio de
PMN, una viscosidad disminuida y unas proteínas elevadas). En este caso la elección está clara,
cualquiera de estas patologías te puede dar un líquido de características inflamatorias menos la
artrosis que te da un líquido de características mecánicas o no inflamatorias.(R3)

443. A un paciente hemofílico de los que usted controla en consulta se le va a realizar el


empaste de una caries, por lo que usted decide administrar una dosis única de
concentrado de factor VIII para prevenir el sangrado. ¿Qué otro agente le parece
razonable asociar al tratamiento?:

1. 1. Uroquinasa.
2. 2. Antitrombina III.
3. 3. Acido epsilon-aminocaproico (EACA).
4. 4. Corticoides.
Gráfico de respuestas
Comentario

 
 
 
 
El tema de las hemofilias es poco frecuente en el MIR, y en concreto preguntas tan específicas
sobre su manejo no han aparecido nunca. Basta con conocer que el déficit de factor VIII constituye
la hemofilia A mientras que el déficit de factor IX constituye la hemofilia B. Ambas son trastornos
ligados al cromosoma X y debemos sospecharlas ante la presencia de hemartros, hematomas en
tejidos blandos, con antecedentes familiares, ante lo cual se realiza un estudio de la coagulación
donde la alteración típica es un TP normal con un TTPa alargado, pero recuerda que el diagnóstico
definitivo nos lo da la cuantificación de factores. El tratamiento se basa en la utilización de
crioprecipitados o concentrados liofilizados o incluso factores obtenidos por ingeniería genética.
Pero además los hemofílicos precisan tratamiento antes de someterse a cirugías o manipulaciones
dentales. En este caso suele bastar con un goteo de crioprecipitados o concentrados de factor VIII
combinado con la administración de 4- 6g de ácido epsilon- aminocaproico cuatro veces al día
durante 72 a 96 horas después de la intervención. El EACA es un potente antifibrinolítico que
inhibe los activadores del plasminógeno que contienen las secreciones bucales y estabiliza la
formación del coágulo en los tejidos de la boca. El tratamiento debe comenzar inmediatamente
antes de proceder a la intervención y hay que mantenerlo como mínimo 48- 72 horas.(R3)

444. Un paciente fumador presenta en la espirometría forzada un volumen espiratorio


forzado en el primer segundo (FEV1) del 31% del predicho, una capacidad vital forzada
(FVC) del 80% del predicho y una relación FEV1/FVC del 40%. Según estos resultados, el
paciente presenta:

1. 1. Una obstrucción al flujo aéreo leve.


2. 2. Una obstrucción al flujo aéreo grave.
3. 3. Una obstrucción al flujo aéreo muy grave.
4. 4. Una obstrucción al flujo aéreo grave asociada a restricción al menos moderada.
Gráfico de respuestas
Comentario

Una pregunta difícil, ya que exige conocimientos memorísticos, aunque los niveles de gravedad de
la EPOC ya habían aparecido en convocatorias previas a ésta.

En primer lugar, no existe ningún dato que nos haga pensar en un trastorno restrictivo añadido, por
lo que la respuesta 4 puede descartarse de entrada. Respecto a las otras, debe saber que la
consideración de grave o muy grave depende sobre todo del VEMS (FEV1). Se dice que es grave
cuando el VEMS se encuentra entre un 30% y un 50% del esperado, por lo que la respuesta
correcta es la 2. También nos ofrecen como dato un índice de Tiffeneau (FEV1/CVF) menor del
40%, dato menos importante para considerar la gravedad pero que orienta claramente a un
trastorno obstructivo.(R2)

445. ¿Cuál es la mejor combinación de fármacos para el tratamiento de la insuficiencia


cardiaca post-infarto una vez estabilizado el paciente?:

1. 1. IECA, ARA-II, betabloqueante y nitrato.


IECA, ARA-II, antagonista de aldosterona (si función renal y potasio normales) y
2. 2.
betabloqueante.
3. 3. IECA, betabloqueante y antialdosterónico.
4. 4. No existe una combinación idónea preestablecida.
Gráfico de respuestas
Comentario
Todo paciente que ha tenido un IAM tiene que estar bajo tratamiento con betabloqueantes e IECA.
Si al paciente como consecuencia del IAM le ha quedado una FEVI disminuida y en el ingreso tuvo
IC está indicada la adición al tratamiento de antagonistas de la aldosterona (el que se ha estudiado

 
 
 
 
en este contexto es la eplerrenona). La asociación de IECA con ARA- II no ha demostrado
beneficio con respecto a cada uno de ellos por separado y sí más efectos secundarios (retención
de potasio, empeoramiento de la función renal). Por tanto, salvo casos de IC refractaria en el que
lo busquemos sea un mayor grado de bloqueo del SRAA, no utilizaremos esa combinación. Lo que
está absolutamente contraindicado es el bloqueo del SRAA con 3 fármacos (IECA + ARA- II +
antagonista de aldosterona) por el riesgo tan alto de hiperpotasemia. La digoxina, los nitratos y los
diuréticos no tienen efecto sobre el pronóstico.(R3)

446. Acude a consulta de urgencias un niño con babeo,


dificultad respiratoria y estridor inspiratorio. Cuando intenta acostarlo para explorarlo,
empeora su disnea, encontrándose mejor sentado, apoyado sobre sus manos. Usted
sospecha una epiglotitis. ¿Cuál es la actitud menos indicada?

1. 1. Valoración en unidad de críticos o UCI pediátrica.


Examen laríngeo en quirófano, para visualizar epiglotis con aspecto característico "rojo
2. 2.
cereza".
3. 3. Radiografía lateral de faringe. Hemocultivo y laboratorios.
Exploración en consulta con abatelenguas para descartar otras causas que puedan simular
4. 4.
la clínica, como faringitis o amigdalitis.
Gráfico de respuestas
Comentario

Ante una sospecha de epiglotitis no debe manipularse la cavidad oral. La epiglotis se encuentra
adyacente a la base de la lengua, y en los niños la epiglotis es muy alta, dado que tienen una
faringe corta. La manipulación de la epiglotis, puede desencadenar un aumento de la inflamación,
resultando en una situación crítica.

Ante una sospecha leve puede iniciar el estudio con radiografía lateral, buscando tumefacción.
Hemocultivo y exámenes de laboratorio (reactantes de fase aguda).

En caso de alta sospecha, mantendrá al niño en una unidad en la que pueda intubarse en caso de
necesidad. Y se realizará una exploración en quirófano. Si se comprueba la epiglotitis, se
procederá a la intubación.(R4)

 
 
 
 
447. An 18-year-old African girl with a history of recurrent colds with fever and sore throat
during the last five years, who is otherwise healthy (she plays tennis three times a week
at school) is seen at a routine health check-up. Cardiac auscultation shows a diastolic
murmur with a loud first sound. The rest of physical examination is unremarkable. Which
of the following is the treatment of choice for this patient?

1. 1. Monthly intramuscular injections of Penicillin G


2. 2. Voriconazole
3. 3. Beta blockers
4. 4. Balloon angioplasty

(R1)

448. Respecto a los objetivos de control metabólico propuestos por la ADA en 2010, ¿cuál
es falso?

1. 1. Colesterol LDL < 100 mg/dl.


2. 2. HbA1C < 6,5%.
3. 3. HDL > 40 en varones y > 50 en mujeres.
4. 4. Triglicéridos < 150 mg/dl.
Gráfico de respuestas
Comentario
La ADA propone una HbA1C menor de 7%. El resto de respuestas son correctas, además se
insiste en que el control tensional mantenga cifras menores de 130/80 mmHg.(R2)

449. Paciente de 36 años, con antecedentes de artritis migratoria no deformante y


lesiones cutáneas faciales fotosensibles, que consulta por presentar dolor, tumefacción
y aumento de temperatura en la pierna izquierda. En la analítica destaca un TTPA de 50
sobre un control de 28. En la exploración, el signo de Homans es positivo. El tratamiento
adecuado es:

1. 1. AINEs.
2. 2. Corticoides a dosis bajas.
3. 3. Antiagregantes.
4. 4. Anticoagulantes.
Gráfico de respuestas
Comentario
En este caso clínico te orientan claramente a sospechar que se trata de un lupus eritematoso
sistémico con anticuerpos antifosfolídos. Los síntomas propios del lupus son la artritis migratoria,
las lesiones cutáneas faciales (rash facial o eritema en alas de mariposa. Por otro lado el paciente
presenta un alargamiento evidente del TTPA sobre el control, es decir presenta un anticoagulante
lúpico, debido a la presencia de anticuerpos antifosfolípido. Esta situación predispone a la
aparición de fenómenos trombóticos, sobretodo venosos y abortos de repetición. Es evidente que
se tratará de una trombosis venosa profunda el dolor y aumento de partes blandas en la pierna.
Desde el punto de vista terapéutico debemos anticoagular al paciente, inicialmente con heparina
subcutánea y mantener posteriormente anticoagulado de forma indefinida con anticoagulantes
orales manteniendo un INR entre 2.5 y 3.5.(R4)

450. La ictericia fisiológica del recién nacido es:

1. 1. Una hiperblirrubinemia no conjugada.

 
 
 
 
2. 2. Es más común en aterminos que en preterminos.
3. 3. Es poco común en niños de raza negra.
4. 4. Generalmente asociado a anemia moderada y reticulocitosis.
Gráfico de respuestas
Comentario

La ictericia fisiológica del RN es a expensas de B. indirecta. Hay que tener en cuenta que cualquier
aumento de B. directa en el RN se debe considerar patológico y no fisiológico.(R1)

451. NO es característico de la clínica de la Depresión:

1. 1. Somatizaciones.
2. 2. Pérdida de interés.
3. 3. Ansiedad.
4. 4. Ideas delirantes de tipo expansivo.
Gráfico de respuestas
Comentario
Las ideas delirantes de fondo expansivo son características de la manía (5 falsa). Las
somatizaciones y las preocupaciones hipocondríacas son muy frecuentes en la depresión, sobre
todo en los ancianos. La ansiedad ocurre como síntoma accesorio en el 50- 80% de las
depresiones (más en la "neuróticas", menos o nada en las melancólicas).(R4)

452. An 11-month-old child is brought to the pediatrician's office by his worried mother,
who thinks that he has an impairment in development because his cousins are taller than
him. Physical examination shows height 70 cm and weight 9 kg. He uses his hands to
bang four cubes together, and uses pincer grasping. He says "mama" and exclamations
such as "oh, oh!" . He tries to imitate words. He is learning to drink from a cup and he
enjoys imitating people when playing. Which of the following is the best advice to his
mother?

1. 1. His development is normal


2. 2. His height is normal, but he is underweight.
3. 3. Genetic test must be performed in order to rule out an inherited condition.
4. 4. He is overweight but taller than expected
Gráfico de respuestas
Comentario
His development is normal. The child described in this case report presents normal development. At
eleven months the child is able to respond to his own name and waves bye-bye. He can also play
pat-a-cake, understands “no-no!” and gives and takes objects. The height and weight are normal
according to percentile curves.(R1)

453. Un niño de 6 años de edad, presenta un desarrollo puberal precoz con cuadros de
hiperfagia y un gran aumento de peso, En la anamnesis destaca poliuria y polidipsia,
¿Qué sospecharía?

1. 1. Craneofaringioma.
2. 2. Glioma óptico.
3. 3. Histiocitosis X.
4. 4. Hemangiomas.
Gráfico de respuestas
Comentario

 
 
 
 
Cuadro compatible con craneofaringioma y alteración de hipotálamo, lo que explica la hiperfagia, el
aumento de peso y la diabetes insípida con poliuria y polidipsia, respuesta 3 correcta.(R1)

454. Mujer de 59 años sin antecedentes personales ni familiares de interés, que realiza su
revisión ginecológica anual. Menarquia a los 13 años; 2 partos eutócicos; lactancia
artificial por pezón invertido; menopausia a los 52 años, desde entonces asintomática.
En mastografía de control anual se describe la aparición de un grupo de 8
microcalcificaciones finas, agrupadas y situadas en CSE. No se detectan imágenes
nodulares. Señale la actitud más CORRECTA:

1. 1. Realizar biopsia diferida guiada con arpón.


2. 2. Realizar biopsia intraoperatoria seguida de protocolo quirúrgico completo.
3. 3. Observación 6 meses, período tras la cuál se realizará nueva mastografía.
4. 4. Ultrasonido.
Gráfico de respuestas
Comentario

Pregunta importante sobre el cáncer de mama radiológico. Tenemos el caso clínico de una
paciente que en el screening mastográfico se observa una lesión altamente sospechosa de
malignidad (>5 microcalcificaciones finas agrupadas). Lo primero que debemos hacer es obtener
material histológico. Si se palpara un nódulo podríamos realizar una PAAF o una PAAG; pero como
no existe nódulo palpable, la única forma de obtener una biopsia de la zona con
microcalcificaciones (que es la sospechosa) es colocar un arpón con control radiológico, y extirpar
la zona en la que se encuentra dicho arpón. Así, se hará estudio histológico de la zona extirpada
(que guiará la actitud a seguir), que en ningún momento se palpaba.(R1)

455. Señale la INCORRECTA en relación a los leiomiomas gástricos:

1. 1. Tienen tendencia a ulcerarse.


2. 2. La biopsia endoscópica tiene una gran sensibilidad.
3. 3. La distinción con el leiomiosarcoma puede ser difícil.
4. 4. Pueden alcanzar dimensiones de hasta 20 cm.
Gráfico de respuestas
Comentario
Los leiomiomas son los tumores gástricos benignos más frecuentes. Aunque habitualmente son
asintomáticos, pueden dar lugar a dolor abdominal o, incluso, sangrado, por ulceración de la
mucosa. Suelen ser hallazgos necrópsicos, por no dar síntomas, pudiendo alcanzar grandes
dimensiones. Pero la biopsia endoscópica es poco rentable, porque a veces, son muy difíciles de
distinguir de un leiomiosarcoma.(R2)

456. En un niño de 15 meses diagnosticado de enfermedad celíaca, ¿qué cereal, de los


siguientes, recomendaría porque NO se asocia a la producción de este cuadro?

1. 1. Trigo.
2. 2. Cebada.
3. 3. Arroz.
4. 4. Avena.
Gráfico de respuestas
Comentario

 
 
 
 
Es muy importante que el día del ENARM no olvide los cereales que contienen gluten: trigo,
cebada, centeno y posiblemente en avena (por contaminación cruzada). Para recordar qué
alimentos se pueden dar, recuerde que terminan con la letra z: maíz y arroz. Respuesta 3
correcta.(R3)

457. Paciente de 26 años, fumadora, sin hijos, sin pareja


estable, es remitida a su consulta de ginecología por citología positiva para HSIL (lesión
intraepitelial de alto grado). Usted realiza una colposcopía ante dicho hallazgo,
obteniéndose la imagen que se muestra. ¿Cuál es la conducta más indicada a
continuación?

1. 1. Tratamiento con antiinflamatorios locales.


2. 2. Estudio de extensión de la enfermedad.
3. 3. Biopsia de la lesión.
4. 4. Conización cervical.
Gráfico de respuestas
Comentario

Pregunta de dificultad media acerca del manejo de las displasias cervicales (simplemente
recordando el algoritmo se contesta la pregunta).

Ante una citología positiva debemos confirmarlo mediante una colcoscopía (tal como comentan en
el enunciado de la pregunta). La imagen corresponde a epitelio acetoblanco (mediante uso de
ácido acético las lesiones se tiñen de blanco, por lo que el siguiente paso será biopsiar la lesión e
estadificar (recuerde que entre los hallazgos colposcópicos anormales se encuentra el epitelio
acetoblanco, epitelio yodonegativo, cambios de coloración o leucoplasia, neoformación vascular y
superficie irregular con pérdida de epitelio normal). Lógicamente, no se realizará directamente una
conización sin confirmación histológica previa. La citología (y éste es el dato que nos dan en el
enunciado) es una técnica muy útil como screening, pero tiene sus limitaciones y, antes de tomar la
decisión terapéutica definitiva, necesitamos una muestra histológica (biopsia), que es una prueba
más específica, para asegurar el resultado anormal.(R3)

458. Con respecto a la patología que presenta la paciente ¿cuál de los siguientes
hallazgos NO sería sugestivo de patología cervical?

1. 1. Eritroplasia cervical.
2. 2. Áreas yodonegativas.
3. 3. Superficie ulcerada.
4. 4. Leucoplasia.
Gráfico de respuestas
 
 
 
 
Comentario

A continuación, le recordamos los hallazgos colposcópicos anormales:

1.- Epitelio acetoblanco.

2.- Epitelio yodonegativo.

3.- Cambios de coloración (leucoplasia).

4.- Neoformación vascular.

5.- Superficie irregular, con pérdida del epitelio normal, con ulceración.

La ectopia o eritroplasia cervical consiste en la presencia de epitelio cilíndrico por debajo del orificio
cervical externo, visible en la colposcopía desde la vagina, quedando en contacto con el medio
vaginal hostil. Aunque la mayor parte de las veces son lesiones asintomáticas, pueden originar
leucorrea y hemorragia postcoital. El diagnóstico se realiza mediante colposcopía, y pueden
eliminarse si producen clínica (a veces produce sangrado). No obstante, no se relaciona con
patología cervical maligna (respuesta 1 correcta).(R1)

459. The image shows a chest X-ray film. Which of the following options is correct
regarding the suspected diagnosis?

1. 1. 20-year survival is possible with adequate treatment.


2. 2. Pathology is essential to confirm diagnosis in every single case.
3. 3. It most commonly affects patients 30 to 50 years old.
4. 4. Pneumothorax is a rare complication in patients affected with this disease.
Gráfico de respuestas
Comentario
En la imagen que nos presentan tenemos un patrón retículo-nodular difuso que afecta
preferentemente a los campos inferiores, lo que debería hacernos pensar en una fibrosis pulmonar
idiopática. La respuesta correcta es la 4, ya que efectivamente la presencia de neumotórax sería
una complicación bastante rara en esta patología. Otras enfermedades intersticiales, como la
histiocitosis X, sí que podrían complicarse con neumotórax, pero en ese caso existe una afectación
preferente de los campos superiores.(R4)

 
 
 
 

 
 
 
 

 
 
 
 

 
 
 
 

 
 
 
 

 
 
 
 

 
 
 
 

 
 
 
 

 
 
 
 

 
 
 
 

 
 
 
 

 
 
 
 

 
 
 
 

 
 
 
 

 
 
 
 

 
 

Potrebbero piacerti anche